Big Ideas Math Answers Grade 5 Chapter 12 Patterns in the Coordinate Plane

Big Ideas Math Answers Grade 5 Chapter 12 Patterns in the Coordinate Plane

Big Ideas Math Answers Grade 5 Chapter 12 Patterns in the Coordinate Plane will help the students to improve their math skills easily. Answers are provided for Big Ideas Math Answers Grade 5 Chapter 12 Patterns in the Coordinate Plane along with the explanation. All the solutions are explained clearly step-by-step in a simple manner. Every topic is explained by the math experts and given with real-time examples. Therefore, make your preparation simple by referring to our Big Ideas Math Answers Grade 5 Chapter 12 Patterns in the Coordinate Plane.

Big Ideas Math Book 5th Grade Chapter 12 Patterns in the Coordinate Plane Answer Key

Students who are struggling to solve Big Ideas Math Book 5th Grade Answer Key Chapter 12 Patterns in the Coordinate Plane have reached the correct place. This BIM Book 5th Grade 12th Chapter Answer Key gives the most accurate answers to all the questions related to this chapter. Different methods of solving every question are given on this page. Verify the below links to know the different methods to solve problems. Be the first to learn the concepts and practice all the questions available here.

Lesson: 1 Plot Points in a Coordinate Plane

Lesson: 2 Relate Points in a Coordinate Plane

Lesson: 3 Draw polygons in a Coordinate Plane

Lesson: 4 Graph Data

Lesson: 5 Make and Interpret Line Graphs

Lesson: 6 Numerical Patterns

Lesson: 7 Graph and Analyze Relationships

Chapter: 12 – Patterns in the Coordinate Plane

Lesson 12.1 Plot Points in a Coordinate Plane

Explore and Grow

Choose a location for your buried treasure on the grid. Choose a point where two grid lines My Treasure intersect. An example is shown.
Big Ideas Math Answers Grade 5 Chapter 12 Patterns in the Coordinate Plane 1
Take turns with a partner guessing the location of each other’s buried treasure. Keep track of your guesses on the grid. After each guess, give a clue to help yourMy Guessespartner, such as “my treasure is northwest of your guess.”This is an image Continue to guess until a treasure is located.
Big Ideas Math Answers Grade 5 Chapter 12 Patterns in the Coordinate Plane 2
Reasoning
The point where the horizontal number line and the vertical number line intersect is called the origin. Why do you think it is called that?

Think and Grow: The Coordinate Plane

Key Idea
A coordinate plane is formed by the intersection of a horizontal number line and a vertical number line. An ordered pair is a pair of numbers that is used to locate a point in a coordinate plane.
Big Ideas Math Answers Grade 5 Chapter 12 Patterns in the Coordinate Plane 3
Example
Write the ordered pair that corresponds to point M.
Big Ideas Math Solutions Grade 5 Chapter 12 Patterns in the Coordinate Plane 4
Big Ideas Math Solutions Grade 5 Chapter 12 Patterns in the Coordinate Plane 5
The horizontal distance from the origin to point M is __ units. So, the x-coordinate is __.
The vertical distance from the origin to point M is ___ units. So, the y-coordinate is ___. The ordered pair is __.

Answer:
The horizontal distance from the origin to point M is 5 units. So, the x-coordinate is 5.
The vertical distance from the origin to point M is 5units. So, the y-coordinate is 3. The ordered pair is (5, 3).

Show and Grow

Write the ordered pair corresponding to the point.

Question 1.
Point B
Answer:(4,2)
Big-Ideas-Math-Solutions-Grade-5-Chapter-12-Patterns-in-the-Coordinate-Plane-6 12.1-1
Explanation:
The horizontal distance from the origin to point B is _4_ units.
So, the x-coordinate is _4_.
The vertical distance from the origin to point B is __2_ units.
So, the y-coordinate is _2__. The ordered pair is _(4,2)_.

Question 2.
Point S

Big Ideas Math Solutions Grade 5 Chapter 12 Patterns in the Coordinate Plane 6
Answer:(0,4)
Explanation:
The horizontal distance from the origin to point S is 0  units.
So, the x-coordinate is _0_.
The vertical distance from the origin to point S is __4_ units.
So, the y-coordinate is _4__.
The ordered pair is _(0,4)_.

Plot and label the point in the coordinate plane.

Question 3.
F(5, 4)
Answer:(5,4)
Explanation:
Big-Ideas-Math-Solutions-Grade-5-Chapter-12-Patterns-in-the-Coordinate-Plane-18 3
Start the Origin. Move  Units right and units up. Then plot and label the point.
The point can be labeled as Y,(5,4)

Question 4.
P(3, 0)
Answer:(3,0)
Explanation:
Big-Ideas-Math-Solutions-Grade-5-Chapter-12-Patterns-in-the-Coordinate-Plane-18 4
Start the Origin. Move  Units right and units up. Then plot and label the point.
The point can be labeled as Y,(3,0)

Apply and Grow: Practice

Use the coordinate plane to write the ordered pair corresponding to the point.

Big Ideas Math Solutions Grade 5 Chapter 12 Patterns in the Coordinate Plane 6.1

Question 5.
Point M
Answer:(8,8)
Explanation:
Big Ideas Math Solutions Grade 5 Chapter 12 Patterns in the Coordinate Plane 6.1
The horizontal distance from the origin to point M is  units.
So, the x-coordinate is _8_.
The vertical distance from the origin to point M is __8 units.
So, the y-coordinate is 8_.
The ordered pair is _(8,8)_.

Question 6.
Point Q
Answer:(2,7)
Explanation:
Big Ideas Math Solutions Grade 5 Chapter 12 Patterns in the Coordinate Plane 6.1
The horizontal distance from the origin to point Q is 2  units.
So, the x-coordinate is _2_.
The vertical distance from the origin to point Q is __7_ units.
So, the y-coordinate is _7__.
The ordered pair is _(2,7)_.

Question 7.
Point N
Answer: (7,6)
Explanation:
Big Ideas Math Solutions Grade 5 Chapter 12 Patterns in the Coordinate Plane 6.1
The horizontal distance from the origin to point N is 7 units.
So, the x-coordinate is _7_.
The vertical distance from the origin to point N is _6_ units.
So, the y-coordinate is _6__.
The ordered pair is _(7,6)_.

Question 8.
Point R
Answer:(2,0)
Explanation:
Big Ideas Math Solutions Grade 5 Chapter 12 Patterns in the Coordinate Plane 6.1
The horizontal distance from the origin to point R is 2  units.
So, the x-coordinate is _2_.
The vertical distance from the origin to point R is __0_ units.
So, the y-coordinate is _0_.
The ordered pair is _(2,0)_.

Question 9.
Point P
Answer:(6,1)
Explanation:
Big Ideas Math Solutions Grade 5 Chapter 12 Patterns in the Coordinate Plane 6.1
The horizontal distance from the origin to point P is 6  units.
So, the x-coordinate is _6_.
The vertical distance from the origin to point P is __1_ units.
So, the y-coordinate is _4__.
The ordered pair is _(6,1)_.

Question 10.
Point T
Answer: (0,5)
Explanation:
Big Ideas Math Solutions Grade 5 Chapter 12 Patterns in the Coordinate Plane 6.1
The horizontal distance from the origin to point T is  units.
So, the x-coordinate is _0_.
The vertical distance from the origin to point P is __5_ units.
So, the y-coordinate is _5__.
The ordered pair is _(0,5)_.

Plot and label the point in the coordinate plane above.

Question 11.
S(0, 3)
Answer: The Point can be labeled as (0,3)
Explanation:
Big-Ideas-Math-Solutions-Grade-5-Chapter-12-Patterns-in-the-Coordinate-Plane-18 11
Start the Origin. Move Units right and  3 units up then plot and label the point.
The point can be labeled as Y,(0,3)

Question 12.
F(2, 5)
Answer: (2,5)
Explanation:
Big-Ideas-Math-Solutions-Grade-5-Chapter-12-Patterns-in-the-Coordinate-Plane-18 12
Start the Origin. Move  Units right and  5 units up then plot and label the point.
The point can be labeled as Y,(2,5)

Question 13.
W(0, 0)
Answer:(0,0)
Explanation:
Big-Ideas-Math-Solutions-Grade-5-Chapter-12-Patterns-in-the-Coordinate-Plane-18 13
Start the Origin. Move Units right and  0 units up then plot and label the point.
The point can be labeled as Y,(0,0)

Name the point for the ordered pair.

Question 14.
(5, 2)
Answer: D(5,2)
Explanation:
Start the Origin. Move Units right and  2 units up then label the point.
The point can be Named as D(5,2)

Big Ideas Math Solutions Grade 5 Chapter 12 Patterns in the Coordinate Plane 7.1

Question 15.
(8, 4)
Answer: E(8,4)
Explanation:
Big Ideas Math Solutions Grade 5 Chapter 12 Patterns in the Coordinate Plane 7.1
Start the Origin. Move Units right and  4 units up then label the point.
The point can be Named as E(8,4)

Question 16.
(0, 3)
Answer: F(0,3)
Explanation:
Big Ideas Math Solutions Grade 5 Chapter 12 Patterns in the Coordinate Plane 7.1
Start the Origin. Move Units right and  3 units up then label the point.
The point can be Named as D(5,2)

Question 17.
Reasoning
How are the locations of the points A(0, 4) and B(4, 0) different in a coordinate plane?
Answer: A(0,4) is in Y axis as x is 0 and y is 4. B(4,0) is in X axis as x is 4 and y is 0.

Question 18.
DIG DEEPER!
Newton buries a bone in a park at the location shown. How can he use a coordinate plane to describe its location?
Big Ideas Math Solutions Grade 5 Chapter 12 Patterns in the Coordinate Plane 8

Answer:
He can use the x-coordinate and y-coordinate to describe the location.

Think and Grow: Modeling Real Life

Example
In a video game, you move an aircraft carrier and a tugboat away from your base. Use the directions to plot and label the locations of the aircraft carrier and the tugboat.
Big Ideas Math Solutions Grade 5 Chapter 12 Patterns in the Coordinate Plane 9
• Aircraft carrier: Located 3 miles east and 4 miles north of your base.
• Tugboat: Located 8 miles east and twice as many miles north of your base as the aircraft carrier.
To find the location of the aircraft carrier, start at your base, which is at the origin.

Move __ units east, or right, and ___ units north, or up.
Plot and label the point as A(___, ___ ).
To find the location of the tugboat, start at your base, which is at the origin.
Move ___ units east, or right, and ___ × __ = ___ units north, or up.
Plot the label the point as T(__, ___).
Answer: Mov

Show and Grow

Question 19.
A guidebook describes how to get to various statues in Chicago, Illinois, from Willis Tower. Plot and label the location of each statue on the map.
Dubuffet’s Monument with Standing Beast: Walk 2 blocks east and 5 blocks north.
• Miró’s Sun, Moon, and One Star: Walk twice as many blocks east as you do to get to the Standing Beast, and 3 blocks north.
Big Ideas Math Solutions Grade 5 Chapter 12 Patterns in the Coordinate Plane 10

Answer:
Big-Ideas-Math-Solutions-Grade-5-Chapter-12-Patterns-in-the-Coordinate-Plane-10

Question 20.
DIG DEEPER!
Which statue is closer to Moon, and One Star, Cloud Gate or Flamingo? Explain.

Plot Points in a Coordinate Plane Homework & practice 12.1

Use the coordinate plane to write the ordered pair corresponding to the point.

Big Ideas Math Solutions Grade 5 Chapter 12 Patterns in the Coordinate Plane 11

Question 1.
Point E
Answer: (5,7)
Explanation:
Big Ideas Math Solutions Grade 5 Chapter 12 Patterns in the Coordinate Plane 11
The horizontal distance from the origin to point E is 5  units.
So, the x-coordinate is _5_.
The vertical distance from the origin to point E is __7_ units.
So, the y-coordinate is _7__.
The ordered pair is _(5,7)_.

Question 2.
Point H
Answer: (0,7)
Explanation:
Big Ideas Math Solutions Grade 5 Chapter 12 Patterns in the Coordinate Plane 11
The horizontal distance from the origin to point H is 0  units.
So, the x-coordinate is _0_.
The vertical distance from the origin to point H is __7_ units.
So, the y-coordinate is _7__.
The ordered pair is _(0,7)_.

Question 3.
Point F
Answer: (3,3)
Explanation:
Big Ideas Math Solutions Grade 5 Chapter 12 Patterns in the Coordinate Plane 11
The horizontal distance from the origin to point F is 3  units.
So, the x-coordinate is _3_.
The vertical distance from the origin to point F is __3 units.
So, the y-coordinate is _3__.
The ordered pair is _(3,3)_

Question 4.
Point J
Answer: (1,0)
Explanation:
Big Ideas Math Solutions Grade 5 Chapter 12 Patterns in the Coordinate Plane 11
The horizontal distance from the origin to point J is 1  units.
So, the x-coordinate is _1_.
The vertical distance from the origin to point J is __0_ units.
So, the y-coordinate is _0__.
The ordered pair is _(1,0)_.

Question 5.
Point G
Answer: (7,2)
Explanation:
Big Ideas Math Solutions Grade 5 Chapter 12 Patterns in the Coordinate Plane 11
The horizontal distance from the origin to point G is 7  units.
So, the x-coordinate is _7_.
The vertical distance from the origin to point G is __2_ units.
So, the y-coordinate is _2__.
The ordered pair is _(7,2)_.

Question 6.
Point K
Answer: (8,5)
Explanation:
Big Ideas Math Solutions Grade 5 Chapter 12 Patterns in the Coordinate Plane 11
The horizontal distance from the origin to point K is 8  units.
So, the x-coordinate is _8_.
The vertical distance from the origin to point K is __5_ units.
So, the y-coordinate is _5_.
The ordered pair is _(8,5)_.

Plot and label the point in the coordinate plane above.

Question 7.
Z(8, 0)
Answer: The Point can be labeled as (8,0)
Explanation:
Big-Ideas-Math-Solutions-Grade-5-Chapter-12-Patterns-in-the-Coordinate-Plane-18 E 7
Start the Origin. Move  Units right and  0 units up then plot and label the point.
The point can be labeled as Y,(8,0)

Question 8.
B(5, 5)
Answer: The Point can be labeled as (5,5)
Explanation:
Big-Ideas-Math-Solutions-Grade-5-Chapter-12-Patterns-in-the-Coordinate-Plane-18 E 8
Start the Origin. Move  Units right and  5 units up then plot and label the point.
The point can be labeled as Y,(5,5)

Question 9.
M(1, 2)
Answer: The Point can be labeled as (1,2)
Explanation:
Big-Ideas-Math-Solutions-Grade-5-Chapter-12-Patterns-in-the-Coordinate-Plane-18 E 9
Start the Origin. Move  Units right and  2 units up then plot and label the point.
The point can be labeled as Y,(1,2)

Name the point for the ordered pair.

Big Ideas Math Solutions Grade 5 Chapter 12 Patterns in the Coordinate Plane 12

Question 10.
(5, 4)
Answer: Q(5,2)
Explanation:
Big Ideas Math Solutions Grade 5 Chapter 12 Patterns in the Coordinate Plane 12
Start the Origin. Move Units right and units up then label the point.
The point can be Named as Q(5,4)

Question 11.
(0, 8)
Answer: P(0,8)
Explanation:
Big Ideas Math Solutions Grade 5 Chapter 12 Patterns in the Coordinate Plane 12
Start the Origin. Move Units right and  8units up then label the point.
The point can be Named as P(0,8)

Question 12.
(3, 1)
Answer: N(5,2)
Explanation:
Big Ideas Math Solutions Grade 5 Chapter 12 Patterns in the Coordinate Plane 12
Start the Origin. Move Units right and  1 units up then label the point.
The point can be Named as N(3,1)

Question 13.
Open-Ended
Use the coordinate plane above. Point T is 3 units from point M. Name two possible ordered pairs for point T.
Answer: T(7,8), (7,0)&(0,8)
Explanation:
Big-Ideas-Math-Solutions-Grade-5-Chapter-12-Patterns-in-the-Coordinate-Plane-18 Q13
The horizontal distance from the origin to point M is 4  units.
So, the x-coordinate is _4_.
The vertical distance from the origin to point M is _5_ units.
So, the y-coordinate is _5_.
The ordered pair is _(4,5)_.
Point T is 3 units from point M
The horizontal distance from  point M is 4  units to add 
So, the x-coordinate is _4+3=7_.
The vertical distance from the point M is 5  units to add 3 units.
So, the y-coordinate is _5+3=8_.
The ordered pair is _(7,8)_.
The two possible ordered pairs for point T is (0,7)& (8,0).

Question 14.
Writing
Explain why the order of the x- and y-coordinates is important when identifying or plotting points in a coordinate plane.
Answer: Locations on the coordinate plane are described as ordered pairs. An ordered pair tells you the location of a point by relating the point’s location along the x-axis (the first value of the ordered pair) and along the y-axis (the second value of the ordered pair).

Question 15.
To get from the school to the arcade, you walk 4 blocks east and 3 blocks north. To get from the school to the skate park, your friend walks 2 blocks east and twice as many blocks north as you. Plot and label the locations of the arcade and the skate park.
Big Ideas Math Solutions Grade 5 Chapter 12 Patterns in the Coordinate Plane 13

Question 16.
DIG DEEPER!
Which building is closer to the bus station, the library or the post office? Explain.

Review & Refresh

Multiply.

Question 17.
Big Ideas Math Solutions Grade 5 Chapter 12 Patterns in the Coordinate Plane 14
Answer: 5/3=1 2/3

Explanation: First Multiply with the whole number(1) with the denominator(4) then add numerator (4+1=5)
and denominator is same as original. i.e, 5/4
then same as next mixed fraction Multiply with the whole number(1) with the denominator(3) then add numerator (3+1=4)
and denominator is same as original. i.e, 4/3
Multiply 5/4*4/3=20/12
This fraction can be reduced by dividing both the numerator and denominator by the Greatest Common Factor of 20 and 12 using 4 i.e, 5/3=1 2/3

Question 18.
Big Ideas Math Solutions Grade 5 Chapter 12 Patterns in the Coordinate Plane 15

Answer: 7/2= 3 1/2

Explanation: First Multiply with the whole number(1) with the denominator(5) then add numerator (5+2=7)
and denominator is same as original. i.e, 7/5
then same as next mixed fraction Multiply with the whole number(2) with the denominator(2) then add numerator (4+1=5)
and denominator is same as original. i.e, 5/2
Multiply 7/5*5/2=35/10
This fraction can be reduced by dividing both the numerator and denominator by the Greatest Common Factor of 35 and 10 using 5 i.e, 7/2=3 1/2

Question 19.
Big Ideas Math Solutions Grade 5 Chapter 12 Patterns in the Coordinate Plane 16

Answer: 527/48=10 47/48

Explanation: First Multiply with the whole number(2) with the denominator(6) then add numerator (12+5=17)
and denominator is same as original. i.e, 17/6
then same as next mixed fraction Multiply with the whole number(3) with the denominator(8) then add numerator (24+7=31)
and denominator is same as original. i.e, 31/8
Multiply 17/6*31/8= 527/48.
therefore we can write it as 10 47/48.

Lesson 12.2 Relate Points in a Coordinate Plane

Plot and label the points in the coordinate plane.
Big Ideas Math Solutions Grade 5 Chapter 12 Patterns in the Coordinate Plane 17
Draw a line segment to connect each pair of points.
Big Ideas Math Solutions Grade 5 Chapter 12 Patterns in the Coordinate Plane 18
Big Ideas Math Solutions Grade 5 Chapter 12 Patterns in the Coordinate Plane 19
Plot and label more points that lie on the line segments you drew. What do you notice about the coordinates?
A and C
Explanation:
Big-Ideas-Math-Solutions-Grade-5-Chapter-12-Patterns-in-the-Coordinate-Plane-18 12-2 01
In A and C line segment we observed that the y- Coordinate of A and C is same.

B and G
Explanation:
Big-Ideas-Math-Solutions-Grade-5-Chapter-12-Patterns-in-the-Coordinate-Plane-18 12-2 0
In B and G line segment we observed that the X- Coordinate of B and G is same.
D and E
Explanation:
Big-Ideas-Math-Solutions-Grade-5-Chapter-12-Patterns-in-the-Coordinate-Plane-18 12-2 03
In D and E line segment we observed that the Y- Coordinate of D and E is same.
F and H
Explanation:
Big-Ideas-Math-Solutions-Grade-5-Chapter-12-Patterns-in-the-Coordinate-Plane-18 12-2 04
In F and H line segment we observed that the X- Coordinate of F and H is same.

Construct Arguments
How can you find the distance between each pair of points? Explain your reasoning.

Think and Grow: Relate Points in a Coordinate Plane

Key Idea
Points on a horizontal line have the same-coordinates. Points on a vertical line have the same x-coordinates.
Big Ideas Math Solutions Grade 5 Chapter 12 Patterns in the Coordinate Plane 20
You can count units or use subtraction to find the distance between two points when they lie on the same horizontal line or vertical line.

Example
Find the distance between points G and H.
Big Ideas Math Solutions Grade 5 Chapter 12 Patterns in the Coordinate Plane 21
One Way: Count units.
Step 1: Identify the locations of the points: Point G is located at (2, 3). Point H is located at (8, 3).
Step 2: Draw a line segment to connect the points.
Step 3: Count horizontal units: There are __ units between points G and H.
So, the distance between points G and H is ___.

Another Way: Use subtraction

Points G and H have the same y-coordinates. They lie on a horizontal line. Subtract the x-coordinates to find the distance.
8 – 2 = _6__
So, the distance between points G and H is 6___.

Show and Grow

Find the distance between the points in the coordinate plane above.

Question 1.
E and F
Answer: 4 Units
Explanation
Big-Ideas-Math-Solutions-Grade-5-Chapter-12-Patterns-in-the-Coordinate-Plane-21 12.2 -1A
Points E and F have the same X-coordinates. They lie on a Vertical line. Subtract the Y-coordinates to find the distance.
5 – 1 = __4_
So, the distance between points E and F is _4__ units

Question 2.
P and Q
Answer: 3 Units
Explanation:
Big-Ideas-Math-Solutions-Grade-5-Chapter-12-Patterns-in-the-Coordinate-Plane-21 12.2 -1B
Points P and Q have the same Y-coordinates. They lie on a Vertical line. Subtract the X-coordinates to find the distance.
3 – 0 = __3_
So, the distance between points P and Q is _3__ units.

Question 3.
S and T
Answer: 7 Units
Explanation:
Big-Ideas-Math-Solutions-Grade-5-Chapter-12-Patterns-in-the-Coordinate-Plane-21 12.2 -1C
Points S and T have the same y-coordinates. They lie on a horizontal line. Subtract the x-coordinates to find the distance.
7 – 0 = __7_
So, the distance between points S and T is _7_ Units.

Apply and Grow: Practice

Find the distance between the points in the coordinate plane.

Question 4.
E and F
Answer: 7 Units
Explanation:
Big-Ideas-Math-Solutions-Grade-5-Chapter-12-Patterns-in-the-Coordinate-Plane-21 12.2 -1 4
Points E and F have the same X-coordinates. They lie on a Vertical line. Subtract the Y-coordinates to find the distance.
8 – 3 = __5
So, the distance between points E and F is _5_ units

Question 5.
J and G
Answer: 7 Units
Explanation:
Big-Ideas-Math-Solutions-Grade-5-Chapter-12-Patterns-in-the-Coordinate-Plane-21 12.2 -1 4 B
Points J and G have the same y-coordinates. They lie on a horizontal line. Subtract the x-coordinates to find the distance.
8 – 0 = __8_
So, the distance between points J and G is _8_ Units.

Question 6.
F and K
Answer: 7 Units
Explanation:

Points F and K have the same y-coordinates. They lie on a horizontal line. Subtract the x-coordinates to find the distance.
8 – 5 = __3_
So, the distance between points F and K is _3_ Units.

Big Ideas Math Solutions Grade 5 Chapter 12 Patterns in the Coordinate Plane 23

Question 7.
Which is longer, \(\overline{J M}\) or \(\overline{H R}\)?

Find the distance between the points.

Question 8.
(1, 7) and (7, 7)
Answer: 6 Units
Explanation:
Big-Ideas-Math-Solutions-Grade-5-Chapter-12-Patterns-in-the-Coordinate-Plane-18 12-2 08A
Lets take point A is (1,7) and Point B is (7,7)
Points A and B have the same y-coordinates. They lie on a horizontal line. Subtract the x-coordinates to find the distance.
7 – 1 = __6_
So, the distance between points A and B is _6_ Units.

Question 9.
(0, 1) and (3, 1)
Answer: 3 Units
Explanation:

Lets take point C is (0,1) and Point D is (3,1)
Points C and D have the same y-coordinates. They lie on a horizontal line. Subtract the x-coordinates to find the distance.
3 – 0 = __3_
So, the distance between points C and D is _3_ Units.

Question 10.
(0, 0) and (6, 0)
Answer: 7 Units
Explanation:

Lets take point E is (0,0) and Point F is (6,0)
Points E and F have the same y-coordinates. They lie on a horizontal line. Subtract the x-coordinates to find the distance.
6 – 0 = __6_
So, the distance between points E and F is _6_ Units.

A line passes through the given points. Name two other points that lie on the line.

Question 11.
(0, 6) and (5, 6)
Answer: (1,6),(2,6),(3,6),(4,6)
Explanation:
Lets take point A is (0,6) and Point B is (5,6)
Points A and B have the same y-coordinates. They lie on a horizontal line.
So the other points lie on the line are (1,6),(2,6),(3,6),(4,6)

Question 12.
(4, 2) and (4, 8)
Answer: (4,3),(4,4),(4,5),(4,6)
Explanation:
Lets take point C is (4,2) and Point D is (4,8)
Points C and D have the same X-coordinates. They lie on a Vertical line.
So the other points lie on the line are (4,3),(4,4),(4,5),(4,6) etc

Question 13.
(3, 3) and (3, 6)
Answer: (3,4),(3,5)
Explanation:
Lets take point E is (3,3) and Point F is (3,6)
Points E and F have the same X-coordinates. They lie on a Vertical line.
So the other points lie on the line are (3,4),(3,5)

Question 14.
YOU BE THE TEACHER
Newton plots the points (2, 7) and (6, 7) and connects them with a line segment. Descartes says that (10, 7) also lies on the line segment. Is he correct? Explain.
Answer: Yes
Explanation:
Points on a Vertical line have the same-coordinates. They lie on a horizontal line.

Question 15.
DIG DEEPER!
Which pair of points does not lie on a line that is parallel to x-axis? Explain.
Big Ideas Math Solutions Grade 5 Chapter 12 Patterns in the Coordinate Plane 23.1
Answer:  (1,2) and (1,6)
Explanation:
Points on a Vertical line have the same-coordinates. They lie on a horizontal line. Except (1,2) and (1,6) , these two
Points on a horizontal line have the same-coordinates. They lie on a Vertical line.

Think and Grow: Modeling Real Life

Example
An archaeologist uses rope to section off a rectangular dig site. How many meters of rope does the archaeologist use?
Big Ideas Math Solutions Grade 5 Chapter 12 Patterns in the Coordinate Plane 24
To find how many meters of rope the archaeologist uses, find the perimeter of the rectangular dig site.
Big Ideas Math Solutions Grade 5 Chapter 12 Patterns in the Coordinate Plane 25
Big Ideas Math Solutions Grade 5 Chapter 12 Patterns in the Coordinate Plane 26
Use a formula to find the perimeter of the site.
Big Ideas Math Solutions Grade 5 Chapter 12 Patterns in the Coordinate Plane 27
So, the archaeologist uses __ meters of rope.

Show and Grow

Question 16.
The owner of an animal shelter uses fencing to create a rectangular dog pen. How many yards of fencing does the owner use?
Big Ideas Math Solutions Grade 5 Chapter 12 Patterns in the Coordinate Plane 28
Answer:
yards of fencing the owner use
To find how many yards of fencing the owner use, to find the perimeter of the rectangular dog pen.

Question 17.
DIG DEEPER!
You run 5 laps around the edges of the volleyball court. How far do you run in feet? in yards?
Big Ideas Math Solutions Grade 5 Chapter 12 Patterns in the Coordinate Plane 29

Answer:
10 × 6 = 60 feet
Convert from foot to yards
1 feet = 0.33 yards
60 feet = 20 yards

Relate Points in a Coordinate Plane Homework & Practice 12.2

Find the distance between the points in the coordinate plane.

Question 1.
P and M
Answer: 6 Units
Explanation:
Big-Ideas-Math-Solutions-Grade-5-Chapter-12-Patterns-in-the-Coordinate-Plane-21 12.2 -P1 A
Points P and M have the same y-coordinates. They lie on a horizontal line. Subtract the x-coordinates to find the distance.
7 – 1 = __6_
So, the distance between points P and M is _6_ Units.

Question 2.
B and Z
Answer: 3 Units
Explanation:

Points B and Z have the same X-coordinates. They lie on a Vertical line. Subtract the Y-coordinates to find the distance.
3 – 0 = _0 
So, the distance between points B and Z is _0_ units

Question 3.
K and T
Answer: 5 Units
Explanation:

Points K and T have the same y-coordinates. They lie on a horizontal line. Subtract the x-coordinates to find the distance.
6 – 1 = __5_
So, the distance between points K and T is _5_ Units.

Big Ideas Math Solutions Grade 5 Chapter 12 Patterns in the Coordinate Plane 30

Question 4.
Which is longer, \(\overline{C D}\) or \(\overline{K P}\)?

Answer: \(\overline{C D}\) longer than \(\overline{K P}\)

Find the distance between the points.

Question 5.
(1, 5) and (6, 5)
Answer: 5 Units
Explanation:

Lets take point A is (1,5) and Point B is (6,5)
Points A and B have the same y-coordinates. They lie on a horizontal line. Subtract the x-coordinates to find the distance.
6 – 1 = _5_
So, the distance between points A and B is _5_ Units.

Question 6.
(3, 4) and (3, 6)
Answer: 2 Units
Explanation:

Lets take point C is (3,4) and Point D is (3,6)
Points C and D have the same X-coordinates. They lie on a Vertical line. Subtract the Y-coordinates to find the distance.
6 – 4 = _2 
So, the distance between points C and D is _2_ Units.

Question 7.
(0, 2) and (0, 9)
Answer: 7 Units
Explanation:

Lets take point E is (0,2) and Point F is (0,9)
Points E and F have the same X-coordinates. They lie on a Vertical line. Subtract the Y-coordinates to find the distance.
9 – 2 = _7 
So, the distance between points E and F is _7_ Units.

A line passes through the given points. Name two other points that lie on the line.

Question 8.
(6, 0) and (6, 7)
Answer: (6,1),(6,2),6,3)
Explanation:
Lets take point A is (6,0) and Point B is (6,7)
Points A and B have the same X-coordinates. They lie on a Vertical line.
So the other points lie on the line are (6,1),(6,2),(6,3)

Question 9.
(5, 3) and (1, 3)
Answer: (2,3),(3,3)(4,3)
Explanation:
Lets take point C is (5,3) and Point D is (1,3)
Points C and D have the same y-coordinates. They lie on a horizontal line.
So the other points lie on the line are (2,3),(3,3)(4,3)

Question 10.
(2, 2) and (2, 9)
Answer: (2,3),(2,4),(2,5)
Explanation:
Lets take point E is (2,2) and Point F is (2,9)
Points E and F have the same X-coordinates. They lie on a Vertical line.
So the other points lie on the line are (2,3),(2,4),(2,5)

Question 11.
Structure
Name four different points that are 3 units away from (5, 4).

Answer:
A distance of 5 units from the origin represents the hypotenuse of a right triangle with sides 3- 4 – 5
If the x-coordinate is 4 then the y-coordinate is | 3 | = ± 3
the coordinates are (4, 3 ) and (4, – 3 )

Question 12.
Number Sense
Which point is farther from (3, 4)? Explain.
Big Ideas Math Solutions Grade 5 Chapter 12 Patterns in the Coordinate Plane 31

Answer: K(9, 4)

Question 13.
Modeling Real Life
A farmer builds a coop for his chickens. He uses poultry netting to enclose the coop. How many feet of netting does he use?
Big Ideas Math Solutions Grade 5 Chapter 12 Patterns in the Coordinate Plane 32

Answer: 6 feet

Question 14.
Modeling Real Life
A giant chessboard is painted on the ground in a park. How many square yards of space does the chessboard occupy?
Big Ideas Math Solutions Grade 5 Chapter 12 Patterns in the Coordinate Plane 33

Answer: 81 yards

Review & Refresh

Find the quotient. Then check your answer.

Question 15.
23.6 ÷ 4 = ___

Answer:
Divide 23.6 by 4
We get
23.6/4 = 5.9
Thus the quotient is 5.9

Question 16.
36.9 ÷ 3 = ___

Answer:
Divide the two numbers 36.9 and 3.
36.9 ÷ 3 = 12.3

Question 17.
114.87 ÷ 7 = ___

Answer:
Divide the two numbers 114.87 and 7
114.87 ÷ 7 = 16.41

Lesson 12.3 Draw polygons in a Coordinate Plane

Explore and Grow

Plot and label three points in which two of the ordered pairs have the same x-coordinates and two of the ordered pairs have the same y-coordinates.
Big Ideas Math Solutions Grade 5 Chapter 12 Patterns in the Coordinate Plane 34
The points represent the vertices of a polygon. Describe the polygon.

Structure
Explain how you can plot another point above to form a rectangle.

Think and Grow: Draw Polygons in a Coordinate Plane

Key Idea
You can use ordered pairs to represent vertices of polygons. To draw a polygon in a coordinate plane, plot and connect the vertices.
Example
The vertices of a polygon are A (2, 2), B(3, 5), C(6, 6), and D(6, 2). Draw the polygon in a coordinate plane. Then identify it.
Step 1: Plot and label the vertices.
Step 2: Draw line segments to connect the points. Be sure to connect the points in order to draw the polygon.
Big Ideas Math Solutions Grade 5 Chapter 12 Patterns in the Coordinate Plane 35
Big Ideas Math Solutions Grade 5 Chapter 12 Patterns in the Coordinate Plane 352

Show and Grow

Draw the polygon with the given vertices in a coordinate plane. Then identify it.

Question 1.
J(0, 8), K(4, 7), L(5, 0)
Big Ideas Math Solutions Grade 5 Chapter 12 Patterns in the Coordinate Plane 36
Answer: Triangle
Explanation:
Big-Ideas-Math-Solutions-Grade-5-Chapter-12-Patterns-in-the-Coordinate-Plane-6 12.3-1A
Step 1: Plot and label the vertices.
Step 2: Draw line segments to connect the points. Be sure to connect the points in order to draw the polygon.
Polygon J,K,L is a Triangle.

Question 2.
P(1, 4), Q(2, 7), R(6, 7), S(7, 4), T(4, 1)
Big Ideas Math Solutions Grade 5 Chapter 12 Patterns in the Coordinate Plane 37
Answer: Pentagon.
Explanation:

Step 1: Plot and label the vertices.
Step 2: Draw line segments to connect the points. Be sure to connect the points in order to draw the polygon.
Polygon P,Q,R,S,T is a Pentagon.

Apply and Grow: Practice

Draw the polygon with the given vertices in a coordinate plane. Then identify it.

Question 3.
C(1, 6), D(4, 6), E(4, 1), F(1, 1)
Big Ideas Math Solutions Grade 5 Chapter 12 Patterns in the Coordinate Plane 37.1
Answer: Rectangle.
Explanation:
Big-Ideas-Math-Solutions-Grade-5-Chapter-12-Patterns-in-the-Coordinate-Plane-6 12.3-1D
Step 1: Plot and label the vertices.
Step 2: Draw line segments to connect the points. Be sure to connect the points in order to draw the polygon.
Polygon C,D,E,F is a Rectangle.

Question 4.
J(2, 2), K(2, 4), L(4, 6), M(6, 4), N(6, 2), P(4, 1)
Big Ideas Math Solutions Grade 5 Chapter 12 Patterns in the Coordinate Plane 38
Answer: Hexagon.
Explanation:

Step 1: Plot and label the vertices.
Step 2: Draw line segments to connect the points. Be sure to connect the points in order to draw the polygon.
Polygon J,K,L,M,N,P is a Hexagon. 

Identify the polygon with the given vertices.

Question 5.
A(2, 6), B(6, 2), C(3, 2)
Answer: Triangle.
Explanation:

Step 1: Plot and label the vertices.
Step 2: Draw line segments to connect the points. Be sure to connect the points in order to draw the polygon.
Polygon A ,B , C is a  Tringle.

Question 6.
G(0, 3), H(6, 3), I(4, 1), J(2, 1)
Answer: Isosceles Trapezoid
Explanation:

Step 1: Plot and label the vertices.
Step 2: Draw line segments to connect the points. Be sure to connect the points in order to draw the polygon.
Polygon G,H,I,J is a Isosceles Trapezoid .

Question 7.
P(1, 1), Q(1, 6), R(6, 6), S(6, 1)
Answer: Square
Explanation:
Big-Ideas-Math-Solutions-Grade-5-Chapter-12-Patterns-in-the-Coordinate-Plane-6 12.3- 7
Step 1: Plot and label the vertices.
Step 2: Draw line segments to connect the points. Be sure to connect the points in order to draw the polygon.
Polygon P, Q ,R, S is a Rectangle.

Question 8.
X(0, 0), Y(0, 7), Z(2, 0)
Answer: Triangle.
Explanation:

Step 1: Plot and label the vertices.
Step 2: Draw line segments to connect the points. Be sure to connect the points in order to draw the polygon.
Polygon X ,Y, Z is a Triangle. 

Plot (6, 3), (6, 8), and (9, 3) in a coordinate plane. Plot another point to form the given quadrilateral. Name the point.

Question 9.
rectangle
Answer:
Explanation:
Big-Ideas-Math-Solutions-Grade-5-Chapter-12-Patterns-in-the-Coordinate-Plane-6 12.3-9

Question 10.
trapezoid
Answer:
Explanation:

Question 11.
Open-Ended
Write four ordered pairs that represent the vertices of a square

Answer:
(2, 2), (4, 2), (2, 4), (4, 4)

Question 12.
YOU BE THE TEACHER
Your friend draws the polygon shown. She names the polygon. Is your friend correct? Explain.
Big Ideas Math Solutions Grade 5 Chapter 12 Patterns in the Coordinate Plane 39

Answer:
No your friend is incorrect because the given figure has 4 sides so it is a quadrilateral not a polygon.

Think and Grow: Modeling Real Life

Example
You and a friend use computer software to create a symmetric company logo using a coordinate plane. Your friend completes one half of the logo as shown. Draw the other half. Then list the vertices of the logo.
Big Ideas Math Solutions Grade 5 Chapter 12 Patterns in the Coordinate Plane 40
Step 1: Plot the vertices for the other half of the logo on the opposite side of the line of symmetry.
Step 2: Draw line segments to connect the points.
Big Ideas Math Solutions Grade 5 Chapter 12 Patterns in the Coordinate Plane 41

Answer:

Big-Ideas-Math-Solutions-Grade-5-Chapter-12-Patterns-in-the-Coordinate-Plane-40

Show and Grow

Draw the other half of the symmetric logo. Then list its vertices.

Question 13.
Big Ideas Math Solutions Grade 5 Chapter 12 Patterns in the Coordinate Plane 42

Answer:
Big-Ideas-Math-Solutions-Grade-5-Chapter-12-Patterns-in-the-Coordinate-Plane-42

Question 14.
Big Ideas Math Solutions Grade 5 Chapter 12 Patterns in the Coordinate Plane 43

Answer:
Big-Ideas-Math-Solutions-Grade-5-Chapter-12-Patterns-in-the-Coordinate-Plane-43

Question 15.
DIG DEEPER!
One half of the design for a symmetric flower garden is shown in the coordinate plane. The line of symmetry is represented by the walkway. Draw the other half of the design for the flower garden. Then list its vertices.
Big Ideas Math Solutions Grade 5 Chapter 12 Patterns in the Coordinate Plane 44

Answer:
Big-Ideas-Math-Solutions-Grade-5-Chapter-12-Patterns-in-the-Coordinate-Plane-44

Draw polygons in a Coordinate Plane Homework & Practice 12.3

Draw the polygon with the given vertices in a coordinate plane. Then identify it.

Question 1.
A(2, 3), B(2, 6), C(5, 6), D(5, 3)
Big Ideas Math Answers Grade 5 Chapter 12 Patterns in the Coordinate Plane 45
Answer:
Big-Ideas-Math-Solutions-Grade-5-Chapter-12-Patterns-in-the-Coordinate-Plane-6 12.3-1 A

Step 1: Plot and label the vertices.
Step 2: Draw line segments to connect the points. Be sure to connect the points in order to draw the polygon.
Polygon A, B, C, D is a Trapezoid..

Question 2.
J(3, 2), K(3, 5), L(6, 5)
Big Ideas Math Answers Grade 5 Chapter 12 Patterns in the Coordinate Plane 46
Answer: Triangle
Explanation:

Step 1: Plot and label the vertices.
Step 2: Draw line segments to connect the points. Be sure to connect the points in order to draw the polygon.
Polygon J, K, L is a Triangle.

Identify the polygon with the given vertices.

Question 3.
M(2, 6), N(4, 4), P(4, 0), Q(2, 2)
Answer:
Explanation:
Big-Ideas-Math-Solutions-Grade-5-Chapter-12-Patterns-in-the-Coordinate-Plane-6 12.3-1 A3

Question 4.
A(1, 2), B(1, 6), C(4, 6), D(6, 4), E(4, 2)
Answer:
Explanation:

Question 5.
P(4, 1), Q(0, 1), R(1, 4), S(5, 5)
Answer:
Explanation:

Question 6.
E(1, 2), F(1, 3), G(6, 3), H(6, 2)

Plot (1, 2), (4, 2), and (3, 4) in a coordinate plane. Plot another point to form the given quadrilateral. Name the point.

Question 7.
trapezoid

Question 8.
parallelogram

Question 9.
Open-Ended
Write the coordinates of the vertices of a rectangle that has a perimeter of 12 units and an area of 5 square units.

Question 10.
Reasoning
Five ordered pairs represent the vertices of a polygon. Will the polygon always be a pentagon?

Question 11.
Modeling Real Life
Draw the other half of the symmetric logo. Then list its vertices.
Big Ideas Math Answers Grade 5 Chapter 12 Patterns in the Coordinate Plane 47

Question 12.
DIG DEEPER!
You complete one fourth of an image with graphic design software. The computer generates the rest of the image with the two lines of symmetry. Draw the rest of the image.
Big Ideas Math Answers Grade 5 Chapter 12 Patterns in the Coordinate Plane 48

Review & Refresh

Estimate the sum or difference.

Question 13.
Big Ideas Math Answers Grade 5 Chapter 12 Patterns in the Coordinate Plane 49
Answer: 1/4

Question 14.
Big Ideas Math Answers Grade 5 Chapter 12 Patterns in the Coordinate Plane 50

Answer:
The fractions have, unlike denominators. First, find the Least Common Denominator and rewrite the fractions with the common denominator.
77/80

Question 15.
Big Ideas Math Answers Grade 5 Chapter 12 Patterns in the Coordinate Plane 51

Answer:
The fractions have, unlike denominators. First, find the Least Common Denominator and rewrite the fractions with the common denominator.
= 3/40

Lesson 12.4 Graph Data

Explore and Grow

The table shows the amount of snow that falls each day for 7 days. Show how you can use ordered pairs in the coordinate plane to represent this information. Explain.
Big Ideas Math Answers Grade 5 Chapter 12 Patterns in the Coordinate Plane 52
What conclusions can you make from your data display?
Big Ideas Math Answers Grade 5 Chapter 12 Patterns in the Coordinate Plane 53
Answer:(1,2), (2,4), (3,6),(4,6),(,5,8),(6,10),(7,14)
Explanation:
Big-Ideas-Math-Answers-Grade-5-Chapter-12-Patterns-in-the-Coordinate-Plane-59 12.4.00 A
Lets take A(1,2), B(2,4), C(3,6), D(4,6), E(,5,8), F(6,10), G(7,14)
The horizontal distance from the origin to points A.B.C.D.E.F.G.H  are 1,2,3,4,5,6,7_ units respectively. So, the x-coordinates are 1,2,3,4,5,6,7 .
The vertical distance from the origin to points  A.B.C.D.E.F.G.H are 2,4,6,6,8,10,14 _ units. So, the y-coordinate are __2,4,6,6,8,10,14_. The ordered pair is _A(1,2), B(2,4), C(3,6), D(4,6), E(,5,8), F(6,10), G(7,14)_.
Reasoning
On Day 8, 1 inch of snow falls. How can you represent this information in the coordinate plane?
https://ccssmathanswers.com/wp-content/uploads/2020/12/Big-Ideas-Math-Answers-Grade-5-Chapter-12-Patterns-in-the-Coordinate-Plane-59-12.4.00-A1-1.png

Answer: (8,16)
Explanation:
Start the Origin. Move  Units right and  16 units up. label the point.
The point can be labeled as H,(8,16).

Think and Grow: Graph Data

Key Idea
Data are values collected from observations or measurements. You can use a coordinate plane to graph and interpret two categories of related data.
Example
The table shows how many gold bars you collect at each level of a video game. Graph the data in a coordinate plane. In how many levels do you collect more than 30 gold bars?
Big Ideas Math Answers Grade 5 Chapter 12 Patterns in the Coordinate Plane 54
Step 1: Write the ordered pairs from the table.
Big Ideas Math Answers Grade 5 Chapter 12 Patterns in the Coordinate Plane 55
Step 2: For each axis, choose appropriate numbers to represent the data in the table.
Step 3: Write a title for the graph and label each axis.
Step 4: Plot a point for each ordered pair.
Three points are above the grid line that represents 30 bars. So, you collect more than 30 gold bars in __ levels.

Show and Grow

Question 1.
The table shows the water levels of a portion of a river during a flood. Graph the data.
Big Ideas Math Answers Grade 5 Chapter 12 Patterns in the Coordinate Plane 57
What does the point (5, 7) represent?
Answer: In 5 hours time span 7 feet water level during flood.
Explanation:
https://ccssmathanswers.com/wp-content/uploads/2020/12/Big-Ideas-Math-Answers-Grade-5-Chapter-12-Patterns-in-the-Coordinate-Plane-59-12.4.1A.png

Apply and Grow: Practice

Question 2.
The table shows how many cars a salesman sells in each of 6 months. Graph the data.
Big Ideas Math Answers Grade 5 Chapter 12 Patterns in the Coordinate Plane 58
What does the point (1, 7) represent?
Big Ideas Math Answers Grade 5 Chapter 12 Patterns in the Coordinate Plane 59
What is the difference of the greatest number of cars sold and the least number of cars sold? Explain.

Use the graph.
Answer: in month of span salesman sells 7 cars.
Explanation:
Big-Ideas-Math-Answers-Grade-5-Chapter-12-Patterns-in-the-Coordinate-Plane-59 12.4.2A

Question 3.
The graph shows how many receiving yards a football player has in each of seven games. How many receiving yards does he have in Game 3?
Big Ideas Math Answers Grade 5 Chapter 12 Patterns in the Coordinate Plane 60
How many times as many receiving yards does he have in Game 4 as in Game 2?
In how many games does he have more than 40 receiving yards?
Answer:
Explanation:
In each game receiving yards are lets label it as A,B,C,D,E,F,G.
A(1,30), B(2,20), C(3,50), D(4,80), E(5,60), F(6,40), G(7,100).
receiving yards a football player has in each of game 30,20,50,80,60,40,100.
In Game 3Receiving yards are 50
So, the distance between points Game 4 and game 2 is 80-60=60
60 times as many receiving yards does he have in Game 4 as in Game 2.
There are 4 games more than 40 receiving yards.

Question 4.
DIG DEEPER!
The player has 75 receiving yards in Game 8. The player has \(\frac{1}{5}\) of this number of receiving yards in Game 9. Graph the data in the coordinate plane above.

Think and Grow: Modeling Real Life

Example
The table shows the ages of eight students and the time they spend on the Internet for 1 week. Graph the data. Of the students who spend more than 15 hours on the Internet, how many are older than 10?
Big Ideas Math Answers Grade 5 Chapter 12 Patterns in the Coordinate Plane 61
Step 1: Write the ordered pairs from the table.
Big Ideas Math Answers Grade 5 Chapter 12 Patterns in the Coordinate Plane 62
Step 2: For each axis, choose appropriate numbers to represent the data in the table.
Big Ideas Math Answers Grade 5 Chapter 12 Patterns in the Coordinate Plane 63
Step 3: Write a title for the graph and label each axis.
Step 4: Plot a point for each ordered pair.
Five points are above the grid line that represents points represents 15 hours. Of those, ___ points represents students older than 10.
So, __ students older than 10 spend more than 15 hours on the Internet.

Show and Grow

Question 5.
The table show much five students sleep the night before a quiz and their quiz scores. Graph the data. Of the students who sleep more than 7 hours, how many score higher than 8 points?
Big Ideas Math Answers 5th Grade Chapter 12 Patterns in the Coordinate Plane 64
Big Ideas Math Answers 5th Grade Chapter 12 Patterns in the Coordinate Plane 65
Answer:
Explanation:
Big-Ideas-Math-Answers-Grade-5-Chapter-12-Patterns-in-the-Coordinate-Plane-59 12.4.5A
The students who sleep more than 7 hours are 3(B,C,D)
3 members score higher than 8 points

Graph Data Homework & Practice 12.4

Question 1.
The table shows how many students are in a choir club in each of 6 years. Graph the data.
Big Ideas Math Answers 5th Grade Chapter 12 Patterns in the Coordinate Plane 66
Big Ideas Math Answers 5th Grade Chapter 12 Patterns in the Coordinate Plane 67
What does the point (2, 20) represent?

What is the difference of the greatest number of students and least number of students? Explain.
Answer: In 2 years 20 students are joined in a choir club
Explanation:
Big-Ideas-Math-Answers-5th-Grade-Chapter-12-Patterns-in-the-Coordinate-Plane-67 12.4.Paractice1 A
What is the difference of the greatest number of students and least number of students
Grates number of students joined in the year of 6th that is 35 members
least number of students joined in the year of 1st that is 15 members
The difference of the greatest number(6,65) of students and least number of students(1,15) is 35-15=20

Use the graph.

Question 2.
The graph shows how many students earn an A on each of seven tests. How many students earn an A on Test 4?
Big Ideas Math Answers 5th Grade Chapter 12 Patterns in the Coordinate Plane 68
How many times as many students earn an A on Test 6 as on Test 2?
On how many tests do fewer than 20 students earn an A? more than 20 students?

Answer: students earn an A on Test 4 is 18 students
students earn an A on Test 6 as on Test 2 is 10
5 students  are fewer than 20students earn an A
1 test only More than 20 students earn A.

Question 3.
DIG DEEPER!
Twenty-five students take Test 1. How many students do not earn an A on the test?
Answer:
25 – 12 = 13

Question 4.
Modeling Real Life
The table shows the ages of five students and how many baby teeth each of them has lost. Graph the data. Of the students who are older than 10 years, how many lost more than 18 baby teeth?
Big Ideas Math Answers 5th Grade Chapter 12 Patterns in the Coordinate Plane 69
Big Ideas Math Answers 5th Grade Chapter 12 Patterns in the Coordinate Plane 70
Answer:
Big-Ideas-Math-Answers-5th-Grade-Chapter-12-Patterns-in-the-Coordinate-Plane-67 12.4.Paractice4 AAA
Baby teeth lost from each team A,B ,C,D.E respectively 18,20,18,14,20
B,C,E are older than 10 years
2 (B&E) students are lost more than 18 baby teeth.

Review & Refresh

Find the quotient. Then check your answer.

Question 5.
5 ÷ 0.8 = ___

Answer:
Divide two numbers 5 and 0.8
5 ÷ 0.8 = 6.25

Question 6.
91.2 ÷ 15 = __

Answer:
Divide two numbers 91.2 and 15
91.2 ÷ 15 = 6.08

Question 7.
14.4 ÷ 3.2 = ___

Answer:
Divide two numbers 14.4 and 3.2
14.4 ÷ 3.2 = 4.5

Lesson 12.5 Make and Interpret Line Graphs

Explore and Grow

The table shows the heights of a bamboo plant over several days. Show how you can use a coordinate plane to represent this information. Explain.
Big Ideas Math Answers 5th Grade Chapter 12 Patterns in the Coordinate Plane 71
Big Ideas Math Answers 5th Grade Chapter 12 Patterns in the Coordinate Plane 72
How can you use your graph to estimate the height of the plant on Day 4? Explain.
Big Ideas Math Answers 5th Grade Chapter 12 Patterns in the Coordinate Plane 73

Reasoning
What could the height of the bamboo plant be on Day 10? Explain your reasoning.

Answer 42 on day 10

Think and Grow: Make and Interpret Line Graphs

Key Idea
A line graph is a graph that uses line segments to show how data values change over time.
Example
The table shows the weights of a dog over 6 months. Make a line graph of the data. Between which two months of age does the dog gain the most weight?
Big Ideas Math Answers 5th Grade Chapter 12 Patterns in the Coordinate Plane 74
Step 1: Write the ordered pairs from the table.
(1, 10), (2, 20), (3, 30), (4, 50), (5, 55), (6, 58)
Step 2: For each axis, choose appropriate numbers to represent the data in the table.
Step 3: Write a title for the graph and label each axis.
Step 4: Plot a point for each ordered pair. Then connect the points with line segments.
Big Ideas Math Answers 5th Grade Chapter 12 Patterns in the Coordinate Plane 75
The greatest difference in weights occurs between the points (___, ___ ) and (___, ___ ).
So, the dog gains the most weight between ___ and ___ months of age.
Answer: 3&4 months age
Explanation:
Big-Ideas-Math-Answers-5th-Grade-Chapter-12-Patterns-in-the-Coordinate-Plane-75 12.5 KA
The greatest difference in weights occurs between the points (_3__, __30_ ) and (__4_, __50_ ).
So, the dog gains the most weight between __3_ and _4__ months of age.

Show and Grow

Use the graph above.

Question 1.
Between which two months of age does the dog gain the least amount of weight? Explain.
Answer: 55 & 58
Explanation:
The Least difference in weights occurs between the points (5__, __55_ ) and (__6_,  _58_ ).
So, the dog gains the least amount of weight __5 and _6_ months of age.

Question 2.
How much do you think the dog weighs when it is 7 months of age? Explain your reasoning.
Answer: 60

Apply and Grow: Practice

Use the graph.

Question 3.
The table shows the height of a seedling over 7 days. Make a line graph of the data.
Big Ideas Math Answers 5th Grade Chapter 12 Patterns in the Coordinate Plane 76
Between which two days did the seedling grow the most? Explain.
Big Ideas Math Answers 5th Grade Chapter 12 Patterns in the Coordinate Plane 77
How tall do you think the seedling will be after 8 days? Explain.
Answer: 3& 4 days,
Explanation
Big-Ideas-Math-Answers-5th-Grade-Chapter-12-Patterns-in-the-Coordinate-Plane-75 12.5-3 A
The greatest difference in seedling grow occurs between the points (_3__, __8_ ) and (__4_, _18_ ).
So, the Seeding grow between __3_ and _4__ days.
After 8 days Seeding may decrease.

Question 4.
Reasoning
Interpret the point (0, 0) in the context of the situation.

Use the graph.

Question 5.
The graph shows the total numbers of likes a social media page has over 8 days. How many likes does the page have after 4 days?
Big Ideas Math Answers 5th Grade Chapter 12 Patterns in the Coordinate Plane 78
What is the difference of likes on Day 7 and Day 3?
Answer: After 4 days Social media likes are 225.
Difference between 7th day Social media likes and 3rd day Social media likes are=175.
Explanation:
Given that 8th day Social media likes are 300
4th day Social media likes are 75
Difference between 8th day Social media likes and 4th day Social media likes =300-75=225
After 4 days Social media likes are 225.
Given that 7th day Social media likes are 225
3rd day Social media likes are 50
Difference between 7th day Social media likes and 3rd day Social media likes are =225-50=175.

Question 6.
DIG DEEPER!
You track the likes between Days 7 and 8 by each hour. Does the total number of likes at every hour fall between 225 and 300? Explain.
Answer: Yes:
Explanation:
The graph shows between 7th and 8th day the social media likes are increased 225 to 300.
So, the total number of likes at every hour fall between 225 and 300.

Think and Grow: Modeling Real Life

Example
The table shows your heart rate during an exercise routine. Make a line graph of the data. Use the graph to estimate your heart rate after exercising for 15 minutes.
Big Ideas Math Answers 5th Grade Chapter 12 Patterns in the Coordinate Plane 80
Big Ideas Math Answers 5th Grade Chapter 12 Patterns in the Coordinate Plane 81
Step 1: Write the ordered pairs from the table.
(0, 80), (10, 110), (20, 140), (30, 148), (35, 135)
Step 2: For each axis, choose appropriate numbers to represent the data in the table. You can show a break in the vertical axis between 0 and 80 because there are no data values between 0 and 80.
Big Ideas Math Answers 5th Grade Chapter 12 Patterns in the Coordinate Plane 81.1
Step 3: Write a title for the graph and label each axis.
Step 4: Plot a point for each ordered pair. Then connect the points with line segments. Use the line segment that connects (10, 110) and (20, 140) to estimate your heart rate after exercising for 15 minutes.
After exercising for 15 minutes, your heart rate is about __ beats per minute.

Show and Grow

Question 7.
The table shows how many views a video has over several hours. Make a line graph of the data. Use the graph to estimate the total number of views the video has after 2 hours.
Big Ideas Math Answers 5th Grade Chapter 12 Patterns in the Coordinate Plane 82
Big Ideas Math Answers 5th Grade Chapter 12 Patterns in the Coordinate Plane 83

Answer:
Big-Ideas-Math-Answers-5th-Grade-Chapter-12-Patterns-in-the-Coordinate-Plane-83

Make and Interpret Line Graphs Homework & Practice 12.5

Example
The table shows the temperatures of a city over several hours during a snowstorm. Make a line graph of the data.
Big Ideas Math Answers 5th Grade Chapter 12 Patterns in the Coordinate Plane 84
Between which two hours does the temperature decrease the most? Explain.
Big Ideas Math Answers 5th Grade Chapter 12 Patterns in the Coordinate Plane 85
The greatest difference in temperatures occurs between the points (2, 28) and (3, 24). So, the temperature decreases the most between Hours 2 and 3.
Estimate the temperature at 4 hours and 30 minutes.
23 degrees Fahrenheit

Question 1.
The table shows the total number of pieces of beach glass you find during an hour at the beach. Make a line graph of the data.
Big Ideas Math Answers 5th Grade Chapter 12 Patterns in the Coordinate Plane 86
Between which two times did you find the most pieces of beach glass? Explain.
Big Ideas Math Answers 5th Grade Chapter 12 Patterns in the Coordinate Plane 87
Estimate how many pieces you had after 25 minutes.

Use the graph.

Question 2.
The graph shows the total amounts of money your class raises over 8 days. How much money does your class raise after 6 days?
How much money does your class raise between Days 2 and 7?
Big Ideas Math Answer Key Grade 5 Chapter 12 Patterns in the Coordinate Plane 88

Answer:
120 – 30 = 90

Question 3.
Logic
Your friend says that your class raises $115 after 9 days. Explain why your friend’s statement does not make sense.

Answer Your friend’s statement makes sense.

Use the graph.

Question 4.
Modeling Real Life
The table shows a bald eagle’s heights above the ground after several seconds. Make a line graph of the data. Use the graph to estimate the eagle’s height above the ground after 6 seconds.
Big Ideas Math Answer Key Grade 5 Chapter 12 Patterns in the Coordinate Plane 89
Big Ideas Math Answer Key Grade 5 Chapter 12 Patterns in the Coordinate Plane 90

Question 5.
DIG DEEPER!
The eagle flies past her nest, which is 120 feet above the ground. After how many seconds do you think the eagle flies past her nest? Explain.

Review & Refresh

Question 6.
Big Ideas Math Answer Key Grade 5 Chapter 12 Patterns in the Coordinate Plane 91

Answer: 8

Question 7.
Big Ideas Math Answers 5th Grade Chapter 12 Patterns in the Coordinate Plane 92

Answer: 12

Question 8.
Big Ideas Math Answers 5th Grade Chapter 12 Patterns in the Coordinate Plane 93

Answer: 50

Lesson 12.6 Numerical Paterns

Explore and Grow

Newton saves $10 each month. Descartes saves $30 each month. Complete each table. What patterns do you notice?
Newton:
Big Ideas Math Answer Key Grade 5 Chapter 12 Patterns in the Coordinate Plane 94

Answer:
Big-Ideas-Math-Answer-Key-Grade-5-Chapter-12-Patterns-in-the-Coordinate-Plane-94
Descartes:
Big Ideas Math Answer Key Grade 5 Chapter 12 Patterns in the Coordinate Plane 95

Answer:
Big-Ideas-Math-Answer-Key-Grade-5-Chapter-12-Patterns-in-the-Coordinate-Plane-95

Repeated Reasoning
How much will Newton have saved when Descartes has saved $300? Explain your reasoning.

Answer: 10 months

Think and Grow: Numerical Paterns

Example
You use 2 pounds of beef to make a batch of empanadas. Each batch makes eight servings. Complete the rule that relates the number of servings to the number of pounds of beef.
Big Ideas Math Answer Key Grade 5 Chapter 12 Patterns in the Coordinate Plane 96
Big Ideas Math Answer Key Grade 5 Chapter 12 Patterns in the Coordinate Plane 97
Step 1: Create each pattern and complete the table.
Use the rule “Add __” to find the number of pounds of beef.
0, 2, 4, ___, ___, ___
Use the rule “Add __” to find the number of servings.
0, 8, 16, ___, ___, ___
Step 2: Write ordered pairs that relate the number of servings to the number of pounds of beef.
(0, 0), (8, 2), (16, 4), ___, ___, ___
Step 3: Write a rule. As you make each batch, the number of pounds of beef is always __ as much as the number of servings.
Big Ideas Math Answer Key Grade 5 Chapter 12 Patterns in the Coordinate Plane 97.1
So, divide the number of servings by __ to find the number of pounds of beef.

Answer:
Step 1: Create each pattern and complete the table.
Use the rule “Add __” to find the number of pounds of beef.
0, 2, 4, 6, 8, 10
Use the rule “Add __” to find the number of servings.
0, 8, 16, 24, 32, 40
Step 2: Write ordered pairs that relate the number of servings to the number of pounds of beef.
(0, 0), (8, 2), (16, 4), (24, 6), (32, 8), (40, 10)
Step 3: Write a rule. As you make each batch, the number of pounds of beef is always __ as much as the number of servings.

Show and Grow

Question 1.
Use the given rules to complete the table. Then complete the rule that relates the number of hours worked to the amount earned.
Big Ideas Math Answer Key Grade 5 Chapter 12 Patterns in the Coordinate Plane 98
Multiply the number of hours worked by __ to find the amount earned.

Answer:
Big-Ideas-Math-Answer-Key-Grade-5-Chapter-12-Patterns-in-the-Coordinate-Plane-98 (1)
Multiply the number of hours worked by 8 to find the amount earned.

Apply and Grow: Practice

Use the given rules to complete the table. Then complete the rule.

Question 2.
Big Ideas Math Answer Key Grade 5 Chapter 12 Patterns in the Coordinate Plane 99
Divide the number of cups of water __ by to find the number of cups of lemon juice.

Answer:
Big-Ideas-Math-Answer-Key-Grade-5-Chapter-12-Patterns-in-the-Coordinate-Plane-99
Divide the number of cups of water 7 by to find the number of cups of lemon juice.

Question 3.
Big Ideas Math Answer Key Grade 5 Chapter 12 Patterns in the Coordinate Plane 100
Multiply the number of push-ups by __ to find the number of sit-ups.

Answer:
Big-Ideas-Math-Answer-Key-Grade-5-Chapter-12-Patterns-in-the-Coordinate-Plane-100
Multiply the number of push-ups by 2 to find the number of sit-ups.

Question 4.
Complete the rule. Then use the rule to complete the table.
Multiply the amount of money that Newton saves by __ to find the amount of money that Descartes saves.
Big Ideas Math Answer Key Grade 5 Chapter 12 Patterns in the Coordinate Plane 101

Answer:
Multiply the amount of money that Newton saves by 3 to find the amount of money that Descartes saves.

Question 5.
Structure
The ordered pairs (3, 2), (6, 4), and (9, 6) relate the number of avocados to the number of plum tomatoes in a guacamole recipe. Use the relationship to complete the table.
Big Ideas Math Answer Key Grade 5 Chapter 12 Patterns in the Coordinate Plane 102

Answer:
Big-Ideas-Math-Answer-Key-Grade-5-Chapter-12-Patterns-in-the-Coordinate-Plane-102

Think and Grow: Modeling Real Life

Example
For each $1 bill you pay, you get 4 tokens and can play 2 games. You have 60 tokens. How many games can you play?
Big Ideas Math Answer Key Grade 5 Chapter 12 Patterns in the Coordinate Plane 103
Think: What do you know? What do you need to find? How will you solve?
Use a rule to create each pattern. Use a table to organize the information.
Big Ideas Math Answer Key Grade 5 Chapter 12 Patterns in the Coordinate Plane 104
Write ordered pairs that relate the number of tokens to the number of games you can play.
(4, 2), ___, ___, ___
Write a rule. The number of games you can play is always ___ as much as the number of tokens.
So, divide the number of tokens by __ to find the number of games you can play.
60 ÷ __ = ___
So, you can play games.

Answer:
Big-Ideas-Math-Answer-Key-Grade-5-Chapter-12-Patterns-in-the-Coordinate-Plane-104
(4, 2), (8,2), (12,6), (16,8)
Write a rule. The number of games you can play is always half as much as the number of tokens.
So, divide the number of tokens by 2 to find the number of games you can play.
60 ÷ 2= 30
So, you can play 30 games.

Show and Grow

Question 6.
Each day, you read 33 pages and your friend reads 11 pages. How many pages does your friend read when you read 396 pages?

Answer: 132 pages

Explanation:
Given that,
Each day, you read 33 pages and your friend reads 11 pages.
33/11 = 3
396/3 = 132
Thus your friend read 132 pages when you read 396 pages.

Question 7.
DIG DEEPER!
Each pack of trading cards has 1 hero card, 5 combination cards, and 30 action cards. You buy packs of trading cards and get 35 combination cards. How many hero cards and action cards do you get?

Answer: 7 hero cards

Numerical Paterns Homework & Practice 12.6

Question 1.
Use the given rules to complete the table. Then complete the rule.
Big Ideas Math Answer Key Grade 5 Chapter 12 Patterns in the Coordinate Plane 105
Multiply the number of candles sold by __ to find the amount of money raised.

Answer:
Big-Ideas-Math-Answer-Key-Grade-5-Chapter-12-Patterns-in-the-Coordinate-Plane-105
Multiply the number of candles sold by 8 to find the amount of money raised.

Question 2.
Big Ideas Math Answer Key Grade 5 Chapter 12 Patterns in the Coordinate Plane 106
Multiply the number of servings by __ to find the number of pretzels.

Answer:
Big-Ideas-Math-Answer-Key-Grade-5-Chapter-12-Patterns-in-the-Coordinate-Plane-106
Multiply the number of servings by 20 to find the number of pretzels.

Question 3.
Complete the rule. Then use the rule to complete the table.
Divide the number of contestants by __ to find the number of winners.
Big Ideas Math Answer Key Grade 5 Chapter 12 Patterns in the Coordinate Plane 107

Answer:
Divide the number of contestants by 4 to find the number of winners.

Question 4.
DIG DEEPER!
Draw Figure 4. How many red squares are in Figure 8? How many yellow squares are in Figure 8? Explain your reasoning.
Big Ideas Math Answer Key Grade 5 Chapter 12 Patterns in the Coordinate Plane 108

Answer:
Big-Ideas-Math-Answer-Key-Grade-5-Chapter-12-Patterns-in-the-Coordinate-Plane-108

Question 5.
Modeling Real Life
Each person at a baseball game receives 3 raffle tickets and a $2 certificate for the team store. A group of people receives 39 raffle tickets. How much money in certificates does the group receive?

Answer:
Given,
Each person at a baseball game receives 3 raffle tickets and a $2 certificate for the team store. A group of people receives 39 raffle tickets.
39 × 2 = $78

Question 6.
DIG DEEPER!
Write a rule that relates the number of months to the cost of a gym membership. What is the cost of a 1-year membership?
Big Ideas Math Answer Key Grade 5 Chapter 12 Patterns in the Coordinate Plane 109

Answer:
Given,
1 month $15
12 months = 15 × 12
= $180

Review & Refresh

Convert the mass.

Question 7.
7 g = __ mg

Answer:

Question 8.
92 g = ___ kg

Answer:
convert from grams to kgs.
92 grams = 0.92 kg

Convert the capacity

Question 9.
800 mL = __ L

Answer:
Convert from ml to l
1 ml = 0.001
800 ml = 0.8 liters

Question 10.
3 L = __ mL

Answer:
Convert from liters to ml
1 liter = 1000 ml
3 liter = 3 × 1000 ml = 3000 ml

Lesson 12.7 Graph and Analyze Relationships

Explore and Grow

Complete each table and graph the data in the coordinate plane. What do you notice about the points?
Big Ideas Math Answer Key Grade 5 Chapter 12 Patterns in the Coordinate Plane 110
Structure
How can you use the graphs to find the number of feet in 7 yards and the number of pints in 6 gallons? Explain your reasoning.

Think and Grow: Graph and Analyze Relationships

Example
For each glass of iced tea Newton makes, he uses 2 spoonfuls of iced-tea mix and 10 fluid ounces of water. Newton uses 16 spoonfuls of iced-tea mix. How many fluid ounces of water does he use?
Step 1: Find the first several numbers in the numerical patterns for the amounts of iced-tea mix and water.
Big Ideas Math Answer Key Grade 5 Chapter 12 Patterns in the Coordinate Plane 112
Step 2: Write the ordered pairs from the table.
Big Ideas Math Answer Key Grade 5 Chapter 12 Patterns in the Coordinate Plane 113
Big Ideas Math Answer Key Grade 5 Chapter 12 Patterns in the Coordinate Plane 114
Step 3: Plot the ordered pairs. Connect the points with line segments.
Big Ideas Math Answer Key Grade 5 Chapter 12 Patterns in the Coordinate Plane 115
Because the ordered pairs follow a pattern, you can extend the line to the point where the x-coordinateis 16.
When the x-coordinate is 16, the y-coordinate is ___
So, Newton uses __ fluid ounces of water.

Answer:
Big-Ideas-Math-Answer-Key-Grade-5-Chapter-12-Patterns-in-the-Coordinate-Plane-112
Big Ideas Math Answer Key Grade 5 Chapter 12 Patterns in the Coordinate Plane 113
(2, 10), (4, 20), (6, 30), (8, 40), (10, 50), (12, 60), (14, 70), (16, 80)
Because the ordered pairs follow a pattern, you can extend the line to the point where the x-coordinate is 16.
When the x-coordinate is 16, the y-coordinate is 80
So, Newton uses 80 fluid ounces of water.

Show and Use

Question 1.
Use the graph above. Newton uses 18 spoonfuls of iced-tea mix. How many fluid ounces of water does he use? Explain your reasoning

Answer: 100
When the x-coordinate is 18, the y-coordinate is 100
So, Newton uses 100 fluid ounces of water.

Apply and Grow: Practice

Use the given information to complete the table. Describe the relationship between the two numerical patterns and plot the points.

Question 2.
A slime recipes calls for 120 milliliters of vegetable oil and 30 grams of cornstarch. You measure 600 milliliters of vegetable oil. How many grams of cornstarch do you need?
Big Ideas Math Answer Key Grade 5 Chapter 12 Patterns in the Coordinate Plane 116
Big Ideas Math Answer Key Grade 5 Chapter 12 Patterns in the Coordinate Plane 117

Answer:
Big-Ideas-Math-Answer-Key-Grade-5-Chapter-12-Patterns-in-the-Coordinate-Plane-116
Big-Ideas-Math-Answer-Key-Grade-5-Chapter-12-Patterns-in-the-Coordinate-Plane-117

Question 3.
A sponsor donates $5 for every 4 laps walked around a track. How much money does the sponsor donate for 28 laps walked?
Big Ideas Math Answer Key Grade 5 Chapter 12 Patterns in the Coordinate Plane 118
Big Ideas Math Answer Key Grade 5 Chapter 12 Patterns in the Coordinate Plane 119

Answer:
Big-Ideas-Math-Answer-Key-Grade-5-Chapter-12-Patterns-in-the-Coordinate-Plane-118
Big-Ideas-Math-Answer-Key-Grade-5-Chapter-12-Patterns-in-the-Coordinate-Plane-119

Question 4.
Writing
How can you use the graph to determine the number of cups in 4 gallons?
Big Ideas Math Answer Key Grade 5 Chapter 12 Patterns in the Coordinate Plane 120

Answer: (4, 64)

Question 5.
Number Sense
What does the ordered pair (0, 0) represent in the graph?

Answer: Origin

Question 6.
DIG DEEPER!
Use the graph to determine the number of cups in 2\(\frac{1}{2}\) gallons.

Answer: 40 cups

Think and Grow: Modeling Real Life

Example
Some friends plan to go to a trampoline park for 2 hours. They want to go to the park that costs less money. Which park should they choose? What is the cost for each person?
Big Ideas Math Answer Key Grade 5 Chapter 12 Patterns in the Coordinate Plane 121
Graph the relationship between time and cost at both parks. Park A has been done for you.
Step 1: Make a table for time and cost at Park B.
Big Ideas Math Answer Key Grade 5 Chapter 12 Patterns in the Coordinate Plane 122
Big Ideas Math Answer Key Grade 5 Chapter 12 Patterns in the Coordinate Plane 123
Step 2: Write the ordered pairs from the table.
Big Ideas Math Answer Key Grade 5 Chapter 12 Patterns in the Coordinate Plane 124
Step 3: Plot the ordered pairs. Connect the points with line segments.

Use the graph to compare the costs for 2 hours at the parks.
Big Ideas Math Answer Key Grade 5 Chapter 12 Patterns in the Coordinate Plane 125
So, the group of friends should choose Trampoline Park __
The cost for each person is $ ___.

Answer:
Big-Ideas-Math-Answer-Key-Grade-5-Chapter-12-Patterns-in-the-Coordinate-Plane-122
So, the group of friends should choose Trampoline Park A.
The cost for each person is $ 15.

Show and Grow

Question 7.
On your map, every centimeter represents 20 kilometers. On your friend’s map, every 2 centimeters represents 50 kilometers. On whose map does 6 centimeters represent a greater distance? How much greater? Explain.
Answer:
Given that,
On your map, every centimeter represents 20 kilometers.
On your friend’s map, every 2 centimeters represents 50 kilometers.
4 cm = 100 kilometers
6 cm = 150 kilometers

Graph and Analyze Relationships Homework & Practice 12.7

Question 1.
Use the graph above. You plan to park your car for 140 minutes. How much money do you put into the meter?

Answer: $35 for 140 minutes

Question 2.
A boxer exercises by jumping rope. He completes 150 repetitions every minute. He completes 750 repetitions. For how many minutes does he jump rope?
Big Ideas Math Answer Key Grade 5 Chapter 12 Patterns in the Coordinate Plane 126

Answer:
Big-Ideas-Math-Answer-Key-Grade-5-Chapter-12-Patterns-in-the-Coordinate-Plane-126

Question 3.
YOU BE THE TEACHER
Your friend says a baker makes 60 plain bagels in 5 hours. Is your friend correct? Explain.
Big Ideas Math Answer Key Grade 5 Chapter 12 Patterns in the Coordinate Plane 127
Big Ideas Math Answer Key Grade 5 Chapter 12 Patterns in the Coordinate Plane 128

Answer: Yes, your friend is correct

Question 4.
Modeling Real Life
Some friends plan to rent bicycles for 6 hours. They want to choose the option that costs less money. Which option should they choose? What is the cost for each person?
Big Ideas Math Answer Key Grade 5 Chapter 12 Patterns in the Coordinate Plane 129
Big Ideas Math Answer Key Grade 5 Chapter 12 Patterns in the Coordinate Plane 129.1

Answer: They should choose option B.
Big-Ideas-Math-Answer-Key-Grade-5-Chapter-12-Patterns-in-the-Coordinate-Plane-129.1

Review & Refresh

Convert the length.

Question 5.
Big Ideas Math Answer Key Grade 5 Chapter 12 Patterns in the Coordinate Plane 130

Answer:
Convert from feet to inches
3.3 feet = 39.6 inches

Question 6.
6 mi = __ yd

Answer:
Convert from miles to yards
1 mile = 1760 yards
6 miles = 10560 yards

Patterns in the Coordinate Plane Performance Task 12

You use a series of commands on an app to create an animation of Descartes dancing and jumping.

Question 1.
a. Complete the animation commands to moveDescartes. Plot the points to show his movement.
Big Ideas Math Answer Key Grade 5 Chapter 12 Patterns in the Coordinate Plane 131
b. Connect the points. Describe the animation in your own words.

Question 2.
You play the animation commands to make Descartes dance.
a. It takes 4 seconds for Descartes to move through the animation commands 1 time. Complete the table and graph the data in the coordinate plane.
Big Ideas Math Answer Key Grade 5 Chapter 12 Patterns in the Coordinate Plane 132
Big Ideas Math Answer Key Grade 5 Chapter 12 Patterns in the Coordinate Plane 133
b. You want Descartes to dance for an exact number of seconds. How can you find the number of times to play the animation commands? Use an example to justify your reasoning.

Answer:
Big-Ideas-Math-Answer-Key-Grade-5-Chapter-12-Patterns-in-the-Coordinate-Plane-132
Big-Ideas-Math-Answer-Key-Grade-5-Chapter-12-Patterns-in-the-Coordinate-Plane-133

Patterns in the Coordinate Plane Treasure Hunt

Directions:

  1. Each player arranges four Treasure Hunt Gold Bars on the My Treasure coordinate plane, horizontally or vertically.
  2. On your turn, name an ordered pair in the coordinate plane. If your partner says you found part of a gold bar, then plot the ordered pair in red. Otherwise, plot the ordered pair in black. Your turn is over.
  3. On your partner’s turn, if your partner finds part of a gold bar, then plot a red on the ordered X pair in the coordinate plane. Tell your partner when all parts of a gold bar have been found.
  4. The first player to find all parts of the partner’s gold bars wins!

Big Ideas Math Answer Key Grade 5 Chapter 12 Patterns in the Coordinate Plane 134

Patterns in the Coordinate Plane Chapter Practice 12

12.1 Plot Points in a Coordinate Plane

Use the coordinate plane to write the ordered pair corresponding to the point.

Question 1.
Point A

Answer: The ordered pair of Point A is (4, 6)

Question 2.
Point D

Answer: The ordered pair of Point B is (0, 0)

Question 3.
Point B

Big Ideas Math Answer Key Grade 5 Chapter 12 Patterns in the Coordinate Plane 135

Answer: The ordered pair of Point B is (2, 3)

Question 4.
Point E

Answer: The ordered pair of Point E is (2, 0)

Question 5.
Point C

Answer: The ordered pair of Point C is (8, 3)

Question 6.
Point F

Answer: The ordered pair of Point F is (8, 7)

Plot and label the point in the coordinate plane above.

Question 7.
N(3, 0)

Answer:
Big-Ideas-Math-Answer-Key-Grade-5-Chapter-12

Question 8.
P(1, 5)

Answer:
Big-Ideas-Math-Answer-Key-Grade-5-Chapter-12 img_2

Question 9.
R(2, 1)

Answer:
Big-Ideas-Math-Answer-Key-Grade-5-Chapter-12 img_3

Name the point for the ordered pair.

Question 10.
(0, 8)

Big Ideas Math Answer Key Grade 5 Chapter 12 Patterns in the Coordinate Plane 136

Answer: G is the point for the ordered pair (0, 8)

Question 11.
(5, 6)

Answer: J is the point for the ordered pair (5, 6)

Question 12.
(4, 2)

Answer: H is the point for the ordered pair (4, 2)

Question 13.
Open-Ended
Use the coordinate plane above. Point S is 2 units from point J. Name two possible ordered pairs for point S.

Answer: The two possible ordered pairs for point S are (6, 2) or (4, 4)

12.2 Relate Points in a Cooordinate Plane

Find the distance between the points in the coordinate grid.

Big Ideas Math Answer Key Grade 5 Chapter 12 Patterns in the Coordinate Plane 136.1

Question 14.
A and B

Answer:
The distance between the two points A and B is 2 units.

Question 15.
E and F

Answer: The distance between the two points E and F is 3 units.

Question 16.
C and D

Answer: The distance between the two points C and D is 7 units.

Question 17.
Which is longer, \(\overline{A C}\) or \(\overline{G E}\) ?

Answer: \(\overline{A C}\) is longer than \(\overline{G E}\).

Find the distance between the points.

Question 18.
(0, 0) and (0, 4)

Answer:
The formula for distance between the points is √(x2 – x1)² + (y2 – y1)²
= √(0 – 0)² + (4 – 0)²
= √16
= 4
Thus the distance between the points is 4.

Question 19.
(3, 2) and (3, 9)

Answer:
The formula for distance between the points is √(x2 – x1)² + (y2 – y1)²
= √(3 – 3)² + (9 – 2)²
= √49
= 7
Thus the distance between the points is 7.

Question 20.
(0, 5) and (7, 5)

Answer:
The formula for distance between the points is √(x2 – x1)² + (y2 – y1)²
= √(7 – 0)² + (5 – 5)²
= √49
= 7
Thus the distance between the points is 7.

A line passes through the given points. Name two other points that lie on the line.

Question 21.
(0, 1) and (0, 7)

Answer: (0, 5) and (0, 6) these two points lie on the same line.

Question 22.
(5, 2) and (5, 8)

Answer: (5, 3) and (5, 7) these two points lie on the same line.

Question 23.
(6, 3) and (0, 3)

Answer: (4, 3) and (3, 3) these two points lie on the same line.

12.3 Draw Polygons in a Coordinate Plane

Draw the polygon with the given vertices in a coordinate plane. Then identify it.

Question 24.
A(2, 5), B(5, 5), C(5, 0), D(2, 0)
Big Ideas Math Answer Key Grade 5 Chapter 12 Patterns in the Coordinate Plane 137

Answer:
Big-Ideas-Math-Answer-Key-Grade-5-Chapter-12-Patterns-in-the-Coordinate-Plane-137

Question 25.
D(1, 3), E(1, 5), F(3, 6), G(5, 5), H(5, 3), J(3, 2)
Big Ideas Math Answer Key Grade 5 Chapter 12 Patterns in the Coordinate Plane 138

Answer:
Big-Ideas-Math-Answer-Key-Grade-5-Chapter-12-Patterns-in-the-Coordinate-Plane-137

12.4 Graph Data

Question 26.
The table shows how many home runs your team scores in each of six kickball games. Graph the data.
Big Ideas Math Answer Key Grade 5 Chapter 12 Patterns in the Coordinate Plane 139
What does the point (6, 3) represent? What is the difference of the greatest number of home runs and the least number of home runs? Explain.

Answer:
Big-Ideas-Math-Answer-Key-Grade-5-Chapter-12-Patterns-in-the-Coordinate-Plane-139
(2, 1) is the least number of home runs.

12.5 Make an Interpret Line Graphs

Question 27.
The table shows the total numbers of coupon books you sell over 7 days. Make a line graph of the data.
Big Ideas Math Answer Key Grade 5 Chapter 12 Patterns in the Coordinate Plane 140
Big Ideas Math Answer Key Grade 5 Chapter 12 Patterns in the Coordinate Plane 141
On which day do you sell the most books? Explain.

Answer: 7th day
How many books do you think you sell after 9 days? Explain.

Answer: 29 books

12.7 Numerical Patterns

Question 28.
Use the given rules to complete the table. Then complete the rule.
Big Ideas Math Answer Key Grade 5 Chapter 12 Patterns in the Coordinate Plane 142
Multiply the number of ounces of pudding mix by __ to find the number of ounces of pumpkin.

Answer:
Big-Ideas-Math-Answer-Key-Grade-5-Chapter-12-Patterns-in-the-Coordinate-Plane-142

12.7 Graph and Analyze Relationships

Question 29.
An employee earns $80 every 8 hours. How much money does she earn after 40 hours?
Big Ideas Math Answer Key Grade 5 Chapter 12 Patterns in the Coordinate Plane 143
Big Ideas Math Answer Key Grade 5 Chapter 12 Patterns in the Coordinate Plane 144

Answer:
Big-Ideas-Math-Answer-Key-Grade-5-Chapter-12-Patterns-in-the-Coordinate-Plane-143
Big-Ideas-Math-Answer-Key-Grade-5-Chapter-12-Patterns-in-the-Coordinate-Plane-144

Question 30.
Modeling Real Life
A group of friends wants to play laser tag for 60 minutes. They want to go to the facility that costs less money. Which facility should they choose? What is the cost for each person?
Big Ideas Math Answer Key Grade 5 Chapter 12 Patterns in the Coordinate Plane 145
Big Ideas Math Answer Key Grade 5 Chapter 12 Patterns in the Coordinate Plane 146

Answer: They need to choose Facility B.

Final Words:

Learn the Big Ideas Math Book 5th Grade Solution Key Chapter 12 Patterns in the Coordinate Plane provided and improve your math as well as problem-solving skills. You can achieve greater heights and fall in love with Math with our Big Ideas Math Answers Grade 5 Chapter 12 Patterns in the Coordinate Plane. Bookmark our ccssmathanswers.com to get the solutions of Big Ideas Math Grade 5 Chapters from 1 to 13.

Go Math Grade 8 Answer Key Chapter 1 Real Numbers

go-math-grade-8-chapter-1-real-numbers-answer-key

The Solutions for Go Math Grade 8 Answer Key Chapter 1 Real Numbers are given in detail here. Get the step by step explanations for all the question in Go Math Grade 8 Chapter 1 Real Numbers Answer Key and start your practice today. You can find the best ways to learn maths by using Go Math Grade 8 Answer Key. So, Download Go Math Grade 8 Chapter 1 Real Numbers Solution Key and make use of the given resources.

Go Math Grade 8 Chapter 1 Real Numbers Answer Key

It is essential for the students to choose the best material to practice the questions. Because by practicing only you can score the highest marks in the exams. Download HMH Go Math Grade 8 Answer Key Chapter 1 Real Numbers PDF for free. Quick learning is possible with our Go Math Grade 8 Chapter 1 Real Numbers Answer Key. Find a better way to make your learning simple by clicking on the below-provided links.

Lesson 1: Rational and Irrational Numbers

Lesson 2: Sets of real Numbers

Lesson 3: Ordering Real Numbers

Model Quiz

Mixed Review

Guided Practice – Rational and Irrational Numbers – Page No. 12

Write each fraction or mixed number as a decimal.

Question 1.
\(\frac{2}{5}\) =

Answer:
0.4

Explanation:
\(\frac{2}{5}\) = \(\frac{2 × 2}{5 × 2}\) = \(\frac{4}{10}\) = 0.4

Question 2.
\(\frac{8}{9}\) =

Answer:
0.88

Explanation:
\(\frac{8}{9}\) = \(\frac{8 × 10}{9 × 10}\) = \(\frac{80}{9 × 10}\) = \(\frac{8.88}{10}\) = 0.88

Question 3.
3 \(\frac{3}{4}\) =

Answer:
3.75

Explanation:
3 \(\frac{3}{4}\) =\(\frac{15}{4}\) = 3.75

Question 4.
\(\frac{7}{10}\) =

Answer:
0.7

Explanation:
\(\frac{7}{10}\) = 0.7

Question 5.
2 \(\frac{3}{8}\) =

Answer:
2.375

Explanation:
2 \(\frac{3}{8}\) = \(\frac{19}{8}\) = 2.375

Question 6.
\(\frac{5}{6}\) =

Answer:
0.833

Explanation:
\(\frac{5}{6}\) = \(\frac{5 × 10}{6 × 10}\) = \(\frac{50}{6 × 10}\) = \(\frac{8.33}{10}\) = 0.833

Write each decimal as a fraction or mixed number in simplest form

Question 7.
0.675
\(\frac{□}{□}\)

Answer:
\(\frac{27}{40}\)

Explanation:
\(\frac{0.675 × 1000}{1 × 1000}\) = \(\frac{675}{1000}\) = \(\frac{675/25}{1000/25}\) = \(\frac{27}{40}\)

Question 8.
5.6
______ \(\frac{□}{□}\)

Answer:
5 \(\frac{3}{5}\)

Explanation:
\(\frac{5.6 × 10}{10}\) = \(\frac{56}{10}\) = 5 \(\frac{6}{10}\) = 5 \(\frac{6/2}{10/2}\) = 5 \(\frac{3}{5}\)

Question 9.
0.44
\(\frac{□}{□}\)

Answer:
\(\frac{11}{25}\)

Explanation:
\(\frac{0.44 × 100}{1 × 100}\) = \(\frac{44}{100}\) = \(\frac{44/4}{100/4}\) = \(\frac{11}{25}\)

Question 10.
0.\(\bar{4}\)
\(\frac{□}{□}\)

Answer:
\(\frac{4}{9}\)

Explanation:
Let x = 0.\(\bar{4}\)
Now, 10x = 4.\(\bar{4}\)
10x – x = 4.\(\bar{4}\) – 0.\(\bar{4}\)
9x = 4
x = \(\frac{4}{9}\)

Question 11.
0.\(\overline { 26 } \)
\(\frac{□}{□}\)

Answer:
\(\frac{26}{99}\)

Explanation:
Let x = 0.\(\overline {26}\)
Now, 100x = 26.\(\overline{26}\)
100x – x = 26.\(\overline{26}\) – 0.\(\overline {26}\)
99x = 26
x = \(\frac{26}{99}\)

Question 12.
0.\(\overline { 325 } \)
\(\frac{□}{□}\)

Answer:
\(\frac{325}{999}\)

Explanation:
Let x = 0.\(\overline {325}\)
Now, 1000x = 325.\(\overline{325}\)
1000x – x = 325.\(\overline{325}\) – 0.\(\overline {325}\)
999x = 325
x = \(\frac{325}{999}\)

Solve each equation for x

Question 13.
x2 = 144
± ______

Answer:
x=±12

Explanation:
x2 = 144
Taking square roots on both the sides
x2=±144
x = ±12

Question 14.
x2 = \(\frac{25}{289}\)
± \(\frac{□}{□}\)

Answer:
x = ±\(\frac{5}{17}\)

Explanation:
x2 = \(\frac{25}{289}\)
Taking square roots on both the sides
x2=±√\(\frac{25}{289}\)
x = ±\(\frac{5}{17}\)

Question 15.
x3 = 216
______

Answer:
x = 6

Explanation:
x3 = 216
Taking cube roots on both the sides
3x3= 3√216
x = 6

Approximate each irrational number to two decimal places without a calculator.

Question 16.
\(\sqrt { 5 } \) ≈ ______

Answer:
2.236

Explanation:
x = \(\sqrt { 5 } \)
The 5 is in between 4 and 6
Take square root of each year
√4 < √5 < √6
2 < √5 < 3
√5 = 2.2
(2.2)² = 4.84
(2.25)² = 5.06
(2.5)³ = 5.29
A good estimate for √5 is 2.25

Question 17.
\(\sqrt { 3 } \) ≈ ______

Answer:
1.75

Explanation:
\(\sqrt { 3 } \)
1 < 3 < 4
√1 < √3 < √4
1 < √3 < 2
√3 = 1.6
(1.65)² = 2.72
(1.7)² = 2.89
(1.75)² = 3.06
A good estimate for √3 is 1.75

Question 18.
\(\sqrt { 10 } \) ≈ ______

Answer:
3.15

Explanation:
\(\sqrt { 10 } \)
9 < 10 < 16
√9 < √10 < √16
3 < √10 < 4
√10 = 3.1
(3.1)² = 9.61
(3.15)² = 9.92
(3.2)² = 10.24
A good estimate for √10 is 3.15

Question 19.
What is the difference between rational and irrational numbers?
Type below:
_____________

Answer:

Rational number can be expressed as a ration of two integers such as 5/2
Irrational number cannot be expressed as a ratio of two integers such as √13

Explanation:
A rational number is a number that can be express as the ratio of two integers. A number that cannot be expressed that way is irrational.

1.1 Independent Practice – Rational and Irrational Numbers – Page No. 13

Question 20.
A \(\frac{7}{16}\)-inch-long bolt is used in a machine. What is the length of the bolt written as a decimal?
______ -inch-long

Answer:
0.4375 inch

Explanation:
The length of the bolt is \(\frac{7}{16}\)-inch
Let, x = \(\frac{7}{16}\)
Multiplying by 125 on both nominator and denominator
x = \(\frac{7×125}{16×125}\) = \(\frac{875}{2000}\) =\(\frac{437.5}{1000}\) = 0.4375

Question 21.
The weight of an object on the moon is \(\frac{1}{6}\) its weight on Earth. Write \(\frac{1}{6}\) as a decimal.
______

Answer:
0.1666

Explanation:
The weight of the object on the moon is \(\frac{1}{6}\)
Let, x = \(\frac{1}{6}\)
Multiplying by 100 on both nominator and denominator
x = \(\frac{1×100}{6×100}\) = \(\frac{16.6}{100}\) =0.166

Question 22.
The distance to the nearest gas station is 2 \(\frac{4}{5}\) kilometers. What is this distance written as a decimal?
______

Answer:
2.8

Explanation:
The distance of the nearest gas station is 2 \(\frac{4}{5}\)
Let, x = 2 \(\frac{4}{5}\)
Multiplying by 100 on both nominator and denominator
x = 2 \(\frac{4×100}{5×100}\) = \(\frac{80}{100}\) =0.8

Question 23.
A baseball pitcher has pitched 98 \(\frac{2}{3}\) innings. What is the number of innings written as a decimal?
______

Answer:
98.6

Explanation:
A baseball pitcher has pitched 98 \(\frac{2}{3}\) innings.
98 \(\frac{2}{3}\) = 98 + 2/3
= (294/3) + (2/3)
296/3
98.6

Question 24.
A heartbeat takes 0.8 second. How many seconds is this written as a fraction?
\(\frac{□}{□}\)

Answer:
\(\frac{4}{5}\)

Explanation:
A heartbeat takes 0.8 seconds.
0.8
There are 8 tenths.
8/10 = 4/5

Question 25.
There are 26.2 miles in a marathon. Write the number of miles using a fraction.
\(\frac{□}{□}\)

Answer:
26\(\frac{1}{5}\)

Explanation:
There are 26.2 miles in a marathon.
26.2 miles
262/10
131/5
26 1/5 miles

Question 26.
The average score on a biology test was 72.\(\bar{1}\). Write the average score using a fraction.
\(\frac{□}{□}\)

Answer:
80 \(\frac{1}{9}\)

Explanation:
The average score on a biology test was 72.\(\bar{1}\).
72.\(\bar{1}\)
Let x = 72.\(\bar{1}\)
10x = 10(72.\(\bar{1}\))
10x = 721.1
-x = -0.1
9x = 721
x = 721/9
x = 80 1/9

Question 27.
The metal in a penny is worth about 0.505 cent. How many cents is this written as a fraction?
\(\frac{□}{□}\)

Answer:
\(\frac{101}{200}\)

Explanation:
The metal in a penny is worth about 0.505 cent.
0.505 cent
505 thousandths
505/1000
101/200 cents

Question 28.
Multistep An artist wants to frame a square painting with an area of 400 square inches. She wants to know the length of the wood trim that is needed to go around the painting.
Go Math Grade 8 Answer Key Chapter 1 Real Numbers Lesson 1: Rational and Irrational Numbers img 1
a. If x is the length of one side of the painting, what equation can you set up to find the length of a side?
x2 = ______

Answer:
x² = 400

Explanation:
The area of a square is the square of its equal side, x
x² = 400

Question 28.
b. Solve the equation you wrote in part a. How many solutions does the equation have?
x = ± ______

Answer:
x = ± 20

Explanation:
Take the square root on both sides. Solve
x = ± 20

Question 28.
c. Do all of the solutions that you found in part b make sense in the context of the problem? Explain.
Type below:
_____________

Answer:
No. Both values of x do not make sense.

Explanation:
The length cannot be negative, hence negative value does not make sense.
No. Both values of x do not make sense.

Question 28.
d. What is the length of the wood trim needed to go around the painting?
P = ______ inches

Answer:
Length P = 20 + 2y

Rational and Irrational Numbers – Page No. 14

Question 29.
Analyze Relationships To find \(\sqrt { 15 } \), Beau found 32 = 9 and 42 = 16. He said that since 15 is between 9 and 16, \(\sqrt { 15 } \) must be between 3 and 4. He thinks a good estimate for \(\sqrt { 15 } \) is \(\frac { 3+4 }{ 2 } \) = 3.5. Is Beau’s estimate high, low, or correct? Explain.
_____________

Answer:
3.85

Explanation:
15 is closer to 16
√15 is closer to √16
Beau’s estimate is low.
(3.8)² = 14.44
(3.85)² = 14.82
(3.9)² = 15.21
√15 is 3.85

Question 30.
Justify Reasoning What is a good estimate for the solution to the equation x3 = 95? How did you come up with your estimate?
x ≈ ______

Answer:
x ≈  4.55

Explanation:
3√x = 95
x = 3√95
64 < 95 < 125
Take the cube root of each number
3√64 < 3√95  < 3√125
4 < 3√95 < 5
3√95 = 4.6
(4.5)³ = 91.125
(4.55)³ = 94.20
(4.6)³ = 97.336
3√95 = 4.55

Question 31.
The volume of a sphere is 36π ft3. What is the radius of the sphere? Use the formula V = \(\frac { 4 }{ 3 } \)πr3 to find your answer.
Go Math Grade 8 Answer Key Chapter 1 Real Numbers Lesson 1: Rational and Irrational Numbers img 2
r = ______

Answer:
r = 3

Explanation:
V = 4/3 πr³
36π = 4/3 πr³
r³ = 36π/π . 3/4
r³ = 27
r = 3√27
r = 3

FOCUS ON HIGHER ORDER THINKING

Question 32.
Draw Conclusions Can you find the cube root of a negative number? If so, is it positive or negative? Explain your reasoning.
_____________

Answer:
Yes

Explanation:
Yes. The cube root of a negative number would be negative. Because the product of three negative signs is always negative.

Question 33.
Make a Conjecture Evaluate and compare the following expressions.
\(\sqrt { \frac { 4 }{ 25 } } \) and \(\frac { \sqrt { 4 } }{ \sqrt { 25 } } \) \(\sqrt { \frac { 16 }{ 81 } } \) and \(\frac { \sqrt { 16 } }{ \sqrt { 81 } } \) \(\sqrt { \frac { 36 }{ 49 } } \) and\(\frac { \sqrt { 36 } }{ \sqrt { 49 } } \)
Use your results to make a conjecture about a division rule for square roots. Since division is multiplication by the reciprocal, make a conjecture about a multiplication rule for square roots.
Expressions are: _____________

Answer:
Evaluating and comparing
√4/25 = 2/5
√16/81 = 4/9
√36/49 = 6/7
Conjecture about a division rule for square roots
√a/√b = √(a/b)
Conjecture about a multiplication rule for square roots
√a × √b

Question 34.
Persevere in Problem Solving
The difference between the solutions to the equation x2 = a is 30. What is a? Show that your answer is correct.
_____

Answer:
30

Explanation:
x2 = a
x = ±√a
√a – (-√a) = 30
√a + √a = 30
2√a = 30
√a = 15
a = 225
x2 = 225
x = ±225
x = ±15
15 – (-15) = 15 + 15 = 30

Guided Practice – Sets of real Numbers – Page No. 18

Write all names that apply to each number.

Question 1.
\(\frac{7}{8}\)
Options:
a. Real Numbers
b. Rational Numbers
c. Integers
d. Whole Numbers
e. Irrational Numbers

Answer:
a. Real Numbers
b. Rational Numbers

Question 2.
\(\sqrt { 36 } \)
Options:
a. Real Numbers
b. Rational Numbers
c. Integers
d. Whole Numbers
e. Irrational Numbers

Answer:
a. Real Numbers
b. Rational Numbers
c. Integers
d. Whole Numbers

Explanation:
\(\sqrt { 36 } \) = 6

Question 3.
\(\sqrt { 24 } \)
Options:
a. Real Numbers
b. Rational Numbers
c. Integers
d. Whole Numbers
e. Irrational Numbers

Answer:
a. Real Numbers
e. Irrational Numbers

Question 4.
0.75
Options:
a. Real Numbers
b. Rational Numbers
c. Integers
d. Whole Numbers
e. Irrational Numbers

Answer:
a. Real Numbers
b. Rational Numbers

Question 5.
0
Options:
a. Real Numbers
b. Rational Numbers
c. Integers
d. Whole Numbers
e. Irrational Numbers

Answer:
a. Real Numbers
b. Rational Numbers
c. Integers
d. Whole Numbers

Question 6.
−\(\sqrt { 100 } \)
Options:
a. Real Numbers
b. Rational Numbers
c. Integers
d. Whole Numbers
e. Irrational Numbers

Answer:
a. Real Numbers
b. Rational Numbers
c. Integers

Explanation:
−\(\sqrt { 100 } \) = – 10

Question 7.
5.\(\overline { 45 } \)
Options:
a. Real Numbers
b. Rational Numbers
c. Integers
d. Whole Numbers
e. Irrational Numbers

Answer:
a. Real Numbers
b. Rational Numbers

Question 8.
−\(\frac{18}{6}\)
Options:
a. Real Numbers
b. Rational Numbers
c. Integers
d. Whole Numbers
e. Irrational Numbers

Answer:
a. Real Numbers
b. Rational Numbers
c. Integers

Explanation:
−\(\frac{18}{6}\) = -3

Tell whether the given statement is true or false. Explain your choice.

Question 9.
All whole numbers are rational numbers.
i. True
ii. False

Answer:
i. True

Explanation:
All whole numbers are rational numbers.
Whole numbers are a subset of the set of rational numbers and can be written as ratio of the whole number to 1.

Question 10.
No irrational numbers are whole numbers.
i. True
ii. False

Answer:
i. True

Explanation:
True. Whole numbers are ration numbers.

Identify the set of numbers that best describes each situation. Explain your choice.

Question 11.
the change in the value of an account when given to the nearest dollar
Options:
a. Real Numbers
b. Rational Numbers
c. Integer Numbers
d. Whole Numbers
e. Irrational Numbers

Answer:
c. Integer Numbers

Explanation:
The change can be a whole dollar amount and can be positive, negative or zero.

Question 12.
the markings on a standard ruler
Go Math Grade 8 Answer Key Chapter 1 Real Numbers Lesson 2: Sets of real Numbers img 3
Options:
a. Real Numbers
b. Rational Numbers
c. Integer Numbers
d. Whole Numbers
e. Irrational Numbers

Answer:
b. Rational Numbers

Explanation:
The ruler is marked every 1/16t inch.

ESSENTIAL QUESTION CHECK-IN

Question 13.
What are some ways to describe the relationships between sets of numbers?

Answer:
There are two ways that we have been using until now to describe the relationships between sets of numbers

  • Using a scheme or a diagram as the one on page 15.
  • Verbal description, for example, “All irrational numbers are real numbers.”

1.2 Independent Practice – Sets of real Numbers – Page No. 19

Write all names that apply to each number. Then place the numbers in the correct location on the Venn diagram.
Go Math Grade 8 Answer Key Chapter 1 Real Numbers Lesson 2: Sets of real Numbers img 4

Question 14.
\(\sqrt { 9 } \)
Options:
a. Real Numbers
b. Rational Numbers
c. Integer Numbers
d. Whole Numbers
e. Irrational Numbers

Answer:
a. Real Numbers
b. Rational Numbers
c. Integer Numbers
d. Whole Numbers

Explanation:
\(\sqrt { 9 } \) = 3

Question 15.
257
Options:
a. Real Numbers
b. Rational Numbers
c. Integer Numbers
d. Whole Numbers
e. Irrational Numbers

Answer:
a. Real Numbers
b. Rational Numbers
c. Integer Numbers
d. Whole Numbers

Question 16.
\(\sqrt { 50 } \)
Options:
a. Real Numbers
b. Rational Numbers
c. Integer Numbers
d. Whole Numbers
e. Irrational Numbers

Answer:
a. Real Numbers
e. Irrational Numbers

Question 17.
8 \(\frac{1}{2}\)
Options:
a. Real Numbers
b. Rational Numbers
c. Integer Numbers
d. Whole Numbers
e. Irrational Numbers

Answer:
a. Real Numbers
b. Rational Numbers

Question 18.
16.6
Options:
a. Real Numbers
b. Rational Numbers
c. Integer Numbers
d. Whole Numbers
e. Irrational Numbers

Answer:
a. Real Numbers
b. Rational Numbers

Question 19.
\(\sqrt { 16 } \)
Options:
a. Real Numbers
b. Rational Numbers
c. Integer Numbers
d. Whole Numbers
e. Irrational Numbers

Answer:
a. Real Numbers
b. Rational Numbers
c. Integer Numbers
d. Whole Numbers

Explanation:
\(\sqrt { 16 } \) = 4

Identify the set of numbers that best describes each situation. Explain your choice.

Question 20.
the height of an airplane as it descends to an airport runway
Options:
a. Real Numbers
b. Rational Numbers
c. Integer Numbers
d. Whole Numbers
e. Irrational Numbers

Answer:
d. Whole Numbers

Explanation:
Whole. The height of an airplane as it descents to an airport runway is a whole number greater than 0

Question 21.
the score with respect to par of several golfers: 2, – 3, 5, 0, – 1
Options:
a. Real Numbers
b. Rational Numbers
c. Integer Numbers
d. Whole Numbers
e. Irrational Numbers

Answer:
c. Integer Numbers

Explanation:
Integers. The scores are counting numbers, their opposites, and zero.

Question 22.
Critique Reasoning Ronald states that the number \(\frac{1}{11}\) is not rational because, when converted into a decimal, it does not terminate. Nathaniel says it is rational because it is a fraction. Which boy is correct? Explain.
i. Ronald
ii. Nathaniel

Answer:
ii. Nathaniel

Explanation:
Nathaniel is correct.
A fraction is a rational real number, even if it is not a terminating decimal.

Sets of real Numbers – Page No. 20

Question 23.
Critique Reasoning The circumference of a circular region is shown. What type of number best describes the diameter of the circle? Explain your answer.
Go Math Grade 8 Answer Key Chapter 1 Real Numbers Lesson 2: Sets of real Numbers img 5
Options:
a. Real Numbers
b. Rational Numbers
c. Irrational Numbers
d. Integers
e. Whole Numbers

Answer:
e. Whole Numbers

Explanation:
Circumference of the circle
A = 2πr
π = 2πr
Diameter is twice the radius
2r = 1
Whole

Question 24.
Critical Thinking A number is not an integer. What type of number can it be?
Options:
a. Real Numbers
b. Rational Numbers
c. Integers
d. Whole Numbers
e. Irrational Numbers

Answer:
b. Rational Numbers
e. Irrational Numbers

Question 25.
A grocery store has a shelf with half-gallon containers of milk. What type of number best represents the total number of gallons?
Options:
a. Real Numbers
b. Rational Numbers
c. Integers
d. Whole Numbers
e. Irrational Numbers

Answer:
b. Rational Numbers

FOCUS ON HIGHER ORDER THINKING

Question 26.
Explain the Error Katie said, “Negative numbers are integers.” What was her error?
Type below:
_______________

Answer:
Her error is that she stated that all negative numbers are integers. Some negative numbers are integers such as -4 but some are not such an -0.8

Question 27.
Justify Reasoning Can you ever use a calculator to determine if a number is rational or irrational? Explain.
Type below:
_______________

Answer:
Not always.

Explanation:
Not always.
If the calculator shows a terminating decimal, the number is rational but otherwise, it is not possible as you can only see a few digits.

Question 28.
Draw Conclusions The decimal 0.\(\bar{3}\) represents \(\frac{1}{3}\). What type of number best describes 0.\(\bar{9}\) , which is 3 × 0.\(\bar{3}\)? Explain.
Type below:
_______________

Answer:
1

Explanation:
let x = 0.9999999
10x = 9.99999999
10x = 9 + 0.999999999
10x = 9 + x
9x = 9
x=1.

Question 29.
Communicate Mathematical Ideas Irrational numbers can never be precisely represented in decimal form. Why is this?

Answer:
Because irrational numbers are nonrepeating, otherwise they could be represented as a fraction. Although a potential counter-example to this claim is that some irrational numbers can only be represented in decimal form, for example, 0.1234567891011121314151617…, 0.24681012141618202224…, 0.101101110111101111101111110… are all irrational numbers.

Guided Practice – Ordering Real Numbers – Page No. 24

Compare. Write <, >, or =.

Question 1.
\(\sqrt { 3 } \) + 2 ________ \(\sqrt { 3 } \) + 3

Answer:
\(\sqrt { 3 } \) + 2 < \(\sqrt { 3 } \) + 3

Explanation:
\(\sqrt { 3 } \) is between 1 and 2
\(\sqrt { 3 } \) + 2 is between 3 and 4
\(\sqrt { 3 } \) + 3 is between 4 and 5
\(\sqrt { 3 } \) + 2 < \(\sqrt { 3 } \) + 3

Question 2.
\(\sqrt { 11 } \) + 15 _______ \(\sqrt { 8 } \) + 15

Answer:
\(\sqrt { 11 } \) + 15 > \(\sqrt { 8 } \) + 15

Explanation:
\(\sqrt { 11 } \) is between 3 and 4
\(\sqrt { 8 } \) is between 2 and 3
\(\sqrt { 11 } \) + 15 is between 18 and 19
\(\sqrt { 8 } \) + 15 is between 17 and 18
\(\sqrt { 11 } \) + 15 > \(\sqrt { 8 } \) + 15

Question 3.
\(\sqrt { 6 } \) + 5 _______ 6 + \(\sqrt { 5 } \)

Answer:
\(\sqrt { 6 } \) + 5 < 6 + \(\sqrt { 5 } \)

Explanation:
\(\sqrt { 6 } \) is between 2 and 3
\(\sqrt { 5 } \) is between 2 and 3
\(\sqrt { 6 } \) is between 7 and 8
\(\sqrt { 5 } \) is between 8 and 9
\(\sqrt { 6 } \) + 5 < 6 + \(\sqrt { 5 } \)

Question 4.
\(\sqrt { 9 } \) + 3 _______ 9 + \(\sqrt { 3 } \)

Answer:
\(\sqrt { 9 } \) + 3 < 9 + \(\sqrt { 3 } \)

Explanation:
\(\sqrt { 9 } \) + 3
9 + \(\sqrt { 3 } \)
\(\sqrt { 3 } \) is between 1 and 2
\(\sqrt { 9 } \) + 3 = 3 + 3 = 6
9 + \(\sqrt { 3 } \) is between 10 and 11
\(\sqrt { 9 } \) + 3 < 9 + \(\sqrt { 3 } \)

Question 5.
\(\sqrt { 17 } \) – 3 _______ -2 + \(\sqrt { 5 } \)

Answer:
\(\sqrt { 17 } \) – 3 > -2 + \(\sqrt { 5 } \)

Explanation:
\(\sqrt { 17 } \) is between 4 and 5
\(\sqrt { 5 } \) is between 2 and 3
\(\sqrt { 17 } \) – 3 is between 1 and 2
-2 + \(\sqrt { 5 } \) is between 0 and 1
\(\sqrt { 17 } \) – 3 > -2 + \(\sqrt { 5 } \)

Question 6.
10 – \(\sqrt { 8 } \) _______ 12 – \(\sqrt { 2 } \)

Answer:
10 – \(\sqrt { 8 } \) < 12 – \(\sqrt { 2 } \)

Explanation:
\(\sqrt { 8 } \) is between 2 and 3
\(\sqrt { 2 } \) is between 1 and 2
10 – \(\sqrt { 8 } \) is between 8 and 7
12 – \(\sqrt { 2 } \) is between 11 and 10
10 – \(\sqrt { 8 } \) < 12 – \(\sqrt { 2 } \)

Question 7.
\(\sqrt { 7 } \) + 2 _______ \(\sqrt { 10 } \) – 1

Answer:
\(\sqrt { 7 } \) + 2 > \(\sqrt { 10 } \) – 1

Explanation:
\(\sqrt { 7 } \) is between 2 and 3
\(\sqrt { 10 } \) is between 3 and 4
\(\sqrt { 7 } \) + 2 is between 4 and 5
\(\sqrt { 10 } \) – 1 is between 2 and 3
\(\sqrt { 7 } \) + 2 > \(\sqrt { 10 } \) – 1

Question 8.
\(\sqrt { 17 } \) + 3 _______ 3 + \(\sqrt { 11 } \)

Answer:
\(\sqrt { 17 } \) + 3 > 3 + \(\sqrt { 11 } \)

Explanation:
\(\sqrt { 17 } \) is between 4 and 5
\(\sqrt { 11 } \) is between 3 and 4
\(\sqrt { 17 } \) + 3 is between 7 and 8
3 + \(\sqrt { 11 } \) is between 6 and 7
\(\sqrt { 17 } \) + 3 > 3 + \(\sqrt { 11 } \)

Question 9.
Order \(\sqrt { 3 } \), 2 π, and 1.5 from least to greatest. Then graph them on the number line.
\(\sqrt { 3 } \) is between _________ and _____________ , so \(\sqrt { 3 } \) ≈ ____________.
π ≈ 3.14, so 2 π ≈ _______________.
Go Math Grade 8 Answer Key Chapter 1 Real Numbers Lesson 3: Ordering Real Numbers img 6
From least to greatest, the numbers are ______________, _____________________ ,_________________.
Type below:
___________

Answer:
1.5, \(\sqrt { 3 } \), 2 π

Explanation:
\(\sqrt { 3 } \) is between 1.7 and 1.75
π = 3.14; 2 π = 6.28
grade 8 chapter 1 image 1
1.5, \(\sqrt { 3 } \), 2 π

Question 10.
Four people have found the perimeter of a forest using different methods. Their results are given in the table. Order their calculations from greatest to least.
Go Math Grade 8 Answer Key Chapter 1 Real Numbers Lesson 3: Ordering Real Numbers img 7
Type below:
___________

Answer:
\(\sqrt { 17 } \) – 2, 1+ π/2, 2.5, 12/5

Explanation:
\(\sqrt { 17 } \) – 2
\(\sqrt { 17 } \) is between 4 and 5
Since, 17 is closer to 16, the estimated value is 4.1
1+ π/2
1 + (3.14/2) = 2.57
12/5 = 2.4
2.5
\(\sqrt { 17 } \) – 2, 1+ π/2, 2.5, 12/5

ESSENTIAL QUESTION CHECK-IN

Question 11.
Explain how to order a set of real numbers.
Type below:
___________

Answer:
Evaluate the given numbers and write in decimal form. Plot on number line and arrange the numbers accordingly.

Independent Practice – Ordering Real Numbers – Page No. 25

Order the numbers from least to greatest.

Question 12.
\(\sqrt { 7 } \), 2, \(\frac { \sqrt { 8 } }{ 2 } \)
Type below:
____________

Answer:
\(\frac { \sqrt { 8 } }{ 2 } \), 2, \(\sqrt { 7 } \)

Explanation:
\(\sqrt { 7 } \), 2, \(\frac { \sqrt { 8 } }{ 2 } \)
\(\sqrt { 7 } \) is between 2 and 3
Since 7 is closer to 9, (2.65)² = 7.02, hence the estimated value is 2.65
\(\frac { \sqrt { 8 } }{ 2 } \)
\(\sqrt { 8 } \) is between 2 and 3
Since 8 is closer to 9, (2.85)² = 8.12, hence the estimated value is 2.85
2.85/2 = 1.43
grade 8 chapter 1 image 3
\(\frac { \sqrt { 8 } }{ 2 } \), 2, \(\sqrt { 7 } \)

Question 13.
\(\sqrt { 10 } \), π, 3.5
Type below:
____________

Answer:
π, \(\sqrt { 10 } \), 3.5

Explanation:
\(\sqrt { 10 } \), π, 3.5
\(\sqrt { 10 } \) is between 3 and 4
Since, 10 is closer to 9, (3.15)² = 9.92, hence the estimated value is 3.15
π = 3.14
3.5
grade 8 chapter 1 image 4
π, \(\sqrt { 10 } \), 3.5

Question 14.
\(\sqrt { 220 } \), −10, \(\sqrt { 100 } \), 11.5
Type below:
____________

Answer:
-10, √100, 11.5, √220

Explanation:
\(\sqrt { 220 } \), −10, \(\sqrt { 100 } \), 11.5
196 < 220 < 225
√196 < √220 < √225
14 < √220 < 15
√220 = 14.5
√100 = 10
grade 8 chapter 1 image 5
-10, √100, 11.5, √220

Question 15.
\(\sqrt { 8 } \), −3.75, 3, \(\frac{9}{4}\)
Type below:
____________

Answer:
−3.75, \(\frac{9}{4}\), \(\sqrt { 8 } \)

Explanation:
\(\sqrt { 8 } \), −3.75, 3, \(\frac{9}{4}\)
\(\sqrt { 8 } \) is between 2 and 3
Since, 8 is closer to 9, (2.85)² = 8.12, hence the estimated value is 2.85
-3.75 = 3
9/4 = 2.25
grade 8 chapter 1 image 6
−3.75, \(\frac{9}{4}\), \(\sqrt { 8 } \)

Question 16.
Your sister is considering two different shapes for her garden. One is a square with side lengths of 3.5 meters, and the other is a circle with a diameter of 4 meters.
a. Find the area of the square.
_______ m2

Answer:
(3.5)² = 12.25

Explanation:
Area of the square = x²
Area = (3.5)² = 12.25

Question 16.
b. Find the area of the circle.
_______ m2

Answer:
π(2)² = 12.56

Explanation:
Area of the circle = πr² where r = d/2 = 4/2 = 2
Area = π(2)² = 12.56

Question 16.
c. Compare your answers from parts a and b. Which garden would give your sister the most space to plant?
___________

Answer:
12.25 < 12.56
The circle will give more space

Question 17.
Winnie measured the length of her father’s ranch four times and got four different distances. Her measurements are shown in the table.
a. To estimate the actual length, Winnie first approximated each distance to the nearest hundredth. Then she averaged the four numbers. Using a calculator, find Winnie’s estimate.
Go Math Grade 8 Answer Key Chapter 1 Real Numbers Lesson 3: Ordering Real Numbers img 8
______

Answer:
7.4815

Explanation:
\(\sqrt { 60 } \) = 7.75
58/8 = 7.25
7.3333
7 3/5 = 7.60
Average = (7.75 + 7.25 + 7.33 + 7.60)/4 = 7.4815

Question 17.
b. Winnie’s father estimated the distance across his ranch to be \(\sqrt { 56 } \) km. How does this distance compare to Winnie’s estimate?
____________

Answer:
They are nearly identical

Explanation:
\(\sqrt { 56 } \) = 7.4833
They are nearly identical

Give an example of each type of number.

Question 18.
a real number between \(\sqrt { 13 } \) and \(\sqrt { 14 } \)
Type below:
____________

Answer:
A real number between \(\sqrt { 13 } \) and \(\sqrt { 14 } \)
Example: 3.7

Explanation:
\(\sqrt { 13 } \) = 3.61
\(\sqrt { 13 } \) = 3.74
A real number between \(\sqrt { 13 } \) and \(\sqrt { 14 } \)
Example: 3.7

Question 19.
an irrational number between 5 and 7
Type below:
____________

Answer:
An irrational number between 5 and 7
Example: \(\sqrt { 29 } \)

Explanation:
5² = 25 and 7² = 49
An irrational number between 5 and 7
Example: \(\sqrt { 29 } \)

Ordering Real Numbers – Page No. 26

Question 20.
A teacher asks his students to write the numbers shown in order from least to greatest. Paul thinks the numbers are already in order. Sandra thinks the order should be reversed. Who is right?
Go Math Grade 8 Answer Key Chapter 1 Real Numbers Lesson 3: Ordering Real Numbers img 9
_____________

Answer:
Neither are correct

Explanation:
\(\sqrt { 115 } \), 115/11, 10.5624
\(\sqrt { 115 } \) is between 10 and 11
Since, 115 is closer to 121, (10.7)² = 114.5, hence the estimated value is 10.7
115/11 = 10.4545
10.5624
Neither are correct

Question 21.
Math History
There is a famous irrational number called Euler’s number, symbolized with an e. Like π, its decimal form never ends or repeats. The first few digits of e are 2.7182818284.
a. Between which two square roots of integers could you find this number?
Type below:
_____________

Answer:
The square of e lies between 7 and 8
2.718281828
(2.72)² = 7.3984
Hence, it lies between \(\sqrt { 7 } \) = 2.65 and \(\sqrt { 8 } \) = 2.82

Question 21.
b. Between which two square roots of integers can you find π?
Type below:
_____________

Answer:
3.142
(3.14)² = 9.8596
Hence. it lies between \(\sqrt { 9 } \) = 3 and \(\sqrt { 10 } \) = 3.16

H.O.T.

FOCUS ON HIGHER ORDER THINKING

Question 22.
Analyze Relationships
There are several approximations used for π, including 3.14 and \(\frac{22}{7}\). π is approximately 3.14159265358979 . . .
a. Label π and the two approximations on the number line.
Go Math Grade 8 Answer Key Chapter 1 Real Numbers Lesson 3: Ordering Real Numbers img 10
Type below:
_____________

Answer:
grade 8 chapter 1 image 7

Question 22.
b. Which of the two approximations is a better estimate for π? Explain.
Type below:
_____________

Answer:
As we can see from the number line, 22/7 is closer to π, so we can conclude that 22/7 is a better estimation for π.

Question 22.
c. Find a whole number x so that the ratio \(\frac{x}{113}\) is a better estimate for π than the two given approximations.
Type below:
_____________

Answer:
355/113 is a better estimation for π, because 355/113 = 3.14159292035 = 3.14159265358979 = π

Question 23.
Communicate Mathematical Ideas
What is the fewest number of distinct points that must be graphed on a number line, in order to represent natural numbers, whole numbers, integers, rational numbers, irrational numbers, and real numbers? Explain.
_______ points

Answer:
2 points

Explanation:
There need to be plotting of at least 2 points because a rational number can never be equal to an irrational number. So let’s say 5 points are the same among six but the 6th will be different as there both rational numbers and irrational numbers included.

Question 24.
Critique Reasoning
Jill says that 12.\(\bar{6}\) is less than 12.63. Explain her error.
Type below:
_____________

Answer:
12.\(\bar{6}\) = 12.666
12.\(\bar{6}\) > 12.63

1.1 Rational and Irrational Numbers – Model Quiz – Page No. 27

Write each fraction as a decimal or each decimal as a fraction.

Question 1.
\(\frac{7}{20}\)
_______

Answer:
0.35

Explanation:
\(\frac{7}{20}\) = 0.35

Question 2.
1.\(\overline { 27} \)
______ \(\frac{□}{□}\)

Answer:
1\(\frac{28}{99}\)

Explanation:
1.\(\overline { 27} \)
x = 1.\(\overline { 27} \)
100x = 100(1.\(\overline { 27} \))
100x = 127(\(\overline { 27} \))
x = .\(\overline { 27} \)
99x = 127
x = 127/99
x = 1 28/99

Question 3.
1 \(\frac{7}{8}\)
______

Answer:
1.875

Explanation:
1 \(\frac{7}{8}\)
1 + 7/8
8/8 + 7/8
15/8 = 1.875

Solve each equation for x.

Question 4.
x2 = 81
± ______

Answer:
± 9

Explanation:
x2 = 81
x = ± 81
x = ± 9

Question 5.
x3 = 343
______

Answer:
x = 7

Explanation:
x3 = 343
x = 7

Question 6.
x2 = \(\frac{1}{100}\)
± \(\frac{□}{□}\)

Answer:
± \(\frac{1}{10}\)

Explanation:
x2 = \(\frac{1}{100}\)
x = ± \(\frac{1}{10}\)

Question 7.
A square patio has an area of 200 square feet. How long is each side of the patio to the nearest 0.05?
______ feet

Answer:
14.15 feet

Explanation:
The area of a square is found by multiplying the side of the square by itself. Therefore, to find the side of the square, we have to take the square root of the area.
Let’s denote with A the area of the patio and with s each side of the square.
We have:
A = 200
A = s.s
s = \(\sqrt { A } \) = \(\sqrt { 200 } \)
Following the steps as in “Explore Activity” on page 9, we can make an estimation for the irrational number:
196 < 200 < 225
\(\sqrt { 196 } \) < \(\sqrt { 200 } \) < \(\sqrt { 225 } \)
14 < \(\sqrt { 200 } \) < 15
We see that 200 is much closer to 196 than to 225, therefore the square root of it should be between 14 and 14.5. To make a better estimation, we pick some numbers between 14 and 14.5 and calculate their squares:
(14.1)² = 198.81
(14.2)² = 201.64
14.1 < \(\sqrt { 200 } \) < 14.2
\(\sqrt { 200 } \) = 14.15
We see that 200 is much closer to 14.1 than to 14.2, therefore the square root of it should be between 14.1 and 14.15. If we round to the nearest 0.05, we have:
s = 14.15

1.2 Sets of Real Numbers

Write all names that apply to each number.

Question 8.
\(\frac { 121 }{ \sqrt { 121 } }\)
Type below:
___________

Answer:
Rational, whole, integer, real numbers

Explanation:
\(\frac { 121 }{ \sqrt { 121 } }\)
121/11 = 11

Question 9.
\(\frac{π}{2}\)
Type below:
___________

Answer:
Irrational, real numbers

Question 10.
Tell whether the statement “All integers are rational numbers” is true or false. Explain your choice.
___________

Answer:
True

Explanation:
“All integers are rational numbers” is true, because every integer can be expressed as a fraction with a denominator equal to 1. The set of integer A a subset of rational numbers.

1.3 Ordering Real Numbers

Compare. Write <, >, or =.

Question 11.
\(\sqrt { 8 }\) + 3 _______ 8 + \(\sqrt { 3 }\)

Answer:
\(\sqrt { 8 }\) + 3 < 8 + \(\sqrt { 3 }\)

Explanation:
4 < 8 < 9
\(\sqrt { 4 }\) < \(\sqrt { 8 }\) < \(\sqrt { 9 }\)
2 < \(\sqrt { 8 }\) < 3
1 < 3 < 4
\(\sqrt { 1 }\) < \(\sqrt { 3 }\) < \(\sqrt { 4 }\)
1 < \(\sqrt { 3 }\) < 2
\(\sqrt { 8 }\) + 3 is between 5 and 6
8 + \(\sqrt { 3 }\) is between 9 and 10
\(\sqrt { 8 }\) + 3 < 8 + \(\sqrt { 3 }\)

Question 12.
\(\sqrt { 5 }\) + 11 _______ 5 + \(\sqrt { 11 }\)

Answer:
\(\sqrt { 5 }\) + 11 > 5 + \(\sqrt { 11 }\)

Explanation:
\(\sqrt { 5 }\) lies in between 2 and 3
\(\sqrt { 11 }\) lies in between 3 and 4
\(\sqrt { 5 }\) + 11 lies in between 13 and 14
5 + \(\sqrt { 11 }\) lies in between 8 and 9
\(\sqrt { 5 }\) + 11 > 5 + \(\sqrt { 11 }\)

Order the numbers from least to greatest.

Question 13.
\(\sqrt { 99 }\), π2, 9.\(\bar { 8 }\)
Type below:
_______________

Answer:
π2, 9.\(\bar { 8 }\), \(\sqrt { 99 }\)

Explanation:
\(\sqrt { 99 }\), π2, 9.\(\bar { 8 }\)
99 lies between 9² and 10²
99 is closer to 100, hence \(\sqrt { 99 }\) is closer to 10
(9.9)² = 98.01
(9.95)² = 99.0025
(10)² = 100
\(\sqrt { 99 }\) = 9.95
π² = 9.86
9.88888 = 9.89
grade 8 chapter 1 image 8
π2, 9.\(\bar { 8 }\), \(\sqrt { 99 }\)

Question 14.
\(\sqrt { \frac { 1 }{ 25 } } \), \(\frac{1}{4}\), 0.\(\bar { 2 }\)
Type below:
____________

Answer:
\(\sqrt { \frac { 1 }{ 25 } } \), 0.\(\bar { 2 }\), \(\frac{1}{4}\)

Explanation:
\(\sqrt { \frac { 1 }{ 25 } } \), \(\frac{1}{4}\), 0.\(\bar { 2 }\)
\(\sqrt { \frac { 1 }{ 25 } } \) = 1/5 = 0.2
1/4 = 0.25
0.\(\bar { 2 }\) = 0.222 = 0.22
grade 8 chapter 1 image 9
\(\sqrt { \frac { 1 }{ 25 } } \), 0.\(\bar { 2 }\), \(\frac{1}{4}\)

Essential Question

Question 15.
How are real numbers used to describe real-world situations?
Type below:
_______________

Answer:
In real-world situations, we use real numbers to count or make measurements. They can be seen as a convention for us to quantify things around, for example, the distance, the temperature, the height, etc.

Selected Response – Mixed Review – Page No. 28

Question 1.
The square root of a number is 9. What is the other square root?
Options:
a. -9
b. -3
c. 3
d. 81

Answer:
a. -9

Explanation:
We know that every positive number has two square roots, one positive and one negative. We are given the principal square root (9), so the other square root would be its negative (-9). To prove that, we square both numbers and we compare the results:
9 • 9 = 81
(-9). (-9)= 81

Question 2.
A square acre of land is 4,840 square yards. Between which two integers is the length of one side?
Options:
a. between 24 and 25 yards
b. between 69 and 70 yards
c. between 242 and 243 yards
d. between 695 and 696 yards

Answer:
b. between 69 and 70 yards

Explanation:
The area of a square is found by multiplying the side of the square by itself. Therefore, to Bud the side of the square, we have to take the square root of the area.
Let’s denote with A the area of the land and with each side of the square. We have:
A = 4840
A = s . s
A = s²
s = √A = √4840
Following the steps as in °Explore Activity on page 9, we can make an estimation for the irrational number:
4761 < 4840 < 4900
\(\sqrt { 4761 }\) < \(\sqrt { 4840 }\) < \(\sqrt { 4900 }\)
69 < \(\sqrt { 4840 }\) < 70
Each side of the land is between 69 and 70 yards.

Question 3.
Which of the following is an integer but not a whole number?
Options:
a. -9.6
b. -4
c. 0
d. 3.7

Answer:
b. -4

Explanation:
Whole numbers are not negative
-4 is an integer but not a whole number

Question 4.
Which statement is false?
Options:
a. No integers are irrational numbers.
b. All whole numbers are integers.
c. No real numbers are irrational numbers.
d. All integers greater than 0 are whole numbers.

Answer:
c. No real numbers are irrational numbers.

Explanation:
Rational and irrational numbers are real numbers.

Question 5.
Which set of numbers best describes the displayed weights on a digital scale that shows each weight to the nearest half pound?
Options:
a. whole numbers
b. rational numbers
c. real numbers
d. integers

Answer:
b. rational numbers

Explanation:
The scale weighs nearest to 1/2 pound.

Question 6.
Which of the following is not true?
Options:
a. π2 < 2π + 4
b. 3π > 9
c. \(\sqrt { 27 }\) + 3 > 172
d. 5 – \(\sqrt { 24 }\) < 1

Answer:
c. \(\sqrt { 27 }\) + 3 > 172

Explanation:
a. π2 < 2π + 4
(3.14)² < 2(3.14) + 4
9.86 < 10.28
True
b. 3π > 9
9.42 > 9
True
c. \(\sqrt { 27 }\) + 3 > 172
5.2 + 3 > 8.5
8.2 > 8.5
False
d. 5 – \(\sqrt { 24 }\) < 1
5 – 4.90 < 1
0.1 < 1
True

Question 7.
Which number is between \(\sqrt { 21 }\) and \(\frac{3π}{2}\) ?
Options:
a. \(\frac{14}{3}\)
b. 2 \(\sqrt { 6 }\)
c. 5
d. π + 1

Answer:

Explanation:
a. \(\sqrt { 21 }\) and \(\frac{3π}{2}\)
\(\sqrt { 21 }\) = 4.58
\(\frac{3π}{2}\) = 4.71
14/3 = 4.67
b. 2\(\sqrt { 6 }\) = 4.90
c. 5
d. π + 1 = 3.14 + 1 = 4.14

Question 8.
What number is shown on the graph?
Go Math Grade 8 Answer Key Chapter 1 Real Numbers Mixed Review img 11
Options:
a. π+3
b. \(\sqrt { 4 }\) + 2.5
c. \(\sqrt { 20 }\) + 2
d. 6.\(\overline { 14 } \)

Answer:
c. \(\sqrt { 20 }\) + 2

Explanation:
6.48
a. π+3 = 3.14 + 3 = 6.14
b. \(\sqrt { 4 }\) + 2.5 = 2 + 2.5 = 4.5
c. \(\sqrt { 20 }\) + 2 = 4.47 + 2 = 6.47
d. 6.\(\overline { 14 } \) = 6.1414

Question 9.
Which is in order from least to greatest?
Options:
a. 3.3, \(\frac{10}{3}\), π, \(\frac{11}{4}\)
b. \(\frac{10}{3}\), 3.3, \(\frac{11}{4}\), π
c. π, \(\frac{10}{3}\), \(\frac{11}{4}\), 3.3
d. \(\frac{11}{4}\), π, 3.3, \(\frac{10}{3}\)

Answer:
d. \(\frac{11}{4}\), π, 3.3, \(\frac{10}{3}\)

Explanation:
10/3 = 3.3333333
11/4 = 2.75
grade 8 chapter 1 image 10

Mini-Task

Question 10.
The volume of a cube is given by V = x3, where x is the length of an edge of the cube. The area of a square is given by A = x2, where x is the length of a side of the square. A given cube has a volume of 1728 cubic inches.
a. Find the length of an edge.
______ inches

Answer:
12 inches

Explanation:
V = x3
A = x2
1728 = x3
x = 12
The length of an edge = 12 in

Question 10.
b. Find the area of one side of the cube.
______ in2

Answer:
144 in2

Explanation:
A = (12)² = 144
Area of the side of the cube = 144 in2

Question 10.
c. Find the surface area of the cube.
______ in2

Answer:
864 in2

Explanation:
SA = 6 (12)² = 864
The surface area of the cube = 864 in2

Question 10.
d. What is the surface area in square feet?
______ ft2

Answer:
6 ft2

Explanation:
SA = 864/144 = 6
The surface area of the cube = 6 ft2

Conclusion:

We hope the details prevailed in this Grade 8 Go Math Answer Key Chapter 1 Real Numbers is helpful for you guys. Make use of the above links and try to solve all the problems. This HMH Go Math Grade 8 Answer Key also helps to complete the homework within the time without any mistakes. Stick to our ccssmathanswers.com site to get the pdf links of all the chapters. Feel free to post your comments in the below box. We will try to clarify your doubts as early as possible. All the Best Guys!!!

Go Math Grade 6 Answer Key Chapter 5 Model Percents

go-math-grade-6-chapter-5-model-percents-answer-key

Go Math Grade 6 Answer Key Chapter 5 Model Percents Pdf is available here. So, the pupils who are in search of the solutions of Chapter 5 Model Percents can get them on this page along with images. Relate the questions in real-time and make your practice best. Students who are preparing for exams must have the best material. Our team will provide step by step explanations for all the questions on Go Math Grade 6 Answer Key.

Go Math Grade 6 Chapter 5 Model Percents Answer Key

Make yourself comfortable by using HMH Go math Grade 6 Answer Key Chapter 5 Model Percents. So, make use of the resources of Go Math Answer Key to score good marks in the exams. Test your skills by solving the problems given at the end of the chapter. Just click on the links and start solving the problems.

Lesson 1: Investigate • Model Percents

Lesson 2: Write Percents as Fractions and Decimals

Lesson 3: Write Fractions and Decimals as Percents

Mid-Chapter Checkpoint

Lesson 4: Percent of a Quantity

Lesson 5: Problem Solving • Percents

Lesson 6: Find the Whole from a Percent

Chapter 5 Review/Test

Share and Show – Page No. 271

Write a ratio and a percent to represent the shaded part.

Question 1.
Go Math Grade 6 Answer Key Chapter 5 Model Percents img 1
Type below:
_____________

Answer:
53% and \(\frac{53}{100}\)

Explanation:
53 squares are shaded out of 100.
So, 53% and 35/100 are the answers.

Question 2.
Go Math Grade 6 Answer Key Chapter 5 Model Percents img 2
Type below:
_____________

Answer:
1% and \(\frac{100}{100}\)

Explanation:
100 out of 100 squares are shaded
So, So, 1% and 100/100 are the answers.

Question 3.
Go Math Grade 6 Answer Key Chapter 5 Model Percents img 3
Type below:
_____________

Answer:
40% and \(\frac{40}{100}\)

Explanation:
40 squares are shaded out of 100.
So, 40% and 40/100 are the answers.

Model the percent and write it as a ratio.

Question 4.
30%
\(\frac{□}{□}\)

Answer:
grade 6 chapter 5 image 1

Explanation:
30% is 30 out of 100
30 out of 100 squares is 30/100
30% = \(\frac{30}{100}\)

Question 5.
5%
\(\frac{□}{□}\)

Answer:
grade 6 chapter 5 image 2

Explanation:
5% is 5 out of 100
5 out of 100 squares is 5/100
5% = \(\frac{5}{100}\)

Question 6.
75%
\(\frac{□}{□}\)

Answer:
grade 6 chapter 5 image 3

Explanation:
75% is 75 out of 100
75 out of 100 squares is 75/100
75% = \(\frac{75}{100}\)

Problem Solving + Applications

Question 7.
Use a Concrete Model Explain how to model 32% on a 10-by-10 grid. How does the model represent the ratio of 32 to 100?
Type below:
_____________

Answer:
grade 6 chapter 5 image 4

Question 8.
A floor has 100 tiles. There are 24 black tiles and 35 brown tiles. The rest of the tiles are white. What percent of the tiles are white?
_______ %

Answer:
41%

Explanation:
A floor has 100 tiles. There are 24 black tiles and 35 brown tiles.
24 + 35 = 59
100 – 59 = 41 tiles are white
41 tiles out of 100 are white tiles

Pose a Problem – Page No. 272

Question 9.
Javier designed a mosaic wall mural using 100 tiles in 3 different colors: yellow, blue, and red. If 64 of the tiles are yellow, what percent of the tiles are either red or blue?
Go Math Grade 6 Answer Key Chapter 5 Model Percents img 4
To find the number of tiles that are either red or blue, count the red and blue squares. Or subtract the number of yellow squares, 64, from the total number of squares, 100.
36 out of 100 tiles are red or blue.
The ratio of red or blue tiles to all tiles is \(\frac{36}{100}\).
So, the percent of the tiles that are either red or blue is 36%.
Write another problem involving a percent that can be solved by using the mosaic wall mural.
Type below:
_____________

Answer:
Sam designed a mosaic wall mural using 100 squares using two colors. She represented the squares with red and blue colors. She has 54 red tiles. What percent of other tiles she can use with blue color?
100 – 54 = 46 blue tiles.

Question 10.
Select the 10-by-10 grids that model 45%. Mark all that apply.
Options:
a. Go Math Grade 6 Answer Key Chapter 5 Model Percents img 5
b. Go Math Grade 6 Answer Key Chapter 5 Model Percents img 6
c. Go Math Grade 6 Answer Key Chapter 5 Model Percents img 7
d. Go Math Grade 6 Answer Key Chapter 5 Model Percents img 8
e. Go Math Grade 6 Answer Key Chapter 5 Model Percents img 9

Answer:
a. Go Math Grade 6 Answer Key Chapter 5 Model Percents img 5
c. Go Math Grade 6 Answer Key Chapter 5 Model Percents img 7
e. Go Math Grade 6 Answer Key Chapter 5 Model Percents img 9

Model Percents – Page No. 273

Write a ratio and a percent to represent the shaded part.

Question 1.
Go Math Grade 6 Answer Key Chapter 5 Model Percents img 10
Type below:
_____________

Answer:
31% and \(\frac{31}{100}\)

Explanation:
31 squares are shaded out of 100.
So, 31% and 31/100 are the answers.

Question 2.
Go Math Grade 6 Answer Key Chapter 5 Model Percents img 11
Type below:
_____________

Answer:
70% and \(\frac{70}{100}\)

Explanation:
70 squares are shaded out of 100.
So, 70% and 70/100 are the answers.

Question 3.
Go Math Grade 6 Answer Key Chapter 5 Model Percents img 12
Type below:
_____________

Answer:
48% and \(\frac{48}{100}\)

Explanation:
48 squares are shaded out of 100.
So, 48% and 48/100 are the answers.

Model the percent and write it as a ratio.

Question 4.
97%
\(\frac{□}{□}\)

Answer:
grade 6 chapter 5 image 5

Explanation:
97% is 97 out of 100
97 out of 100 squares is 97/100
97% = \(\frac{97}{100}\)

Question 5.
24%
\(\frac{□}{□}\)

Answer:
grade 6 chapter 5 image 6

Explanation:
24% is 24 out of 100
24 out of 100 squares is 24/100
24% = \(\frac{24}{100}\)

Question 6.
50%
\(\frac{□}{□}\)

Answer:
grade 6 chapter 5 image 7

Explanation:
50% is 50 out of 100
50 out of 100 squares is 50/100
50% = \(\frac{50}{100}\)

Problem Solving

The table shows the pen colors sold at the school supply store one week. Write the ratio comparing the number of the given color sold to the total number of pens sold. Then shade the grid.
Go Math Grade 6 Answer Key Chapter 5 Model Percents img 13

Question 7.
Black
\(\frac{□}{□}\)

Answer:
\(\frac{49}{100}\)
grade 6 chapter 5 image 8

Explanation:
The total number of pens sold = 36 + 49 + 15 = 100
Black : total number of pens sold = 49:100
49 out of 100 squares need to shade the grid

Question 8.
Not Blue
\(\frac{□}{□}\)

Answer:
\(\frac{64}{100}\)
grade 6 chapter 5 image 9

Explanation:
Not Blue = Black + Red = 49 + 15 = 64

Question 9.
Is every percent a ratio? Is every ratio a percent? Explain.
Type below:
_____________

Answer:
Every percent is a ratio but not all ratios are percent. All ratios can be expressed as percents, decimals, or fractions or in ratio form.

Lesson Check – Page No. 274

Question 1.
What percent of the large square is shaded?
Go Math Grade 6 Answer Key Chapter 5 Model Percents img 14
_______ %

Answer:
63%

Explanation:
63 squares are shaded out of 100.
So, 63% and 63/100 are the answers.

Question 2.
Write a ratio to represent the shaded part.
Go Math Grade 6 Answer Key Chapter 5 Model Percents img 15
\(\frac{□}{□}\)

Answer:
\(\frac{10}{100}\)

Explanation:
63 squares are shaded out of 100.
63/100 is the answer.

Spiral Review

Question 3.
Write a number that is less than −2 \(\frac{4}{5}\) and greater than −3 \(\frac{1}{5}\).
Type below:
_____________

Answer:
-2.9, -3.0, -3.1

Explanation:
−2 \(\frac{4}{5}\) = -14/5 = -2.8
−3 \(\frac{1}{5}\) = -16/5 = -3.2
-2.9, -3.0, -3.1 are the numbers less than −2 \(\frac{4}{5}\) and greater than −3 \(\frac{1}{5}\)

Question 4.
On a coordinate grid, what is the distance between (2, 4) and (2, –3)?
_______ units

Answer:
7 units

Explanation:
|-3| = 3
4+ 0 = 4; 0 + 3 = 3
4 + 3 = 7

Question 5.
Each week, Diana spends 4 hours playing soccer and 6 hours babysitting. Write a ratio to compare the time Diana spends playing soccer to the time she spends babysitting.
\(\frac{□}{□}\)

Answer:
\(\frac{2}{3}\)

Explanation:
Each week, Diana spends 4 hours playing soccer and 6 hours babysitting.
The ratio to compare the time Diana spends playing soccer to the time she spends babysitting is 4:6 or 4/6 = 2/3

Question 6.
Antwone earns money at a steady rate mowing lawns. The points (1, 25) and (5, 125) appear on a graph of the amount earned versus number of lawns mowed. What are the coordinates of the point on the graph with an x-value of 3?
Type below:
_____________

Answer:
(3, 75)

Explanation:
y2-y1/x2-x1.
Y2 is 125, Y1 is 25, X2 is 5, and X1 is 1.
You then plug the numbers in, 125-25=100. 5-1=4.
Then you divide 100/4, in which you get 25. So you time 25 by 3, getting 75.

Share and Show – Page No. 277

Write the percent as a fraction.

Question 1.
80%
\(\frac{□}{□}\)

Answer:
\(\frac{80}{100}\)

Explanation:
80% is 80 out of 100
80 out of 100 squares is 80/100

Question 2.
150%
______ \(\frac{□}{□}\)

Answer:
1\(\frac{1}{2}\)

Explanation:
150% is 150 out of 100
150 out of 100 squares is 150/100 = 3/2 = 1 1/2

Question 3.
0.2%
\(\frac{□}{□}\)

Answer:
\(\frac{2}{1,000}\)

Explanation:
0.2% is 0.2 out of 100
0.2 out of 100 squares is 0.2/100 = 2/1,000

Write the percent as a decimal.

Question 4.
58%
______

Answer:
0.58

Explanation:
58% is 58 out of 100
58 out of 100 squares is 58/100
58/100 = 0.58

Question 5.
9%
______

Answer:
0.09

Explanation:
9% is 9 out of 100
9 out of 100 squares is 9/100
9/100 = 0.09

On Your Own

Write the percent as a fraction or mixed number.

Question 6.
17%
\(\frac{□}{□}\)

Answer:
\(\frac{17}{100}\)

Explanation:
17% is 17 out of 100
17 out of 100 squares is 17/100

Question 7.
20%
\(\frac{□}{□}\)

Answer:
\(\frac{1}{5}\)

Explanation:
20% is 20 out of 100
20 out of 100 squares is 20/100 = 2/10 = 1/5

Question 8.
125%
______ \(\frac{□}{□}\)

Answer:
1\(\frac{1{4}\)

Explanation:
125% is 125 out of 100
125 out of 100 squares is 125/100 = 1 1/4

Question 9.
355%
______ \(\frac{□}{□}\)

Answer:
3\(\frac{11}{20}\)

Explanation:
355% is 355 out of 100
355 out of 100 squares is 355/100 = 3 11/20

Question 10.
0.1%
\(\frac{□}{□}\)

Answer:
\(\frac{1}{1,000}\)

Explanation:
0.1% is 0.1 out of 100
0.1 out of 100 squares is 0.1/100 = 1/1,000

Question 11.
2.5%
\(\frac{□}{□}\)

Answer:
\(\frac{1}{40}\)

Explanation:
2.5% is 2.5 out of 100
2.5 out of 100 squares is 2.5/100 = 25/1,000 = 1/40

Write the percent as a decimal.

Question 12.
89%
______

Answer:
0.89

Explanation:
89% is 89 out of 100
89 out of 100 squares is 89/100
89/100 = 0.89

Question 13.
30%
______

Answer:
0.3

Explanation:
30% is 30 out of 100
30 out of 100 squares is 30/100
30/100 = 0.3

Question 14.
2%
______

Answer:
0.02

Explanation:
2% is 2 out of 100
2 out of 100 squares is 2/100
2/100 = 0.02

Question 15.
122%
______

Answer:
1.22

Explanation:
122% is 122 out of 100
122 out of 100 squares is 122/100
122/100 = 1.22

Question 16.
3.5%
______

Answer:
0.035

Explanation:
3.5% is 3.5 out of 100
3.5 out of 100 squares is 3.5/100
3.5/100 = 0.035

Question 17.
6.33%
______

Answer:
0.0633

Explanation:
6.33% is 6.33 out of 100
6.33 out of 100 squares is 6.33/100
6.33/100 = 0.0633

Question 18.
Use Reasoning Write <, >, or =.
21.6% ______ \(\frac{1}{5}\)

Answer:
21.6% > \(\frac{1}{5}\)

Explanation:
1/5 × 100/100 = 100/500 = 0.2/100 = 0.2%
21.6% > 0.2%

Question 19.
Georgianne completed 60% of her homework assignment. Write the portion of her homework that she still needs to complete as a fraction.
\(\frac{□}{□}\)

Answer:
\(\frac{2}{5}\)

Explanation:
Georgianne completed 60% of her homework assignment.
60/100
She needs to complete 40% of her homework = 40/100 = 2/5

Problem Solving + Applications – Page No. 278

Use the table for 20 and 21.
Go Math Grade 6 Answer Key Chapter 5 Model Percents img 16

Question 20.
What fraction of computer and video game players are 50 years old or more?
\(\frac{□}{□}\)

Answer:
\(\frac{13}{50}\)

Explanation:
computer and video game players,
50 or more are of 26% = 26/100 = 13/50

Question 21.
What fraction of computer and video game players are 18 years old or more?
\(\frac{□}{□}\)

Answer:
\(\frac{49}{100}\)

Explanation:
18 years old or more are of 49% = 49/100

Question 22.
Box A and Box B each contain black tiles and white tiles. They have the same total number of tiles. In Box A, 45% of the tiles are black. In Box B, \(\frac{11}{20}\) of the tiles are white. Compare the number of black tiles in the boxes. Explain your reasoning.
Type below:
_____________

Answer:
In Box A, 45% of the tiles are black.
In Box B, \(\frac{11}{20}\) of the tiles are white.
11/20 = 0.55 = 55/100 = 55%
100 – 55 = 45%
Both Box A and Box B have an equal number of black tiles

Question 23.
Mr. Truong is organizing a summer program for 6th grade students. He surveyed students to find the percent of students interested in each activity. Complete the table by writing each percent as a fraction or decimal.
Go Math Grade 6 Answer Key Chapter 5 Model Percents img 17
Type below:
_____________

Answer:
Sports = 48% = 48/100 = 0.48
Cooking = 23% = 23/100
Music = 20% = 20/100
Art = 9% = 9/100 = 0.09

Write Percents as Fractions and Decimals – Page No. 279

Write the percent as a fraction or mixed number.

Question 1.
44%
\(\frac{□}{□}\)

Answer:
\(\frac{11}{25}\)

Explanation:
44% is 44 out of 100
44 out of 100 squares is 44/100 = 11/25

Question 2.
32%
\(\frac{□}{□}\)

Answer:
\(\frac{8}{25}\)

Explanation:
32% is 32 out of 100
32 out of 100 squares is 32/100 = 8/25

Question 3.
116%
______ \(\frac{□}{□}\)

Answer:
1 \(\frac{4}{25}\)

Explanation:
116% is 116 out of 100
116 out of 100 squares is 116/100 = 1 4/25

Question 4.
250%
______ \(\frac{□}{□}\)

Answer:
2\(\frac{1}{2}\)

Explanation:
250% is 250 out of 100
250 out of 100 squares is 250/100 = 2 1/2

Question 5.
0.3%
\(\frac{□}{□}\)

Answer:
\(\frac{3}{1,000}\)

Explanation:
0.3% is 0.3 out of 100
0.3 out of 100 squares is 0.3/100
3/1,000

Question 6.
0.4%
\(\frac{□}{□}\)

Answer:
\(\frac{1}{250}\)

Explanation:
0.4% is 0.4 out of 100
0.4 out of 100 squares is 0.4/100 = 4/1,000 = 1/250

Question 7.
1.5%
\(\frac{□}{□}\)

Answer:
\(\frac{3}{200}\)

Explanation:
1.5% is 1.5 out of 100
1.5 out of 100 squares is 1.5/100 = 15/1,000 = 3/200

Question 8.
12.5%
\(\frac{□}{□}\)

Answer:
\(\frac{1}{8}\)

Explanation:
12.5% is 12.5 out of 100
12.5 out of 100 squares is 12.5/100 = 125/1,000 = 25/200 = 5/40 = 1/8

Write the percent as a decimal.

Question 9.
63%
______

Answer:
0.63

Explanation:
63% is 63 out of 100
63 out of 100 squares is 63/100
63/100 = 0.63

Question 10.
110%
______

Answer:
1.1

Explanation:
110% is 110 out of 100
110 out of 100 squares is 110/100 = 1.1

Question 11.
42.15%
______

Answer:
0.4215

Explanation:
42.15% is 42.15 out of 100
42.15 out of 100 squares is 42.15/100 = 0.4215

Question 12.
0.1%
______

Answer:
0.001

Explanation:
0.1% is 0.1 out of 100
0.1 out of 100 squares is 0.1/100  = 0.001

Problem Solving

Question 13.
An online bookstore sells 0.8% of its books to foreign customers. What fraction of the books are sold to foreign customers?
\(\frac{□}{□}\)

Answer:
\(\frac{1}{125}\)

Explanation:
An online bookstore sells 0.8% of its books to foreign customers.
0.8% = 0.8/100 = 8/1,000 = 1/125

Question 14.
In Mr. Klein’s class, 40% of the students are boys. What decimal represents the portion of the students that are girls?
______

Answer:
0.4

Explanation:
In Mr. Klein’s class, 40% of the students are boys.
40/100 = 0.4

Question 15.
Explain how percents, fractions, and decimals are related. Use a 10-by-10 grid to make a model that supports your explanation.
Type below:
_____________

Answer:
Go Math Grade 6 Answer Key Chapter 5 Model Percents img 1
53 squares are shaded out of 100.
53% or \(\frac{53}{100}\) or 0.53

Lesson Check – Page No. 280

Question 1.
The enrollment at Sonya’s school this year is 109% of last year’s enrollment. What decimal represents this year’s enrollment compared to last year’s?
______

Answer:
1.09 represents this year’s enrollment compared to last year’s

Explanation:
The enrollment at Sonya’s school this year is 109% of last year’s enrollment.
109% = 109/100 = 1.09

Question 2.
An artist’s paint set contains 30% watercolors and 25% acrylics. What fraction represents the portion of the paints that are watercolors or acrylics? Write the fraction in simplest form.
\(\frac{□}{□}\)

Answer:
\(\frac{11}{20}\)

Explanation:
An artist’s paint set contains 30% watercolors and 25% acrylics.
30 + 25 = 55% = 55/100 = 11/20

Spiral Review

Question 3.
Write the numbers in order from least to greatest.
-5.25 1.002 -5.09
Type below:
_____________

Answer:
-5.25, -5.09, 1.002

Question 4.
On a coordinate plane, the vertices of a rectangle are (2, 4), (2, −1), (−5, −1), and ( −5, 4). What is the perimeter of the rectangle?
______ units

Answer:
24 units

Explanation:
(2, 4) to (2, −1) is 4 + 1 = 5
(2, −1) to (−5, −1) is 2 + 5 = 7
5 + 7 + 5 + 7 = 24

Question 5.
The table below shows the widths and lengths, in feet, for different playgrounds. Which playgrounds have equivalent ratios of width to length?
Go Math Grade 6 Answer Key Chapter 5 Model Percents img 18
Type below:
_____________

Answer:
12/20 and 16.5/27.5 are equal

Explanation:
12/20 = 0.6
15/22.5 = 0.666
20/25 = 0.8
16.5/27.5 = 0.6

Question 6.
What percent represents the shaded part?
Go Math Grade 6 Answer Key Chapter 5 Model Percents img 19
_______ %

Answer:
85%

Explanation:
85 squares are shaded out of 100.
85%

Share and Show – Page No. 283

Write the fraction or decimal as a percent.

Question 1.
\(\frac{3}{25}\)
_______ %

Answer:
12%

Explanation:
3/25 ÷ 25/25 = 0.12/1 = 12/100 = 12%

Question 2.
\(\frac{3}{10}\)
_______ %

Answer:
30%

Explanation:
3/10 ÷ 10/10 = 0.3 = 0.3 × 100/100 = 30/100 = 30%

Question 3.
0.717
_______ %

Answer:
71.7%

Explanation:
0.717 = 717/100 = 71.7%

Question 4.
0.02
_______ %

Answer:
2%

Explanation:
0.02 = 2/100 = 2%

On Your Own

Write the number in two other forms ( fraction, decimal, or percent). Write the fraction in simplest form.

Question 5.
0.01
Type below:
_____________

Answer:
1% and \(\frac{1}{100}\)

Explanation:
0.01 as a fraction 1/100
0.01 as percent 1%

Question 6.
\(\frac{13}{40}\)
Type below:
_____________

Answer:
0.325 and 32.5%

Explanation:
\(\frac{13}{40}\) as decimal 0.325
\(\frac{13}{40}\) as percent 32.5/100 = 32.5%

Question 7.
\(\frac{6}{5}\)
Type below:
_____________

Answer:
1.2 and 120%

Explanation:
\(\frac{6}{5}\) as decimal 1.2
\(\frac{6}{5}\) as percent 120/100 = 120%

Question 8.
0.08
Type below:
_____________

Answer:
8% and \(\frac{8}{100}\)

Explanation:
0.08 as a fraction 8/100
0.08 as percent 8%

The table shows the portion of Kim’s class that participates in each sport. Use the table for 9–10.
Go Math Grade 6 Answer Key Chapter 5 Model Percents img 20

Question 9.
Do more students take part in soccer or in swimming? Explain your reasoning.
Type below:
_____________

Answer:
Soccer = 1/5 = 0.2
Swimming = 0.09
0.2 > 0.09
more students take part in Soccer

Question 10.
Explain What percent of Kim’s class participates in one of the sports listed? Explain how you found your answer
_______ %

Answer:
23%

Explanation:
Kim’s class participates in Baseball that is mentioned with 23%

Question 11.
For their reading project, students chose to either complete a character study, or write a book review. \(\frac{1}{5}\) of the students completed a character study, and 0.8 of the students wrote a book review. Joia said that more students wrote a book review than completed a character study. Do you agree with Joia? Use numbers and words to support your answer
Type below:
_____________

Answer:
1/5 = 0.2
0.2 < 0.8
More students completed writing a book review.
I agree with Joia

Sand Sculptures – Page No. 284

Every year, dozens of teams compete in the U.S. Open Sandcastle Competition. Recent winners have included complex sculptures in the shape of flowers, elephants, and racing cars.

Teams that participate in the contest build their sculptures using a mixture of sand and water. Finding the correct ratios of these ingredients is essential for creating a stable sculpture.

The table shows the recipes that three teams used. Which team used the greatest percent of sand in their recipe?
Go Math Grade 6 Answer Key Chapter 5 Model Percents img 21
Convert to percents. Then order from least to greatest.
Go Math Grade 6 Answer Key Chapter 5 Model Percents img 22
From least to greatest, the percents are 75%, 84%, 95%.
So, Team B used the greatest percent of sand.
Solve.

Question 12.
Which team used the greatest percent of water in their recipe?
Type below:
_____________

Answer:
Team A used the greatest percent of water in their recipe

Explanation:
Team A, 10/10+30 = 10/40 = 0.25 = 25%
Team B, 1/20 × 5/5 = 5/100 = 5%
Team C, 0.16 = 16%

Question 13.
Some people say that the ideal recipe for sand sculptures contains 88.9% sand. Which team’s recipe is closest to the ideal recipe?
Type below:
_____________

Answer:
Team C

Question 14.
Team D used a recipe that consists of 20 cups of sand, 2 cups of flour, and 3 cups of water. How does the percent of sand in Team D’s recipe compare to that of the other teams?
Type below:
_____________

Answer:
Total number of cups together = 20 + 2+ 3 =25 cups
20/25 × 100 = 80/100 = 80%

Write Fractions and Decimals as Percents – Page No. 285

Write the fraction or decimal as a percent.

Question 1.
\(\frac{7}{20}\)
_______ %

Answer:
35%

Explanation:
7/20 = 0.35 = 35%

Question 2.
\(\frac{3}{50}\)
_______ %

Answer:
6%

Explanation:
3/50 = 0.06 = 6%

Question 3.
\(\frac{1}{25}\)
_______ %

Answer:
4%

Explanation:
1/25 = 0.04 = 4%

Question 4.
\(\frac{5}{5}\)
_______ %

Answer:
0.01%

Explanation:
5/5 = 1 = 0.01%

Question 5.
0.622
_______ %

Answer:
6.22%

Explanation:
0.622 = 6.22/100 = 6.22%

Question 6.
0.303
_______ %

Answer:
3.03%

Explanation:
0.303 = 3.03/100 = 3.03%

Question 7.
0.06
_______ %

Answer:
6%

Explanation:
0.06 = 6/100 = 6%

Question 8.
2.45
_______ %

Answer:
245%

Explanation:
2.45 × 100/100 = 245/100 = 245%

Write the number in two other forms (fraction, decimal, or percent). Write the fraction in simplest form

Question 9.
\(\frac{19}{20}\)
Type below:
_____________

Answer:
0.95 and 95%

Explanation:
\(\frac{19}{20}\) as a decimal 0.95
\(\frac{19}{20}\) as a percentage 95%

Question 10.
\(\frac{9}{16}\)
Type below:
_____________

Answer:
0.5625 and 56.25%

Explanation:
\(\frac{9}{16}\) as a decimal 0.5625
\(\frac{9}{16}\) as a percentage 56.25%

Question 11.
0.4
Type below:
_____________

Answer:
\(\frac{2}{5}\) and 40%

Explanation:
0.4 as a fraction 2/5
0.4 as a percentage 40/100 = 40%

Question 12.
0.22
Type below:
_____________

Answer:
\(\frac{11}{50}\) and 22%

Explanation:
0.22 as a fraction 11/50
0.22 as a percentage 22/100 = 22%

Problem Solving

Question 13.
According to the U.S. Census Bureau, \(\frac{3}{25}\) of all adults in the United States visited a zoo in 2007. What percent of all adults in the United States visited a zoo in 2007?
_______ %

Answer:
12%

Explanation:
According to the U.S. Census Bureau, \(\frac{3}{25}\) of all adults in the United States visited a zoo in 2007.
\(\frac{3}{25}\) = 0.12 = 12%

Question 14.
A bag contains red and blue marbles. Given that \(\frac{17}{20}\) of the marbles are red, what percent of the marbles are blue?
_______ %

Answer:
15%

Explanation:
The total number of marbles = 20
If 17 marbles are red, the remaining 3 marbles out of 20 are blue marbles
3/20 = 0.15 = 15%

Question 15.
Explain two ways to write \(\frac{4}{5}\) as a percent.
Type below:
_____________

Answer:
Decimal =0.8.
Percentage =80%

Explanation:
4/5 = 0.8 = 80/100 = 80%

Lesson Check – Page No. 286

Question 1.
The portion of shoppers at a supermarket who pay by credit card is 0.36. What percent of shoppers at the supermarket do NOT pay by credit card?
_______ %

Answer:
36%

Explanation:
The portion of shoppers at a supermarket who pay by credit card is 0.36.
0.36 = 0.36 × 100/100 = 36/100 = 36%

Question 2.
About \(\frac{23}{40}\) of a lawn is planted with Kentucky bluegrass. What percent of the lawn is planted with Kentucky bluegrass?
_______ %

Answer:
57.5%

Explanation:
About \(\frac{23}{40}\) of a lawn is planted with Kentucky bluegrass.
23/40 = 0.575 = 0.575 × 100/100 = 57.5/100 = 57.5%

Spiral Review

Question 3.
A basket contains 6 peaches and 8 plums. What is the ratio of peaches to total pieces of fruit?
Type below:
_____________

Answer:
6:14

Explanation:
total pieces of fruit 6 + 8 = 14
the ratio of peaches to total pieces of fruit is 6:14

Question 4.
It takes 8 minutes for 3 cars to move through a car wash. At the same rate, how many cars can move through the car wash in 24 minutes?
_______ cars

Answer:
9 cars

Explanation:
It takes 8 minutes for 3 cars to move through a car wash.
3/8 × 24 = 9 cars

Question 5.
A 14-ounce box of cereal sells for $2.10. What is the unit rate?
$ _______ per ounce

Answer:
$0.15 per ounce

Explanation:
$2.10/14 × 14/14 = $0.15 per ounce

Question 6.
A model railroad kit contains curved tracks and straight tracks. Given that 35% of the tracks are curved, what fraction of the tracks are straight? Write the fraction in simplest form.
\(\frac{□}{□}\)

Answer:
\(\frac{7}{20}\)

Explanation:
A model railroad kit contains curved tracks and straight tracks. Given that 35% of the tracks are curved,
35% = 35/100 = 7/20

Vocabulary – Page No. 287

Choose the best term from the box to complete the sentence.
Go Math Grade 6 Answer Key Chapter 5 Model Percents img 23

Question 1.
A _____ is a ratio that compares a quantity to 100.
Type below:
_____________

Answer:
percent

Concepts and Skills

Write a ratio and a percent to represent the shaded part.

Question 2.
Go Math Grade 6 Answer Key Chapter 5 Model Percents img 24
Type below:
_____________

Answer:
17% and \(\frac{17}{100}\)

Explanation:
17 squares are shaded out of 100.
So, 17% and 17/100 are the answers.

Question 3.
Go Math Grade 6 Answer Key Chapter 5 Model Percents img 25
Type below:
_____________

Answer:
60% and \(\frac{60}{100}\)

Explanation:
60 squares are shaded out of 100.
So, 60% and 60/100 are the answers.

Question 4.
Go Math Grade 6 Answer Key Chapter 5 Model Percents img 26
Type below:
_____________

Answer:
7% and \(\frac{7}{100}\)

Explanation:
7 squares are shaded out of 100.
So, 7% and 7/100 are the answers.

Question 5.
Go Math Grade 6 Answer Key Chapter 5 Model Percents img 27
Type below:
_____________

Answer:
11% and \(\frac{11}{100}\)

Explanation:
11 squares are shaded out of 100.
So, 11% and 11/100 are the answers.

Question 6.
Go Math Grade 6 Answer Key Chapter 5 Model Percents img 28
Type below:
_____________

Answer:
82% and \(\frac{82}{100}\)

Explanation:
82 squares are shaded out of 100.
So, 82% and 82/100 are the answers.

Question 7.
Go Math Grade 6 Answer Key Chapter 5 Model Percents img 29
Type below:
_____________

Answer:
36% and \(\frac{36}{100}\)

Explanation:
36 squares are shaded out of 100.
So, 36% and 36/100 are the answers.

Write the number in two other forms (fraction, decimal, or percent).

Write the fraction in simplest form.

Question 8.
0.04
Type below:
_____________

Answer:
\(\frac{1}{25}\) and 4%

Explanation:
0.04 as a fraction 4/100 = 1/25
0.04 as a decimal 0.04 × 100/100 = 4/100 = 4%

Question 9.
\(\frac{3}{10}\)
Type below:
_____________

Answer:
0.3 and 30%

Explanation:
\(\frac{3}{10}\) as a decimal 0.3
\(\frac{3}{10}\) as a percentage 0.3 × 100/100 = 30/100 = 30%

Question 10.
1%
Type below:
_____________

Answer:
\(\frac{1}{100}\) and 0.01

Explanation:
1% as a fraction 1/100
1% as a decimal 1/100 = 0.01

Question 11.
1 \(\frac{1}{5}\)
Type below:
_____________

Answer:
1.2 and 120%

Explanation:
1 \(\frac{1}{5}\) as a decimal = 6/5 = 1.2
1 \(\frac{1}{5}\) as a percentage 1.2 × 100/100 = 120/100 = 120%

Question 12.
0.9
Type below:
_____________

Answer:
\(\frac{90}{100}\) and 90%

Explanation:
0.9 as a fraction 0.9 × 100/100 = 90/100 = 90%

Question 13.
0.5%
Type below:
_____________

Answer:
\(\frac{5}{1,000}\) and 0.005

Explanation:
0.5% as a fraction = 0.5/100 = 5/1,000
0.5% as a decimal = 0.5/100 = 0.005

Question 14.
\(\frac{7}{8}\)
Type below:
_____________

Answer:
0.875 and 87.5%

Explanation:
\(\frac{7}{8}\) as a decimal 0.875
\(\frac{7}{8}\) as a percentage 87.5/100 = 87.5%

Question 15.
355%
Type below:
_____________

Answer:
\(\frac{71}{20}\) and 35.5

Explanation:
355% as a decimal 355/100 = 71/20 = 35.5

Page No. 288

Question 16.
About \(\frac{9}{10}\) of the avocados grown in the United States are grown in California. About what percent of the avocados grown in the United States are grown in California?
_______ %

Answer:
90%

Explanation:
About \(\frac{9}{10}\) of the avocados grown in the United States are grown in California.
9/10 × 10/10 = 90/100 = 90%

Question 17.
Morton made 36 out of 48 free throws last season. What percent of his free throws did Morton make?
_______ %

Answer:
75%

Explanation:
Morton made 36 out of 48 free throws last season.
36/48 = 0.75 = 75/100 = 75%

Question 18.
Sarah answered 85% of the trivia questions correctly. What fraction describes this percent?
\(\frac{□}{□}\)

Answer:
\(\frac{17}{20}\)

Explanation:
Sarah answered 85% of the trivia questions correctly.
85% = 85/100 = 17/20

Question 19.
About \(\frac{4}{5}\) of all the orange juice in the world is produced in Brazil. About what percent of all the orange juice in the world is produced in Brazil?
_______ %

Answer:
80%

Explanation:
About \(\frac{4}{5}\) of all the orange juice in the world is produced in Brazil.
4/5 = 0.8 × 100/100 = 80/100 = 80%

Question 20.
If you eat 4 medium strawberries, you get 48% of your daily recommended amount of vitamin C. What fraction of your daily amount of vitamin C do you still need?
\(\frac{□}{□}\)

Answer:
\(\frac{13}{25}\)

Explanation:
If you eat 4 medium strawberries, you get 48% of your daily recommended amount of vitamin C.
48% = 48/100
100 – 48 = 52
52% = 52/100 = 13/25 of your daily amount of vitamin C do you still need

Share and Show – Page No. 290

Find the percent of the quantity.

Question 1.
25% of 320
_______

Answer:
80

Explanation:
Write the percent as a rate per 100
25% = 25/100
25/100 × 320 = 80

Question 2.
80% of 50
_______

Answer:
40

Explanation:
Write the percent as a rate per 100
80% = 80/100
80/100 × 50 = 40

Question 3.
175% of 24
_______

Answer:
42

Explanation:
Write the percent as a rate per 100
175% = 175/100
175/100 × 24 = 42

Question 4.
60% of 210
_______

Answer:
126

Explanation:
Write the percent as a rate per 100
60% = 60/100
60/100 × 210 = 126

Question 5.
A jar contains 125 marbles. Given that 4% of the marbles are green, 60% of the marbles are blue, and the rest are red, how many red marbles are in the jar?
_______ marbles

Answer:
45 marbles

Explanation:
A jar contains 125 marbles.
4% of the marbles are green = 125 × 4/100 = 5
60% of the marbles are blue = 125 × 60/100 = 75
Red Marbles = Total Number of Marbles -[Number of Green Marbles + Number of Blue Marbles]
Red Marbles = 125 – (5 + 75) = 125 – 80 = 45

Question 6.
There are 32 students in Mr. Moreno’s class and 62.5% of the students are girls. How many boys are in the class?
_______ students

Answer:
12 students

Explanation:
There are 32 students in Mr. Moreno’s class
62.5% of the students are girls = 32 × 62.5/100 = 20
boys = 32 – 20 = 12

On Your Own – Page No. 291

Find the percent of the quantity.

Question 7.
60% of 90
_______

Answer:
54

Explanation:
Write the percent as a rate per 100
60% = 60/100
60/100 × 90 = 54

Question 8.
25% of 32.4
_______

Answer:
8.1

Explanation:
Write the percent as a rate per 100
25% = 25/100
25/100 × 32.4 = 8.1

Question 9.
110% of 300
_______

Answer:
330

Explanation:
Write the percent as a rate per 100
110% = 110/100
110/100 × 300 = 330

Question 10.
0.2% of 6500
_______

Answer:
13

Explanation:
Write the percent as a rate per 100
0.2% = 0.2/100
0.2/100 × 6500 = 13

Question 11.
A baker made 60 muffins for a cafe. By noon, 45% of the muffins were sold. How many muffins were sold by noon?
_______ muffins

Answer:
27 muffins

Explanation:
A baker made 60 muffins for a cafe. By noon, 45% of the muffins were sold.
60 × 45%
60 × 45/100 = 27

Question 12.
There are 30 treasures hidden in a castle in a video game. LaToya found 80% of them. How many of the treasures did LaToya find?
_______ treasures

Answer:
24 treasures

Explanation:
There are 30 treasures hidden in a castle in a video game.
LaToya found 80% of them.
30 × 80/100 = 24

Question 13.
A school library has 260 DVDs in its collection. Given that 45% of the DVDs are about science and 40% are about history, how many of the DVDs are about other subjects?
_______ DVDs

Answer:
39 DVDs

Explanation:
A school library has 260 DVDs in its collection.
45% of the DVDs are about science = 260 × 45/100 = 117
40% are about history = 260 × 40/100 = 104
other subjects = 260 – (117 + 104) = 260 – 221 = 39

Question 14.
Mitch planted cabbage, squash, and carrots on his 150-acre farm. He planted half the farm with squash and 22% with carrots. How many acres did he plant with cabbage?
_______ acres

Answer:

Explanation:
Mitch planted cabbage, squash, and carrots on his 150-acre farm.
He planted half the farm with squash 150/2 = 75
22% with carrots = 150 × 22/100 = 33
cabbage = 150 – (75 + 33) = 150 – 108 = 42

Question 15.
45% of 60 _______ 60% of 45

Answer:
45% of 60 = 60% of 45

Explanation:
45% of 60
45/100 × 60 = 27
60% of 45
60/100 × 45 = 27
45% of 60 = 60% of 45

Question 16.
10% of 90 _______ 90% of 100

Answer:
10% of 90 _______ 90% of 100

Explanation:
10% of 90
10/100 × 90 = 9
90% of 100
90/100 × 100 = 90
10% of 90 < 90% of 100

Question 17.
75% of 8 _______ 8% of 7.5

Answer:
75% of 8 > 8% of 7.5

Explanation:
75% of 8
75/100 × 8 = 6
8% of 7.5
8/100 × 7.5 = 0.6
75% of 8 > 8% of 7.5

Question 18.
Sarah had 12 free throw attempts during a game and made at least 75% of the free throws. What is the greatest number of free throws Sarah could have missed during the game?
_______ free throws

Answer:
3 free throws

Explanation:
Sarah had 12 free throw attempts during a game and made at least 75% of the free throws.
So, she missed 25% of the free throws.
12 × 25/100 = 3

Question 19.
Chrissie likes to tip a server in a restaurant a minimum of 20%. She and her friend have a lunch bill that is $18.34. Chrissie says the tip will be $3.30. Her friend says that is not a minimum of 20%. Who is correct? Explain.
Type below:
_____________

Answer:
100% = $18.34
10% = $18.34 / 10 = 1.834
20% = 1.834 × 2 = 3.66800 = $3.70
Her friend is correct because $3.70 is more than $3.30.

Unlock The Problem – Page No. 292

Question 20.
One-third of the juniors in the Linwood High School Marching Band play the trumpet. The band has 50 members and the table shows what percent of the band members are freshmen, sophomores, juniors, and seniors. How many juniors play the trumpet?
Go Math Grade 6 Answer Key Chapter 5 Model Percents img 30
a. What do you need to find?
Type below:
_____________

Answer:
The percent of the band members are freshmen, sophomores, juniors, and seniors. How many juniors play the trumpet

Question 20.
b. How can you use the table to help you solve the problem?
Type below:
_____________

Answer:
percent of the band members that are Juniors: 24%
In 50 members of the band, 50×24/100 = 12 are Juniors. One-third of them play the trumpet, which makes 12×(1/3) = 4 members.

Question 20.
c. What operation can you use to find the number of juniors in the band?
Type below:
_____________

Answer:
percent of the band members that are Juniors: 24%
In 50 members of the band, 50×24/100 = 12 are Juniors.

Explanation:

Question 20.
d. Show the steps you use to solve the problem.
Type below:
_____________

Answer:
percent of the band members that are Juniors: 24%
In 50 members of the band, 50×24/100 = 12 are Juniors. One-third of them play the trumpet, which makes 12×(1/3) = 4 members.

Question 20.
e. Complete the sentences.
The band has _____ members. There are _____ juniors in the band. The number of juniors who play the trumpet is _____.
Type below:
_____________

Answer:
The band has 50 members. There are 12 juniors in the band. The number of juniors who play the trumpet is 4.

Question 21.
Compare. Circle <, >, or =.
a. 25% of 44 Ο 20% of 50
b. 10% of 30 Ο 30% of 100
c. 35% of 60 Ο 60% of 35
25% of 44 _____ 20% of 50
10% of 30 _____ 30% of 100
35% of 60 _____ 60% of 35

Answer:
25% of 44 >  20% of 50
10% of 30 < 30% of 100
35% of 60 = 60% of 35

Explanation:
25% of 44 = 25/100 × 44 = 11
20% of 50 = 20/100 × 50 = 1000/100 = 10
25% of 44  > 20% of 50
10% of 30 = 10/100 × 30 = 3
30% of 100 = 30/100 × 100 = 30
10% of 30 < 30% of 100
35% of 60 = 35/100 × 60 = 21
60% of 35 = 60/100 × 35 = 21
35% of 60 = 60% of 35

Percent of a Quantity – Page No. 293

Find the percent of the quantity.

Question 1.
60% of 140
_____

Answer:
84

Explanation:
60% of 140
60/100 × 140 = 84

Question 2.
55% of 600
_____

Answer:
330

Explanation:
55% of 600
55/100 × 600 = 330

Question 3.
4% of 50
_____

Answer:
2

Explanation:
4% of 50
4/100 × 50 = 2

Question 4.
10% of 2,350
_____

Answer:
235

Explanation:
10% of 2,350
10/100 × 2,350 = 235

Question 5.
160% of 30
_____

Answer:
48

Explanation:
160% of 30
160/100 × 30 = 48

Question 6.
105% of 260
_____

Answer:
273

Explanation:
105% of 260
105/100 × 260 = 273

Question 7.
0.5% of 12
_____

Answer:
0.06

Explanation:
0.5% of 12
0.5/100 × 12 = 0.06

Question 8.
40% of 16.5
_____

Answer:
6.6

Explanation:
40% of 16.5
40/100 × 16.5 =  6.6

Problem Solving

Question 9.
The recommended daily amount of vitamin C for children 9 to 13 years old is 45 mg. A serving of a juice drink contains 60% of the recommended amount. How much vitamin C does the juice drink contain?
_____ mg

Answer:
27 mg

Explanation:
The recommended daily amount of vitamin C for children 9 to 13 years old is 45 mg. A serving of a juice drink contains 60% of the recommended amount.
45% of 60 = 45/100 × 60 = 27

Question 10.
During a 60-minute television program, 25% of the time is used for commercials and 5% of the time is used for the opening and closing credits. How many minutes remain for the program itself?
_____ minutes

Answer:
42 minutes

Explanation:
60 minutes of tv
25% + 5% = 30%
30%= 0.30
60 times 0.30= 18
60-18=42
inly 42 minutes are used for the program itself

Question 11.
Explain two ways you can find 35% of 700.
Type below:
_____________

Answer:
First way
700 : 100 = x : 35
x = 700 × 35 : 100
x = 245
Second way
700 : 100 × 35 =
245

Lesson Check – Page No. 294

Question 1.
A store has a display case with cherry, peach, and grape fruit chews. There are 160 fruit chews in the display case. Given that 25% of the fruit chews are cherry and 40% are peach, how many grape fruit chews are in the display case?
_____ grape fruit chews

Answer:
56 grape fruit chews

Explanation:
A store has a display case with cherry, peach, and grape fruit chews. There are 160 fruit chews in the display case. Given that 25% of the fruit chews are cherry and 40% are peach,
25% + 40% +?% = 100%
65% + ?% = 100%
?% = 35%
.35×160 = 56

Question 2.
Kelly has a ribbon that is 60 inches long. She cuts 40% off the ribbon for an art project. While working on the project, she decides she only needs 75% of the piece she cut off. How many inches of ribbon does Kelly end up using for her project?
_____ inches

Answer:
18 inches

Explanation:
Length of ribbon = 60 inches
Part of ribbon cut off for an art project = 40%
So, the Length of the ribbon remains is given by
40% of 60 = 40/100 × 60 = 24
Part of a piece she only needs from cut off = 75%
so, the Length of ribbon she need end up using in her project is given by
75/100 × 24 = 18

Spiral Review

Question 3.
Three of the following statements are true. Which one is NOT true?
|−12| > 1      |0| > −4      |20| > |−10|        6 < |−3|
Type below:
_____________

Answer:
|−12| > 1
12 > 1; True
|0| > −4
0 > -4; True
|20| > |−10|
20 > 10; True
6 < |−3|
6 < 3; False

Question 4.
Miyuki can type 135 words in 3 minutes. How many words can she expect to type in 8 minutes?
_____ words

Answer:
360 words

Explanation:
Miyuki can type 135 words in 3 minutes.
135/3 = 45
45 × 8 = 360

Question 5.
Which percent represents the model?
Go Math Grade 6 Answer Key Chapter 5 Model Percents img 31
_____ %

Answer:
63%

Explanation:
63 squares are shaded out of 100
63%

Question 6.
About \(\frac{3}{5}\) of the students at Roosevelt Elementary School live within one mile of the school. What percent of students live within one mile of the school?
_____ %

Answer:
60%

Explanation:
About \(\frac{3}{5}\) of the students at Roosevelt Elementary School live within one mile of the school.
3/5 × 100/100 = 60/100 = 60%

Share and Show – Page No. 297

Question 1.
A geologist visits 40 volcanoes in Alaska and California. 15% of the volcanoes are in California. How many volcanoes does the geologist visit in California and how many in Alaska?
Type below:
_____________

Answer:
40 volcanoes = 100% of them
100 – 15% = 85%
Number of volcanoes in California = 15% of 40 volcanoes = 0.15 x 40 = 6
Number of volcanoes in Alaska = 85% of 40 volcanoes 0.85 x 40 = 34

Question 2.
What if 30% of the volcanoes were in California? How many volcanoes would the geologist have visited in California and how many in Alaska?
Type below:
_____________

Answer:
Number of volcanoes in California = 30% of 40 = 30/100 x 40 = 12
Number of volcanoes in Alaska = 70% of 40 = 70/100 x 40 = 28

Question 3.
Ricardo has $25 to spend on school supplies. He spends 72% of the money on a backpack and the rest on a large binder. How much does he spend on the backpack? How much does he spend on the binder?
Type below:
_____________

Answer:
$18 on Backpack $7 on binder.
If you turn the percent into a decimal .72 and multiply .72 by 25 you get 18 which is the cost of the backpack.
subtract 18 from 25 and you get $7 left meaning the binder was $7

Question 4.
Kevin is hiking on a trail that is 4.2 miles long. So far, he has hiked 80% of the total distance. How many more miles does Kevin have to hike in order to complete the trail?
Type below:
_____________

Answer:
0.84 miles

Explanation:
Kevin is hiking on a trail that is 4.2 miles long. So far, he has hiked 80% of the total distance.
80% of 4.2 = 80/100 x 4.2 = 3.36
4.2 – 3.36 = 0.84 miles

On Your Own – Page No. 298

Question 5.
Jordan takes 50% of the cherries from a bowl. Then Mei takes 50% of the remaining cherries. Finally, Greg takes 50% of the remaining cherries. There are 3 cherries left. How many cherries were in the bowl before Jordan arrived?
_____ cherries

Answer:
24 cherries

Explanation:
Let total cherries in a bowl=x
Jordan takes cherries=50% of x = 50x/100
Remaining cherries = x – 50x/100 = x/2
Mei takes cherries=50% of 50x/100 = x/4
remaining cherries= x/2 – x/4 = x/4
Greg takes cherries=50% of x/4 = x/8
remaining cherries = x/4 – x/8 = x/8
Now,remaining cherries in a bowl=3
x/8 =3
x = 8 × 3 = 24

Question 6.
Each week, Tasha saves 65% of the money she earns babysitting and spends the rest. This week she earned $40. How much more money did she save than spend this week?
$ _____

Answer:
Tasha saved $26 and spent $14

Explanation:
Since 65% of 40 is 26, that’s how much Tasha saves. Then do 40 – 26 to get 14, which is how much she spends.
So Tasha saved $26 and spent $14.

Question 7.
An employee at a state park has 53 photos of animals found at the park. She wants to arrange the photos in rows so that every row except the bottom row has the same number of photos. She also wants there to be at least 5 rows. Describe two different ways she can arrange the photos
Type below:
_____________

Answer:
5 rows of 10 photos and last row with 3 photos,
6 rows of 8 photos and last row with 5 photos,
7 rows of 7 photos and last row with 4 photos,
Also, reverse the rows and photos in each row (ex 5 rows 10 photos=10 rows 5 photos) to get another 3 sets.

Question 8.
Explain a Method Maya wants to mark a length of 7 inches on a sheet of paper, but she does not have a ruler. She has pieces of wood that are 4 inches, 5 inches, and 6 inches long. Explain how she can use these pieces to mark a length of 7 inches.
Type below:
_____________

Answer:
Maya can put the 5 and 6-inch pieces together to get 11 inches. She can then subtract the length of the 4-inch piece to get 7 inches.

Question 9.
Pierre’s family is driving 380 miles from San Francisco to Los Angeles. On the first day, they drive 30% of the distance. On the second day, they drive 50% of the distance. On the third day, they drive the remaining distance and arrive in Los Angeles. How many miles did Pierre’s family drive each day? Write the number of miles in the correct box.
Go Math Grade 6 Answer Key Chapter 5 Model Percents img 32
Type below:
_____________

Answer:
76 miles

Explanation:
Pierre’s family is driving 380 miles from San Francisco to Los Angeles.
On the first day, they drive 30% of the distance. 380 × 30/100 = 114
On the second day, they drive 50% of the distance. 380 × 50/100 = 190
They traveled 80%.
On the third day, they drive the remaining distance and arrive in Los Angeles.
380 × 20/100 = 76 miles

Problem Solving Percents – Page No. 299

Read each problem and solve.

Question 1.
On Saturday, a souvenir shop had 125 customers. Sixty-four percent of the customers paid with a credit card. The other customers paid with cash. How many customers paid with cash?T
_____ costumers

Answer:
45 costumers

Explanation:
On Saturday, a souvenir shop had 125 customers. Sixty-four percent of the customers paid with a credit card.
125 × 64/100 = 80
100 – 64 = 36
125 × 36/100 = 45

Question 2.
A carpenter has a wooden stick that is 84 centimeters long. She cuts off 25% from the end of the stick. Then she cuts the remaining stick into 6 equal pieces. What is the length of each piece?
_____ cm

Answer:
10 1/2 cm

Explanation:
A carpenter has a wooden stick that is 84 centimeters long. She cuts off 25% from the end of the stick. Then she cuts the remaining stick into 6 equal pieces.
84 × 75/100 = 63
63/6 = 10 1/2

Question 3.
A car dealership has 240 cars in the parking lot and 17.5% of them are red. Of the other 6 colors in the lot, each color has the same number of cars. If one of the colors is black, how many black cars are in the lot?
_____ black cars

Answer:
33 black cars

Explanation:
number of red cars 17.5% × 240 = 42
number of cars of other colors = 240 – 42 = 198
number of black cars 1/6 × 198 = 33

Question 4.
The utilities bill for the Millers’ home in April was $132. Forty-two percent of the bill was for gas, and the rest was for electricity. How much did the Millers pay for gas, and how much did they pay for electricity?
Type below:
_____________

Answer:
Amount of money paid for gas = 132 * (42/100) dollars
= 5544/100 dollars
= 55.44 dollars
Then
The amount of money paid for electricity = (132 – 55.44) dollars
= 76.56 dollars
So the Millers paid 55.44 dollars for gas and 76.56 dollars for electricity in the month of April.

Question 5.
Andy’s total bill for lunch is $20. The cost of the drink is 15% of the total bill and the rest is the cost of the food. What percent of the total bill did Andy’s food cost? What was the cost of his food?
Type below:
_____________

Answer:
$17

Explanation:
Andy paid $20 total for his lunch (100%).
15% is for drink.
Therefore, 100 – 15 = 85% is the percent that was constituted by the food.
85% of 20 is equal to 0.85 × 20 is equal to:
17 × 20/20 = 17
Andy’s food cost $17.

Question 6.
Write a word problem that involves finding the additional amount of money needed to purchase an item, given the cost and the percent of the cost already saved.
Type below:
_____________

Answer:
Each week, Tasha saves 65% of the money she earns babysitting and spends the rest. This week she earned $40. How much more money did she save than spend this week?
Tasha saved $26 and spent $14

Lesson Check – Page No. 300

Question 1.
Milo has a collection of DVDs. Out of 45 DVDs, 40% are comedies and the remaining are action-adventures. How many actionadventure DVDs does Milo own?
_____ DVDs

Answer:
27 DVDs

Explanation:
100%-40%=60%
60/100*45=27
27 DVD’s are action-adventure

Question 2.
Andrea and her partner are writing a 12-page science report. They completed 25% of the report in class and 50% of the remaining pages after school. How many pages do Andrea and her partner still have to write?
_____ pages

Answer:
9 pages

Explanation:
first 50% + 25% = 75%
then you can do 75% of 12
75% = 0.75
of = multiplication
0.75 • 12 which should equal 9
so they have 9 pages left

Spiral Review

Question 3.
What is the absolute value of \(\frac{-4}{25}\)?
\(\frac{□}{□}\)

Answer:
\(\frac{4}{25}\)

Explanation:
|\(\frac{-4}{25}\)| = 4/25

Question 4.
Ricardo graphed a point by starting at the origin and moving 5 units to the left. Then he moved up 2 units. What is the ordered pair for the point he graphed?
Type below:
_____________

Answer:
(-5, 2)

Explanation:
In a coordinate system, the coordinates of the origin are (0, 0).
If he moves 5 units to the left, he is moving in the negative direction along the x-axis, and x takes the value -5.
If he moves up 2 units, he is moving in the positive direction along the y-axis, and y takes the value 2.
The ordered pair (x, y) is (-5, 2).

Question 5.
The population of birds in a sanctuary increases at a steady rate. The graph of the population over time has the points (1, 105) and (3, 315). Name another point on the graph.
Type below:
_____________

Answer:
You could do (2, 210) or (4, 420) or (5, 525)

Question 6.
Alicia’s MP3 player contains 1,260 songs. Given that 35% of the songs are rock songs and 20% of the songs are rap songs, how many of the songs are other types of songs?
_____ songs

Answer:
567 songs

Explanation:
Since 55% of the songs are rock and rap, 45% of the songs are other.
To find 45% of 1260 we multiply by the decimal:
1260 x 0.45 = 567
Therefore 567 of the songs are other.

Share and Show – Page No. 303

Find the unknown value.

Question 1.
9 is 25% of _____.
_____

Answer:
36

Explanation:
25/100 ÷ 25/25 = 1/4
1/4 = 9/s
1/4 × 9/9 = 9/36
the unknown value is 36

Question 2.
14 is 10% of _____.
_____

Answer:
140

Explanation:
10/100 ÷ 10/10 = 1/10
1/10 = 14/s
1/10 × 14/14 = 14/140
the unknown value is 140

Question 3.
3 is 5% of _____.
_____

Answer:
6

Explanation:
5/10 ÷ 5/5 = 1/2
1/2 × 3/3 = 3/6
the unknown value is 6

Question 4.
12 is 60% of _____.
_____

Answer:
20

Explanation:
60/100 ÷ 60/60 = 60/100
60/100 ÷ 5/5 = 12/20
the unknown value is 20

On Your Own

Find the unknown value.

Question 5.
16 is 20% of _____.
_____

Answer:
80

Explanation:
20/100 ÷ 20/20 = 1/5
1/5 × 16/16 = 16/80
the unknown value is 80

Question 6.
42 is 50% of _____.
_____

Answer:
84

Explanation:
50/100 ÷ 50/50 = 1/2
1/2 × 42/42 = 42/84
the unknown value is 84

Question 7.
28 is 40% of _____.
_____

Answer:
70

Explanation:
40/100 ÷ 40/40 = 1/2.5
1/2.5 × 28/28 = 28/70
the unknown value is 70

Question 8.
60 is 75% of _____.
_____

Answer:
80

Explanation:
75/100 ÷ 75/75 = 60/s
60 × 100 = 6000/75 = 80
the unknown value is 80

Question 9.
27 is 30% of _____.
_____

Answer:
90

Explanation:
30/100 ÷ 30/30 = 3/10
3/10 × 9/9 = 27/90
the unknown value is 90

Question 10.
21 is 60% of _____.
_____

Answer:
35

Explanation:
60/100 ÷ 60/60 = 3/5
3/5 × 7/7 = 21/35
the unknown value is 35

Question 11.
12 is 15% of _____.
_____

Answer:
80

Explanation:
15/100 ÷ 15/15 = 3/20
3/20 × 4/4 = 12/80
the unknown value is 80

Solve.

Question 12.
40% of the students in the sixth grade at Andrew’s school participate in sports. If 52 students participate in sports, how many sixth graders are there at Andrew’s school?
_____ students

Answer:
130 students

Explanation:
52/s = 40%
52/s = 40/100
s = 40/100 × 52 = 130

Question 13.
There were 136 students and 34 adults at the concert. If 85% of the seats were filled, how many seats are in the auditorium?
_____ seats

Answer:
80 seats

Explanation:
There are 170 seats filled total. 170 is 85% of 200. There are 200 seats in the auditorium.
If you were to solve for x in the equation 40% = 32/x, you would get x = 80.

Use Reasoning Algebra Find the unknown value.

Question 14.
40% = \(\frac{32}{?}\)
_____

Answer:
80

Explanation:
40/100 = 32/?
40/100 ÷ 40/40 = 2/5
2/5 × 16/16 = 32/80
the unknown value is 80

Question 15.
65% = \(\frac{91}{?}\)
_____

Answer:
140

Explanation:
65/100 = 91/?
65/100 ÷ 65/65 = 13/20
13/20 × 7/7 = 91/140
the unknown value is 140

Question 16.
45% = \(\frac{54}{?}\)
_____

Answer:
120

Explanation:
45/100 ÷ 45/45 = 9/20
9/20 × 6/6 = 54/120

Problem Solving + Applications – Page No. 304

Use the advertisement for 17 and 18.
Go Math Grade 6 Answer Key Chapter 5 Model Percents img 33

Question 17.
Corey spent 20% of his savings on a printer at Louie’s Electronics. How much did Corey have in his savings account before he bought the printer?
$ _____

Answer:
$800

Explanation:
(printer cost) = 0.20 * (savings)
(printer cost)/0.20 = (savings)
savings = 5*(printer cost)
Corey’s savings was 5 times that amount.
savings = 5 × 160 = 800

Question 18.
Kai spent 90% of his money on a laptop that cost $423. Does he have enough money left to buy a scanner? Explain.
Type below:
_____________

Answer:
$42.3

Explanation:
He spent 90% of his money. So, he left 10% of money with him.
423 × 10/100 = 42.3 left to buy a scanner

Question 19.
Maurice has completed 17 pages of the research paper he is writing. That is 85% of the required length of the paper. What is the required length of the paper?
_____ pages

Answer:
20 pages

Explanation:
Maurice has completed 17 pages of the research paper he is writing. That is 85% of the required length of the paper.
85%=17 ? what about 100%
100multiplied by 17 divided by 85% =20

Question 20.
Of 250 seventh-grade students, 175 walk to school. What percent of seventh-graders do not walk to school?
_____ %

Answer:
30%

Explanation:
it’s either 30 percent or 70. 70 percent walks to school and 30 percent DO NOT walk to school

Question 21.
What’s the Error? Kate has made 20 free throws in basketball games this year. That is 80% of the free throws she has attempted. To find the total number of free throws she attempted, Kate wrote the equation \(\frac{80}{100}=\frac{?}{20}\). What error did Kate make?
Type below:
_____________

Answer:
20 free throws is 80% of the total attempted
80% to decimal is:
80/100 = 0.8
If total attempted is x, we can say:
20 is 80% (0.8) of x
We can now write an algebraic equation:
20 = 0.8x
We simply solve this for x, that is the number of free throws she attempted:
20 = 0.8x
x = 20/0.8 = 25

Question 22.
Maria spent 36% of her savings to buy a smart phone. The phone cost $90. How much money was in Maria’s savings account before she purchased the phone? Find the unknown value.
$ _____

Answer:
$ 250

Explanation:
let her savings be A
A/Q-
36% of A = $90
36/100 of A = $90
A = 90×100/36
A= $ 250

Find the Whole from a Percent – Page No. 305

Find the Whole from a Percent

Question 1.
9 is 15% of _____.
_____

Answer:
60

Explanation:
15/100 ÷ 15/15 = 3/20
3/20 × 3/3 = 9/60
the unknown value is 60

Question 2.
54 is 75% of _____.
_____

Answer:
72

Explanation:
75/100 ÷ 75/75 = 3/4
3/4 × 18/18 = 54/72
the unknown value is 72

Question 3.
12 is 2% of _____.
_____

Answer:
600

Explanation:
2/100 = 1/50
1/50 × 12/12 = 12/600
the unknown value is 600

Question 4.
18 is 50% of _____.

Answer:
36

Explanation:
50/100 = 1/2
1/2 × 18/18 = 18/36
the unknown value is 36

Question 5.
16 is 40% of _____.
_____

Answer:
40

Explanation:
40/100 = 2/5
2/5 × 8/8 = 16/40
the unknown value is 40

Question 6.
56 is 28% of _____.
_____

Answer:
200

Explanation:
28/100 = 14/50 = 7/25
7/25 × 8/8 = 56/200
the unknown value is 200

Question 7.
5 is 10% of _____.
_____

Answer:
50

Explanation:
10/100 = 1/10
1/10 × 5/5 = 5/50
the unknown value is 50

Question 8.
24 is 16% of _____.
_____

Answer:
150

Explanation:
16/100 = 4/25
4/25 × 6/6 = 24/150
the unknown value is 150

Question 9.
15 is 25% of _____.
_____

Answer:
60

Explanation:
25/100 = 1/4
1/4 × 15/15 = 15/60
the unknown value is 60

Problem Solving

Question 10.
Michaela is hiking on a weekend camping trip. She has walked 6 miles so far. This is 30% of the total distance. What is the total number of miles she will walk?
_____ miles

Answer:
20 miles

Explanation:
Since 6mi=30%,
You should find ten percent.
This is how, divide both sides by 3, and this gives you
2m=10% (2m being 2 miles)
So, to find 100%, you need to multiply both sides by 10
20m=100%
So now, Michaela will walk 20 miles this weekend

Question 11.
A customer placed an order with a bakery for muffins. The baker has completed 37.5% of the order after baking 81 muffins. How many muffins did the customer order?
_____ muffins

Answer:
216 muffins

Explanation:
A customer placed an order with a bakery for muffins. The baker has completed 37.5% of the order after baking 81 muffins.
37.5/100=0.375 and 81/0.375=216
so the answer is 216

Question 12.
Write a question that involves finding what number is 25% of another number. Solve using a double number line and check using equivalent ratios. Compare the methods.
Type below:
_____________

Answer:
25% of 15 = 25/100 × 15 = 375/100 = 3.75

Lesson Check – Page No. 306

Question 1.
Kareem saves his coins in a jar. 30% of the coins are pennies. If there are 24 pennies in the jar, how many coins does Kareem have?
_____ coins

Answer:
80 coins

Explanation:
24=30%
find 100%
24=30%
diivde by 3
8=10%
multiply 10
80=100%
80 coins

Question 2.
A guitar shop has 19 acoustic guitars on display. This is 19% of the total number of guitars. What is the total number of guitars the shop has?
_____ guitars

Answer:
100 guitars

Explanation:
Let’s find out how much 1% is worth first.
19 guitars = 19%
therefore 19 ÷ 19 = [ 1 guitar = 1% ]
The total number of guitars is going to be 100%,
so if 1% × 100 = 100%, then 1 guitar × 100 = 100 guitars total.

Spiral Review

Question 3.
On a coordinate grid, in which quadrant is the point (−5, 4) located?
Type below:
_____________

Answer:
Quadrant II

Explanation:
(-5, 4)
-5 is the negative point of the x coordinate
4 is the positive point of the y coordinate
Quadrant II

Question 4.
A box contains 16 cherry fruit chews, 15 peach fruit chews, and 12 plum fruit chews. Which two flavors are in the ratio 5 to 4?
Type below:
_____________

Answer:
peach fruit chews and plum fruit chews are in the ratio 5 to 4

Explanation:
15 peach fruit chews, and 12 plum fruit chews
15/12 = 5/4

Question 5.
During basketball season, Marisol made \(\frac{19}{25}\) of her free throws. What percent of her free throws did Marisol make?
_____ %

Answer:
76%

Explanation:
During the basketball season, Marisol made \(\frac{19}{25}\) of her free throws.
(19 ÷ 25) × 100 = 76%. Marisol made 76% of her free throws.

Question 6.
Landon is entering the science fair. He has a budget of $115. He has spent 20% of the money on new materials. How much does Landon have left to spend?
$ _____

Answer:
$92

Explanation:
Landon has $92 left because if you divide 115/.20 you get 23 and then you subtract 115-23=92 or $92.

Chapter 5 Review/Test – Page No. 307

Question 1.
What percent is represented by the shaded part?
Go Math Grade 6 Answer Key Chapter 5 Model Percents img 34
Options:
a. 46%
b. 60%
c. 64%
d. 640%

Answer:
c. 64%

Explanation:
64 squares are shaded out of 100.
So, 64% and 64/100 are the answers.

Question 2.
Write a percent to represent the shaded part.
Go Math Grade 6 Answer Key Chapter 5 Model Percents img 35
_____ %

Answer:
42%

Explanation:
42 squares are shaded out of 100.
So, 42% and 42/100 are the answers.

Question 3.
Rosa made a mosaic wall mural using 42 black tiles, 35 blue tiles and 23 red tiles. Write a percent to represent the number of red tiles in the mural.
_____ %

Answer:
23%

Explanation:
42+35+23= 100
So plug it in.
23/100
23%
Your answer is 23%.

Question 4.
Model 39%.
Type below:
_____________

Answer:
grade 6 chapter 5 image 10

Explanation:
39 squares out of 100 need to shaded

Page No. 308

Question 5.
For 5a–5d, choose Yes or No to indicate whether the percent and the fraction represent the same amount.
5a. 50% and \(\frac{1}{2}\)
5b. 45% and \(\frac{4}{5}\)
5c. \(\frac{3}{8}\) and 37.5%
5d. \(\frac{2}{10}\) and 210%
5a. _____________
5b. _____________
5c. _____________
5d. _____________

Answer:
5a. Yes
5b. No
5c. Yes
5d. No

Explanation:
1/2 = 0.5 × 100/100 = 50/100 = 50%
4/5 = 0.8 × 100/100 = 80/100 = 80%
3/8 = 0.375 × 100/100 = 37.5/100 = 37.5%
2/10 = 0.2 × 100/100 = 20/100 = 20%

Question 6.
The school orchestra has 25 woodwind instruments, 15 percussion instruments, 30 string instruments, and 30 brass instruments. Select the portion of the instruments that are percussion. Mark all that apply.
Options:
a. 15%
b. 1.5
c. \(\frac{3}{20}\)
d. 0.15

Answer:
a. 15%
c. \(\frac{3}{20}\)
d. 0.15

Explanation:
25 + 15 + 30 + 30 = 100
15 percussion instruments = 15/100 = 15% = 0.15

Question 7.
For a science project, \(\frac{3}{4}\) of the students chose to make a poster and 0.25 of the students wrote a report. Rosa said that more students made a poster than wrote a report. Do you agree with Rosa? Use numbers and words to support your answer
Type below:
_____________

Answer:
Yes, because 3/4 is equal to 0.75 and 0.75 > 0.25
Or 0.25 is equal to 1/4, and 1/4 < 3/4

Question 8.
Select other ways to write 0.875. Mark all that apply.
Options:
a. 875%
b. 87.5%
c. \(\frac{7}{8}\)
d. \(\frac{875}{100}\)

Answer:
c. \(\frac{7}{8}\)

Explanation:
0.875 = 8.75/100 = 8.75%

Page No. 309

Question 9.
There are 88 marbles in a bin and 25% of the marbles are red.
There are _____________ red marbles in the bin.

Answer:
There are 22 red marbles in the bin.

Explanation:
88 × 25% = 88 × 25/100 = 22

Question 10.
Harrison has 30 CDs in his music collection. If 40% of the CDs are country music and 30% are pop music, how many CDs are other types of music?
_____ CDs

Answer:
9 CDs

Explanation:
Harrison has 30 CDs in his music collection. If 40% of the CDs are country music and 30% are pop music,
40 + 30 = 70
100 – 70 = 30%
30 × 30/100 = 9

Question 11.
For numbers 11a–11b, choose <, >, or =.
11a. 30% of 90 Ο 35% of 80
11b. 25% of 16 Ο 20% of 25
30% of 90 _____ 35% of 80
25% of 16 _____ 20% of 25

Answer:
30% of 90 < 35% of 80
25% of 16 < 20% of 25

Explanation:
30% of 90 = 30/100 × 90 = 27
35% of 80 = 35/100 × 80 = 28
30% of 90 < 35% of 80
25% of 16 = 25/100 × 16 = 4
20% of 25 = 20/100 × 25 = 5
25% of 16 < 20% of 25

Question 12.
There were 200 people who voted at the town council meeting. Of these people, 40% voted for building a new basketball court in the park. How many people voted against building the new basketball court? Use numbers and words to explain your answer.
Type below:
_____________

Answer:
There were 200 people who voted at the town council meeting. Of these people, 40% voted for building a new basketball court in the park.
100 – 40% = 60%
200 × 60/100 = 120 people

Page No. 310

Question 13.
James and Sarah went out to lunch. The price of lunch for both of them was $20. They tipped their server 20% of that amount. How much did each person pay if they shared the price of lunch and the tip equally?
$ _____

Answer:
$12

Explanation:
James and Sarah went out to lunch. The price of lunch for both of them was $20. They tipped their server 20% of that amount.
20% of 20 = 20/100 × 20 = 4
20 + 4 = 24
24/2 = 12
$12

Question 14.
A sandwich shop has 30 stores and 60% of the stores are in California. The rest of the stores are in Nevada.
Part A
How many stores are in California and how many are in Nevada?
Type below:
_____________

Answer:
30 × 60/100 = 18 stores in California
30 – 18 = 12 stores in Nevada

Question 14.
Part B
The shop opens 10 new stores. Some are in California, and some are in Nevada. Complete the table.
Go Math Grade 6 Answer Key Chapter 5 Model Percents img 36
Type below:
_____________

Answer:
grade 6 chapter 5 image 11

Explanation:
100 – 45 = 55%
55% of 40 = 55/100 × 40 = 22
45% of 40 = 45/100 × 40 = 18

Question 15.
Juanita has saved 35% of the money that she needs to buy a new bicycle. If she has saved $63, how much money does the bicycle cost? Use numbers and words to explain your answer
$ _____

Answer:
$180

Explanation:
Juanita has saved 35% of the money that she needs to buy a new bicycle. If she has saved $63,
35/100 = 7/20
7/20 × 9/9 = 63/180
The bicycle cost is $180

Page No. 311

Question 16.
For 16a–16d, choose Yes or No to indicate whether the statement is correct.
16a. 12 is 20% of 60.
16b. 24 is 50% of 48.
16c. 14 is 75% of 20.
16d. 9 is 30% of 30.
16a. _____________
16b. _____________
16c. _____________
16d. _____________

Answer:
16a. Yes
16b. Yes
16c. No
16d. Yes

Explanation:
20% of 60 = 20/100 × 60 = 12
50% of 48 = 50/100 × 48 = 24
75% of 20 = 75/100 × 20 = 15
30% of 30 = 30/100 × 30 = 9

Question 17.
Heather and her family are going to the grand opening of a new amusement park. There is a special price on tickets this weekend. Tickets cost $56 each. This is 70% of the cost of a regular price ticket
Part A
What is the cost of a regular price ticket? Show your work.
$ _____

Answer:
$80

Explanation:
70/100 = 56/s
s = 56 × 100/70 = 80

Question 17.
Part B
Heather’s mom says that they would save more than $100 if they buy 4 tickets for their family on opening weekend. Do you agree or disagree with Heather’s mom? Use numbers and words to support your answer. If her statement is incorrect, explain the correct way to solve it.
Type below:
_____________

Answer:
80 × 4 = 320
56 × 4 = 224
320 – 224 = 96
$96

Question 18.
Elise said that 0.2 equals 2%. Use words and numbers to explain her mistake.
Type below:
_____________

Answer:
0.2 × 100/100 = 20/100 = 2%

Page No. 312

Question 19.
Write 18% as a fraction.
\(\frac{□}{□}\)

Answer:
\(\frac{9}{50}\)

Explanation:
18% = 18/100 = 9/50

Question 20.
Noah wants to put a variety of fish in his new fish tank. His tank is large enough to hold a maximum of 70 fish.
Part A
Complete the table.
Go Math Grade 6 Answer Key Chapter 5 Model Percents img 37
Type below:
_____________

Answer:
grade 6 chapter 5 image 12

Explanation:
70 × 20/100 = 14
70 × 40/100 = 28
70 × 30/100 = 21

Question 20.
Part B
Has Noah put the maximum number of fish in his tank? Use numbers and words to explain how you know. If he has not put the maximum number of fish in the tank, how many more fish could he put in the tank?
Type below:
_____________

Answer:
No, since 20% + 40% + 30% = 90%, he can add 10% in the tank.

Conclusion:

Test your knowledge by solving the problems from Go Math Grade 6 Answer Key Chapter 5 Model Percents. Get the solutions for Mid Chapter Checkpoint and Review Test along with the exercise problems in Go Math Grade 6 Chapter 5 Model Percents Solution Key. Quick learning and best practice come in a single hand with our Go Math Grade 6 Solution Key Chapter 5 Model Percents @ ccssmathanswers.com

Go Math Grade 8 Answer Key Chapter 8 Solving Systems of Linear Equations

go-math-grade-8-chapter-8-solving-systems-of-linear-equations-answer-key

Students of Grade 8 can get a detailed explanation for all the questions in Go Math Answer Key Chapter 8 Solving Systems of Linear Equations. In addition to the exercise problems we also provide the solutions for the review test. So, go through all the answers and explanations provided by the math experts in Go Math Grade 8 Chapter 8 Solving Systems of Linear Equations Answer Key. Our aim is to provide easy and simple tricks to solve the problems in Go Math Grade 8 Answer Key Chapter 8 Solving Systems of Linear Equations.

Download Go Math Grade 8 Chapter 8 Solving Systems of Linear Equations Answer Key Pdf

Students who are interested to secure the highest marks in the exams are suggested to download the Go Math Grade 8 Answer Key Chapter 8 Solving Systems of Linear Equations pdf. All the solutions are provided in the pdf format as per the list of the chapters provided in the latest edition. Hence refer to Go Math 8th Grade Solution Key to learning the easy way of maths practice. Check the list of the topics covered in Chapter 8 Solving Systems of Linear Equations from the following section.

Lesson 1: Solving Systems of Linear Equations by Graphing

Lesson 2: Solving Systems by Substitution

Lesson 3: Solving Systems by Elimination

Lesson 4: Solving Systems by Elimination with Multiplication

Lesson 5: Solving Solving Special Systems

Model Quiz

Review

Guided Practice – Solving Systems of Linear Equations by Graphing – Page No. 232

Solve each system by graphing.

Question 1.
\(\left\{\begin{array}{l}y=3 x-4 \\y=x+2\end{array}\right.\)
Go Math Grade 8 Answer Key Chapter 8 Solving Systems of Linear Equations Lesson1: Solving Systems of Linear Equations by Graphing img 1
Type below:
______________

Answer:
grade 8 chapter 8 image 1

Explanation:
y = 3x – 4
y = x + 2
The solution of thr linear system of equations is the intersection point of the two equations.
(3, 5) is the solution of the system of equations.
If x = 3, y = 3(3) – 4 = 9 – 4 = 5; y = 3 + 2 = 5
5 = 5; True

Question 2.
\(\left\{\begin{array}{l}x-3 y=2 \\-3x+9y=-6\end{array}\right.\)
Go Math Grade 8 Answer Key Chapter 8 Solving Systems of Linear Equations Lesson1: Solving Systems of Linear Equations by Graphing img 2
Type below:
______________

Answer:
grade 8 chapter 8 image 2
Infinitely many solutions

Explanation:
x – 3y = 2
-3x + 9y = -6
x – 3y – x = -x + 2
-3y = -x + 2
y = 1/3 . x – 2/3
-3x + 9y + 3x = 3x – 6
9y = 3x – 6
y = 3/9 . x – 6/9
y = 1/3 . x – 2/3
The solution of the linear system of equations is the intersection of the two equations.
Infinitely many solutions

Question 3.
Mrs. Morales wrote a test with 15 questions covering spelling and vocabulary. Spelling questions (x) are worth 5 points and vocabulary questions (y) are worth 10 points. The maximum number of points possible on the test is 100.
a. Write an equation in slope-intercept form to represent the number of questions on the test.
Go Math Grade 8 Answer Key Chapter 8 Solving Systems of Linear Equations Lesson1: Solving Systems of Linear Equations by Graphing img 3
Type below:
______________

Answer:
y = -x + 15

Explanation:
Mrs. Morales wrote a test with 15 questions covering spelling and vocabulary. Spelling questions (x) are worth 5 points and vocabulary questions (y) are worth 10 points.
x + y = 15
x + y – x = -x + 15
y = -x + 15

Question 3.
b. Write an equation in slope-intercept form to represent the total number of points on the test.
Type below:
______________

Answer:
y = -1/2 . x + 10

Explanation:
The total number of points on test is 100
5x + 10y = 100
5x + 10y – 5x = -5x + 100
10y = -5x + 100
y = -5/10 . x + 100/10
y = -1/2 . x + 10

Question 3.
c. Graph the solutions of both equations.
Type below:
______________

Answer:
grade 8 chapter 8 image 3

Question 3.
d. Use your graph to tell how many of each question type are on the test.
_________ spelling questions
_________ vocabulary questions

Answer:
10 spelling questions
5 vocabulary questions

ESSENTIAL QUESTION CHECK-IN

Question 4.
When you graph a system of linear equations, why does the intersection of the two lines represent the solution of the system?
Type below:
______________

Answer:
To solve a system of linear equations means finding the solutions that satisfy all the equations of that system. When we graph a system of linear equations, the intersection point lies on the line of each equation, which means that satisfies all the equations. Therefore, it is considered to be the solution to that system.

Solving Systems of Linear Equations by Graphing – Page No. 233

Question 5.
Vocabulary
A_________________ is a set of equations that have the same variables.
______________

Answer:
system of equations

Explanation:
A system of equations is a set of equations that have the same variables.

Question 6.
Eight friends started a business. They will wear either a baseball cap or a shirt imprinted with their logo while working. They want to spend exactly $36 on the shirts and caps. Shirts cost $6 each and caps cost $3 each.
a. Write a system of equations to describe the situation. Let x represent the number of shirts and let y represent the number of caps.
______________

Answer:
6x + 3y = 36

Explanation:
The sum of caps and shirts is 8. The total cost of caps and shirts is $36.
x + y = 8
6x + 3y = 36

Question 6.
b. Graph the system. What is the solution and what does it represent?
Go Math Grade 8 Answer Key Chapter 8 Solving Systems of Linear Equations Lesson1: Solving Systems of Linear Equations by Graphing img 4
Type below:
______________

Answer:
The solution is (4, 4)
grade 8 chapter 8 image 4

Explanation:
x + y – x = -x + 8
y = -x + 8
6x + 3y – 6x = -6x + 36
3y = -6x + 36
y = -6/2 . x + 36/3
y = -2x + 12
(4, 4). They should order 4 shirts and 4 caps.

Question 7.
Multistep The table shows the cost for bowling at two bowling alleys.
Go Math Grade 8 Answer Key Chapter 8 Solving Systems of Linear Equations Lesson1: Solving Systems of Linear Equations by Graphing img 5
a. Write a system of equations, with one equation describing the cost to bowl at Bowl-o-Rama and the other describing the cost to bowl at Bowling Pinz. For each equation, let x represent the number of games played and let y represent the total cost.
Type below:
______________

Answer:
y = 2.5x + 2
y = 2x + 4

Explanation:
Cost at Bowl-o-Rama => y = 2.5x + 2
Cost at Bowling Pinz => y = 2x + 4

Question 7.
b. Graph the system. What is the solution and what does it represent?
Go Math Grade 8 Answer Key Chapter 8 Solving Systems of Linear Equations Lesson1: Solving Systems of Linear Equations by Graphing img 6
Type below:
______________

Answer:
grade 8 chapter 8 image 5

Explanation:
The solution of the linear system of equations is the intersection of the two equations.
(4, 12)
When 4 games are played, the total cost is $12.

Solving Systems of Linear Equations by Graphing – Page No. 234

Question 8.
Multi-Step Jeremy runs 7 miles per week and increases his distance by 1 mile each week. Tony runs 3 miles per week and increases his distance by 2 miles each week. In how many weeks will Jeremy and Tony be running the same distance? What will that distance be?
Type below:
______________

Answer:
After 4 weeks Jeremy and Tony will be running the same distance and that distance would be 11 miles.

Explanation:
Multi-Step Jeremy runs 7 miles per week and increases his distance by 1 mile each week.
y = x + 7
Tony runs 3 miles per week and increases his distance by 2 miles each week.
y = 2x + 3
grade 8 chapter 8 image 6
The solution of the system of linear equation is (4, 11) which means that after 4 weeks Jeremy and Tony will be running the same distance and that distance would be 11 miles.

Question 9.
Critical Thinking Write a real-world situation that could be represented by the system of equations shown below.
\(\left\{\begin{array}{l}y=4 x+10 \\y=3x+15\end{array}\right.\)
Type below:
______________

Answer:
The entry fee of the first gym is $10 and for every hour that you spend there, you pay an extra $4. If we denote with x the number of hours that somebody spends at the gym and with y the total cost is
y = 4x + 10
The entry fee of the second gym is $15 and for every hour that you spend there, you pay an extra $3. If we denote with x the number of hours that somebody spends at the gym and with y the total cost is
y = 3x + 15
y = 4x + 10
y = 3x + 15

FOCUS ON HIGHER ORDER THINKING

Question 10.
Multistep The table shows two options provided by a high-speed Internet provider.
Go Math Grade 8 Answer Key Chapter 8 Solving Systems of Linear Equations Lesson1: Solving Systems of Linear Equations by Graphing img 7
a. In how many months will the total cost of both options be the same? What will that cost be?
________ months
$ ________

Answer:
5 months
$ 200

Explanation:
Let y be the total cost after x month
y = 30x + 50
Let y be the total cost after x month
y = 40x
Substitute y = 40x in y = 30x + 50
40x = 30x + 50
40x – 30x = 50
10x = 50
x = 50/10
x = 5
The total cost of both options will be the same after 5 months. Total cost would be y = 40(5) = $200.

Question 10.
b. If you plan to cancel your Internet service after 9 months, which is the cheaper option? Explain.
______________

Answer:
When x = 9 months
y = 30(9) + 50 = $320
y = 40(9) = $360
$320 < $360
Option 1 is cheaper as the total cost is less for option 1

Question 11.
Draw Conclusions How many solutions does the system formed by x − y = 3 and ay − ax + 3a = 0 have for a nonzero number a? Explain.
Type below:
______________

Answer:
x – y = 3
ay – ax + 3a =0
ay – ax + 3a – 3a = 0 – 3a
ay – ax = – 3a
a(y – x) = -3a
y – x = -3
x – y = 3
Both equations are the same. The system of linear equations have infinitely many solutions.

Guided Practice – Solving Systems by Substitution – Page No. 240

Solve each system of linear equations by substitution.

Question 1.
\(\left\{\begin{array}{l}3x-2y=9 \\y=2x-7\end{array}\right.\)
x = ________
y = ________

Answer:
x = 5
y = 3

Explanation:
\(\left\{\begin{array}{l}3x-2y=9 \\y=2x-7\end{array}\right.\)
Substitute 2x – 7 in 3x – 2y = 9
3x – 2(2x – 7) = 9
3x – 4x + 14 = 9
-x + 14 = 9
-x + 14 – 14 = 9 – 14
-x = -5
x = -5/-1 = 5
y = 2(5) – 7 = 3
Solution is (5, 3)

Question 2.
\(\left\{\begin{array}{l}y=x-4 \\2x+y=5\end{array}\right.\)
x = ________
y = ________

Answer:
x = 3
y = -1

Explanation:
\(\left\{\begin{array}{l}y=x-4 \\2x+y=5\end{array}\right.\)
2x + x – 4 = 5
3x – 4 = 5
3x – 4 + 4 = 5 + 4
3x = 9
x = 9/3 = 3
y = 3 – 4 = -1
The solution is (3, -1)

Question 3.
\(\left\{\begin{array}{l}x+4y=6 \\y=-x+3\end{array}\right.\)
x = ________
y = ________

Answer:
x = 2
y = 1

Explanation:
\(\left\{\begin{array}{l}x+4y=6 \\y=-x+3\end{array}\right.\)
Substitute y = -x + 3 in x + 4y = 6
x + 4(-x + 3) = 6
x – 4x + 12 = 6
-3x + 12 = 6
-3x + 12 – 12 = 6 – 12
-3x = -6
x = -6/-3 = 2
y = -2 + 3 = 1
The solution is (2, 1)

Question 4.
\(\left\{\begin{array}{l}x+2y=6 \\x-y=3\end{array}\right.\)
x = ________
y = ________

Answer:
x = 4
y = 1

Explanation:
\(\left\{\begin{array}{l}x+2y=6 \\x-y=3\end{array}\right.\)
y = x – 3
Substitute y = x – 3 in x + 2y = 6
x + 2(x – 3) = 6
x + 2x – 6 = 6
3x = 12
x = 12/3
x = 4
4 – y = 3
-y = 3 – 4
-y = -1
y = 1
The solution is (4, 1)

Solve each system. Estimate the solution first.

Question 5.
\(\left\{\begin{array}{l}6x+y=4 \\x-4y=19\end{array}\right.\)
Estimate ______________
Solution ______________
Type below:
______________

Answer:
Estimate (2, -5)
Solution (1.4, -4.4)

Explanation:
\(\left\{\begin{array}{l}6x+y=4 \\x-4y=19\end{array}\right.\)
Let’s find the estimation by graphing the equations
Estimate: (2, -5)
grade 8 chapter 8 image 7
x = 4y + 19
6(4y + 19) + y = 4
24y + 114 + y = 4
25y + 114 = 4
25y = 4 – 114
25y = -110
y = -110/25
y = -4.4
x + 4(-4.4) = 19
x + 17.6 = 19
x = 19 – 17.6
x = 1.4
The solution is (1.4, -4.4)

Question 6.
\(\left\{\begin{array}{l}x+2y=8 \\3x+2y=6\end{array}\right.\)
Estimate ______________
Solution ______________
Type below:
______________

Answer:
Estimate (-1, 5)
Solution (-1, 4.5)

Explanation:
\(\left\{\begin{array}{l}x+2y=8 \\3x+2y=6\end{array}\right.\)
Let’s find the estimation by graphing the equations
Estimate: (-1, 5)
grade 8 chapter 8 image 8
x = -2y + 8
Substitute the equation x = -2y + 8 in 3x + 2y = 6
3(-2y + 8) + 2y = 6
-6y + 24 + 2y = 6
-4y = 6 – 24
-4y = -18
y = -18/-4
y = 4.5
x + 2(4.5) = 8
x + 9 = 8
x = 8 – 9
x = -1
The solution is (-1, 4.5)

Question 7.
\(\left\{\begin{array}{l}3x+y=4 \\5x-y=22\end{array}\right.\)
Estimate ______________
Solution ______________
Type below:
______________

Answer:
Estimate (3, -6)
Solution (3.25, -5.75)

Explanation:
\(\left\{\begin{array}{l}3x+y=4 \\5x-y=22\end{array}\right.\)
Find the Estimation using graphing the equations.
Estimate: (3, -6)
grade 8 chapter 8 image 9
y = -3x + 4
Substitute y = -3x + 4 in 5x – y = 22
5x – (-3x + 4) = 22
5x + 3x -4 = 22
8x = 26
x = 26/8
x = 3.25
3(3.25) + y = 4
9.75 + y = 4
y = 4 – 9.75
y = -5.75
The solution is (3.25, -5.75)

Question 8.
\(\left\{\begin{array}{l}2x+7y=2 \\x+y=-1\end{array}\right.\)
Estimate ______________
Solution ______________
Type below:
______________

Answer:
Estimate (-2, 1)
Solution (-1.8, 0.8)

Explanation:
\(\left\{\begin{array}{l}2x+7y=2 \\x+y=-1\end{array}\right.\)
Find the Estimation using graphing the equations.
Estimate: (-2, 1)
grade 8 chapter 8 image 10
y = -x -1
Substitute y = -x – 1 in 2x + 7y = 2
2x + 7(-x – 1) = 2
2x – 7x -7 = 2
-5x = 2 + 7
-5x = 9
x = -9/5
x = -1.8
-1.8 + y = -1
y = -1 + 1.8
y = 0.8
The solution is (-1.8, 0.8)

Question 9.
Adult tickets to Space City amusement park cost x dollars. Children’s tickets cost y dollars. The Henson family bought 3 adult and 1 child tickets for $163. The Garcia family bought 2 adult and 3 child tickets for $174.
a. Write equations to represent the Hensons’ cost and the Garcias’ cost.
Hensons’ cost: ________________
Garcias’ cost:__________________
Type below:
______________

Answer:
Hensons’ cost: 3x + y = 163
Garcias’ cost: 2x + 3y = 174

Explanation:
Henson’s cost
3x + y = 163
Garcia’s cost
2x + 3y = 174

Question 9.
b. Solve the system.
adult ticket price: $ _________
Garcias’ cost: $ _________

Answer:
adult ticket price: $ 45
Garcias’ cost: $ 28

Explanation:
y = -3x + 163
Substitute y = -3x + 163 in 2x + 3y = 174
2x + 3(-3x + 163) = 174
2x -9x + 489 = 174
-7x = -315
x = -315/-7 = 45
3(45) + y = 163
135 + y = 163
y = 163 – 135
y = 28
adult ticket price: $ 45
Garcias’ cost: $ 28

ESSENTIAL QUESTION CHECK-IN

Question 10.
How can you decide which variable to solve for first when you are solving a linear system by substitution?
Type below:
______________

Answer:
The variable with the unit coefficient should be solved first when solving a linear system by substitution.

8.2 Independent Practice – Solving Systems by Substitution – Page No. 241

Question 11.
Check for Reasonableness Zach solves the system
\(\left\{\begin{array}{l}x+y=-3 \\x-y=1\end{array}\right.\)
and finds the solution (1, -2). Use a graph to explain whether Zach’s solution is reasonable.
Go Math Grade 8 Answer Key Chapter 8 Solving Systems of Linear Equations Lesson 2: Solving Systems by Substitution img 8
Type below:
______________

Answer:
grade 8 chapter 8 image 11

Explanation:
\(\left\{\begin{array}{l}x+y=-3 \\x-y=1\end{array}\right.\)
The x coordinate of the solution is negative, hence Zach’s solution is not reasonable.

Represent Real-World Problems Angelo bought apples and bananas at the fruit stand. He bought 20 pieces of fruit and spent $11.50. Apples cost $0.50 and bananas cost $0.75 each.
a. Write a system of equations to model the problem. (Hint: One equation will represent the number of pieces of fruit. A second equation will represent the money spent on the fruit.)
Go Math Grade 8 Answer Key Chapter 8 Solving Systems of Linear Equations Lesson 2: Solving Systems by Substitution img 9
Type below:
______________

Answer:
x + y = 20
0.5x + 0.75y = 11.5

Explanation:
x + y = 20
0.5x + 0.75y = 11.5
where c is the number of Apples and y is the number of Bananas.

Question 12.
b. Solve the system algebraically. Tell how many apples and bananas Angelo bought.
________ apples
________ bananas

Answer:
14 apples
6 bananas

Explanation:
y = -x + 20
Substitute y = -x + 20 in 0.5x + 0.75y = 11.5
0.5x + 0.75(-x + 20) = 11.5
0.5x – 0.75x + 15 = 11.5
-0.25x + 15 = 11.5
-0.25x = 11.5 – 15
-0.25x = -3.5
x = -3.5/-0.25
x = 14
14 + y = 20
y = 6
Angelo bought 14 apples and 6 bananas.

Question 13.
Represent Real-World Problems A jar contains n nickels and d dimes. There is a total of 200 coins in the jar. The value of the coins is $14.00. How many nickels and how many dimes are in the jar?
________ nickels
________ dimes

Answer:
120 nickels
80 dimes

Explanation:
A jar contains n nickels and d dimes. There is a total of 200 coins in the jar. The value of the coins is $14.00.
$14 = 1400 cents
n + d = 200
5n + 10d = 1400
d = -n + 200
5n + 10(-n + 200) = 1400
5n – 10n + 2000 = 1400
-5n = -600
n = -600/-5
n = 120
120 + d = 200
d = 200 – 120
d = 80
There are 120 nickles and 80 dimes in the jar.

Question 14.
Multistep The graph shows a triangle formed by the x-axis, the line 3x−2y=0, and the line x+2y=10. Follow these steps to find the area of the triangle.
a. Find the coordinates of point A by solving the system
\(\left\{\begin{array}{l}3x-2y=0 \\x-2y=10\end{array}\right.\)
Point A: ____________________
Go Math Grade 8 Answer Key Chapter 8 Solving Systems of Linear Equations Lesson 2: Solving Systems by Substitution img 10
Type below:
______________

Answer:
Point A: (2.5, 3.75)Coordinate of A is (2.5, 3.75)

Explanation:
\(\left\{\begin{array}{l}3x-2y=0 \\x-2y=10\end{array}\right.\)
x = -2y + 10
Substitute x = -2y + 10 in 3x – 2y = 0
3(-2y + 10) -2y = 0
-6y + 30 – 2y = 0
-8y = -30
y = -30/-8 = 3.75
x + 2(3.75) = 10
x + 7.5 = 10
x = 10 – 7.5
x = 2.5
Coordinate of A is (2.5, 3.75)

Question 14.
b. Use the coordinates of point A to find the height of the triangle.
height:__________________
height: \(\frac{□}{□}\) units

Answer:
height: 3.75
height: \(\frac{15}{4}\) units

Explanation:
Height of the triangle is the y coordinate of A
Height = 3.75

Question 14.
c. What is the length of the base of the triangle?
base:________________
base: ______ units

Answer:
base: 10 units

Explanation:
Length of the base = 10

Question 14.
d. What is the area of the triangle?
A = ______ \(\frac{□}{□}\) square units

Answer:
A = 18.75 square units
A = 18 \(\frac{3}{4}\) square units

Explanation:
Area of the triangle = 1/2 . Height . Base
Area = 1/2 . 3.75 . 10 = 18.75

Solving Systems by Substitution – Page No. 242

Question 15.
Jed is graphing the design for a kite on a coordinate grid. The four vertices of the kite are at A(−\(\frac{4}{3}\), \(\frac{2}{3}\)), B(\(\frac{14}{3}\), −\(\frac{4}{3}\)), C(\(\frac{14}{3}\), −\(\frac{16}{3}\)), and D(\(\frac{2}{3}\), −\(\frac{16}{3}\)). One kite strut will connect points A and C. The other will connect points B and D. Find the point where the struts cross.
Go Math Grade 8 Answer Key Chapter 8 Solving Systems of Linear Equations Lesson 2: Solving Systems by Substitution img 11
Type below:
______________

Answer:
The struts cross as (8/3, 10/3)

Explanation:
1. From AC
Slope = (y2 – y1)/(x2 – x1) = [(-16/3)-(2/3)] ÷ [(14/3) – (-4/3)] = (-18/3) ÷ (18/3) = -1
y = mx + b
2/3 = -1(-4/3) + b
2/3 = 4/3 + b
1. From BD
Slope = (y2 – y1)/(x2 – x1) = [(-16/3)-(-4/3)] ÷ [(2/3) – (144/3)] = (-12/3) ÷ (-12/3) = 1
y = mx + b
-4/3 = 1(14/3) + b
-4/3 = 14/3 + b
-18/3 = b
-6 = b
y = mx + b
y = x -6
3. y = -x -2/3
y = x – 6
4. y = -x – 2/3
x – 6 = -x – 2/3
x = -x – 2/3 + 6
x = – x + 16/3
2x = 16/3
x = 16/6
x = 8/3
then y = x – 6
y = 8/3 – 18/3
y = -10/3
The struts cross as (8/3, 10/3)

FOCUS ON HIGHER ORDER THINKING

Question 16.
Analyze Relationships Consider the system
\(\left\{\begin{array}{l}6x-3y=15 \\x+3y=-8\end{array}\right.\)
Describe three different substitution methods that can be used to solve this system. Then solve the system.
Type below:
______________

Answer:
(1, -3) is the answer.

Explanation:
As there are three different substitution methods, we can write
Solve for y in the first equation, then substitute that value into the second equation.
Solve for x in the second equation, then substitute that value into the first equation.
Solve either equation for 3y, then substitute that value into the other equation.
From the Second method,
x + 3y = -8
x = -3y – 8
6x – 3y = 15
6 (-3y – 8) -3y = 15
-18y – 48 -3y = 15
-21y – 48 = 15
-21y = 63
y = -3
x + 3y = -8
x + 3(-3) = -8
x – 9 = -8
x = 1
(1, -3) is the answer.

Question 17.
Communicate Mathematical Ideas Explain the advantages, if any, that solving a system of linear equations by substitution has over solving the same system by graphing.
Type below:
______________

Answer:
The advantage of solving a system of linear equations by graphing is that it is relatively easy to do and requires very little algebra.

Question 18.
Persevere in Problem Solving Create a system of equations of the form
\(\left\{\begin{array}{l}Ax+By=C \\Dx+Ey=F\end{array}\right.\)
that has (7, −2) as its solution. Explain how you found the system.
Type below:
______________

Answer:
x + y = 5
x – y = 9
solves in :
x = (5+9)/2 = 7
y = 5-9)/2 = -2
A=1, B=2, C= 5
D=1, E= -1, F=9
x = 7
y = -2
IS a system (even if it is a trivial one) of equations so this answer would be acceptable.
The target for a system is to find it SOLUTION SET and not to conclude with x=a and y=b

Guided Practice – Solving Systems by Elimination – Page No. 248

Question 1.
Solve the system
\(\left\{\begin{array}{l}4x+3y=1 \\x-3y=-11\end{array}\right.\)
by adding.
Go Math Grade 8 Answer Key Chapter 8 Solving Systems of Linear Equations Lesson 3: Solving Systems by Elimination img 12
Type below:
______________

Answer:
4x + 3y = 1
x – 3y = -11
Add the above two equations
4x + 3y = 1
+(x – 3y = -11)
Add to eliminate the variable y
5x + 0y = -10
Simplify and solve for x
5x = -10
Divide both sided by 5
x = -10/5 = -2
Substitute into one of the original equations and solve for y.
4(-2) + 3y = 1
-8 + 3y = 1
3y = 9
y = 9/3 = 3
So, (-2, 3) is the solution of the system.

Solve each system of equations by adding or subtracting.

Question 2.
\(\left\{\begin{array}{l}x+2y=-2 \\-3x+2y=-10\end{array}\right.\)
x = ________
y = ________

Answer:
x = 2
y = -2

Explanation:
\(\left\{\begin{array}{l}x+2y=-2 \\-3x+2y=-10\end{array}\right.\)
Subtract the equations
x + 2y = -2
-(-3x + 2y = -10)
y is eliminated as it has reversed coefficients. Solve for x
x + 2y + 3x – 2y = -2 + 10
4x = 8
x = 8/4 = 2
Substituting x in either of the equation to find y
2 + 2y = -2
2 + 2y -2 = -2 -2
2y = -4
y = -4/2 = -2
(2, -2) is the answer.

Question 3.
\(\left\{\begin{array}{l}3x+y=23 \\3x-2y=8\end{array}\right.\)
(________ , ________)

Answer:
(6, 5)

Explanation:
\(\left\{\begin{array}{l}3x+y=23 \\3x-2y=8\end{array}\right.\)
Subtract the equations
3x + y = 23
-(3x – 2y = 8)
x is eliminated as it has reversed coefficients. Solve for y
3x + y – 3x + 2y = 23 – 8
3y = 15
y = 15/3 = 5
Substituting y in either of the equation to find x
3x + 5 = 23
3x + 5 – 5 = 23 – 5
3x = 18
x = 18/3 = 6
Solution is (6, 5)

Question 4.
\(\left\{\begin{array}{l}-4x-5y=7 \\3x+5y=-14\end{array}\right.\)
(________ , ________)

Answer:
(7, -7)

Explanation:
\(\left\{\begin{array}{l}-4x-5y=7 \\3x+5y=-14\end{array}\right.\)
Add the equations
-4x – 5y = 7
+(3x + 5y = -14)
y is eliminated as it has reversed coefficients. Solve for x
-4x -5y +3x + 5y = 7 -14
-x = -7
x = -7/-1 = 7
Substituting x in either of the equation to find y
3(7) + 5y = -14
21 + 5y -21 = -14 -21
5y = -35
y = -35/5 = -7
The answer is (7, -7)

Question 5.
\(\left\{\begin{array}{l}x-2y=-19 \\5x+2y=1\end{array}\right.\)
(________ , ________)

Answer:
(-3, 8)

Explanation:
\(\left\{\begin{array}{l}x-2y=-19 \\5x+2y=1\end{array}\right.\)
Add the equations
x – 2y = -19
+(5x + 2y = 1)
y is eliminated as it has reversed coefficients. Solve for x
x – 2y + 5x + 2y = -19 + 1
6x = -18
x = -18/6 = -3
Substituting x in either of the equation to find y
-3 -2y = -19
-3 -2y + 3 = -19 + 3
-2y = -16
y = -16/-2 = 8
The answer is (-3, 8)

Question 6.
\(\left\{\begin{array}{l}3x+4y=18 \\-2x+4y=8\end{array}\right.\)
(________ , ________)

Answer:
(2, 3)

Explanation:
\(\left\{\begin{array}{l}3x+4y=18 \\-2x+4y=8\end{array}\right.\)
Subtract the equations
3x + 4y = 18
-(-2x + 4y = 8)
y is eliminated as it has reversed coefficients. Solve for x
3x + 4y + 2x – 4y = 18 – 8
5x = 10
x = 10/5 = 2
Substituting x in either of the equation to find y
3(2) + 4y = 18
6 + 4y – 6 = 18 – 6
4y = 12
y = 12/4 =3
Solution is (2, 3)

Question 7.
\(\left\{\begin{array}{l}-5x+7y=11 \\-5x+3y=19\end{array}\right.\)
(________ , ________)

Answer:
(-5, -2)

Explanation:
\(\left\{\begin{array}{l}-5x+7y=11 \\-5x+3y=19\end{array}\right.\)
Subtract the equations
-5x + 7y = 11
-(-5x + 3y = 19)
x is eliminated as it has reversed coefficients. Solve for y
-5x + 7y + 5x – 3y = 11 – 19
4y = -8
y = -8/4 = -2
Substituting y in either of the equation to find x
-5x + 7(-2) = 11
-5x -14 + 14 = 11 + 14
-5x = 25
x = 25/-5 = -5
Solution is (-5, -2)

Question 8.
The Green River Freeway has a minimum and a maximum speed limit. Tony drove for 2 hours at the minimum speed limit and 3.5 hours at the maximum limit, a distance of 355 miles. Rae drove 2 hours at the minimum speed limit and 3 hours at the maximum limit, a distance of 320 miles. What are the two speed limits?
a. Write equatios to represent Tony’s distance and Rae’s distance.
Type below:
______________

Answer:
Tony’s distance: 2x + 3.5y = 355
Rae’s distance: 2x + 3y = 320
where x is the minimum speed and y is the maximum speed.

Question 8.
b. Solve the system.
minimum speed limit:______________
maximum speed limit______________
minimum speed limit: ________ mi/h
maximum speed limit: ________ mi/h

Answer:
minimum speed limit:55
maximum speed limit70
minimum speed limit: 55mi/h
maximum speed limit: 70mi/h

Explanation:
Subtract the equations
2x + 3.5y = 355
-(2x + 3y = 320)
x is eliminated as it has reversed coefficients. Solve for y
2x + 3.5y – 2x – 3y = 355 – 320
0.5y = 35
y = 35/0.5 = 70
Substituting y in either of the equation to find x
2x + 3(70) = 320
2x + 210 – 210 = 320 – 210
2x = 110
x = 110/2 = 55
Minimum speed limit: 55 miles per hour
Maximum speed limit: 70 miles per hour

ESSENTIAL QUESTION CHECK-IN

Question 9.
Can you use addition or subtraction to solve any system? Explain.
________

Answer:
No. One of the variables should have the same coefficient in order to add or subtract the system.

8.3 Independent Practice – Solving Systems by Elimination – Page No. 249

Question 10.
Represent Real-World Problems Marta bought new fish for her home aquarium. She bought 3 guppies and 2 platies for a total of $13.95. Hank also bought guppies and platies for his aquarium. He bought 3 guppies and 4 platies for a total of $18.33. Find the price of a guppy and the price of a platy.
Go Math Grade 8 Answer Key Chapter 8 Solving Systems of Linear Equations Lesson 3: Solving Systems by Elimination img 13
Guppy: $ ________
Platy: $ ________

Answer:
Guppy: $ 3.19
Platy: $ 2.19

Explanation:
3x + 2y = 13.95
3x + 4y = 18.33
where x is the unit price of guppy and y is the unit price of platy
Subtract the equations
3x + 2y = 13.95
-(3x + 4y = 18.33)
x is eliminated as it has reversed coefficients. Solve for y
3x + 2y – 3x – 4y = 13.95 – 18.33
-2y = -4.38
y = -4.38/-2 = 2.19
Substituting y in either of the equation to find x
3x + 2(2.19) = 13.95
3x + 4.38 – 4.38 = 13.95 – 4.38
3x = 9.57
x = 9.57/3 = 3.19
The price of a guppy is $3.19 and price of a platy is $2.19

Question 11.
Represent Real-World Problems The rule for the number of fish in a home aquarium is 1 gallon of water for each inch of fish length. Marta’s aquarium holds 13 gallons and Hank’s aquarium holds 17 gallons. Based on the number of fish they bought in Exercise 10, how long is a guppy and how long is a platy?
Length of a guppy = ________ inches
Length of a platy = ________ inches

Answer:
Length of a guppy = 3 inches
Length of a platy = 2 inches

Explanation:
3x + 2y = 13
3x + 4y = 17
where x is the length of guppy and y is the length of a platy
Subtract the equations
3x + 2y = 13
-(3x + 4y = 17)
x is eliminated as it has reversed coefficients. Solve for y
3x + 2y – 3x – 4y = 13 – 17
-2y = -4
y = -4/-2 = 2
Substituting y in either of the equation to find x
3x + 2(2) = 13
3x + 4 – 4 = 13 – 4
3x = 9
x = 9/3 = 3
The length of a guppy is 3 inches and price of a platy is 2 inches

Question 12.
Line m passes through the points (6, 1) and (2, -3). Line n passes through the points (2, 3) and (5, -6). Find the point of intersection of these lines.
Type below:
________________

Answer:
The intersection of these lines is (3.5, -1.5)

Explanation:
Find the slope of line m = (y2 – y1)/(x2 – x1) where (x2, y2) = (2, -3) and (x1, y1) = (6, 1)
Slope = (-3 -1)/(2 – 6) = -4/-4 = 1
Substitute the value of m and any of the given ordered pair (x, y) in point-slope form of equation: y – y1 = m(x – x1)
y – 1 = 1(x – 6)
y – 1 = x – 6
y = x – 6 + 1
x – y = 5
Find the slope of line n = (y2 – y1)/(x2 – x1) where (x2, y2) = (5, -6) and (x1, y1) = (2, 3)
Slope = (-6 -3)/(5 – 2) = -9/3 = -3
Substitute the value of m and any of the given ordered pair (x, y) in point-slope form of equation: y – y1 = m(x – x1)
y – 3 = -3(x – 2)
y – 3 = -3x + 6
y = -3x + 6 + 3
3x + y = 9
Add the equations
x – y = 5
+(3x + y = 9)
y is eliminated as it has reversed coefficients. Solve for x
x – y + 3x + y = 5 + 9
4x = 14
x = 14/4 = 3.5
Substituting x in either of the equation to find y
3.5 – y = 5
3.5 – y – 3.5 = 5 – 3.5
-y = 1.5
y = -1.5
The intersection of these lines is (3.5, -1.5)

Question 13.
Represent Real-World Problems Two cars got an oil change at the same auto shop. The shop charges customers for each quart of oil plus a flat fee for labor. The oil change for one car required 5 quarts of oil and cost $22.45. The oil change for the other car required 7 quarts of oil and cost $25.45. How much is the labor fee and how much is each quart of oil?
Labor fee: $ ________
Quart of oil: $ ________

Answer:
Labor fee: $ 14.95
Quart of oil: $ 1.5

Explanation:
5x + y = 22.45
7x + y = 25.45
where x is the unit cost of quarts of oil and y is the flat fee for labor
Subtract the equations
5x + y = 22.45
-(7x + y = 25.45)
y is eliminated as it has reversed coefficients. Solve for x
5x + y – 7x – y = 22.45 – 25.45
-2x = -3
x = -3/-2 = 1.5
Substituting x in either of the equation to find y
5(1.5) + y = 22.45
7.5 + y – 7.5 = 22.45 – 7.5
y = 14.95
Labor fee is $14.95 and unit cost of quart of oil is $1.5

Question 14.
Represent Real-World Problems A sales manager noticed that the number of units sold for two T-shirt styles, style A and style B, was the same during June and July. In June, total sales were $2779 for the two styles, with A selling for $15.95 per shirt and B selling for $22.95 per shirt. In July, total sales for the two styles were $2385.10, with A selling at the same price and B selling at a discount of 22% off the June price. How many T-shirts of each style were sold in June and July combined?
________ T-shirts of style A and style B were sold in June and July.

Answer:
15.95x + 22.95y = 2779
15.95x + 17.9y = 2385.10
where x is number of style A shirt and y is the number of style B shirt
In July, the price of style B shirt is 22% of the price of style B shirt in June, hence 0.78(22.95) = 17.90
Subtract the equations
15.95x + 22.95y = 2779
-(15.95x + 17.9y = 2385.10)
x is eliminated as it has reversed coefficients. Solve for y
15.95x + 22.95 – 15.95x – 17.9y = 2779 – 2385.10
5.05y = 393.9
y = 393.9/5.05 = 78
Substituting y in either of the equation to find x
15.95x +22.95(78) = 2779
15.95x + 1790.1 – 1790.1 = 2779 – 1790.1
15.95x = 988.9
x = 988.9/15.95 = 62
The number of style A T shirt sold in June is 62.
Since the number of T-shirts sold in both numbers is the same, the total number = 2. 62 = 124.
The number of style B T-shirts sold in June is 78.
Since the number of T-shirts sold in both numbers is the same, the total number = 2. 78 = 156.

Question 15.
Represent Real-World Problems Adult tickets to a basketball game cost $5. Student tickets cost $1. A total of $2,874 was collected on the sale of 1,246 tickets. How many of each type of ticket were sold?
Go Math Grade 8 Answer Key Chapter 8 Solving Systems of Linear Equations Lesson 3: Solving Systems by Elimination img 14img 14
________ adult tickets
________ student tickets

Answer:
407 adult tickets
839 student tickets

Explanation:
x + y = 1246
5x + y = 2874
where x is the number of adult tickets sold and y is the number of student tickets sold.
Subtract the equations
x + y = 1246
-(5x + y = 2874)
y is eliminated as it has reversed coefficients. Solve for x
x + y – 5x – y = 1246 – 2874
-4x = -1628
x = -1628/-4 = 407
Substituting x in either of the equation to find y
407 + y = 1246
407 + y – 407 = 1246 – 407
y = 839
The number of adult tickets sold is 407 and student tickets sold is 839.

FOCUS ON HIGHER ORDER THINKING – Solving Systems by Elimination – Page No. 250

Question 16.
Communicate Mathematical Ideas Is it possible to solve the system
\(\left\{\begin{array}{l}3x-2y=10 \\x+2y=6\end{array}\right.\)
by using substitution? If so, explain how. Which method, substitution or elimination, is more efficient? Why?
________

Answer:
The system can be solved by substitution as x in equation 2 can be isolated.
3x – 2y = 10
x + 2y = 6
Solve the equation for x in the equation.
x = -2y + 6
Substitute the expression for x in the other equation and solve.
3(-2y + 6) -2y = 10
-6y + 18 – 2y = 10
-8y + 18 = 10
-8y = -8
y = -8/-8 = 1
Substitute the values of y into one of the equations and solve for the other variable x.
x + 2(1) = 6
x = 4
The solution is (4, 1)
As the cofficient if variable y is opposite, it will be eliminated and solved for x in less number of steps.
Elimination would be more efficient.

Question 17.
Jenny used substitution to solve the system
\(\left\{\begin{array}{l}2x+y=8 \\x-y=1\end{array}\right.\). Her solution is shown below.
Step 1: y = -2x + 8               Solve the first equation for y.
Step 2: 2x + (-2x + 8) = 8     Substitute the value of y in an original equation.
Step 3: 2x – 2x + 8 = 8          Use the Distributive Property.
Step 4: 8 = 8                         Simplify.
a. Explain the Error Explain the error Jenny made. Describe how to correct it.
Type below:
______________

Answer:
2x + y = 8
x – y = 1
Rewritten equation should be substituted in the other original equation
Error is that Jenny solved for y in the first equation and substitute it in the original equation.
x – (-2x + 8) = 1
3x – 8 = 1
3x = 9
x = 9/3 = 3
x = 3

Question 17.
b. Communicate Mathematical Ideas Would adding the equations have been a better method for solving the system? If so, explain why.
________

Answer:
Yes

Explanation:
As the coefficient, if variable y is the opposite, it will be eliminated and solved for x in less number of steps.

Guided Practice – Solving Systems by Elimination with Multiplication – Page No. 256

Question 1.
Solve the system
\(\left\{\begin{array}{l}3x-y=8 \\-2x+4y=-12\end{array}\right.\)
by multiplying and adding.
Go Math Grade 8 Answer Key Chapter 8 Solving Systems of Linear Equations Lesson 4: Solving Systems by Elimination with Multiplication img 15
Type below:
______________

Answer:
\(\left\{\begin{array}{l}3x-y=8 \\-2x+4y=-12\end{array}\right.\)
Multiply each term in the first equation by 4 to get opposite coefficients for the y-terms.
4(3x – y = 8)
12x – 4y = 32
Add the second equation to the new equation
12x – 4y = 32
+(-2x + 4y = -12)
Add to eliminate the variable y
10x = 20
Divide both sides by 10
x = 20/10 = 2
Substitue into one of the original equations and solve for y
y = 3(2) – 8 = -1
S0, (2, -2)is the solution of the system.

Solve each system of equations by multiplying first.

Question 2.
\(\left\{\begin{array}{l}x+4y=2 \\2x+5y=7\end{array}\right.\)
(________ , ________ )

Answer:
(6, -1)

Explanation:
x + 4y = 2
2x + 5y = 7
To eliminate x terms, multiply the 2nd equation by 2
2(x + 4y = 2)
2x + 8y = 4
Subtract the equations
2x + 8y = 4
-(2x + 5y = 7)
x is eliminated as it has reversed coefficients. Solve for y
2x + 8y – 2x – 5y = 4 – 7
3y = -3
y = -3/3 = -1
Substituting y in either of the equation to find x
x + 4(-1) = 2
x – 4 + 4 = 2 + 4
x = 6
Solution: (6, -1)

Question 3.
\(\left\{\begin{array}{l}3x+y=-1 \\2x+3y=18\end{array}\right.\)
(________ , ________ )

Answer:
(-3, 8)

Explanation:
\(\left\{\begin{array}{l}3x+y=-1 \\2x+3y=18\end{array}\right.\)
To eliminate y terms, multiply the 1st equation by 3
3(3x + y = -1)
9x + 3y = -3
Subtract the equations
9x + 3y = -3
-(2x + 3y = 18)
y is eliminated as it has reversed coefficients. Solve for x
9x + 3y – 2x – 3y = -3 -18
7x = -21
x = -21/7
x = -3
Substituting x in either of the equation to find y
3(-3) + y = -1
-9 + y + 9 = -1 + 9
y = 8
Solution: (-3, 8)

Question 4.
\(\left\{\begin{array}{l}2x+8y=21 \\6x-4y=14\end{array}\right.\)
Type below:
______________

Answer:
The soultion is (3.5, 1.75)

Explanation:
\(\left\{\begin{array}{l}2x+8y=21 \\6x-4y=14\end{array}\right.\)
To eliminate y terms, multiply the 2nd equation by 2
2(6x – 4y = 14)
2x + 8y = 21
Add the equations
2x + 8y = 21
+(12x – 8y = 28)
y is eliminated it has reversed coefficients. Solve for x
2x + 8y + 12x – 8y = 21 + 28
14x = 49
x = 49/14 = 3.5
Substituting x in either of the equation to find y
6(3.5) – 4y = 14
21 – 4y – 21 = 14 – 21
-4y = -7
y = -7/-4 = 1.75
The soultion is (3.5, 1.75)

Question 5.
\(\left\{\begin{array}{l}2x+y=3 \\-x+3y=-12\end{array}\right.\)
(________ , ________ )

Answer:

Explanation:
\(\left\{\begin{array}{l}2x+y=3 \\-x+3y=-12\end{array}\right.\)
To eliminate x terms, multiply the 2nd equation by 2
2(-x + 3y = -12)
-2x + 6y = -24
Add the equations
2x + y = 3
+(-2x + 6y = -24)
x is eliminated it has reversed coefficients. Solve for y
2x + y – 2x + 6y = 3 – 24
7y = -21
y = -21/7 = -3
Substituting y in either of the equation to find x
-x + 3(-3) = -12
-x -9 + 9 = -12 + 9
-x = -3
x = 3
The soultion is (3, -3)

Question 6.
\(\left\{\begin{array}{l}6x+5y=19 \\2x+3y=5\end{array}\right.\)
(________ , ________ )

Answer:
The soultion is (4, -1)

Explanation:
\(\left\{\begin{array}{l}6x+5y=19 \\2x+3y=5\end{array}\right.\)
To eliminate x terms, multiply the 2nd equation by 3
3(2x + 3y = 5)
6x + 9y = 15
Subtract the equations
6x + 5y = 19
-(6x + 9y = 15)
x is eliminated it has reversed coefficients. Solve for y
6x + 5y – 6x – 9y = 19 – 15
-4y = 4
y = 4/-4 = -1
Substituting y in either of the equation to find x
2x + 3(-1) = 5
2x – 3 + 3 = 5 + 3
2x = 8
x = 8/2 = 4
The soultion is (4, -1)

Question 7.
\(\left\{\begin{array}{l}2x+5y=16 \\-4x+3y=20\end{array}\right.\)
(________ , ________ )

Answer:
The soultion is (-2, 4)

Explanation:
\(\left\{\begin{array}{l}2x+5y=16 \\-4x+3y=20\end{array}\right.\)
To eliminate x terms, multiply the 1st equation by 2
2(2x + 5y = 16)
4x + 10y = 32
Add the equations
4x + 10y = 32
+(-4x + 3y = 20)
x is eliminated it has reversed coefficients. Solve for y
10y + 3y = 32 + 20
13y = 52
y = 52/13 = 4
Substituting y in either of the equation to find x
2x + 5(4) = 16
2x + 20 – 20 = 16 – 20
2x = -4
x = -4/2 = -2
The soultion is (-2, 4)

Question 8.
Bryce spent $5.26 on some apples priced at $0.64 each and some pears priced at $0.45 each. At another store he could have bought the same number of apples at $0.32 each and the same number of pears at $0.39 each, for a total cost of $3.62. How many apples and how many pears did Bryce buy?
a. Write equations to represent Bryce’s expenditures at each store
First store: _____________
Second store: _____________
Type below:
_____________

Answer:
First store: 0.64x + 0.45y = 5.26
Second store: 0.32x + 0.39y = 3.62

Explanation:
First store = 0.64x + 0.45y = 5.26
Second store = 0.32x + 0.39y = 3.62
where x is the number of apples and y is the number of pears.

Question 8.
b. Solve the system.
Number of apples: _______
Number of pears: _______

Answer:
Number of apples: 4
Number of pears: 6

Explanation:
First store = 0.64x + 0.45y = 5.26
Second store = 0.32x + 0.39y = 3.62
Multiply by 100
64x + 45y = 526
32x + 39y = 362
To eliminate x terms, multiply the 2nd equation by 2
2(32x + 39y = 362)
64x + 45y = 526
Subtract the equations
64x + 45y = 526
-(64x + 78y = 724)
x is eliminated it has reversed coefficients. Solve for y
64x + 45y – 64x – 78y = 526 – 724
-33y = -198
y = -198/-33 = 6
Substituting y in either of the equation to find x
32x + 39(6) = 362
32x + 234 – 234 = 362 – 234
32x = 128
x = 128/32 = 4
He bought 4 apples and 6 pears.

ESSENTIAL QUESTION CHECK-IN

Question 9.
When solving a system by multiplying and then adding or subtracting, how do you decide whether to add or subtract?
Type below:
_____________

Answer:
If the variable with the same coefficient but reversed sign, we add and if they have the same sign, we subtract.

Solving Systems by Elimination with Multiplication – Page No. 257

Question 10.
Explain the Error Gwen used elimination with multiplication to solve the system
\(\left\{\begin{array}{l}2x+6y=3 \\x-3y=-1\end{array}\right.\)
Her work to find x is shown. Explain her error. Then solve the system.
2(x − 3y) = -1
2x − 6y = -1
+2x + 6y = 3
_____________
4x + 0y = 2
x = \(\frac{1}{2}\)
Type below:
____________

Answer:
2x + 6y = 3
x – 3y = -1
To eliminate x terms, multiply the 2nd equation by 2
2(x – 3y = -1)
2x – 6y = -2
Error is the Gnew did not multiply the entire expression with 2.
Add the equations
2x + 6y = 3
+(2x – 6y = -2)
y is eliminated it has reversed coefficients. Solve for x
2x + 6y + 2x – 6y = 3 – 2
4x = 1
x = 1/4
Substituting x in either of the equation to find y
x – 3y = -1
1/4 – 3y – 1/4 = -1 -1/4
-3y = -5/4
y = -5/4(-3) = 5/12

Question 11.
Represent Real-World Problems At Raging River Sports, polyester-fill sleeping bags sell for $79. Down-fill sleeping bags sell for $149. In one week the store sold 14 sleeping bags for $1,456.
a. Let x represent the number of polyester-fill bags sold and let y represent the number of down-fill bags sold. Write a system of equations you can solve to find the number of each type sold.
Go Math Grade 8 Answer Key Chapter 8 Solving Systems of Linear Equations Lesson 4: Solving Systems by Elimination with Multiplication img 16
Type below:
____________

Answer:
x + y = 14
79x + 149y = 1456
where x is the polyster-fill bags and y is the number of down-fill bags

Question 11.
b. Explain how you can solve the system for y by multiplying and subtracting.
Type below:
____________

Answer:
x + y = 14
79x + 149y = 1456
Multiply the second equation by 79. Subtract the new equation from the first equation and solve the resulting equation for y.

Question 11.
c. Explain how you can solve the system for y using substitution.
Type below:
____________

Answer:
Solve the second equation for x. Substitute the expression for x , in the first equation and solve the resulting equation for y.

Question 11.
d. How many of each type of bag were sold?
_______ polyester-fill
_______ down-fill

Answer:
9 polyester-fill
5 down-fill

Explanation:
x + y = 14
79x + 149y = 1456
To eliminate x terms, multiply the 2nd equation by 2
79(x + y = 14)
79x + 149y = 1456
Subtract the equations
79x + 79y = 1106
-(79x + 149y = 1456)
x is eliminated it has reversed coefficients. Solve for y
79x + 79y – 79x – 149y = 1106 – 1456
-70y = -350
y = -350/-70 = 5
Substituting y in either of the equation to find x
x + 5 = 14
x = 14 – 5
x = 9
There were 9 polyster-fill bags and 5 down-fill bags sold.

Question 12.
Twice a number plus twice a second number is 310. The difference between the numbers is 55. Find the numbers by writing and solving a system of equations. Explain how you solved the system.
x = _______
y = _______

Answer:
x = 105
y = 50

Explanation:
2x + 2y = 310
x – y = 55
To eliminate y terms, multiply the 2nd equation by 2
2(x – y = 55)
2x – 2y = 110
Add the equations
2x + 2y = 310
+ (2x – 2y = 110)
y is eliminated it has reversed coefficients. Solve for x
2x + 2y + 2x – 2y = 310 + 110
4x = 420
x = 420/4 = 105
Substituting x in either of the equation to find y
105 – y = 55
y = 105 – 55
y = 50
The solution is (105, 50)

Solving Systems by Elimination with Multiplication – Page No. 258

Question 13.
Represent Real-World Problems A farm stand sells apple pies and jars of applesauce. The table shows the number of apples needed to make a pie and a jar of applesauce. Yesterday, the farm picked 169 Granny Smith apples and 95 Red Delicious apples. How many pies and jars of applesauce can the farm make if every apple is used?
Go Math Grade 8 Answer Key Chapter 8 Solving Systems of Linear Equations Lesson 4: Solving Systems by Elimination with Multiplication img 17
_______ pies
_______ jars of applesauce

Answer:
21 pies
16 jars of applesauce

Explanation:
5x + 4y = 169
3x + 2y = 95
where x is the number of apples needed for pie and y is the number of apples for jar of applesauce
To eliminate y terms, multiply the 2nd equation by 2
2(3x + 2y = 95)
6x + 4y = 190
Subtract the equations
5x + 4y = 169
– (6x + 4y = 190)
y is eliminated it has reversed coefficients. Solve for x
5x + 4y – 6x – 4y = 169 – 190
-x = -21
x = -21/-1 = 21
Substituting x in either of the equation to find y
5(21) + 4y = 169
105 + 4y – 105 = 169 – 105
4y = 64
y = 64/4 = 16
The number of apples needed for pie is 21 and the number of apples for jar of applesauce is 16.

FOCUS ON HIGHER ORDER THINKING

Question 14.
Make a Conjecture Lena tried to solve a system of linear equations algebraically and in the process found the equation 5 = 9. Lena thought something was wrong, so she graphed the equations and found that they were parallel lines. Explain what Lena’s graph and equation could mean.
Type below:
____________

Answer:
Lena’s graph is a parallel line which means the graph does not intersect each other, hence they have no solutions. Equation 5 = 9 means variables are eliminated and this statement is not true. This linear system has no solution.

Question 15.
Consider the system
\(\left\{\begin{array}{l}2x+3y=6 \\3x+7y=-1\end{array}\right.\)
a. Communicate Mathematical Ideas Describe how to solve the system by multiplying the first equation by a constant and subtracting. Why would this method be less than ideal?
Type below:
____________

Answer:
Multiplying the first equation by a constant and subtracting
2x + 3y = 6
3x + 7y = -1
Multiply the first equation by 1.5 and subtract. This would be less than ideal because you would introduce decimals into the solution process.

Question 15.
b. Draw Conclusions Is it possible to solve the system by multiplying both equations by integer constants? If so, explain how.
Type below:
____________

Answer:
Yes

Explanation:
Multiply the first equation by 3 and the second equation by 2. Both x-term coefficients would be 6. Solve by eliminating the x-terms using subtraction.

Question 15.
c. Use your answer from part b to solve the system.
(_______ , _______)

Answer:
(9, -4)

Explanation:
2x + 3y = 6
3x + 7y = -1
Multiply the first equation by 3 and the second equation by 2.
3(2x + 3y = 6)
2(3x + 7y = -1)
Subtract the equations
6x + 9y = 18
-(6x + 14y = -2)
x is eliminated it has reversed coefficients. Solve for y
6x + 9y – 6x – 14y = 18 + 2
-5y = 20
y = 20/-5 = -4
Substituting y in either of the equation to find x
2x + 3(-4) = 6
2x = 18
x = 18/2 = 9
The solution is (9, -4)

Guided Practice – Solving Solving Special Systems – Page No. 262

Use the graph to solve each system of linear equations

Question 1.
A. \(\left\{\begin{array}{l}4x-2y=-6 \\2x-y=4\end{array}\right.\)
B. \(\left\{\begin{array}{l}4x-2y=-6 \\x+y=6\end{array}\right.\)
C. \(\left\{\begin{array}{l}2x-y=4 \\6x-3y=-12\end{array}\right.\)
STEP 1 Decide if the graphs of the equations in each system intersect, are parallel, or are the same line.
Go Math Grade 8 Answer Key Chapter 8 Solving Systems of Linear Equations Lesson 5: Solving Solving Special Systems img 18
System A: The graphs __________
System B: The graphs __________
System C: The graphs __________

Answer:
System A: The graphs are parallel
System B: The graphs are intersecting
System C: The graphs are  the same line

Explanation:
System A: 4x – 2y = -6
2x – y = 4
System B: 4x – 2y = -6
x + y = 6
System C: 2x – y = 4
6x – 3y = 12

Question 1.
STEP 2 Decide how many points the graphs have in common.
a. Intersecting lines have _______________ point(s) in common.
b. Parallel lines have _______________ point(s) in common.
c. The same lines have ___________ point(s) in common.
a. __________
b. __________
c. __________

Answer:
a. Intersecting lines have one point(s) in common.
b. Parallel lines have no point(s) in common.
c. The same lines have infinitely many points (s) in common.

Explanation:
From the graphs,
Intersecting lines have one point(s) in common
Parallel lines have no point(s) in common
The same lines have infinitely many points (s) in common

Question 1.
STEP 3 Solve each system.
System A has __________ points in common, so it has __________ solution.
System B has __________ point in common. That point is the solution, __________.
System C has __________ points in common. ________ ordered pairs on the line will make both equations true.
Type below:
___________

Answer:
System A has no points in common, so it has no solution. System B has one point in common. That point is the solution, (1,5). System C has an infinite number of points in common. All ordered pairs on the line will make both equations true.

Explanation:
Number of solutions for each system
System A has no points in common, so it has no solution. System B has one point in common. That point is the solution, (1,5). System C has an infinite number of points in common. All ordered pairs on the line will make both equations true.

Solve each system. Tell how many solutions each system has.

Question 2.
\(\left\{\begin{array}{l}x-3y=4 \\-5x+15y=-20\end{array}\right.\)
___________

Answer:
infinitely many solutions

Explanation:
x – 3y = 4
-5x + 15y = -20
To eliminate y terms, multiply the 1st equation by 5
5(x – 3y = 4)
5x – 15y = 20
Add the equations
5x – 15y = 20
+(-5x + 15y = -20)
x and y is eliminated as it has reversed coefficients.
5x – 15y – 5x + 15y = 20 – 20
0 = 0
The statement is true, hence the solution has infinitely many solutions.

Question 3.
\(\left\{\begin{array}{l}6x+2y=-4 \\3x+y=4\end{array}\right.\)
___________

Answer:
no solution

Explanation:
6x + 2y = -4
3x + y = 4
To eliminate y terms, multiply the 2nd equation by 5
2(3x + y = 4)
6x + 2y = 8
Subtract the equations
6x + 2y = -4
-(6x + 2y = 8)
x and y is eliminated as it has reversed coefficients.
6x + 2y – 6x – 2y = -4 -8
0 = -12
The statement is false, hence the solution has no solution.

Question 4.
\(\left\{\begin{array}{l}6x-2y=-10 \\3x+4y=-25\end{array}\right.\)
___________

Answer:
one solution

Explanation:
6x – 2y = -10
3x + 4y = -25
To eliminate y terms, multiply the 1st equation by 2
2(6x – 2y = -10)
12x – 4y = -20
Add the equations
12x – 4y = -20
+(3x + 4y = -25)
y is eliminated as it has reversed coefficients. Solve for x.
12x – 4y + 3x + 4y = -20 – 25
15x = -45
x = -45/15 = -3
Substitute x in any one of the original equations and solve for y
3(-3) + 4y = -25
-9 + 4y + 9 = -25 + 9
4y = -16
y = -16/4
y = -4
There is one solution, (-3, -4)

ESSENTIAL QUESTION CHECK-IN

Question 5.
When you solve a system of equations algebraically, how can you tell whether the system has zero, one, or an infinite number of solutions?
Type below:
___________

Answer:
When x and y are eliminated and the statement is true, the system has infinitely many solutions.
When x and y are eliminated and the statement is false, the system has no solutions.
When the system has one solution by solving, the system has one solution.

8.5 Independent Practice – Solving Solving Special Systems – Page No. 263

Solve each system by graphing. Check your answer algebraically.

Question 6.
\(\left\{\begin{array}{l}-2x+6y=12 \\x-3y=3\end{array}\right.\)
Go Math Grade 8 Answer Key Chapter 8 Solving Systems of Linear Equations Lesson 5: Solving Solving Special Systems img 19
Solution: ______________
___________

Answer:
\(\left\{\begin{array}{l}-2x+6y=12 \\x-3y=3\end{array}\right.\)
Graph the equations on same coordinate plane
No solution as equations are parallel
grade 8 chapter 8 image 1
To eliminate y terms, multiply the 2nd equation by 2
2(x – 3y = 3)
2x – 6y = 6
Add the equations
-2x + 6y = 12
2x – 6y = 6
x and y is eliminated as it has reversed coefficients.
-2x + 6y + 2x – 6y = 12 + 6
0 = 18
The statement is false, hence the system has no solution.

Question 7.
\(\left\{\begin{array}{l}15x+5y=5 \\3x+y=1\end{array}\right.\)
Go Math Grade 8 Answer Key Chapter 8 Solving Systems of Linear Equations Lesson 5: Solving Solving Special Systems img 20
Solution: ______________
___________

Answer:
\(\left\{\begin{array}{l}15x+5y=5 \\3x+y=1\end{array}\right.\)
Graph the equations on same coordinate plane
grade 8 chapter 8 image 2
Infinitely many solutions as equations are overlapping
To eliminate y terms, multiply the 2nd equation by 5
5(3x + y = 1)
15x + 5y = 5
Subtarct the equations
15x + 5y = 5
-(15x + 5y = 5)
x and y is eliminated as it has reversed coefficients.
15x + 5y -15x – 5y = 5 – 5
0 = 0
The statement is true, hence the system has infinitely many solutions.

For Exs. 8–

14, state the number of solutions for each system of linear equations

Question 8.
a system whose graphs have the same slope but different y-intercepts
___________

Answer:
No solutions

Explanation:
Equations are parallel
No solutions

Question 9.
a system whose graphs have the same y-intercepts but different slopes
___________

Answer:
One solution

Explanation:
Equations are intersecting
One solution

Question 10.
a system whose graphs have the same y-intercepts and the same slopes
___________

Answer:
Infinitely many solutions

Explanation:
Equations are overlapping
Infinitely many solutions

Question 11.
a system whose graphs have different y-intercepts and different slopes
___________

Answer:
One solution

Explanation:
Equations are intersecting
One solution

Question 12.
the system
\(\left\{\begin{array}{l}y=2 \\y=-3\end{array}\right.\)
___________

Answer:
No solutions

Explanation:
Equations are parallel
No solutions

Question 13.
the system
\(\left\{\begin{array}{l}y=2 \\y=-3\end{array}\right.\)
___________

Answer:
One solution

Explanation:
Equations are intersecting
One solution

Question 14.
the system whose graphs were drawn using these tables of values:
Go Math Grade 8 Answer Key Chapter 8 Solving Systems of Linear Equations Lesson 5: Solving Solving Special Systems img 21
___________

Answer:
No solutions

Explanation:
Equations are parallel The slope is the same for both equations but the y-intercept is different.
No solutions

Question 15.
Draw Conclusions The graph of a linear system appears in a textbook. You can see that the lines do not intersect on the graph, but also they do not appear to be parallel. Can you conclude that the system has no solution? Explain.
___________

Answer:

No; although the lines do not intersect on the graph, they intersect at a point that is not on the graph. To prove that a system has no solution, you must do so algebraically

Solving Solving Special Systems – Page No. 264

Question 16.
Represent Real-World Problems Two school groups go to a roller skating rink. One group pays $243 for 36 admissions and 21 skate rentals. The other group pays $81 for 12 admissions and 7 skate rentals. Let x represent the cost of admission and let y represent the cost of a skate rental. Is there enough information to find values for x and y? Explain.
Go Math Grade 8 Answer Key Chapter 8 Solving Systems of Linear Equations Lesson 5: Solving Solving Special Systems img 22
___________

Answer:
36x + 21y = 243
12x + 7y = 81
where x is the cost of admission and y is the cost of stake rentals.
Although the information can be used to develop a system of linear equation, where each equation has two variables when the system is solved, the number of solutions is infinite, Mee the values of x and y cannot be determined.
No

Question 17.
Represent Real-World Problems Juan and Tory are practicing for a track meet. They start their practice runs at the same point, but Tory starts 1 minute after Juan. Both run at a speed of 704 feet per minute. Does Tory catch up to Juan? Explain.
___________

Answer:
No; Both Juan and Tory-run at the same rate, so the lines representing the distances each has run are parallel. There is no solution to the system

FOCUS ON HIGHER ORDER THINKING

Question 18.
Justify Reasoning A linear system with no solution consists of the equation y = 4x − 3 and a second equation of the form y = mx + b. What can you say about the values of m and b? Explain your reasoning.
Type below:
___________

Answer:
y = 4x – 3
y = mx + b
Since the system has no solutions, the two equations are parallel. The value of the slope, m would be the same i.e. 4. The value of y-intercept, b can be any number except -3 as b is different for parallel lines.

Question 19.
Justify Reasoning A linear system with infinitely many solutions consists of the equation 3x + 5 = 8 and a second equation of the form Ax + By = C. What can you say about the values of A, B, and C? Explain your reasoning.
Type below:
___________

Answer:
3x + 5 = 8
Ax + By = C
Since the system has infinitely many solutions, the values of A, B, and C must all be the same multiple of 3, 5, and 8, respectively. The two equations represent a single line, so the coefficients and constants of one equation must be a multiple of the other.

Question 20.
Draw Conclusions Both the points (2, -2) and (4, -4) are solutions of a system of linear equations. What conclusions can you make about the equations and their graphs?
Type below:
___________

Answer:
If a system has more than one solution, the equations represent the same line and have infinitely many solutions.

Ready to Go On? – Model Quiz – Page No. 265

8.1 Solving Systems of Linear Equations by Graphing

Solve each system by graphing.

Question 1.
\(\left\{\begin{array}{l}y=x-1 \\y=2x-3\end{array}\right.\)
Go Math Grade 8 Answer Key Chapter 8 Solving Systems of Linear Equations Model Quiz img 23
(________ , ________)

Answer:
(2, 1)

Explanation:
y = x – 1
y = 2x – 3
Graph the equations on the same coordinate plane
grade 8 chapter 8 image 3
The solution of the system is the point of intersection
The solution is (2, 1)

Question 2.
\(\left\{\begin{array}{l}x+2y=1 \\-x+y=2\end{array}\right.\)
Go Math Grade 8 Answer Key Chapter 8 Solving Systems of Linear Equations Model Quiz img 24
(________ , ________)

Answer:
(-1, 1)

Explanation:
x + 2y = 1
-x + y = 2
Graph the equations on same coordinate plane
grade 8 chapter 8 image 4
The solution of the system is the point of intersection
The solution is (-1, 1)

8.2 Solving Systems by Substitution

Solve each system of equations by substitution.

Question 3.
\(\left\{\begin{array}{l}y=2x \\x+y=-9\end{array}\right.\)
(________ , ________)

Answer:
(-3, -6)

Explanation:
y = 2x
x + y = -9
Substitute y from equation 1 in the other equation.
x + 2x = -9
3x = -9
x = -9/3
x = -3
Then, y = 2(-3) = -6
The Solution is (-3, -6)

Question 4.
\(\left\{\begin{array}{l}3x-2y=11 \\x+2y=9\end{array}\right.\)
(________ , ________)

Answer:
(5, 2)

Explanation:
3x – 2y = 11
x + 2y = 9
Solve for x in equation 2
x = – 2y + 9
Substitute x from equation 2 in the other equation
3(-2y + 9) – 2y = 11
-6y + 27 -2y = 11
-8y = -16
y = -16/-8 = 2
Substitute y in any of the equations to find x
x + 2(2) = 9
x + 4 – 4 = 9 – 4
x = 5
The solution is (5, 2)

8.3 Solving Systems by Elimination

Solve each system of equations by adding or subtracting.

Question 5.
\(\left\{\begin{array}{l}3x+y=9 \\2x+y=5\end{array}\right.\)
(________ , ________)

Answer:
(4, -3)

Explanation:
\(\left\{\begin{array}{l}3x+y=9 \\2x+y=5\end{array}\right.\)
Subtract the equations
3x + y = 9
-(2x + y = 5)
y is eliminated as it has reversed coefficients. Solve for x
3x + y – 2x – y = 9 – 5
x = 4
Substituting x in either of the equation to find y
2(4) + y = 5
8 + y – 8 = 5 – 8
y = -3
The solution is (4, -3)

Question 6.
\(\left\{\begin{array}{l}-x-2y=4 \\3x+2y=4\end{array}\right.\)
(________ , ________)

Answer:
(4, -4)

Explanation:
\(\left\{\begin{array}{l}-x-2y=4 \\3x+2y=4\end{array}\right.\)
Add the equations
-x – 2y = 4
+(3x + 2y = 4)
y is eliminated as it has reversed coefficients. Solve for x
-x – 2y + 3x + 2y = 4 + 4
2x = 8
x = 8/2 = 4
Substituting x in either of the equation to find y
3(4) + 2y = 4
12 + 2y – 12 = 4 – 12
2y = -8
y = -8/2 = -4
The solution is (4, -4)

8.4 Solving Systems by Elimination with Multiplication

Solve each system of equations by multiplying first.

Question 7.
\(\left\{\begin{array}{l}x+3y=-2 \\3x+4y=-1\end{array}\right.\)
(________ , ________)

Answer:
(1, -1)

Explanation:
\(\left\{\begin{array}{l}x+3y=-2 \\3x+4y=-1\end{array}\right.\)
Subtract the equations
3x + 9y = -6
-(3x + 4y = -1)
x is eliminated as it has reversed coefficients. Solve for y
3x + 9y – 3x – 4y = -6 + 1
5y = -5
y = -5/5
y = -1
Substituting y in either of the equation to find x
x + 3(-1) = -2
x – 3 = -2
x = -2 + 3
x = 1
The solution is (1, -1)

Question 8.
\(\left\{\begin{array}{l}2x+8y=22 \\3x-2y=5\end{array}\right.\)
(________ , ________)

Answer:
(3, 2)

Explanation:
\(\left\{\begin{array}{l}2x+8y=22 \\3x-2y=5\end{array}\right.\)
Multiply equation 2 by 4 so that y can be eliminated
4(3x – 2y = 5)
12x – 8y = 20
Add the equations
2x + 8y = 22
+(12x – 8y = 20)
y is eliminated as it has reversed coefficients. Solve for x
2x + 8y + 12x – 8y = 22 + 20
14x = 42
x = 42/14
x = 3
Substituting y in either of the equation to find x
2(3) + 8y = 22
6 + 8y = 22
8y = 22 – 6
8y = 16
y = 16/8
y = 2
The solution is (3, 2)

8.5 Solving Special Systems

Solve each system. Tell how many solutions each system has.

Question 9.
\(\left\{\begin{array}{l}-2x+8y=5 \\x-4y=-3\end{array}\right.\)
_____________

Answer:
no solution

Explanation:
\(\left\{\begin{array}{l}-2x+8y=5 \\x-4y=-3\end{array}\right.\)
Multiply equation 2 by 2 so that y can be eliminated
2(x – 4y = -3)
2x – 8y = -6
Add the equations
-2x + 8y = 5
+(2x – 8y = -6)
x and y is eliminated
-2x + 8y + 2x – 8y = 5 – 6
0 = -1
The statement is false. Hence, the system has no solution.

Question 10.
\(\left\{\begin{array}{l}6x+18y=-12 \\x+3y=-2\end{array}\right.\)
_____________

Answer:
infinitely many solutions

Explanation:
\(\left\{\begin{array}{l}6x+18y=-12 \\x+3y=-2\end{array}\right.\)
Multiply equation 2 by 6 so that x can be eliminated
6(x + 3y = -2)
6x + 18y = -12
Subtract the equations
6x + 18y = -12
-(6x + 18y = -12)
x and y is eliminated
6x + 18y -6x -18y = -12 + 12
0 = 0
The statement is true. Hence, the system has infinitely many solutions.

ESSENTIAL QUESTION

Question 11.
What are the possible solutions to a system of linear equations, and what do they represent graphically?
Type below:
___________

Answer:
System of linear equations can have no solution, which is represented by parallel lines; one solution, which is represented by intersecting lines; and infinitely many solutions, which is represented by overlapping lines.

Selected Response – Mixed Review – Page No. 266

Question 1.
The graph of which equation is shown?
Go Math Grade 8 Answer Key Chapter 8 Solving Systems of Linear Equations Mixed Review img 25
Options:
A. y = −2x + 2
B. y = −x + 2
C. y = 2x + 2
D. y = 2x + 1

Answer:
C. y = 2x + 2

Explanation:
Option A and B are eliminated as the slope of the graph is 2.
Option D is eliminated as the y-intercept from the graph should be 2.
Option C is the equation of the graph

Question 2.
Which best describes the solutions to the system
\(\left\{\begin{array}{l}x+y=-4 \\-2x-2y=0\end{array}\right.\)
Options:
A. one solution
B. no solution
C. infinitely many
D. (0, 0)

Answer:
B. no solution

Explanation:
\(\left\{\begin{array}{l}x+y=-4 \\-2x-2y=0\end{array}\right.\)
Multply equation 1 by 2 so that x can be eliminated
2(x + y = -4)
2x + 2y = -8
Add the equations
2x + 2y = -8
-2x – 2y = 0
x and y is eliminated
2x + 2y – 2x -2y = -8 + 0
0 = -8
The statement is false. Hence, the system has no solution.

Question 3.
Which of the following represents 0.000056023 written in scientific notation?
Options:
A. 5.6023 × 105
B. 5.6023 × 104
C. 5.6023 × 10-4
D. 5.6023 × 10-5

Answer:
D. 5.6023 × 10-5

Explanation:
Move the decimal 5 points right to get the equation.
D. 5.6023 × 10-5

Question 4.
Which is the solution to
\(\left\{\begin{array}{l}2x-y=1 \\4x+y=11\end{array}\right.\)
Options:
A. (2, 3)
B. (3, 2)
C. (-2, 3)
D. (3, -2)

Answer:
A. (2, 3)

Explanation:
\(\left\{\begin{array}{l}2x-y=1 \\4x+y=11\end{array}\right.\)
Add the equations
2x – y = 1
4x + y = 11
y is eliminated as it has reversed coefficients. Solve for x.
2x – y + 4x + y = 1 + 11
6x = 12
x = 12/6 = 2
Substituting x in either of the equation to find y
4(2) + y = 11
8 + y = 11
y = 11 – 8
y = 3
The solution is (2, 3)

Question 5.
Which expression can you substitute in the indicated equation to solve
\(\left\{\begin{array}{l}3x-y=5 \\x+2y=4\end{array}\right.\)
Options:
A. 2y – 4 for x in 3x – y = 5
B. 4 – x for y in 3x – y = 5
C. 3x – 5 for y in 3x – y = 5
D. 3x – 5 for y in x + 2y = 4

Answer:
D. 3x – 5 for y in x + 2y = 4

Explanation:
\(\left\{\begin{array}{l}3x-y=5 \\x+2y=4\end{array}\right.\)
Solve for y in equation 1
y = 3x – 5
Substitute in other equation x + 2y = 4

Question 6.
What is the solution to the system of linear equations shown on the graph?
Go Math Grade 8 Answer Key Chapter 8 Solving Systems of Linear Equations Mixed Review img 26
Options:
A. -1
B. -2
C. (-1, -2)
D. (-2, -1)

Answer:
C. (-1, -2)

Explanation:
The point of intersection is (-1, -2), which is the solution of the system

Question 7.
Which step could you use to start solving
\(\left\{\begin{array}{l}x-6y=8 \\2x-5y=3\end{array}\right.\)
Options:
A. Add 2x – 5y = 3 to x – 6y = 8.
B. Multiply x – 6y = 8 by 2 and add it to 2x – 5y = 3.
C. Multiply x – 6y = 8 by 2 and subtract it from 2x – 5y = 3.
D. Substitute x = 6y – 8 for x in 2x – 5y = 3.

Answer:
C. Multiply x – 6y = 8 by 2 and subtract it from 2x – 5y = 3.

Explanation:
x – 6y = 8
2x – 5y = 3
Multiply the 1st equation by 2 so that the coefficient of variable x is the same in both equations
Subtract the equations as x has the same sign.

Mini-Task

Question 8.
A hot-air balloon begins rising from the ground at 4 meters per second at the same time a parachutist’s chute opens at a height of 200 meters. The parachutist descends at 6 meters per second.
a. Define the variables and write a system that represents the situation.
Type below:
_____________

Answer:
y represents the distance from the ground and x represents the time in seconds
y = 4x
y = -6x + 200

Question 8.
b. Find the solution. What does it mean?
Type below:
_____________

Answer:
Substitute y from the equation 1 in the equation 2
4x = -6x + 200
4x + 6x = -6x + 200 + 6x
10x = 200
x = 200/10 = 20
Substitute x in any one of the equations and solve for x
y = 4(20) = 80
The solution is (20, 80)
The ballon and parachute meets after 20sec at 80m from the ground.

Final Words:

No Need to go to the tuitions or study hours if you follow our Go Math Grade 8 Answer Key Chapter 8 Solving Systems of Linear Equations PDF. I think the information seen in the Go Math 8th Grade Chapter 8 Solving Systems of Linear Equations is helpful for all the students. Get the step by step explanation for all the questions along with answers from the ccssmathanswers.com site. Bookmark our website to get the solutions for all the chapters in pdf format. All the Best Guys!!!

Go Math Grade 4 Answer Key Chapter 10 Two-Dimensional Figures

Go Math Grade 4 Answer Key Chapter 10 Two-Dimensional Figures will make understanding concepts of Two-Dimensional Figures so easy. So following the Go Math HMH 4th Grade Chapter 10 Solution Key is very necessary to get notified of the topics of Two-dimensional shapes. Go Math Grade 4 Answer Key has specialized solutions to all the questions that are covered in extra practice FL, Homework Practice FL. So it’ll be easy for you to understand the concepts behind each and every lesson.

Go Math Grade 4 Chapter 10 Two-Dimensional Figures Answer Key

The questions asked in Chapter Tests, Practice Tests, Performance Tests, etc. are also covered by the Go Math Grade 4 Answer Key Chapter 10 Two-Dimensional Figures PDF. So that you can cross-check Go Math Grade 4 Chapter 10 Two-Dimensional Figures Solution Key PDF. For more practice questions simply go to the Go Math Grade 4 Solution Key Chapter 10 Two-Dimensional Figures Homework Practice FL Page on our site.

Lesson 1: Lines, Rays, and Angles

Lesson 2: Classify Triangles by Angles

Lesson 3: Parallel Lines and Perpendicular Lines

Lesson 4: Classify Quadrilaterals

Mid-Chapter Checkpoint

Lesson 5: Line Symmetry

Lesson 6: Find and Draw Lines of Symmetry

Lesson 7: Problem Solving • Shape Patterns

Review/Test

Common Core – New – Page No. 553

Lines, Rays, and Angles

Draw and label an example of the figure.

Question 1.
obtuse ∠ABC
Go Math Grade 4 Answer Key Chapter 10 Two-Dimensional Figures Common Core - New img 1
Think: An obtuse angle is greater than a right angle. The middle letter, B, names the vertex of the angle.
_________

Answer:
grade 4 chapter 10 Lines, Rays, and Angles image 1 553

Explanation:
An obtuse angle is greater than a right angle. The middle letter, B, names the vertex of the angle.

Question 2.
\(\overrightarrow{G H}\)
_________

Answer:
grade 4 chapter 10 Lines, Rays, and Angles image 2 553

Explanation:
GH is a ray that has one endpoint and continues without an end in one direction.

Question 3.
acute ∠JKL
_________

Answer:
grade 4 chapter 10 Lines, Rays, and Angles image 3 553

Explanation:
Angle JKL is an acute angle that is less than a right angle.

Question 4.
\(\overline{B C}\)

Answer:
grade 4 chapter 10 Lines, Rays, and Angles image 4 553

Explanation:
BC is a line that continues without an end in both directions.

Use the figure for 5–8.
Go Math Grade 4 Answer Key Chapter 10 Two-Dimensional Figures Common Core - New img 2

Question 5.
Name a line segment.

Answer:
line segment EF

Explanation:

EF line is a straight path of points that continues without an end in both directions.

Question 6.
Name a right angle.
∠ _____

Answer:
∠EJF

Explanation:
EJF is a right angle that forms a square corner.

Question 7.
Name an obtuse angle.
obtuse ∠ _____

Answer:
∠CEJ

Explanation:
CEJ is an obtuse angle that is greater than a right angle.

Question 8.
Name a ray.

Answer:
Ray JD

Explanation:
JD is a ray that has one endpoint and continues without an end in one direction.

Problem Solving

Use the figure at the right for 9–11.
Go Math Grade 4 Answer Key Chapter 10 Two-Dimensional Figures Common Core - New img 3

Question 9.
Classify ∠AFD
_________

Answer:
Obtuse Angle

Explanation:
AFD is an obtuse angle that is greater than a right angle.

Question 10.
Classify ∠CFE.
_________

Answer:
Right Angle

Explanation:
∠CFE is a right angle that forms a square corner.

Question 11.
Name two acute angles.
acute ∠ _____ acute ∠ _____

Answer:
∠AFB and ∠DFE

Explanation:
∠AFB and ∠DFE are two acute angles with less than a right angle.

Common Core – New – Page No. 554

Lesson Check

Question 1.
The hands of a clock show the time 12:25.
Go Math Grade 4 Answer Key Chapter 10 Two-Dimensional Figures Common Core - New img 4
Which best describes the angle between the hands of the clock?
Options:
a. acute
b. right
c. obtuse
d. straight

Answer:
c. obtuse

Explanation:
The hands of the time 12:25 are forming greater than a right angle. So, the answer is the Obtuse angle.

Question 2.
Which of the following name two different figures?
Options:
a. \(\overline{A B} \text { and } \overline{B A}\)
b. \(\stackrel{\longleftrightarrow}{A B}\) and \(\stackrel{\longleftrightarrow}{B A}\)
c. \(\overrightarrow{A B} \text { and } \overrightarrow{B A}\)
d. ∠ABC and ∠CBA

Answer:
c. \(\overrightarrow{A B} \text { and } \overrightarrow{B A}\)

Explanation:
In \(\overrightarrow{A B}\), A is an end point and B continues without end in one direction.
In \(\overrightarrow{B A}\), B is an end point and A continues without an end in one direction.

Spiral Review

Question 3.
Jan’s pencil is 8.5 cm long. Ted’s pencil is longer. Which could be the length of Ted’s pencil?
Options:
a. 0.09 cm
b. 0.8 cm
c. 8.4 cm
d. 9.0 cm

Answer:
d. 9.0 cm

Explanation:
9 ones is greater than 8 ones. So, 9.0 cm > 8.5 cm

Question 4.
Kayla buys a shirt for $8.19. She pays with a $10 bill. How much change should she receive?
Options:
a. $1.81
b. $1.89
c. $2.19
d. $2.81

Answer:
a. $1.81

Explanation:
Kayla buys a shirt for $8.19. She pays with a $10 bill. To find the change she received, $10 – $8.19 = 1.81

Question 5.
Sasha donated \(\frac{9}{100}\) of her class’s entire can collection for the food drive. Which decimal is equivalent to \(\frac{9}{100}\) ?
Options:
a. 9
b. 0.99
c. 0.9
d. 0.09

Answer:
d. 0.09

Explanation:
\(\frac{9}{100}\) is 9 hundredths. So, the decimal is 0.09.

Question 6.
Jose jumped 8 \(\frac{1}{3}\) feet. This was 2 \(\frac{2}{3}\) feet farther than Lila jumped. How far did Lila jump?
Options:
a. 5 \(\frac{1}{3}\)
b. 5 \(\frac{2}{3}\)
c. 6 \(\frac{1}{3}\)
d. 11

Answer:
b. 5 \(\frac{2}{3}\)

Explanation:
Jose jumped 8 \(\frac{1}{3}\) feet. This was 2 \(\frac{2}{3}\) feet farther than Lila jumped.
8 \(\frac{1}{3}\) – 2 \(\frac{2}{3}\) = \(\frac{25}{3}\) – \(\frac{8}{3}\) = \(\frac{7}{3}\) = 5 \(\frac{2}{3}\)

Page No. 557

Question 1.
Name the triangle. Tell whether each angle is acute, right, or obtuse.
A name for the triangle is __________ .
Go Math Grade 4 Answer Key Chapter 10 Two-Dimensional Figures img 5
Name:
∠F is _________
∠G is _________
∠H is _________

Answer:
Right Triangle; Triangle FGH;
∠F and ∠H are acute angles.
∠G is Right angle

Explanation:
∠F and ∠H are acute angles with less than a right angle. ∠G is the Right angle that forms a square corner. A triangle that has one right angle is called a right triangle.

Classify each triangle. Write acute, right, or obtuse.

Question 2.
Go Math Grade 4 Answer Key Chapter 10 Two-Dimensional Figures img 6
_____

Answer:

Obtuse triangle;
Angle B and Angle C are both acute.
Angle A is obtuse.

Explanation:
From triangle ABC, Angle B, and Angle C are both acute with less than a right angle. Angle A is obtuse angle that is greater than a right angle.

Question 3.
Go Math Grade 4 Answer Key Chapter 10 Two-Dimensional Figures img 7
_____

Answer:
grade 4 chapter 10 Lines, Rays, and Angles image 2 557
Obtuse triangle;
Angle A and Angle C are both acute.
Angle B is obtuse.

Explanation:
From triangle ABC, Angle A, and Angle C are both acute with less than a right angle. Angle B is an obtuse angle that is greater than a right angle. A triangle with an obtuse angle is called an obtuse triangle.

Question 4.
Go Math Grade 4 Answer Key Chapter 10 Two-Dimensional Figures img 8
_____

Answer:
grade 4 chapter 10 Lines, Rays, and Angles image 3 557
Acute triangle;
Angle A, Angle B, and Angle C are acute angles.

Explanation:
From triangle ABC, Angle A, Angle B, and Angle C are acute angles with less than a right angle. A triangle with three acute angles called an acute triangle. So, the given triangle is an acute triangle.

Question 5.
Go Math Grade 4 Answer Key Chapter 10 Two-Dimensional Figures img 9
_____

Answer:
grade 4 chapter 10 Lines, Rays, and Angles image 4 557
Right Triangle; Triangle ABC;
∠A and ∠C are acute angles.
∠B is Right angle

Explanation:
∠A and ∠C are acute angles with less than a right angle. ∠B is the Right angle that forms a square corner. A triangle that has one right angle is called a right triangle.

Question 6.
Go Math Grade 4 Answer Key Chapter 10 Two-Dimensional Figures img 10
_____

Answer:
grade 4 chapter 10 Lines, Rays, and Angles image 5 557
Acute triangle;
Angle A, Angle B, and Angle C are acute angles.

Explanation:
From triangle ABC, Angle A, Angle B, and Angle C are acute angles with less than a right angle. A triangle with three acute angles called an acute triangle. So, the given triangle is an acute triangle.

Question 7.
Go Math Grade 4 Answer Key Chapter 10 Two-Dimensional Figures img 11
_____

Answer:
grade 4 chapter 10 Lines, Rays, and Angles image 6 557
Right Triangle;
∠A and ∠C are acute angles.
∠B is Right angle

Explanation:
∠A and ∠C are acute angles with less than a right angle. ∠B is the Right angle that forms a square corner. A triangle that has one right angle is called a right triangle.

Question 8.
Cross out the figure that does not belong. Explain.
Go Math Grade 4 Answer Key Chapter 10 Two-Dimensional Figures img 12
Type below:
_________

Answer:
grade 4 chapter 10 Lines, Rays, and Angles image 7 557
Explanation:
From the given image, 1, 3, and 4 have two acute angles, and one obtuse angle. 2 have three acute angles.

Page No. 558

Use the Venn diagram for 9–10.
Go Math Grade 4 Answer Key Chapter 10 Two-Dimensional Figures img 13

Question 9.
Which triangles do NOT have an obtuse angle? Explain.
_______ triangles

Answer:
4 triangles;
Triangle DEF, Triangle SPN, Triangle ABC, and Triangle GHP are don’t have an obtuse angle. Triangle DEF, Triangle SPN are acute angles. An acute triangle is a triangle with three acute angles. Triangle ABC, and Triangle GHP are right angles. A right triangle is a triangle with one right angle. The sum of the triangle is 180 degrees. A right triangle has 90 degrees. So, the remaining angles must be acute angles.

Question 10.
How many triangles have at least two acute angles? Explain.
_______ triangles

Answer:
4 triangles;
Triangle DEF, Triangle SPN, Triangle ABC, and Triangle GHP at least two acute angles. Triangle DEF, Triangle SPN are acute angles. An acute triangle is a triangle with three acute angles. Triangle ABC, and Triangle GHP are right angles. A right triangle is a triangle with one right angle and two acute angles.

Question 11.
Use the square shown at the right. Draw a line segment from point M to point P. Name and classify the triangles formed by the line segment.
Go Math Grade 4 Answer Key Chapter 10 Two-Dimensional Figures img 14
Type below:
_________

Answer:
grade 4 chapter 10 Lines, Rays, and Angles image 8 557
Angle MNP and Angle MQP

Explanation:
The line segment from M to P forms Angle MNP and Angle MQP.

Question 12.
Write the letter of the triangle under its correct classification.
Go Math Grade 4 Answer Key Chapter 10 Two-Dimensional Figures img 15
Type below:
_________

Answer:
grade 4 chapter 10 Lines, Rays, and Angles image 9 557

Explanation:
Triangle A and triangle B have three acute angles. So, they are acute triangles. Triangle D and triangle F have one obtuse angle. So, they are obtuse triangles.
Triangle C and triangle E have one right angle. So, they are right triangles.

Common Core – New – Page No. 559

Classify Triangles

Classify each triangle. Write acute, right, or obtuse.

Question 1.
Go Math Grade 4 Answer Key Chapter 10 Two-Dimensional Figures Common Core - New img 16
Think: Angles A and C are both acute.
Angle B is obtuse.

Answer:
Obtuse triangle;
Angle A and Angle C are both acute.
Angle B is obtuse.

Explanation:
From triangle ABC, Angle A, and Angle C are both acute with less than a right angle. Angle B is an obtuse angle that is greater than a right angle.

Question 2.
Go Math Grade 4 Answer Key Chapter 10 Two-Dimensional Figures Common Core - New img 17
_________

Answer:
Right Triangle; Triangle DEF;
∠D and ∠F are acute angles.
∠E is Right angle

Explanation:
∠D and ∠F are acute angles with less than a right angle. ∠E is the Right angle that forms a square corner. A triangle that has one right angle is called a right triangle.

Question 3.
Go Math Grade 4 Answer Key Chapter 10 Two-Dimensional Figures Common Core - New img 18
_________

Answer:
Acute triangle;
Angle G, Angle J, and Angle H are acute angles.

Explanation:
From triangle GJH, Angle G, Angle J, and Angle H are acute angles with less than a right angle. A triangle with three acute angles called an acute triangle. So, the given triangle is an acute triangle.

Question 4.
Go Math Grade 4 Answer Key Chapter 10 Two-Dimensional Figures Common Core - New img 19
_________

Answer:
Obtuse triangle;
Angle L and Angle N are both acute.
Angle M is obtuse.

Explanation:
From triangle LMN, Angle L and Angle N are both acute with less than a right angle. Angle M is an obtuse angle that is greater than a right angle. A triangle with an obtuse angle is called an obtuse triangle.

Problem Solving

Question 5.
Use figure ABCD below. Draw a line segment from point B to point D. Name and classify the triangles formed.
Go Math Grade 4 Answer Key Chapter 10 Two-Dimensional Figures Common Core - New img 20
Two _________ triangles
△ _________
△ _________

Answer:
grade 4 chapter 10 Lines, Rays, and Angles image 1 559
Two Acute triangles.
△ ABD
△ BCD

Explanation:
If we draw a line segment from point B to point D, then there are two traingles formed with less than right angles. They are △ ABD and △ BCD.

Question 6.
Use figure ABCD below. Draw a line segment from point A to point C. Name and classify the triangles formed.
Go Math Grade 4 Answer Key Chapter 10 Two-Dimensional Figures Common Core - New img 21
Two _________ triangles
△ _________
△ _________

Answer:
grade 4 chapter 10 Lines, Rays, and Angles image 2 559
Two Acute triangles.
△ ABC
△ ADC

Explanation:
If we draw a line segment from point A to point C, then there are two traingles formed with less than right angles. They are △ ABC and △ ADC.

Common Core – New – Page No. 560

Lesson Check

Question 1.
Stephen drew this triangle. How many obtuse angles does the triangle have?
Go Math Grade 4 Answer Key Chapter 10 Two-Dimensional Figures Common Core - New img 22
Options:
a. 0
b. 1
c. 2
d. 3

Answer:
a. 0

Explanation:
The given image has three acute angles. So, there are 0 obtuse angles.

Question 2.
Joan was asked to draw a right triangle. How many right angles are in a right triangle?
Options:
a. 0
b. 1
c. 2
d. 3

Answer:
b. 1

Explanation:
A right triangle has only one right angle.

Spiral Review

Question 3.
Oliver drew the figure below to show light traveling from the sun to Earth. Name the figure he drew.
Go Math Grade 4 Answer Key Chapter 10 Two-Dimensional Figures Common Core - New img 23
Options:
a. segment SE
b. ray SE
c. line SE
d. ray ES

Answer:
b. ray SE

Explanation:
SE is a ray that has one endpoint and continues without an end in one direction.

Question 4.
Armon added \(\frac{1}{10}\) and \(\frac{8}{100}\). Which is the correct sum?
Options:
a. \(\frac{18}{10}\)
b. \(\frac{9}{10}\)
c. \(\frac{9}{100}\)
d. \(\frac{18}{100}\)

Answer:
d. \(\frac{18}{100}\)

Explanation:
\(\frac{1 X 10}{10 X 10}\) + \(\frac{8}{100}\) = \(\frac{10}{100}\) + \(\frac{8}{100}\) = \(\frac{18}{100}\)

Question 5.
Sam counted out loud by 6s. Jorge counted out loud by 8s. What are the first three numbers both students said?
Options:
a. 8, 16, 24
b. 14, 28, 42
c. 24, 48, 72
d. 48, 96, 144

Answer:
c. 24, 48, 72

Explanation:
Sam counted out loud by 6s = 6, 12, 18, 24, 30, 36, 42, 48, 54, 60, 66, 72.
Jorge counted out loud by 8s = 8, 16, 24, 32, 40, 48, 56, 64, 72, 80.
Both students said the first three numbers are 24, 48, 72.

Question 6.
A basketball team averaged 105 points per game. How many points did the team score in 6 games?
Options:
a. 605 points
b. 630 points
c. 900 points
d. 6,030 points

Answer:
b. 630 points

Explanation:
A basketball team averaged 105 points per game.
They score in 6 games = 6 x 105 = 630 points.

Page No. 563

Question 1.
Draw and label \(\overline{Q R} \| \overline{S T}\).
Think: Parallel lines never intersect. Parallel line segments are parts of parallel lines.
Type below:
_________

Answer:
grade 4 chapter 10 Lines, Rays, and Angles image 1 563
\(\overline{Q R} \| \overline{S T}\)

Explanation:
Parallel lines never intersect. Parallel line segments are parts of parallel lines.

Use the figure for 2 and 3.
Go Math Grade 4 Answer Key Chapter 10 Two-Dimensional Figures img 24

Question 2.
Name two line segments that appear to be parallel.
Type below:
_________

Answer:
Line Segment CB and Line Segment DF.

Explanation:
Line Segment CB and Line Segment DF are parallel lines. The both lines never intersect and are always the same distance apart.

Question 3.
Name two line segments that appear to be perpendicular.
Type below:
_________

Answer:
Line Segment CB and Line Segment BF are perpendicular lines.

Explanation:
Line Segment CB and Line Segment DF are perpendicular lines. Both lines intersect to form four right angles.

Use the figure for 4–5.
Go Math Grade 4 Answer Key Chapter 10 Two-Dimensional Figures img 25

Question 4.
Name a pair of lines that are perpendicular.
Type below:
_________

Answer:
FJ and HG are perpendicular lines.

Explanation:
FJ and HG lines intersect each other and form four right angles.

Question 5.
Name a pair of lines that appear to be parallel.
Type below:
_________

Answer:
DC and FJ are parallel lines.

Explanation:
DC and FJ are never intersected and are always the same distance apart.

Question 6.
\(\overline{R S} \| \overline{T U}\)
Type below:
_________

Answer:
grade 4 chapter 10 Lines, Rays, and Angles image 3 559
\(\overline{R S} \| \overline{T U}\)

Explanation:
Parallel lines never intersect. Parallel line segments are parts of parallel lines.

Question 7.
\(\overrightarrow{K L} \text { and } \overrightarrow{K M}\)
Type below:
_________

Answer:
grade 4 chapter 10 Lines, Rays, and Angles image 4 559

Explanation:
KL and KM are two rays and start at the same point K.

Question 8.
\(\overline{C D} \perp \overline{D E}\)
Type below:
_________

Answer:
grade 4 chapter 10 Lines, Rays, and Angles image 5 559
\(\overline{C D} \perp \overline{D E}\)

Explanation:
\(\overline{C D} \perp \overline{D E}\) are two lines. They are intersect each other and form four right angles.

Question 9.
\(\overset { \longleftrightarrow }{ JK } \) ⊥ \(\overset { \longleftrightarrow }{ LM } \)
Type below:
_________

Answer:
grade 4 chapter 10 Lines, Rays, and Angles image 5 559
\(\overset { \longleftrightarrow }{ JK } \) ⊥ \(\overset { \longleftrightarrow }{ LM } \)

Explanation:
JK and LM are two lines and intersected each other to form right angles.

Question 10.
\(\overset { \longleftrightarrow }{ ST } \) intersecting \(\overset { \longleftrightarrow }{ UV } \) at point X
Type below:
_________

Answer:
grade 4 chapter 10 Lines, Rays, and Angles image 7 559

Explanation:
ST and UV are two lines intersecting at point X.

Question 11.
\(\overset { \longleftrightarrow }{ AB } \) || \(\overset { \longleftrightarrow }{ FG } \)
Type below:
_________

Answer:
grade 4 chapter 10 Lines, Rays, and Angles image 8 559
\(\overset { \longleftrightarrow }{ AB } \) || \(\overset { \longleftrightarrow }{ FG } \)

Explanation:
Parallel lines never intersect. Parallel line segments are parts of parallel lines.

Use the figure for 12–13.
Go Math Grade 4 Answer Key Chapter 10 Two-Dimensional Figures img 26

Question 12.
Dan says that \(\overset { \longleftrightarrow }{ HL } \) is parallel to \(\overset { \longleftrightarrow }{ IM } \). Is Dan correct? Explain.
_____

Answer:
No; HL and IM are not parallel lines. Parallel lines are always the same distance apart. But from the given image, the H and I may intersect if the line is extended.

Question 13.
Name two intersecting line segments that are not perpendicular.
Type below:
_________

Answer:
JM and KG are two intersecting line segments and also not perpendicular.

Page No. 564

Use the house plan at the right for 14–16.
Go Math Grade 4 Answer Key Chapter 10 Two-Dimensional Figures img 27

Question 14.
What geometric term describes a corner of the living room?
_________

Answer:
The corner of the living room is a vertex. Corners of any shape are vertexes.

Question 15.
Name three parts of the plan that show line segments.
_________

Answer:
Kitchen, Living Room, and Master Bedroom.

Question 16.
Name a pair of line segments that appear to be parallel
_________

Answer:
Two sidelines of Living Room are parallel.
Two sidelines of Master Bedroom are parallel.

Use the map at the right for 17–19.
Go Math Grade 4 Answer Key Chapter 10 Two-Dimensional Figures img 28

Question 17.
Name a street that appears to be parallel to S 17th Street.
_________

Answer:
S 18th Street

Explanation:
S 18th Street is parallel to S 17th Street. They never meet each other and are always the same distance apart.

Question 18.
Use Diagrams Name a street that appears to be parallel to Vernon Street.
_________

Answer:
Perry Street

Explanation:
Perry Street is parallel to Vernon Street. They never meet each other and are always the same distance apart.

Question 19.
Name a street that appears to be perpendicular to S 19th Street.
_________

Answer:
Austin Street

Explanation:
Austin Street is perpendicular to S 19th Street. They are intersect with each other and form four right angles.

Question 20.
Choose the labels to make a true statement.
Go Math Grade 4 Answer Key Chapter 10 Two-Dimensional Figures img 29
Go Math Grade 4 Answer Key Chapter 10 Two-Dimensional Figures img 30
Type below:
_________

Answer:
Line AB is perpendicular to Line EF.

Common Core – New – Page No. 565

Parallel Lines and Perpendicular Lines

Use the figure for 1–3.
Go Math Grade 4 Answer Key Chapter 10 Two-Dimensional Figures Common Core - New img 31

Question 1.
Name a pair of lines that appear to be perpendicular.
Think: Perpendicular lines form right angles.
\(\overset { \longleftrightarrow }{ AB } \) and \(\overset { \longleftrightarrow }{ EF } \) appear to form right angles.
\(\overset { \longleftrightarrow }{ AB } \) and \(\overset { \longleftrightarrow }{ EF } \)

Answer:
\(\overset { \longleftrightarrow }{ AB } \) and \(\overset { \longleftrightarrow }{ EF } \)

Explanation:
Perpendicular lines form right angles.
\(\overset { \longleftrightarrow }{ AB } \) and \(\overset { \longleftrightarrow }{ EF } \) appear to form right angles.
\(\overset { \longleftrightarrow }{ AB } \) and \(\overset { \longleftrightarrow }{ EF } \)

Question 2.
Name a pair of lines that appear to be parallel.
_____ and _____

Answer:
\(\overset { \longleftrightarrow }{ AB } \) and \(\overset { \longleftrightarrow }{ CD} \)

Explanation:
Parallel lines never interest each other. \(\overset { \longleftrightarrow }{ AB } \) and \(\overset { \longleftrightarrow }{ CD} \) are parallel lines.

Question 3.
Name another pair of lines that appear to be perpendicular.
_____ and _____

Answer:
\(\overset { \longleftrightarrow }{ CD } \) and \(\overset { \longleftrightarrow }{ EF } \)

Explanation:
Perpendicular lines form right angles.
\(\overset { \longleftrightarrow }{ CD } \) and \(\overset { \longleftrightarrow }{ EF } \) appear to form right angles.
\(\overset { \longleftrightarrow }{ CD } \) and \(\overset { \longleftrightarrow }{ EF } \)

Draw and label the figure described.

Question 4.
\(\overset { \longleftrightarrow }{ MN } \) and \(\overset { \longleftrightarrow }{ PQ } \) intersecting at point R

Answer:
grade 4 chapter 10 Lines, Rays, and Angles image 1 565

Explanation:
MN and PQ are two lines and interesting at point R.

Question 5.
\(\overset { \longleftrightarrow }{ WX } \) || \(\overset { \longleftrightarrow }{ YZ } \)

Answer:
grade 4 chapter 10 Lines, Rays, and Angles image 2 565

Explanation:
WX and YZ are parallel lines and they never intersect with each other.

Question 6.
\(\overset { \longleftrightarrow }{ FH } \) ⊥ \(\overset { \longleftrightarrow }{ JK } \)

Answer:
grade 4 chapter 10 Lines, Rays, and Angles image 3 565

Explanation:
FH and JK are two lines and intersecting each other to form four right angles.

Problem Solving

Use the street map for 7–8.
Go Math Grade 4 Answer Key Chapter 10 Two-Dimensional Figures Common Core - New img 32

Question 7.
Name two streets that intersect but do not appear to be perpendicular.
Type below:
_________

Answer:
Maple and Oak or Oak and Birch

Explanation:
Maple and Oak or Oak and Birch; They are intersecting with each other and not perpendicular.

Question 8.
Name two streets that appear to be parallel to each other.
Type below:
_________

Answer:
Maple and Birch

Explanation:
Maple and Birch are streets and not intersect with each other. They appear to be parallel to each other.

Common Core – New – Page No. 566

Lesson Check

Question 1.
Which capital letter appears to have perpendicular line segments?
Options:
a. N
b. O
c. T
d. V

Answer:
c. T

Explanation:
T has two lines and interesting to form four right angles.

Question 2.
In the figure, which pair of line segments appear to be parallel?
Go Math Grade 4 Answer Key Chapter 10 Two-Dimensional Figures Common Core - New img 33
Options:
a. \(\overline{F G} \text { and } \overline{G H}\)
b. \(\overline{F J} \text { and } \overline{G H}\)
c. \(\overline{F G} \text { and } \overline{J H}\)
d. \(\overline{J H} \text { and } \overline{F J}\)

Answer:
c. \(\overline{F G} \text { and } \overline{J H}\)

Explanation:
\(\overline{F G} \text { and } \overline{J H}\) are parallel lines that never intersect

Spiral Review

Question 3.
Nolan drew a right triangle. How many acute angles did he draw?
Options:
a. 0
b. 1
c. 2
d. 3

Answer:
c. 2

Explanation:
A triangle with one right angle will have two acute angles.

Question 4.
Mike drank more than half the juice in his glass. What fraction of the juice could Mike have drunk?
Options:
a. \(\frac{1}{3}\)
b. \(\frac{2}{5}\)
c. \(\frac{3}{6}\)
d. \(\frac{5}{8}\)

Answer:
d. \(\frac{5}{8}\)

Explanation:
Mike drank more than half the juice in his glass. He drunk \(\frac{5}{8}\) of the juice.

Question 5.
A school principal ordered 1,000 pencils. He gave an equal number to each of 7 teachers until he had given out as many as possible. How many pencils were left?
Options:
a. 2
b. 4
c. 6
d. 142

Answer:
c. 6

Explanation:
A school principal ordered 1,000 pencils. He gave an equal number to each of 7 teachers until he had given out as many as possible. He shared 142 pencils for each of 7 teachers. So, 142 X 7 = 994. The remaining pencils are 6.

Question 6.
A carton of juice contains 64 ounces. Ms. Wilson bought 6 cartons of juice. How many ounces of juice did she buy?
Options:
a. 364 ounces
b. 370 ounces
c. 384 ounces
d. 402 ounces

Answer:
c. 384 ounces

Explanation:
A carton of juice contains 64 ounces. Ms. Wilson bought 6 cartons of juice. 64 X 6 = 384 ounces juice she can buy.

Page No. 569

Question 1.
Tell whether the quadrilateral is also a trapezoid, parallelogram, rhombus, rectangle, or square.
Go Math Grade 4 Answer Key Chapter 10 Two-Dimensional Figures img 34
Think:
____ pairs of parallel sides
____ sides of equal length
____ right angles
Quadrilateral ABCD is also a __________
__________

Answer:
2 pairs of parallel sides
4 sides of equal length
0 right angles.
Quadrilateral ABCD is also a Rhombus

Explanation:
A Rhombus is a quadrilateral that has 2 pairs of parallel sides and 4 sides of equal lengths.

Classify each figure as many ways as possible. Write quadrilateral, trapezoid, parallelogram, rhombus, rectangle, or square.

Question 2.
Go Math Grade 4 Answer Key Chapter 10 Two-Dimensional Figures img 35
__________

Answer:
Quadrilateral

Explanation:
0 pairs of parallel sides
0 sides of equal length
0 right angles.
The given image is quadrilateral.
The quadrilateral doesn’t have a name because it has 0 pairs of parallel sides, 0 sides of equal length, and 0 right angles.

Question 3.
Go Math Grade 4 Answer Key Chapter 10 Two-Dimensional Figures img 36
_________
_________
_________

Answer:
Quadrilateral, Rectangle, and Parallelogram

Explanation:
2 pairs of parallel sides
2 pairs of sides of equal length
4 right angles.
Given quadrilateral is Rectangle and Parallelogram.
A Rectangle is a quadrilateral that has 2 pairs of parallel sides and 2 pairs of sides of equal lengths, and 4 right angles.

Question 4.
Go Math Grade 4 Answer Key Chapter 10 Two-Dimensional Figures img 37
_________
_________
_________

Answer:
Quadrilateral, Parallelogram, and Rhombus

Explanation:
2 pairs of parallel sides
4 sides of equal length
0 right angles.
Given quadrilateral is Rhombus and Parallelogram.
A Rhombus is a quadrilateral that has 2 pairs of parallel sides and 4 sides of equal lengths, and 0 right angles.

Question 5.
Go Math Grade 4 Answer Key Chapter 10 Two-Dimensional Figures img 38
_________
_________

Answer:
Quadrilateral and Parallelogram

Explanation:
2 pairs of parallel sides
2 pairs of sides of equal length
0 right angles.
Given quadrilateral is Parallelogram.
A Parallelogram is a quadrilateral that has 2 pairs of parallel sides and 2 pairs of sides of equal lengths, and 0 right angles.

Question 6.
Go Math Grade 4 Answer Key Chapter 10 Two-Dimensional Figures img 39
_________
_________
_________

Answer:
Quadrilateral and Square

Explanation:
2 pairs of parallel sides
4 sides of equal length
4 right angles.
Given quadrilateral is Square.
A Square is a quadrilateral that has 2 pairs of parallel sides and 4 sides of equal lengths, and 4 right angles.

Question 7.
Go Math Grade 4 Answer Key Chapter 10 Two-Dimensional Figures img 40
_________
_________

Answer:
Quadrilateral and Trapezoid

Explanation:
1 pair of parallel sides
0 sides of equal length
0 right angles.
Given quadrilateral is Trapezoid.
A Square is a quadrilateral that has 1 pair of parallel sides and 0 sides of equal lengths, and 0 right angles.

Page No. 570

Question 8.
Explain how a rhombus and square are alike, and how they are different.
Type below:
_________

Answer:
The rhombus and square have 2 pairs of parallel sides and 4 sides of equal length. But the rhombus has 0 right angles and the square has 4 right angles.

Question 9.
Classify the figure. Select all that apply.
Go Math Grade 4 Answer Key Chapter 10 Two-Dimensional Figures img 41
Options:
a. quadrilateral
b. trapezoid
c. parallelogram
d. rectangle
e. rhombus
f. square

Answer:
a. quadrilateral
b. trapezoid
c. parallelogram

Explanation:
A Parallelogram is a quadrilateral that has 2 pairs of parallel sides and 2 pairs of sides of equal lengths, and 0 right angles.

The Louvre Museum is located in Paris, France. Architect I. M. Pei designed the glass and metal structure at the main entrance of the museum. This structure is called the Louvre Pyramid. Below is a diagram of part of the entrance to the Louvre Pyramid.
Go Math Grade 4 Answer Key Chapter 10 Two-Dimensional Figures img 42
Go Math Grade 4 Answer Key Chapter 10 Two-Dimensional Figures img 43

Question 10.
Describe the quadrilaterals you see in the diagram.
_________
_________

Answer:
Trapezoid and Rhombus

Explanation:
There are 2 quadrilaterals available in the given image. One is Trapezoid with 1 pair of parallel sides. Another one is Rhombus is with 2 pairs of parallel sides and 4 sides of equal lengths, and 0 right angles.

Question 11.
How many triangles do you see in the diagram? Explain.
______ triangles

Answer:
11 triangles

Explanation:
The given image has 11 triangles

Common Core – New – Page No. 571

Classify Quadrilaterals

Classify each figure as many ways as possible. Write quadrilateral, trapezoid, parallelogram, rhombus, rectangle, or square.

Question 1.
Go Math Grade 4 Answer Key Chapter 10 Two-Dimensional Figures Common Core - New img 44
Think: 2 pairs of parallel sides
4 sides of equal length
0 right angles
quadrilateral, parallelogram, rhombus

Answer:
Quadrilateral, Parallelogram, and rhombus.

Explanation:
2 pairs of parallel sides
4 sides of equal length
0 right angles
Quadrilateral, Parallelogram, and rhombus.

Question 2.
Go Math Grade 4 Answer Key Chapter 10 Two-Dimensional Figures Common Core - New img 45
Type below:
_________

Answer:
Quadrilateral, Parallelogram, Rectangle

Explanation:
2 pairs of parallel sides
2 pairs of sides of equal length
4 right angles
Quadrilateral, Parallelogram, Rectangle

Question 3.
Go Math Grade 4 Answer Key Chapter 10 Two-Dimensional Figures Common Core - New img 46
Type below:
_________

Answer:

Explanation:
1 pair of parallel sides
2 sides of equal length
0 right angles
Quadrilateral, Trapezoid

Question 4.
Go Math Grade 4 Answer Key Chapter 10 Two-Dimensional Figures Common Core - New img 47
Type below:
_________

Answer:
Quadrilateral

Explanation:
0 pair of parallel sides
0 sides of equal length
0 right angles
Quadrilateral

Question 5.
Go Math Grade 4 Answer Key Chapter 10 Two-Dimensional Figures Common Core - New img 48
Type below:
_________

Answer:
Quadrilateral, Parallelogram, and rhombus

Explanation:
2 pairs of parallel sides
4 sides of equal length
0 right angles
Quadrilateral, Parallelogram, and rhombus

Question 6.
Go Math Grade 4 Answer Key Chapter 10 Two-Dimensional Figures Go Math Grade 4 Answer Key Chapter 10 Two-Dimensional Figures img 49 img 49
Type below:
_________

Answer:

Explanation:
1 pair of parallel sides
0 sides of equal length
2 right angles
Quadrilateral, Trapezoid

Question 7.
Go Math Grade 4 Answer Key Chapter 10 Two-Dimensional Figures Go Math Grade 4 Answer Key Chapter 10 Two-Dimensional Figures img 50 img 50
Type below:
_________

Answer:

Explanation:
2 pairs of parallel sides
2 pairs of sides of equal length
0 right angles
Quadrilateral, Parallelogram

Problem Solving

Question 8.
Alan drew a polygon with four sides and four angles. All four sides are equal. None of the angles are right angles. What figure did Alan draw
_________

Answer:
Quadrilateral or rhombus

Explanation:
Alan drew a polygon with four sides and four angles. All four sides are equal. None of the angles are right angles. Alan drew Quadrilateral or rhombus

Question 9.
Teresa drew a quadrilateral with 2 pairs of parallel sides and 4 right angles. What quadrilateral could she have drawn?
_________

Answer:
square or rectangle

Explanation:
2 pairs of parallel sides and 4 right angles. she could draw a square or rectangle.

Common Core – New – Page No. 572

Lesson Check

Question 1.
Joey is asked to name a quadrilateral that is also a rhombus. What should be his answer?
Options:
a. square
b. rectangle
c. parallelogram
d. trapezoid

Answer:
a. square

Explanation:
The quadrilateral square is also called a rhombus. Both square and rhombus have 2 pairs of parallel sides and 4 sides of equal length.

Question 2.
Which quadrilateral has exactly one pair of parallel sides?
Options:
a. square
b. rhombus
c. parallelogram
d. trapezoid

Answer:
d. trapezoid

Explanation:
A trapezoid has exactly one pair of parallel sides.

Spiral Review

Question 3.
Terrence has 24 eggs to divide into equal groups. What are all the possible numbers of eggs that Terence could put in each group?
Options:
a. 1, 2, 3, 4
b. 2, 4, 6, 8, 12
c. 1, 2, 3, 4, 6, 8, 12, 24
d. 24, 48, 72, 96

Answer:
c. 1, 2, 3, 4, 6, 8, 12, 24

Explanation:
Terrence has 24 eggs to divide into equal groups. Terence could put in each group in 1, 2, 3, 4, 6, 8, 12, 24 ways.

Question 4.
In a line of students, Jenna is number 8. The teacher says that a rule for a number pattern is add 4. The first student in line says the first term, 7. What number
should Jenna say?
Options:
a. 31
b. 35
c. 39
d. 43

Answer:
b. 35

Explanation:
In a line of students, Jenna is number 8. The teacher says that a rule for a number pattern is add 4. The first student in line says the first term, 7.
7 + 4 = 11
11 + 4 = 15
15 + 4 = 19
19 + 4 = 23
23 + 4 = 27
27 + 4 = 31
31 + 4 = 35.
Jenna says 35.

Question 5.
Lou eats \(\frac{6}{8}\) of a pizza. What fraction of the pizza is left over?
Options:
a. \(\frac{1}{8}\)
b. \(\frac{1}{4}\)
c. \(\frac{1}{2}\)
d. \(\frac{3}{4}\)

Answer:
b. \(\frac{1}{4}\)

Explanation:
Lou eats \(\frac{6}{8}\) of a pizza. So, 6 parts of pizza is finished and remaining 2 parts of pizza is remained. So, the left over pizza is \(\frac{2}{8}\) = \(\frac{1}{4}\).

Question 6.
Which capital letter appears to have parallel lines?
Options:
a. D
b. L
c. N
d. T

Answer:
c. N

Explanation:
N has two parallel lines and never intersect each other.

Page No. 573

Choose the best term from the box to complete the sentence.
Go Math Grade 4 Answer Key Chapter 10 Two-Dimensional Figures img 51

Question 1.
A _______ is part of a line between two endpoints.
_________

Answer:
line segment

Question 2.
A _______ forms a square corner.
_________

Answer:
Right angle

Question 3.
An _______ is greater than a right angle and less than a straight angle.
_________

Answer:
Obtuse angle

Question 4.
The two-dimensional figure that has one endpoint is a ________.
_________

Answer:
ray

Question 5.
An angle that forms a line is called a _______.
_________

Answer:
straight line

Question 6.
On the grid below, draw a polygon that has 2 pairs of parallel sides, 2 pairs of sides equal in length, and 2 acute and 2 obtuse angles. Tell all the possible names for the figure.
Go Math Grade 4 Answer Key Chapter 10 Two-Dimensional Figures img 52
Type below:
_________

Answer:
grade 4 chapter 10 Lines, Rays, and Angles image 1 573
Parallelogram

Explanation:
The possible polygon that has 2 pairs of parallel sides, 2 pairs of sides equal in length, and 2 acute and 2 obtuse angles is Parallelogram.

Draw the figure.

Question 7.
parallel lines
Type below:
_________

Answer:
grade 4 chapter 10 Lines, Rays, and Angles image 1 563

Explanation:
QR and ST are two parallel lines. they never intersect each other.

Question 8.
obtuse ∠ABC
Type below:
_________

Answer:
Go Math Grade 4 Answer Key Chapter 10 Two-Dimensional Figures Common Core - New img 16

Explanation:
From triangle, ABC, Angle A, and Angle C are both acute with less than a right angle. Angle B is an obtuse angle that is greater than a right angle.

Question 9.
intersecting lines that are not perpendicular
Type below:
_________

Answer:

grade 4 chapter 10 Lines, Rays, and Angles image 7 559

Explanation:
ST and UV are two lines intersecting at point X.

Question 10.
acute ∠RST
Type below:
_________

Answer:
grade 4 chapter 10 Lines, Rays, and Angles image 2 575

Page No. 574

Question 11.
Which triangle has one right angle?
_________

Answer:
A right triangle has one right angle.

Question 12.
Which figure has 2 pairs of parallel sides, 2 pairs of sides of equal length, and 4 right angles?
_________

Answer:
A Rectangle has 2 pairs of parallel sides, 2 pairs of sides of equal length, and 4 right angles.

Question 13.
Which quadrilateral can have 2 pairs of parallel sides, all sides with equal length, and no right angles?
_________

Answer:
Rhombus can have 2 pairs of parallel sides, all sides with equal length, and no right angles.

Question 14.
What is the correct name of the figure shown?
Go Math Grade 4 Answer Key Chapter 10 Two-Dimensional Figures img 53
_________

Answer:
Ray

Explanation:
EF is a ray that has one endpoint and continues without an end in one direction.

Question 15.
Describe the angles of an obtuse triangle.
Type below:
_________

Answer:
An obtuse triangle (or obtuse-angled triangle) is a triangle with one obtuse angle (greater than 90°) and two acute angles.

Page No. 577

Tell whether the parts on each side of the line match. Is the line a line of symmetry? Write yes or no.

Question 1.
Go Math Grade 4 Answer Key Chapter 10 Two-Dimensional Figures img 54
____

Answer:
Yes;

Explanation:
The line of symmetry divides a shape into two parts that are the same size and shape.

Question 2.
Go Math Grade 4 Answer Key Chapter 10 Two-Dimensional Figures img 55
____

Answer:
Yes;

Explanation:
The line of symmetry divides a shape into two parts that are the same size and shape.

Question 3.
Go Math Grade 4 Answer Key Chapter 10 Two-Dimensional Figures img 56
____

Answer:
Yes;

Explanation:
The line of symmetry divides a shape into two parts that are the same size and shape.

Question 4.
Go Math Grade 4 Answer Key Chapter 10 Two-Dimensional Figures img 57
____

Answer:
No;

Explanation:
The line of symmetry divides a shape into two parts that are not with the same size and shape.

Tell if the blue line appears to be a line of symmetry. Write yes or no.

Question 5.
Go Math Grade 4 Answer Key Chapter 10 Two-Dimensional Figures img 58
____

Answer:
Yes;

Explanation:
The line of symmetry divides a shape into two parts that are the same size and shape.

Question 6.
Go Math Grade 4 Answer Key Chapter 10 Two-Dimensional Figures img 59
____

Answer:
No;

Explanation:
The line of symmetry divides a shape into two parts that are not with the same size and shape.

Question 7.
Go Math Grade 4 Answer Key Chapter 10 Two-Dimensional Figures img 60
____

Answer:
No;

Explanation:
The line of symmetry divides a shape into two parts that are not with the same size and shape.

Question 8.
Go Math Grade 4 Answer Key Chapter 10 Two-Dimensional Figures img 61
____

Answer:
Yes;

Explanation:
The line of symmetry divides a shape into two parts that are the same size and shape.

Tell if the blue line appears to be a line of symmetry. Write yes or no.

Question 9.
Go Math Grade 4 Answer Key Chapter 10 Two-Dimensional Figures img 62
____

Answer:
No;

Explanation:
The line of symmetry divides a shape into two parts that are not with the same size and shape.

Question 10.
Go Math Grade 4 Answer Key Chapter 10 Two-Dimensional Figures img 63
____

Answer:
Yes;

Explanation:
The line of symmetry divides a shape into two parts that are the same size and shape.

Question 11.
Go Math Grade 4 Answer Key Chapter 10 Two-Dimensional Figures img 64
____

Answer:
No;

Explanation:
The line of symmetry divides a shape into two parts that are not with the same size and shape.

Question 12.
Go Math Grade 4 Answer Key Chapter 10 Two-Dimensional Figures img 65
____

Answer:
Yes;

Explanation:
The line of symmetry divides a shape into two parts that are the same size and shape.

Question 13.
Which best describes the symmetry in the letter I?
Go Math Grade 4 Answer Key Chapter 10 Two-Dimensional Figures img 66
Type below:
________

Answer:
The two parts of the folded I match exactly. The fold line is a line of symmetry.

Explanation:
Take the Horizontal line in the middle of the Letter I. Cut out the tracing. Fold the tracing over a horizontal line. The two parts of the folded I match exactly. The fold line is a line of symmetry.

Page No. 578

Question 14.
Which shape has a correctly drawn line of symmetry?
Go Math Grade 4 Answer Key Chapter 10 Two-Dimensional Figures img 67
Go Math Grade 4 Answer Key Chapter 10 Two-Dimensional Figures img 68
Go Math Grade 4 Answer Key Chapter 10 Two-Dimensional Figures img 69
Go Math Grade 4 Answer Key Chapter 10 Two-Dimensional Figures img 70
a. What do you need to find?
Type below:
________

Answer:
Find the shape that has an exact line of symmetry.

Question 14.
b. How can you tell if the line of symmetry is correct?
Type below:
________

Answer:
If the two parts of the folded match exactly, then the line is a line of symmetry.

Question 14.
c. Tell how you solved the problem.
Type below:
________

Answer:
From fig 1 to 4, the fig 2 is has a line of symmetry that can exactly separate the two parts equally.

Question 14.
d. Circle the correct shape above.
Type below:
________

Answer:
Go Math Grade 4 Answer Key Chapter 10 Two-Dimensional Figures img 68

Question 15.
Reason Abstractly Draw a line of symmetry in the figure shown.
Go Math Grade 4 Answer Key Chapter 10 Two-Dimensional Figures img 71

Answer:
grade 4 chapter 10 Lines, Rays, and Angles image 1 578

Question 16.
Evie’s birthday is on the 18th of May. Since May is the 5th month, Evie wrote the date as shown.
Go Math Grade 4 Answer Key Chapter 10 Two-Dimensional Figures img 72
Evie says all the numbers she wrote have line symmetry. Is she correct? Explain.

Answer:
No; The number 5 doesn’t have a line of symmetry. So, Evie explanation is wrong.

Common Core – New – Page No. 579

Line Symmetry

Tell if the dashed line appears to be a line of symmetry. Write yes or no.

Question 1.
Go Math Grade 4 Answer Key Chapter 10 Two-Dimensional Figures Common Core - New img 73
yes

Answer:
Yes;

Explanation:
The line of symmetry divides a shape into two parts that are the same size and shape.

Question 2.
Go Math Grade 4 Answer Key Chapter 10 Two-Dimensional Figures Common Core - New img 74
____

Answer:
No;

Explanation:
The line of symmetry divides a shape into two parts that are not with the same size and shape.

Question 3.
Go Math Grade 4 Answer Key Chapter 10 Two-Dimensional Figures Common Core - New img 75
____

Answer:
Yes;

Explanation:
The line of symmetry divides a shape into two parts that are the same size and shape.

Question 4.
Go Math Grade 4 Answer Key Chapter 10 Two-Dimensional Figures Common Core - New img 76
____

Answer:
No;

Explanation:
The line of symmetry divides a shape into two parts that are not with the same size and shape.

Question 5.
Go Math Grade 4 Answer Key Chapter 10 Two-Dimensional Figures Common Core - New img 77
____

Answer:
No;

Explanation:
The line of symmetry divides a shape into two parts that are not with the same size and shape.

Question 6.
Go Math Grade 4 Answer Key Chapter 10 Two-Dimensional Figures Common Core - New img 78
____

Answer:
Yes;

Explanation:
The line of symmetry divides a shape into two parts that are the same size and shape.

Question 7.
Go Math Grade 4 Answer Key Chapter 10 Two-Dimensional Figures Common Core - New img 79
____

Answer:
No;

Explanation:
The line of symmetry divides a shape into two parts that are not with the same size and shape.

Question 8.
Go Math Grade 4 Answer Key Chapter 10 Two-Dimensional Figures Common Core - New img 80
____

Answer:
Yes;

Explanation:
The line of symmetry divides a shape into two parts that are the same size and shape.

Complete the design by reflecting over the line of symmetry.

Question 9.
Go Math Grade 4 Answer Key Chapter 10 Two-Dimensional Figures Common Core - New img 81

Answer:
grade 4 chapter 10 Lines, Rays, and Angles image 4 578

Question 10.
Go Math Grade 4 Answer Key Chapter 10 Two-Dimensional Figures Common Core - New img 82

Answer:
grade 4 chapter 10 Lines, Rays, and Angles image 6 578

Problem Solving

Question 11.
Kara uses the pattern below to make paper dolls. The dashed line represents a line of symmetry. A complete doll includes the reflection of the pattern over the line of symmetry. Complete the design to show what one of Kara’s paper dolls looks like.
Go Math Grade 4 Answer Key Chapter 10 Two-Dimensional Figures Common Core - New img 83

Answer:
grade 4 chapter 10 Lines, Rays, and Angles image 7 578

Common Core – New – Page No. 580

Lesson Check

Question 1.
Which best describes the line of symmetry in the letter D?
Go Math Grade 4 Answer Key Chapter 10 Two-Dimensional Figures Common Core - New img 84
Options:
a. horizontal
b. vertical
c. diagonal
d. half turn

Answer:
a. horizontal

Explanation:
The horizontal line of symmetry in the letter D can exactly separate two parts equally.

Question 2.
Which shape has a correctly drawn line of symmetry?
Options:
a. Go Math Grade 4 Answer Key Chapter 10 Two-Dimensional Figures Common Core - New img 85
b. Go Math Grade 4 Answer Key Chapter 10 Two-Dimensional Figures Common Core - New img 86
c. Go Math Grade 4 Answer Key Chapter 10 Two-Dimensional Figures Common Core - New img 87
d. Go Math Grade 4 Answer Key Chapter 10 Two-Dimensional Figures Common Core - New img 88

Answer:
b. Go Math Grade 4 Answer Key Chapter 10 Two-Dimensional Figures Common Core - New img 86

Explanation:
Image b has the line of symmetry that separates two parts equally.

Spiral Review

Question 3.
The class has 360 unit cubes in a bag. Johnnie divides the unit cubes equally among 8 groups. How many unit cubes will each group get?
Options:
a. 40
b. 44
c. 45
d. 48

Answer:
c. 45

Explanation:
The class has 360 unit cubes in a bag. Johnnie divides the unit cubes equally among 8 groups. 360/8= 45.

Question 4.
There are 5,280 feet in one mile. How many feet are there in 6 miles?
Options:
a. 30,680
b. 31,260
c. 31,608
d. 31,680

Answer:
d. 31,680

Explanation:
There are 5,280 feet in one mile. So, for 6 miles = 6 x 5, 280 = 31,680.

Question 5.
Sue has 4 pieces of wood. The lengths of her pieces of wood are \(\frac{1}{3}\) foot, \(\frac{2}{5}\) foot, \(\frac{3}{10}\) foot, and \(\frac{1}{4}\) foot. Which piece of wood is the shortest?
Options:
a. the \(\frac{1}{3}\) foot piece
b. the \(\frac{2}{5}\) foot piece
c. the \(\frac{3}{10}\) foot piece
d. the \(\frac{1}{4}\) foot piece

Answer:
d. the \(\frac{1}{4}\) foot piece

Explanation:
The lengths of \(\frac{1}{4}\) foot piece is less compared to other lengths.

Question 6.
Alice has \(\frac{1}{5}\) as many miniature cars as Sylvester has. Sylvester has 35 miniature cars. How many miniature cars does Alice have?
Options:
a. 7
b. 9
c. 40
d. 175

Answer:
a. 7

Explanation:
Alice has \(\frac{1}{5}\) as many miniature cars as Sylvester has. Sylvester has 35 miniature cars. Alice have \(\frac{1}{5}\) X 35 = 7 miniature cars.

Page No. 583

Question 1.
The shape at the right has line symmetry. Draw the 2 lines of symmetry.
Go Math Grade 4 Answer Key Chapter 10 Two-Dimensional Figures img 89
Type below:
_________

Answer:
grade 4 chapter 10 Lines, Rays, and Angles image 1 583

Tell whether the shape appears to have zero lines, 1 line, or more than 1 line of symmetry. Write zero, 1, or more than 1.

Question 2.
Go Math Grade 4 Answer Key Chapter 10 Two-Dimensional Figures img 90
_________

Answer:
more than 1

Explanation:
grade 4 chapter 10 Lines, Rays, and Angles image 2 583
There is more than 1 line of symmetries that separates two parts equally.

Question 3.
Go Math Grade 4 Answer Key Chapter 10 Two-Dimensional Figures img 91
_________

Answer:
more than 1

Explanation:
grade 4 chapter 10 Lines, Rays, and Angles image 3 583
There is more than 1 lines of symmetries that separates two parts equally.

Question 4.
Go Math Grade 4 Answer Key Chapter 10 Two-Dimensional Figures img 92
_________

Answer:
1 line

Explanation:
grade 4 chapter 10 Lines, Rays, and Angles image 4 583
There is 1 line of symmetry that separates two parts equally.

Question 5.
Go Math Grade 4 Answer Key Chapter 10 Two-Dimensional Figures img 93
_________

Answer:
zero lines

Explanation:
There is no line of symmetries that separates two parts equally.

Tell whether the shape appears to have zero lines, 1 line, or more than 1 line of symmetry. Write zero, 1, or more than 1.

Question 6.
Go Math Grade 4 Answer Key Chapter 10 Two-Dimensional Figures img 94
_________

Answer:
more than 1

Explanation:
grade 4 chapter 10 Lines, Rays, and Angles image 6 583
There is more than 1 lines of symmetries that separate two parts equally.

Question 7.
Go Math Grade 4 Answer Key Chapter 10 Two-Dimensional Figures img 95
_________

Answer:
zero lines

Explanation:
There is no line of symmetries that separates two parts equally.

Question 8.
Go Math Grade 4 Answer Key Chapter 10 Two-Dimensional Figures img 96
_________

Answer:
zero lines

Explanation:
There is no line of symmetries that separates two parts equally.

Question 9.
Go Math Grade 4 Answer Key Chapter 10 Two-Dimensional Figures img 97
_________

Answer:
1 line

Explanation:
grade 4 chapter 10 Lines, Rays, and Angles image 8 583
There is 1 line of symmetry that separates two parts equally.

Practice: Copy and Solve Does the design have line symmetry?
Write yes or no. If your answer is yes, draw all lines of symmetry.

Question 10.
Go Math Grade 4 Answer Key Chapter 10 Two-Dimensional Figures img 98
____

Answer:
Yes;
grade 4 chapter 10 Lines, Rays, and Angles image 8 583

Question 11.
Go Math Grade 4 Answer Key Chapter 10 Two-Dimensional Figures img 99
_____

Answer:
No;

Question 12.
Go Math Grade 4 Answer Key Chapter 10 Two-Dimensional Figures img 100
_____

Answer:
Yes;
grade 4 chapter 10 Lines, Rays, and Angles image 9 578

Question 13.
Go Math Grade 4 Answer Key Chapter 10 Two-Dimensional Figures img 101
_____

Answer:
No;

Question 14.
Draw a figure that has 5 sides and exactly 1 line of symmetry.
Type below:
_________

Answer:

Explanation:
the above 5 sides shape has only 1 line symmetry

Page No. 584

Use the chart for 15–17.
Go Math Grade 4 Answer Key Chapter 10 Two-Dimensional Figures img 102

Question 15.
Which letters appear to have only 1 line of symmetry?
Type below:
_________

Answer:
A, B, C, D, E, T, U, V, W

Explanation:
The letters A, B, C, D, E, T, U, V, W have only 1 line of symmetry.

Question 16.
Which letters appear to have zero lines of symmetry?
Type below:
_________

Answer:
J, N, S

Explanation:
The letters J, N, S have only zero lines of symmetry.

Question 17.
The letter C has horizontal symmetry. The letter A has vertical symmetry. Which letters appear to have both horizontal and vertical symmetry?
Type below:
_________

Answer:
H and I

Explanation:
The letters H and I have both horizontal and vertical symmetry.

Question 18.
Verify the Reasoning of Others Jeff says that the shape has only 2 lines of symmetry.
Does his statement make sense? Explain.
Go Math Grade 4 Answer Key Chapter 10 Two-Dimensional Figures img 103
Type below:
_________

Answer:
No; Jeff’s explanation is wrong. Because the given shape has only 2 lines of symmetry.

Question 19.
Match each figure with the correct number of lines of symmetry it has.
Go Math Grade 4 Answer Key Chapter 10 Two-Dimensional Figures img 104
Type below:
_________

Answer:
grade 4 chapter 10 Lines, Rays, and Angles image 12 583

Common Core – New – Page No. 585

Find and Draw Lines of Symmetry

Tell whether the shape appears to have zero lines, 1 line, or more than 1 line of symmetry. Write zero, 1, or more than 1.

Question 1.
Go Math Grade 4 Answer Key Chapter 10 Two-Dimensional Figures Common Core - New img 105
1

Answer:
more than 1

Explanation:
There is more than 1 line of symmetry that separates two parts equally.

Question 2.
Go Math Grade 4 Answer Key Chapter 10 Two-Dimensional Figures Common Core - New img 106
________

Answer:
more than 1

Explanation:
There is more than 1 line of symmetry that separates two parts equally.

Question 3.
Go Math Grade 4 Answer Key Chapter 10 Two-Dimensional Figures Common Core - New img 107
________

Answer:
Zero

Explanation:
There are 0 lines of symmetries.

Question 4.
Go Math Grade 4 Answer Key Chapter 10 Two-Dimensional Figures Common Core - New img 108
________

Answer:
more than 1

Explanation:
grade 4 chapter 10 Lines, Rays, and Angles image 2 585
There is more than 1 line of symmetry that separates two parts equally.

Does the design have line symmetry? Write yes or no.
If your answer is yes, draw all lines of symmetry.

Question 5.
Go Math Grade 4 Answer Key Chapter 10 Two-Dimensional Figures Common Core - New img 109
_____

Answer:
Yes;
grade 4 chapter 10 Lines, Rays, and Angles image 3 585

Question 6.
Go Math Grade 4 Answer Key Chapter 10 Two-Dimensional Figures Common Core - New img 110
_____

Answer:
Yes;
grade 4 chapter 10 Lines, Rays, and Angles image 5 585

Go Math Grade 4 Answer Key Chapter 10 Two-Dimensional Figures Common Core - New img 111
_____

Answer:
No;

Question 8.
Go Math Grade 4 Answer Key Chapter 10 Two-Dimensional Figures Common Core - New img 112
______

Answer:
Yes;
grade 4 chapter 10 Lines, Rays, and Angles image 6 585

Draw a shape for the statement. Draw the line or lines of symmetry.

Question 9.
zero lines of symmetry
Go Math Grade 4 Answer Key Chapter 10 Two-Dimensional Figures Common Core - New img 113

Answer:
grade 4 chapter 10 Lines, Rays, and Angles image 7 585

Question 10.
1 line of symmetry
Go Math Grade 4 Answer Key Chapter 10 Two-Dimensional Figures Common Core - New img 114

Answer:
grade 4 chapter 10 Lines, Rays, and Angles image 9 585

Question 11.
2 lines of symmetry
Go Math Grade 4 Answer Key Chapter 10 Two-Dimensional Figures Common Core - New img 115

Answer:
grade 4 chapter 10 Lines, Rays, and Angles image 10 585

Problem Solving

Use the chart for 12–13.
Go Math Grade 4 Answer Key Chapter 10 Two-Dimensional Figures Common Core - New img 116

Question 12.
Which number or numbers appear to have only 1 line of symmetry?
_____

Answer:
3

Explanation:
The number 3 has only 1 line of symmetry.

Question 13.
Which number or numbers appear to have 2 lines of symmetry?
_____

Answer:
0 and 8

Explanation:
The numbers 0 and 8 appear to have 2 lines of symmetry.

Common Core – New – Page No. 586

Lesson Check

Question 1.
How many lines of symmetry does this shape appear to have?
Go Math Grade 4 Answer Key Chapter 10 Two-Dimensional Figures Common Core - New img 117
Options:
a. 0
b. 2
c. 6
d. 12

Answer:
c. 6

Explanation:
grade 4 chapter 10 Lines, Rays, and Angles image 1 586
The given shape has 6 lines of symmetries.

Question 2.
Which of the following shapes appears to have exactly 1 line of symmetry?
Options:
a. Go Math Grade 4 Answer Key Chapter 10 Two-Dimensional Figures Common Core - New img 118
b.Go Math Grade 4 Answer Key Chapter 10 Two-Dimensional Figures Common Core - New img 119
c. Go Math Grade 4 Answer Key Chapter 10 Two-Dimensional Figures Common Core - New img 120
d. Go Math Grade 4 Answer Key Chapter 10 Two-Dimensional Figures Common Core - New img 121

Answer:
d. Go Math Grade 4 Answer Key Chapter 10 Two-Dimensional Figures Common Core - New img 121

Explanation:
grade 4 chapter 10 Lines, Rays, and Angles image 2 586
The trapezoid has exactly 1 line of symmetry.

Spiral Review

Question 3.
Richard practiced each of 3 piano solos for \(\frac{5}{12}\) hour. How long did he practice in all?
Options:
a. \(\frac{2}{3}\) hours
b. 1 \(\frac{1}{4}\) hours
c. 1 \(\frac{1}{3}\) hours
d. 1 \(\frac{5}{12}\) hours

Answer:
b. 1 \(\frac{1}{4}\) hours

Explanation:
Richard practiced each of 3 piano solos for \(\frac{5}{12}\) hour. \(\frac{5}{12}\) hour = 1 \(\frac{1}{4}\) hours hours.

Question 4.
Which of the following decimals is equivalent to three and ten hundredths?
Options:
a. 0.30
b. 0.31
c. 3.01
d. 3.1

Answer:
d. 3.1

Explanation:
three and ten hundredths = 310 hundredths = 3.1

Question 5.
Lynne used \(\frac{3}{8}\) cup of flour and \(\frac{1}{3}\) cup of sugar in a recipe. Which number below is a common denominator for \(\frac{3}{8}\) and \(\frac{1}{3}\)?
Options:
a. 8
b. 12
c. 16
d. 24

Answer:
d. 24

Explanation:
Lynne used \(\frac{3}{8}\) cup of flour and \(\frac{1}{3}\) cup of sugar in a recipe. To find the common denominator for \(\frac{3}{8}\) and \(\frac{1}{3}\), multiply 8 X3 and 3 X 8 = 24.

Question 6.
Kevin draws a figure that has four sides. All sides have the same length. His figure has no right angles. What figure does Kevin draw?
Options:
a. square
b. trapezoid
c. rhombus
d. rectangle

Answer:
c. rhombus

Explanation:

Page No. 589

Question 1.
Marisol is making a pattern with blocks. What might the missing shape be?
First, look at the blocks.
Go Math Grade 4 Answer Key Chapter 10 Two-Dimensional Figures img 122
Next, describe the pattern.
Type below:
_________

Answer:
The first image 1 has three line segments. The next shape will have four line segments.

Question 1.
Finally, draw the missing shape.
Go Math Grade 4 Answer Key Chapter 10 Two-Dimensional Figures img 123
Type below:
_________

Answer:
grade 4 chapter 10 Lines, Rays, and Angles image 1 586

Question 2.
Use the shapes to write a number pattern. Then describe the pattern in the numbers.

Answer:
grade 4 chapter 10 Lines, Rays, and Angles image 3 586
Shape 1: Triangle
Shape 2: Square
Shape 3: Pentagon
Shape 4: Hexagon
Shape 5: Heptagon

Question 3.
What if the pattern continued? Write an expression to describe the number of sides the sixth shape has in Marisol’s pattern.
Type below:
_________

Answer:
If the pattern continued, then the next shape will have one more extra line segment to it. The sixth shape will become the octagon.
Shape 6: Octagon

Question 4.
Sahil made a pattern using circles. The first nine circles are shown. Describe the pattern. If Sahil continues the pattern, what might the next three circles be?
Go Math Grade 4 Answer Key Chapter 10 Two-Dimensional Figures img 124
Type below:
_________

Answer:
The pattern is repeated for every three circles. One big circle followed by two small circles.
grade 4 chapter 10 Lines, Rays, and Angles image 6 586

Page No. 590

Use the toy quilt designs for 5–6.
Go Math Grade 4 Answer Key Chapter 10 Two-Dimensional Figures img 125

Question 5.
Lu is making a quilt that is 20 squares wide and has 24 rows. The border of the quilt is made by using each toy design equally as often. Each square can hold one design. How many of each design does she use for the border?
______ times

Answer:
The border will have 20 squares two times, and 24 squares two times as well, that is the perimeter or the border, because a quilt has 4 sides:
20 X 2 + 24 X 2 = 40 + 48 = 88
So, the border will have 88 squares in total. So if the border can have only one design, Lu can use any toy design 88 times if she wants the border to have the same toy design in it.

Question 6.
Communicate Starting in the first square of her quilt, Lu lined up her toy designs in this order: plane, car, fire truck, helicopter, crane, and wagon. Using this pattern unit, which design will Lu place in the fifteenth square? Explain how you found your answer.
_________

Answer:
The answer is fire truck. As the pattern repeats, the fifteenth square will fire truck.

Question 7.
Missy uses 1 hexagonal, 2 rectangular, and 4 triangular pieces of fabric to make 1 bug design for a quilt. If she uses 70 pieces in all to make bug designs, how many of each shape does she use?
Hexagonal: _________ shapes
Rectangular: _________ shapes
Triangular: _________ shapes

Answer:
Hexagonal: 10 shapes
Rectangular: 10 shapes
Triangular: 10 shapes
(1 x 10) + (2 x 10) + (4 x 10) = 10 + 20 + 40 = 70 pieces in all.

Question 8.
Norris drew the pattern shown.
Go Math Grade 4 Answer Key Chapter 10 Two-Dimensional Figures img 126
Label the circles to show the colors in the fourth figure of the pattern.
Type below:
_________

Answer:
grade 4 chapter 10 Lines, Rays, and Angles image 1 591

Common Core – New – Page No. 591

Problem Solving Shape Patterns

Solve each Problem.

Question 1.
Marta is using this pattern to decorate a picture frame. Describe the pattern. Draw what might be the next three figures in the pattern.
Go Math Grade 4 Answer Key Chapter 10 Two-Dimensional Figures Common Core - New img 127
Possible answer: the pattern repeats: one trangle followed by two squares.

Answer:
Go Math Grade 4 Answer Key Chapter 10 Two-Dimensional Figures Common Core - New img 127
The pattern repeats one triangle followed by two squares.

Question 2.
Describe the pattern. Draw what might be the next three figures in the pattern. How many circles are in the sixth figure in the pattern?
Go Math Grade 4 Answer Key Chapter 10 Two-Dimensional Figures Common Core - New img 128
_____ circles

Answer:
grade 4 chapter 10 Lines, Rays, and Angles image 1 591
Add one more column with 1 more circle than in the previous column; 21.

Question 3.
Larry stencils this pattern to make a border at the top of his bedroom walls. Describe the pattern. Draw what might be the missing figure in the pattern.
Go Math Grade 4 Answer Key Chapter 10 Two-Dimensional Figures Common Core - New img 129

Answer:
grade 4 chapter 10 Lines, Rays, and Angles image 2 591
2 triangles placed side to side followed by 2 sets of 2 triangles placed vertex to vertex

Common Core – New – Page No. 592

Lesson Check

Question 1.
What might be the next three figures in this pattern?
Go Math Grade 4 Answer Key Chapter 10 Two-Dimensional Figures Common Core - New img 130
Options:
a. Go Math Grade 4 Answer Key Chapter 10 Two-Dimensional Figures Common Core - New img 131
b. Go Math Grade 4 Answer Key Chapter 10 Two-Dimensional Figures Common Core - New img 132
c. Go Math Grade 4 Answer Key Chapter 10 Two-Dimensional Figures Common Core - New img 133
d. Go Math Grade 4 Answer Key Chapter 10 Two-Dimensional Figures Common Core - New img 134

Answer:
a. Go Math Grade 4 Answer Key Chapter 10 Two-Dimensional Figures Common Core - New img 131

Explanation:
the pattern has odd numbers of up arrows then even number of down arrows. So, the next three figures are Go Math Grade 4 Answer Key Chapter 10 Two-Dimensional Figures Common Core - New img 131.

Question 2.
Which might be the missing figure in the following pattern?
Go Math Grade 4 Answer Key Chapter 10 Two-Dimensional Figures Common Core - New img 135
Options:
a. Go Math Grade 4 Answer Key Chapter 10 Two-Dimensional Figures Common Core - New img 136
b. Go Math Grade 4 Answer Key Chapter 10 Two-Dimensional Figures Common Core - New img 137
c. Go Math Grade 4 Answer Key Chapter 10 Two-Dimensional Figures Common Core - New img 138
d. Go Math Grade 4 Answer Key Chapter 10 Two-Dimensional Figures Common Core - New img 139

Answer:
a. Go Math Grade 4 Answer Key Chapter 10 Two-Dimensional Figures Common Core - New img 136

Explanation:
From the pattern, the missing image will have vertical rectangle with the circle and X mark in it.

Spiral Review

Question 3.
Chad has two pieces of wood. One piece is \(\frac{7}{12}\) foot long. The second piece is \(\frac{5}{12}\) foot longer than the first piece. How long is the second piece?
Options:
a. \(\frac{2}{12}\) foot
b. \(\frac{1}{2}\) foot
c. \(\frac{12}{18}\) foot
d. 1 foot

Answer:
d. 1 foot

Explanation:
\(\frac{7}{12}\) + \(\frac{5}{12}\) = \(\frac{12}{12}\) = 1 foot.

Question 4.
Olivia finished a race in 40.64 seconds. Patty finished the race in 40.39 seconds. Miguel finished the race in 41.44 seconds. Chad finished the race in 40.46 seconds. Who finished the race in the least time?
Options:
a. Olivia
b. Patty
c. Miguel
d. Chad

Answer:
b. Patty

Explanation:
Patty finished the race in 40.39 seconds that is the least time compared to others.

Question 5.
Justin bought 6 ribbons for an art project. Each ribbon is \(\frac{1}{4}\) yard long. How many yards of ribbon did Justin buy?
Options:
a. \(\frac{2}{3}\) yard
b. 1 \(\frac{1}{4}\) yards
c. 1 \(\frac{1}{2}\) yards
d. 1 \(\frac{3}{4}\) yards

Answer:
c. 1 \(\frac{1}{2}\) yards

Explanation:
Justin bought 6 ribbons for an art project. Each ribbon is \(\frac{1}{4}\) yard long. So, 6 X \(\frac{1}{4}\) = \(\frac{3}{2}\) = 1 \(\frac{1}{2}\) yards.

Question 6.
Kyle and Andrea were asked to make a list of prime numbers.
Kyle: 1, 3, 7, 19, 23
Andrea: 2, 3, 5, 7, 11
Whose list is correct?
Options:
a. Only Kyle’s list
b. Only Andrea’s list
c. Both lists are correct.
d. Neither list is correct.

Answer:
b. Only Andrea’s list

Explanation:
1 is not a prime number. So, the answer is Only Andrea’s list is correct.

Page No. 593

Question 1.
Gavin is designing a kite. He sketched a picture of the kite.
How many right angles does the kite appear to have?
Go Math Grade 4 Answer Key Chapter 10 Two-Dimensional Figures img 140
_____ right angles

Answer:
0 right angles

Explanation:
There is no right angles in the given shape.

Question 2.
Write the letter of the triangle under its correct classification.
Go Math Grade 4 Answer Key Chapter 10 Two-Dimensional Figures img 141
Go Math Grade 4 Answer Key Chapter 10 Two-Dimensional Figures img 142

Answer:
grade 4 chapter 10 Lines, Rays, and Angles image 1 593

Explanation:
C and F are Acute angles with less than right angles.
B and D are Obtuse Angles with more than right angles.
A and E are Right Angles.

Question 3.
Select the angles that identify an obtuse triangle. Mark all that apply.
Options:
a. acute, acute, acute
b. acute, acute, obtuse
c. right, acute, acute
d. obtuse, right, acute

Answer:
b. acute, acute, obtuse

Explanation:
An obtuse triangle will have one obtuse angle and two acute angles.

Page No. 594

Question 4.
Write the word that describes the part of Figure A written below.
Go Math Grade 4 Answer Key Chapter 10 Two-Dimensional Figures img 143
Go Math Grade 4 Answer Key Chapter 10 Two-Dimensional Figures img 144
\(\overline{E B}\) _________
\(\overset { \longleftrightarrow }{ AB } \) _________
\(\overrightarrow{G A}\) _________
∠EBG _________
∠CGB _________

Answer:
\(\overline{E B}\) line segment.
\(\overset { \longleftrightarrow }{ AB } \) Line.
\(\overrightarrow{G A}\) Ray.
∠EBG right angle.
∠CGB acute angle.

Explanation:
\(\overline{E B}\) is a line segment that has two endpoints connected to form a line.
\(\overset { \longleftrightarrow }{ AB } \) is a Line that continues without an end in both directions.
\(\overrightarrow{G A}\) is a Ray that has one endpoint and continues without an end in one direction.
∠EBG right angle.
∠CGB is an acute angle with less than the right angle.

Question 5.
What term best describes the figure shown below?
Go Math Grade 4 Answer Key Chapter 10 Two-Dimensional Figures img 145

Answer:
perpendicular lines

Explanation:
The lines are forming four right angles they form squares. So, the both lines are perpendicular lines.

Question 6.
Naomi leaves for her trip to Los Angeles on the 12th day of August. Since August is the 8th month, Naomi wrote the date as shown.
Go Math Grade 4 Answer Key Chapter 10 Two-Dimensional Figures img 146
Naomi says all the numbers she wrote have line symmetry. Is she correct? Explain your thinking.
_______

Answer:
Naomi is incorrect. The number 2 does not have a line of symmetry because if it were cut out, there would be no way to fold it in half so that the two parts matched exactly.

Page No. 595

Question 7.
Max made a pennant that looks like a triangle. How can you classify the triangle based upon its angles?
Go Math Grade 4 Answer Key Chapter 10 Two-Dimensional Figures img 147
The triangle is a(n) ____________ triangle.

Answer:
The triangle is an acute triangle.

Explanation:
The triangle is an acute triangle.  because it has angles with less than right angles.

Question 8.
Choose the labels to make a true statement.
Go Math Grade 4 Answer Key Chapter 10 Two-Dimensional Figures img 148
Go Math Grade 4 Answer Key Chapter 10 Two-Dimensional Figures img 149
_____ is parallel to ______

Answer:
Line AB is parallel to line CD.

Explanation:
From the given image, Line AB is parallel to line CD.

Question 9.
Classify the figure. Select all that apply.
Go Math Grade 4 Answer Key Chapter 10 Two-Dimensional Figures img 150
Options:
a. quadrilateral
b. trapezoid
c. parallelogram
d. rectangle
e. rhombus
f. square

Answer:
a. quadrilateral
b. trapezoid
c. parallelogram
d. rectangle

Explanation:
The given image has 2 parallel sides, 2 pairs of sides of length, and four right angles. So, the possible answers are quadrilateral, trapezoid, parallelogram, and rectangle.

Question 10.
Lily designed a deck in her backyard that looks like a quadrilateral that has only 1 pair of parallel sides. How can you classify the figure?
The quadrilateral is a ________

Answer:
The quadrilateral is a trapezoid

Explanation:
Lily designed a deck in her backyard that looks like a quadrilateral that has only 1 pair of parallel sides. So, the answer is a trapezoid.

Page No. 596

Question 11.
Match each figure with the correct number of lines of symmetry it has.
Go Math Grade 4 Answer Key Chapter 10 Two-Dimensional Figures img 151

Answer:
grade 4 chapter 10 Lines, Rays, and Angles image 1 595

Explanation:
Image 1: 1 line of symmetry
Image 2: 2 lines of symmetry
Image 3: 0 lines of symmetry
Image 4: More than 2 lines of symmetry.

Question 12.
Barb drew the pattern shown.
Go Math Grade 4 Answer Key Chapter 10 Two-Dimensional Figures img 152
Use the square shown to draw the missing pattern. □

Answer:
grade 4 chapter 10 Lines, Rays, and Angles image 1 596

Explanation:
The fourth shape must consist of one extra square box in the top line and bottom line.

Question 13.
Claudia drew the figure below. Draw a line of symmetry on Claudia’s figure.
Go Math Grade 4 Answer Key Chapter 10 Two-Dimensional Figures img 153

Answer:
grade 4 chapter 10 Lines, Rays, and Angles image 2 596

Explanation:
The image can have one line symmetry.

Question 14.
Write the word or words that best describe this figure.
Go Math Grade 4 Answer Key Chapter 10 Two-Dimensional Figures img 154
_________

Answer:
Ray

Explanation:
The ray that has one endpoint and continues without an end in one direction.

Question 15.
How many acute angles does a right triangle have?
A right triangle has ____ acute angles.

Answer:
A right triangle has 2 acute angles.

Page No. 597

Question 16.
Mike drew a figure with opposite sides parallel. Write the pairs of parallel sides. What figure is it?
Go Math Grade 4 Answer Key Chapter 10 Two-Dimensional Figures img 155

Answer:
Line DG is parallel to Line FE and Line DE is parallel to Line GF; the figure is a parallelogram.

Question 17.
Circle the letter that does not have line symmetry.
Go Math Grade 4 Answer Key Chapter 10 Two-Dimensional Figures img 156

Answer:
grade 4 chapter 10 Lines, Rays, and Angles image 4 569

Explanation:
The S does not have line symmetry.

Question 18.
Joseph made a pattern using ovals and rectangles. The first four figures of his pattern are shown. Draw the next figure in the pattern.
Go Math Grade 4 Answer Key Chapter 10 Two-Dimensional Figures img 157

Answer:
grade 4 chapter 10 Lines, Rays, and Angles image 5 596

Question 19.
Jeremy drew Figure 1 and Louisa drew Figure 2.
Go Math Grade 4 Answer Key Chapter 10 Two-Dimensional Figures img 158
Part A
Jeremy says both figures are rectangles. Do you agree with Jeremy?
Support your answer.
_____

Answer:
Yes; both figures have 2 pairs of parallel sides, opposite sides that are equal in length, and 4 right angles.

Question 19.
Part B
Louisa says both figures are rhombuses. Do you agree with Louisa?
Support your answer.
_____

Answer:
No; figure 2 is a rhombus since it has 2 pairs of parallel sides and 4 sides of equal length. Figure 1 does not have 4 sides of equal length so it cannot be a rhombus.

Page No. 598

Question 20.
Veronica found the number of lines of symmetry for the figure below. How many lines of symmetry does it have?
Go Math Grade 4 Answer Key Chapter 10 Two-Dimensional Figures img 159
______ lines of symmetry

Answer:
2 lines of symmetry

Explanation:
the given shape can have 2 lines of symmetry.

Question 21.
Jordan drew the pattern below.
Go Math Grade 4 Answer Key Chapter 10 Two-Dimensional Figures img 160
Part A
Describe the pattern.

Answer:
Each figure has 2 more squares than the preceding figure.

Question 21.
Part B
Write a rule using numbers to find the number of squares in any figure in the pattern.

Answer:
multiply the figure number by 2.

Question 21.
Part C
Draw Figure 5.

Answer:
grade 4 chapter 10 Lines, Rays, and Angles image 1 597

Page No. 603

Tell what fraction of the circle the shaded angle represents.

Question 1.
Go Math Grade 4 Answer Key Chapter 10 Two-Dimensional Figures img 161
\(\frac{□}{□}\)

Answer:
\(\frac{1}{2}\)

Explanation:
Half of the part is shaded out of the circle. So, the answer is 1/2.

Question 2.
Go Math Grade 4 Answer Key Chapter 10 Two-Dimensional Figures img 162
\(\frac{□}{□}\)

Answer:
\(\frac{1}{4}\)

Explanation:
\(\frac{1}{4}\) part of the circle is shaded out of the circle.

Question 3.
Go Math Grade 4 Answer Key Chapter 10 Two-Dimensional Figures img 163
\(\frac{□}{□}\)

Answer:
\(\frac{3}{4}\)

Explanation:
3 parts of the circle is shaded out of the circle

Question 4.
Go Math Grade 4 Answer Key Chapter 10 Two-Dimensional Figures img 164
\(\frac{□}{□}\)

Answer:
\(\frac{1}{1}\) = 1

Explanation:
The complete circle is shaded. So, the answer is 1.

Question 5.
Go Math Grade 4 Answer Key Chapter 10 Two-Dimensional Figures img 165
\(\frac{□}{□}\)

Answer:
\(\frac{1}{2}\)

Explanation:
Half of the part is shaded out of the circle. So, the answer is 1/2.

Question 6.
Go Math Grade 4 Answer Key Chapter 10 Two-Dimensional Figures img 166
\(\frac{□}{□}\)

Answer:
\(\frac{1}{12}\)

Tell whether the angle on the circle shows a \(\frac{1}{4}, \frac{1}{2}, \frac{3}{4}\), or 1 full turn clockwise or counterclockwise.

Question 7.
Go Math Grade 4 Answer Key Chapter 10 Two-Dimensional Figures img 167
Type below:
________

Answer:
\(\frac{3}{4}\); counterclockwise

Explanation:
The image shows the counterclockwise and formed the fraction of \(\frac{3}{4}\).

Question 8.
Go Math Grade 4 Answer Key Chapter 10 Two-Dimensional Figures img 168
Type below:
________

Answer:
\(\frac{1}{2}\); clockwise

Explanation:
The image shows the clockwise and formed the fraction of \(\frac{1}{2}\).

Question 9.
Go Math Grade 4 Answer Key Chapter 10 Two-Dimensional Figures img 169
Type below:
________

Answer:
\(\frac{1}{4}\); clockwise

Explanation:
The image shows the clockwise and formed the fraction of \(\frac{1}{4}\).

Question 10.
Susan watched the game from 1 p.m. to 1:30 p.m. Describe the turn the minute hand made.
Go Math Grade 4 Answer Key Chapter 10 Two-Dimensional Figures img 170
Type below:
________

Answer:
The minute hand made a \(\frac{1}{2}\) turn clockwise.

Question 11.
Compare the angles in Exercises 1 and 5. Does the position of the angle affect the size of the angle? Explain.
_____

Answer:
No; The size of the angle does not depend on the lengths of its sides.

Page No. 604

Question 12.
Malcolm drew this angle on the circle. Which of the following describes the angle? Mark all that apply.
Go Math Grade 4 Answer Key Chapter 10 Two-Dimensional Figures img 171
Options:
a. \(\frac{3}{4}\) turn
b. \(\frac{1}{4}\) turn
c. clockwise
d. counterclockwise

Answer:
a. \(\frac{3}{4}\) turn
d. counterclockwise

Explanation:
The image show the \(\frac{3}{4}\) turn and also the counterclockwise.

Sense or Nonsense?

Question 13.
Whose statement makes sense? Whose statement is nonsense? Explain your reasoning.
Go Math Grade 4 Answer Key Chapter 10 Two-Dimensional Figures img 172
Go Math Grade 4 Answer Key Chapter 10 Two-Dimensional Figures img 173
Type below:
__________

Answer:
The girl’s statement makes sense. The boy’s statement makes non-sense. Because from the figure it is clearly shown that the shaded part is \(\frac{1}{4}\) of the circle. There is no particular direction given to measure the shaded part.

Conclusion:

Finally, you will find detailed solutions to all questions that you’re looking for. So, you can attempt the exam with utmost confidence and secure good grades in the exams. The topics covered in Ch 10 Two-dimensional figures are such Lines, Rays, Angles, Classify Triangles by Angles, Parallel Lines and Perpendicular Lines, etc. Understand how to solve the problems of two-dimensional figures by accessing our Go Math Grade 4 Solution Key Chapter 10 Two-dimensional figures Homework Practice FL.

Go Math Grade 8 Answer Key Chapter 2 Exponents and Scientific Notation

go-math-grade-8-chapter-2-exponents-and-scientific-notation-answer-key

Get Free Access to Download Go Math Grade 8 Answer Key Chapter 2 Exponents and Scientific Notation PDF from here. Start your preparation with the help of Go Math Grade 8 Answer Key. It is essential for all the students to learn the concepts of this chapter in-depth. So, make use of the Go Math Grade 8 Chapter 2 Exponents and Scientific Notation Solution Key links and go through the solutions.

Go Math Grade 8 Chapter 2 Exponents and Scientific Notation Answer Key

Check out the list of the topics before you start your preparation. You can step by step explanation for all the questions in HMH Go Math Grade 8 Answer Key Chapter 2 Exponents and Scientific Notation for free of cost. Quickly download Go Math Grade 8 Chapter 2 Answer Key PDF and fix the timetable to prepare.

Lesson 1: Integer Exponents

Lesson 2: Scientific Notation with Positive Powers of 10

Lesson 3: Scientific Notation with Negative Powers of 10

Lesson 4: Operations with Scientific Notation

Model Quiz

Mixed Review

Guided Practice – Integer Exponents – Page No. 36

Find the value of each power.

Question 1.
8−1 =
\(\frac{□}{□}\)

Answer:
\(\frac{1}{8}\)

Explanation:
Base = 8
Exponent = 1
8−1 = (1/8)1 = 1/8

Question 2.
6−2 =
\(\frac{□}{□}\)

Answer:
\(\frac{1}{36}\)

Explanation:
Base = 6
Exponent = 2
6−2 = (1/6)2 = 1/36

Question 3.
2560 =
______

Answer:
1

Explanation:
2560
Base = 256
Exponent = 0
Anything raised to the zeroth power is 1.
2560 = 1

Question 4.
102 =
______

Answer:
100

Explanation:
Base = 10
Exponent = 2
102 = 10 × 10 = 100

Question 5.
54 =
______

Answer:
625

Explanation:
Base = 5
Exponent = 4
54 = 5 × 5 × 5 × 5 = 625

Question 6.
2−5 =
\(\frac{□}{□}\)

Answer:
\(\frac{1}{32}\)

Explanation:
Base = 2
Exponent = 5
2−5 = (1/2)5 = (1/2) × (1/2) × (1/2) × (1/2) × (1/2) = 1/32

Question 7.
4−5 =
\(\frac{□}{□}\)

Answer:
\(\frac{1}{1,024}\)

Explanation:
Base = 4
Exponent = 5
4−5 = (1/4)5 = (1/4) × (1/4) × (1/4) × (1/4) × (1/4) = 1/1,024

Question 8.
890 =
______

Answer:
1

Explanation:
890
Base = 89
Exponent = 0
Anything raised to the zeroth power is 1.
890 = 1

Question 9.
11−3 =
\(\frac{□}{□}\)

Answer:
\(\frac{1}{1,331}\)

Explanation:
Base = 11
Exponent = 3
11−3 = (1/11)3 = (1/11) × (1/11) × (1/11) = 1/1,331

Use properties of exponents to write an equivalent expression.

Question 10.
4 ⋅ 4 ⋅ 4 = 4?
Type below:
_____________

Answer:
43

Explanation:
The same number 4 is multiplying 3 times.
The number of times a term is multiplied called the exponent.
So the base is 4 and the exponent is 3
4 ⋅ 4 ⋅ 4 = 43

Question 11.
(2 ⋅ 2) ⋅ (2 ⋅ 2 ⋅ 2) = 2? ⋅ 2? = 2?
Type below:
_____________

Answer:
25

Explanation:
The same number 2 is multiplying 5 times.
The number of times a term is multiplied called the exponent.
So the base is 2 and the exponent is 5
(2 ⋅ 2) ⋅ (2 ⋅ 2 ⋅ 2) = 22 ⋅ 23 = 25

Question 12.
\(\frac { { 6 }^{ 7 } }{ { 6 }^{ 5 } } \) = \(\frac{6⋅6⋅6⋅6⋅6⋅6⋅6}{6⋅6⋅6⋅6⋅6}\) = 6?
Type below:
_____________

Answer:
62

Explanation:
\(\frac { { 6 }^{ 7 } }{ { 6 }^{ 5 } } \) = \(\frac{6⋅6⋅6⋅6⋅6⋅6⋅6}{6⋅6⋅6⋅6⋅6}\)
Cancel the common factors
6.6
Base = 6
Exponent = 2
62

Question 13.
\(\frac { { 8 }^{ 12 } }{ { 8 }^{ 9 } } \) = 8?-? = 8?
Type below:
_____________

Answer:
83

Explanation:
\(\frac { { 8 }^{ 12 } }{ { 8 }^{ 9 } } \)
Bases are common. So, the exponents are subtracted
812-9 = 83

Question 14.
510 ⋅ 5 ⋅ 5 = 5?
Type below:
_____________

Answer:
512

Explanation:
Bases are common and multiplied. So, the exponents are added
Base = 5
Exponents = 10 + 1 + 1 = 12
512

Question 15.
78 ⋅ 75 = 7?
Type below:
_____________

Answer:
713

Explanation:
Bases are common and multiplied. So, the exponents are added
Base = 7
Exponents = 8 + 5 = 13
713

Question 16.
(62)4 = (6 ⋅ 6)? = (6 ⋅ 6) ⋅ (6 ⋅ 6) ⋅ (? ⋅ ?) ⋅ ? = 6?
Type below:
_____________

Answer:
68

Explanation:
(62)4 = (6 ⋅ 6)4 = (6 ⋅ 6) ⋅ (6 ⋅ 6) ⋅ (6 ⋅ 6) ⋅ (6 ⋅ 6) = 62 ⋅ 62 . 62 ⋅ 62
Bases are common and multiplied. So, the exponents are added
= 62+2+2+2
68

Question 17.
(33)3 = (3 ⋅ 3 ⋅ 3)3 = (3 ⋅ 3 ⋅ 3) ⋅ (? ⋅ ? ⋅ ?) ⋅ ? = 3?
Type below:
______________

Answer:
39

Explanation:
(3 ⋅ 3 ⋅ 3) ⋅ (3 ⋅ 3 ⋅ 3) ⋅ (3 ⋅ 3 ⋅ 3) = 33 ⋅ 33 ⋅ 33
Bases are common and multiplied. So, the exponents are added
33 + 3 + 3
39

Simplify each expression.

Question 18.
(10 − 6)3⋅42 + (10 + 2)2
______

Answer:
1,168

Explanation:
4³. 4² + (12)² = 45 + (12)² = 45 + (12 . 12)²
45 + (144) = 1,024 + 144 = 1,168

Question 19.
\(\frac { { (12-5) }^{ 7 } }{ { [(3+4)^{ 2 }] }^{ 2 } } \)
________

Answer:
343

Explanation:
77 ÷ (7²)² = 77 ÷ 74
77-4

7 . 7 . 7 = 343

ESSENTIAL QUESTION CHECK-IN

Question 20.
Summarize the rules for multiplying powers with the same base, dividing powers with the same base, and raising a power to a power.
Type below:
______________

Answer:
The exponent “product rule” tells us that, when multiplying two powers that have the same base, you can add the exponents.
The quotient rule tells us that we can divide two powers with the same base by subtracting the exponents.
The “power rule” tells us that to raise a power to a power, just multiply the exponents.

Independent Practice – Integer Exponents – Page No. 37

Question 21.
Explain why the exponents cannot be added in the product 123 ⋅ 113.
Type below:
______________

Answer:
The exponent “product rule” tells us that, when multiplying two powers that have the same base, you can add the exponents.
The bases are not the same in the given problem.
=> (12)³ x (11)³
If we solve this equation following the rule of exponent will get the correct answer:
=> (12 x 12 x 12) x (11 x 11 x 11)
=> 1728 X 1331
=> the answer is 2 299 968
But if we add the exponent, the answer would be wrong
=> (12)³ x (11)³
=> 132^6
=> 5289852801024 which is wrong.

Question 22.
List three ways to express 35 as a product of powers.
Type below:
______________

Answer:
3¹ . 34
3² . 33
3³ . 32

Question 23.
Astronomy
The distance from Earth to the moon is about 224 miles. The distance from Earth to Neptune is about 227 miles. Which distance is the greater distance and about how many times greater is it?
_______ times

Answer:
(22)³ or 10,648 times

Explanation:
The distance from Earth to the moon is about 224 miles. The distance from Earth to Neptune is about 227 miles.
227 – 224  = (22)³
The greatest distance is from Earth to Neptune
The distance from Earth to Neptune is greater by (22)³ or 10,648 miles

Question 24.
Critique Reasoning
A student claims that 83 ⋅ 8-5 is greater than 1. Explain whether the student is correct or not.
______________

Answer:
83 ⋅ 8-5 is = 8-2
(1/8)²
(1/8) . (1/8) = 1/64 = 0.015
The student is not correct.

Find the missing exponent.

Question 25.
(b2)? = b-6
_______

Answer:
(b2)-8

Explanation:
(b2)? = b-6
(b-6) = b2-8
(b2-8) = b2 . b-8
(b2)-8 = b-6

Question 26.
x? ⋅ x6 = x9
_______

Answer:

Explanation:
x? ⋅ x6 = x9
x9 = x3 + 6
x³ x6

Question 27.
\(\frac { { y }^{ 25 } }{ { y }^{ ? } } \) = y6
_______

Answer:
y25 ÷ y16

Explanation:
\(\frac { { y }^{ 25 } }{ { y }^{ ? } } \) = y
y6 = y25 – 16
y25 ÷ y16

Question 28.
Communicate Mathematical Ideas
Why do you subtract exponents when dividing powers with the same base?
Type below:
______________

Answer:
To divide exponents (or powers) with the same base, subtract the exponents. The division is the opposite of multiplication, so it makes sense that because you add exponents when multiplying numbers with the same base, you subtract the exponents when dividing numbers with the same base.

Question 29.
Astronomy
The mass of the Sun is about 2 × 1027 metric tons, or 2 × 1030 kilograms. How many kilograms are in one metric ton?
________ kgs in one metric ton

Answer:
1,000 kgs in one metric ton

Explanation:
The mass of the Sun is about 2 × 1027 metric tons, or 2 × 1030 kilograms.
2 × 1027 metric tons = 2 × 1030 ki
1 metric ton = 2 × 1030 ki ÷ 2 × 1027 = (10)³ = 1,000 kgs in one metric ton

Question 30.
Represent Real-World Problems
In computer technology, a kilobyte is 210 bytes in size. A gigabyte is 230 bytes in size. The size of a terabyte is the product of the size of a kilobyte and the size of a gigabyte. What is the size of a terabyte?
Type below:
______________

Answer:
240 bytes

Explanation:
In computer technology, a kilobyte is 210 bytes in size. A gigabyte is 230 bytes in size. The size of a terabyte is the product of the size of a kilobyte and the size of a gigabyte.
terabyte = 210 bytes × 230 bytes = 210+30 bytes = 240 bytes

Integer Exponents – Page No. 38

Question 31.
Write equivalent expressions for x7 ⋅ x-2 and \(\frac { { x }^{ 7 } }{ { x }^{ 2 } } \). What do you notice? Explain how your results relate to the properties of integer exponents.
Type below:
______________

Answer:
x^a * x^b = x^(a+b)
and
x^-a = 1/x^a
Therefore, x^7 * x^-2 = x^7/x^2 = x^5
or
x^7 * x^-2 = x^(7-2) = x^5
x^7 / x^2 = x^7 * x^-2

A toy store is creating a large window display of different colored cubes stacked in a triangle shape. The table shows the number of cubes in each row of the triangle, starting with the top row.
Go Math Grade 8 Answer Key Chapter 2 Exponents and Scientific Notation Lesson 1: Integer Exponents img 1

Question 32.
Look for a Pattern
Describe any pattern you see in the table.
Type below:
______________

Answer:
As the number of rows increased, the number of cubes in each row by multiple of 3.

Question 33.
Using exponents, how many cubes will be in Row 6? How many times as many cubes will be in Row 6 than in Row 3?
_______ times more cubes

Answer:
(33) times more cubes

Explanation:
For row 6, the number of cubes in each row = (36)
(36) ÷ (33) = (36-3) = (33)
(33) times more cubes

Question 34.
Justify Reasoning
If there are 6 rows in the triangle, what is the total number of cubes in the triangle? Explain how you found your answer.
______ cubes

Answer:
1,092 cubes

Explanation:
(31) + (32) + (33) + (34) + (35) + (36)
3 + 9 + 27 + 81 + 243 + 729 = 1,092

H.O.T.

Focus on Higher Order Thinking

Question 35.
Critique Reasoning
A student simplified the expression \(\frac { { 6 }^{ 2 } }{ { 36 }^{ 2 } } \) as \(\frac{1}{3}\). Do you agree with this student? Explain why or why not.
______________

Answer:
\(\frac { { 6 }^{ 2 } }{ { 36 }^{ 2 } } \)
(62) ÷ (62
(62) ÷ (64)
(62 – 4)
(6-2) = 1/36
I don’t agree with the student

Question 36.
Draw Conclusions
Evaluate –an when a = 3 and n = 2, 3, 4, and 5. Now evaluate (–a)n when a = 3 and n = 2, 3, 4, and 5. Based on this sample, does it appear that –an = (–a)n? If not, state the relationships, if any, between –an and (–a)n.
Type below:
______________

Answer:
–an when a = 3 and n = 2, 3, 4, and 5.
-3n
-(32 )= -9
(–a)n = -3 . -3 = 9
–an = (–a)n are not equal.

Question 37.
Persevere in Problem Solving
A number to the 12th power divided by the same number to the 9th power equals 125. What is the number?
_______

Answer:
Let’s call our number a.
(a12 ) ÷ (a9 )
(a12-9 ) = (a3 )
(a3 ) = 125
a = (125)1/3
a = 5

Guided Practice – Scientific Notation with Positive Powers of 10 – Page No. 42

Write each number in scientific notation.

Question 1.
58,927
(Hint: Move the decimal left 4 places)
Type below:
______________

Answer:
5.8927 × (10)4

Explanation:
58,927
Move the decimal left 4 places
5.8927 × (10)4

Question 2.
1,304,000,000
(Hint: Move the decimal left 9 places.)
Type below:
______________

Answer:
1.304 × (10)9

Explanation:
1,304,000,000
Move the decimal left 9 places
1.304 × (10)9

Question 3.
6,730,000
Type below:
______________

Answer:

Explanation:
6,730,000
Move the decimal left 6 places
6.73 × (10)6

Question 4.
13,300
Type below:
______________

Answer:

Explanation:
13,300
Move the decimal left 4 places
1.33 × (10)4

Question 5.
An ordinary quarter contains about 97,700,000,000,000,000,000,000 atoms.
Type below:
______________

Answer:

Explanation:
97,700,000,000,000,000,000,000
Move the decimal left 22 places
9.77 × (10)22

Question 6.
The distance from Earth to the Moon is about 384,000 kilometers.
Type below:
______________

Answer:
3.84 × (10)6

Explanation:
384,000
Move the decimal left 6 places
3.84 × (10)6

Write each number in standard notation.

Question 7.
4 × 105
(Hint: Move the decimal right 5 places.)
Type below:
______________

Answer:
400,000

Explanation:
4 × 105
Move the decimal right 5 places
400,000

Question 8.
1.8499 × 109
(Hint: Move the decimal right 9 places.)
Type below:
______________

Answer:
1849900000

Explanation:
1.8499 × 109
Move the decimal right 9 places
1849900000

Question 9.
6.41 × 103
Type below:
______________

Answer:
6410

Explanation:
6.41 × 103
Move the decimal right 3 places
6410

Question 10.
8.456 × 107
Type below:
______________

Answer:
84560000

Explanation:
8.456 × 107
Move the decimal right 7 places
84560000

Question 11.
8 × 105
Type below:
______________

Answer:
800,000

Explanation:
8 × 105
Move the decimal right 5 places
800,000

Question 12.
9 × 1010
Type below:
______________

Answer:
90000000000

Explanation:
9 × 1010
Move the decimal right 10 places
90000000000

Question 13.
Diana calculated that she spent about 5.4 × 104 seconds doing her math homework during October. Write this time in standard notation.
Type below:
______________

Answer:
5400

Explanation:
Diana calculated that she spent about 5.4 × 104 seconds doing her math homework during October.
5.4 × 104
Move the decimal right 4 places

5400

Question 14.
The town recycled 7.6 × 106 cans this year. Write the number of cans in standard notation
Type below:
______________

Answer:
7600000

Explanation:
The town recycled 7.6 × 106 cans this year.
7.6 × 106
Move the decimal right 10 places
7600000

ESSENTIAL QUESTION CHECK-IN

Question 15.
Describe how to write 3,482,000,000 in scientific notation.
Type below:
______________

Answer:
3.482 × (10)9

Explanation:
3,482,000,000
Move the decimal left 9 places
3.482 × (10)9

Independent Practice – Scientific Notation with Positive Powers of 10 – Page No. 43

Paleontology

Use the table for problems 16–21. Write the estimated weight of each dinosaur in scientific notation.
Go Math Grade 8 Answer Key Chapter 2 Exponents and Scientific Notation Lesson 2: Scientific Notation with Positive Powers of 10 img 2

Question 16.
Apatosaurus ______________
Type below:
______________

Answer:
6.6 × (10)4

Explanation:
66,000
Move the decimal left 4 places
6.6 × (10)4

Question 17.
Argentinosaurus ___________
Type below:
______________

Answer:
2.2 × (10)5

Explanation:
220,000
Move the decimal left 5 places
2.2 × (10)5

Question 18.
Brachiosaurus ______________
Type below:
______________

Answer:
1 × (10)5

Explanation:
100,000
Move the decimal left 5 places
1 × (10)5

Question 19.
Camarasaurus ______________
Type below:
______________

Answer:
4 × (10)4

Explanation:
40,000
Move the decimal left 4 places
4 × (10)4

Question 20.
Cetiosauriscus ____________
Type below:
______________

Answer:
1.985 × (10)4

Explanation:
19,850
Move the decimal left 4 places
1.985 × (10)4

Question 21.
Diplodocus _____________
Type below:
______________

Answer:
5 × (10)4

Explanation:
50,000
Move the decimal left 4 places
5 × (10)4

Question 22.
A single little brown bat can eat up to 1,000 mosquitoes in a single hour. Express in scientific notation how many mosquitoes a little brown bat might eat in 10.5 hours.
Type below:
______________

Answer:
1.05 × (10)4

Explanation:
(1000 x 10.5) = 10500.
The little brown bat can eat 10500 mosquitoes in 10.5 hours.
1.05 × (10)4

Question 23.
Multistep
Samuel can type nearly 40 words per minute. Use this information to find the number of hours it would take him to type 2.6 × 105 words.
Type below:
______________

Answer:
Samuel can type 40 words per minute.
Then how many hours will it take for him to type 2.6 words times 10 to the power of five words
2.6 words time 10 to the power of 5
2.6 × (10)4
2.6 x 100 000 = 260 000 words in all.
Now, we need to find the number of words Samuel can type in an hour
40 words/minutes, in 1 hour there are 60 minutes
40 x 60
2,400 words /hour
Now, let’s divide the total of words he needs to type to the number of words he can type in an hour
260 000 / 2 400
108.33 hours.

Question 24.
Entomology
A tropical species of mite named Archegozetes longisetosus is the record holder for the strongest insect in the world. It can lift up to 1.182 × 103 times its own weight.
a. If you were as strong as this insect, explain how you could find how many pounds you could lift.
Type below:
______________

Answer:
Number of pounds you can lift by 1.182 × 103 by your weight

Question 24.
b. Complete the calculation to find how much you could lift, in pounds, if you were as strong as an Archegozetes longisetosus mite. Express your answer in both scientific notation and standard notation.
Type below:
______________

Answer:
scientific notation: 1.182 × 105
standard notation: 118200

Explanation:
1.182 × 103 × 102
1.182 × 105
118200

Question 25.
During a discussion in science class, Sharon learns that at birth an elephant weighs around 230 pounds. In four herds of elephants tracked by conservationists, about 20 calves were born during the summer. In scientific notation, express approximately how much the calves weighed all together.
Type below:
______________

Answer:
4.6 × 103

Explanation:
During a discussion in science class, Sharon learns that at birth an elephant weighs around 230 pounds. In four herds of elephants tracked by conservationists, about 20 calves were born during the summer.
Total weight of the claves = 230 × 20 = 4600
Move the decimal left 3 places
4.6 × 103

Question 26.
Classifying Numbers
Which of the following numbers are written in scientific notation?
0.641 × 103          9.999 × 104
2 × 101                 4.38 × 510
Type below:
______________

Answer:
0.641 × 103
4.38 × 510

Scientific Notation with Positive Powers of 10 – Page No. 44

Question 27.
Explain the Error
Polly’s parents’ car weighs about 3500 pounds. Samantha, Esther, and Polly each wrote the weight of the car in scientific notation. Polly wrote 35.0 × 102, Samantha wrote 0.35 × 104, and Esther wrote 3.5 × 104.
a. Which of these girls, if any, is correct?
______________

Answer:
None of the girls is correct

Question 27.
b. Explain the mistakes of those who got the question wrong.
Type below:
______________

Answer:
Polly did not express the number such first part is greater than or equal to 1 and less than 10
Samantha did not express the number such first part is greater than or equal to 1 and less than 10
Esther did not express the exponent of 10 correctly

Question 28.
Justify Reasoning
If you were a biologist counting very large numbers of cells as part of your research, give several reasons why you might prefer to record your cell counts in scientific notation instead of standard notation.
Type below:
______________

Answer:
It is easier to comprehend the magnitude of large numbers when in scientific notation as multiple zeros in the number are removed and express as an exponent of 10.
It is easier to compare large numbers when in scientific notation as numbers are be expressed as a product of a number greater than or equal to 1 and less than 10
It is easier to multiply the numbers in scientific notation.

H.O.T.

Focus on Higher Order Thinking

Question 29.
Draw Conclusions
Which measurement would be least likely to be written in scientific notation: number of stars in a galaxy, number of grains of sand on a beach, speed of a car, or population of a country? Explain your reasoning.
Type below:
______________

Answer:
speed of a car

Explanation:
As we know scientific notation is used to express measurements that are extremely large or extremely small.
The first two are extremely large, then, they could be expressed in scientific notation.
If we compare the speed of a car and the population of a country, it is clear that the larger will be the population of a country.
Therefore, it is more likely to express that in scientific notation, so the answer is the speed of a car.

Question 30.
Analyze Relationships
Compare the two numbers to find which is greater. Explain how you can compare them without writing them in standard notation first.
4.5 × 106              2.1 × 108
Type below:
______________

Answer:
2.1 × 108

Explanation:
2.1 × 108 is greater because the power of 10 is greater in  2.1 × 108

Question 31.
Communicate Mathematical Ideas
To determine whether a number is written in scientific notation, what test can you apply to the first factor, and what test can you apply to the second factor?
Type below:
______________

Answer:
The first term must have one number before the decimal point
the second term (factor) must be 10 having some power.

Guided Practice – Scientific Notation with Negative Powers of 10 – Page No. 48

Write each number in scientific notation.

Question 1.
0.000487
Hint: Move the decimal right 4 places.
Type below:
______________

Answer:
4.87 × 10-4

Explanation:
0.000487
Move the decimal right 4 places
4.87 × 10-4

Question 2.
0.000028
Hint: Move the decimal right 5 places
Type below:
______________

Answer:
2.8 × 10-5

Explanation:
0.000028
Move the decimal right 5 places
2.8 × 10-5

Question 3.
0.000059
Type below:
______________

Answer:
5.9 × 10-5

Explanation:
0.000059
Move the decimal right 5 places
5.9 × 10-5

Question 4.
0.0417
Type below:
______________

Answer:
4.17 × 10-2

Explanation:
0.0417
Move the decimal right 2 places
4.17 × 10-2

Question 5.
Picoplankton can be as small as 0.00002 centimeters.
Type below:
______________

Answer:
2 × 10-5

Explanation:
0.00002
Move the decimal right 5 places
2 × 10-5

Question 6.
The average mass of a grain of sand on a beach is about 0.000015 gram.
Type below:
______________

Answer:
1.5 × 10-5

Explanation:
0.000015
Move the decimal right 5 places
1.5 × 10-5

Write each number in standard notation.

Question 7.
2 × 10-5
Hint: Move the decimal left 5 places.
Type below:
______________

Answer:
0.00002

Explanation:
2 × 10-5
Move the decimal left 5 places
0.00002

Question 8.
3.582 × 10-6
Hint: Move the decimal left 6 places.
Type below:
______________

Answer:
0.000003582

Explanation:
3.582 × 10-6
Move the decimal left 6 places
0.000003582

Question 9.
8.3 × 10-4
Type below:
______________

Answer:
0.00083

Explanation:
8.3 × 10-4
Move the decimal left 4 places
0.00083

Question 10.
2.97 × 10-2
Type below:
______________

Answer:
0.0297

Explanation:
2.97 × 10-2
Move the decimal left 2 places
0.0297

Question 11.
9.06 × 10-5
Type below:
______________

Answer:
0.0000906

Explanation:
9.06 × 10-5
Move the decimal left 5 places
0.0000906

Question 12.
4 × 10-5
Type below:
______________

Answer:
0.00004

Explanation:
4 × 10-5
Move the decimal left 5 places
0.00004

Question 13.
The average length of a dust mite is approximately 0.0001 meters. Write this number in scientific notation.
Type below:
______________

Answer:
1 × 10-4

Explanation:
The average length of a dust mite is approximately 0.0001 meters.
0.0001
Move the decimal right 4 places
1 × 10-4

Question 14.
The mass of a proton is about 1.7 × 10-24 grams. Write this number in standard notation.
Type below:
______________

Answer:
0.000000000000000000000017

Explanation:
The mass of a proton is about 1.7 × 10-24 grams.
1.7 × 10-24
Move the decimal left 24 places
0.000000000000000000000017

ESSENTIAL QUESTION CHECK-IN

Question 15.
Describe how to write 0.0000672 in scientific notation.
Type below:
______________

Answer:
6.72 × 10-5

Explanation:
0.0000672
Move the decimal right 5 places
6.72 × 10-5

Independent Practice – Scientific Notation with Negative Powers of 10 – Page No. 49

Use the table for problems 16–21. Write the diameter of the fibers in scientific notation.
Go Math Grade 8 Answer Key Chapter 2 Exponents and Scientific Notation Lesson 3: Scientific Notation with Negative Powers of 10 img 3

Question 16.
Alpaca _______
Type below:
______________

Answer:
2.77 × 10-3

Explanation:
0.00277
Move the decimal right 3 places
2.77 × 10-3

Question 17.
Angora rabbit _____________
Type below:
______________

Answer:
1.3 × 10-3

Explanation:
0.0013
Move the decimal right 3 places
1.3 × 10-3

Question 18.
Llama ____________
Type below:
______________

Answer:
3.5 × 10-3

Explanation:
0.0035
Move the decimal right 3 places
3.5 × 10-3

Question 19.
Angora goat ____________
Type below:
______________

Answer:
4.5 × 10-3

Explanation:
0.0045
Move the decimal right 3 places
4.5 × 10-3

Question 20.
Orb web spider ___________
Type below:
______________

Answer:
1.5 × 10-2

Explanation:
0.015
Move the decimal right 2 places
1.5 × 10-2

Question 21.
Vicuña __________
Type below:
______________

Answer:
8 × 10-4

Explanation:
0.0008
Move the decimal right 4 places
8 × 10-4

Question 22.
Make a Conjecture
Which measurement would be least likely to be written in scientific notation: the thickness of a dog hair, the radius of a period on this page, the ounces in a cup of milk? Explain your reasoning.
Type below:
______________

Answer:
The ounces in a cup of milk would be least likely to be written in scientific notation. The ounces in a cup of milk is correct.
Scientific notation is used for either very large or extremely small numbers.
The thickness of dog hair is very small as the hair is thin. Hence can be converted to scientific notation.
The radius of a period on this page is also pretty small. Hence can be converted to scientific notation.
The ounces in a cup of milk. There are 8 ounces in a cup, so this is least likely to be written in scientific notation.

Question 23.
Multiple Representations
Convert the length 7 centimeters to meters. Compare the numerical values when both numbers are written in scientific notation
Type below:
______________

Answer:
7 centimeters convert to meters
In every 1 meter, there are 100 centimeters = 7/100 = 0.07
Therefore, in 7 centimeters there are 0.07 meters.
7 cm is a whole number while 0.07 m is a decimal number
Scientific Notation of each number
7 cm = 7 x 10°
7 m = 1 x 10¯²
Scientific notation, by the way, is an expression used by the scientist to make a large number of very small number easy to handle.

Question 24.
Draw Conclusions
A graphing calculator displays 1.89 × 1012 as 1.89E12. How do you think it would display 1.89 × 10-12? What does the E stand for?
Type below:
______________

Answer:
1.89E-12. E= Exponent

Explanation:

Question 25.
Communicate Mathematical Ideas
When a number is written in scientific notation, how can you tell right away whether or not it is greater than or equal to 1?
Type below:
______________

Answer:
A number written in scientific notation is of the form
a × 10-n where 1 ≤ a < 10 and n is an integer
The number is greater than or equal to one if n ≥ 0.

Question 26.
The volume of a drop of a certain liquid is 0.000047 liter. Write the volume of the drop of liquid in scientific notation.
Type below:
______________

Answer:
4.7 × 10-5

Explanation:
The volume of a drop of a certain liquid is 0.000047 liter.
Move the decimal right 5 places
4.7 × 10-5

Question 27.
Justify Reasoning
If you were asked to express the weight in ounces of a ladybug in scientific notation, would the exponent of the 10 be positive or negative? Justify your response.
______________

Answer:
Negative

Explanation:
Scientific notation is used to express very small or very large numbers.
Very small numbers are written in scientific notation using negative exponents.
Very large numbers are written in scientific notation using positive exponents.
Since a ladybug is very small, we would use the very small scientific notation, which uses negative exponents.

Physical Science – Scientific Notation with Negative Powers of 10 – Page No. 50

The table shows the length of the radii of several very small or very large items. Complete the table.
Go Math Grade 8 Answer Key Chapter 2 Exponents and Scientific Notation Lesson 3: Scientific Notation with Negative Powers of 10 img 4

Question 28.
Type below:
______________

Answer:
1.74 × (10)6

Explanation:
The moon = 1,740,000
Move the decimal left 6 places
1.74 × (10)6

Question 29.
Type below:
______________

Answer:
1.25e-10

Explanation:
1.25 × (10)-10
Move the decimal left 10 places
1.25e-10

Question 30.
Type below:
______________

Answer:
2.8 × (10)3

Explanation:
0.0028
Move the decimal left 3 places
2.8 × (10)3

Question 31.
Type below:
______________

Answer:
71490000

Explanation:
7.149 × (10)7
Move the decimal left 7 places
71490000
Question 32.
Type below:
______________

Answer:
1.82 × (10)-10

Explanation:
0.000000000182
Move the decimal right 10 places
1.82 × (10)-10

Question 33.
Type below:
______________

Answer:
3397000

Explanation:
3.397 × (10)6
Move the decimal left 6 places
3397000

Question 34.
List the items in the table in order from the smallest to the largest.
Type below:
______________

Answer:
1.82 × (10)-10
1.25 × (10)-10
2.8 × (10)3
1.74 × (10)6
3.397 × (10)6
7.149 × (10)7

H.O.T.

Focus on Higher Order Thinking

Question 35.
Analyze Relationships
Write the following diameters from least to greatest. 1.5 × 10-2m ; 1.2 × 102 m ; 5.85 × 10-3 m ; 2.3 × 10-2 m ; 9.6 × 10-1 m.
Type below:
______________

Answer:
5.85 × 10-3 m, 1.5 × 10-2m, 2.3 × 10-2 m, 9.6 × 10-1 m, 1.2 × 102 m

Explanation:
1.5 × 10-2m = 0.015
1.2 × 102 m = 120
5.85 × 10-3 m = 0.00585
2.3 × 10-2 m = 0.023
9.6 × 10-1 m = 0.96
0.00585, 0.015, 0.023, 0.96, 120

Question 36.
Critique Reasoning
Jerod’s friend Al had the following homework problem:
Express 5.6 × 10-7 in standard form.
Al wrote 56,000,000. How can Jerod explain Al’s error and how to correct it?
Type below:
______________

Answer:

Explanation:
5.6 × 10-7 in
0.000000056
Al wrote 56,000,000. AI wrote the zeroes to the right side of the 56 which is not correct. As the exponent of 10 is negative zero’s need to add to the left of the number.

Question 37.
Make a Conjecture
Two numbers are written in scientific notation. The number with a positive exponent is divided by the number with a negative exponent. Describe the result. Explain your answer.
Type below:
______________

Answer:
When the division is performed, the denominator exponent is subtracted from the numerator exponent. Subtracting a negative value from the numerator exponent will increase its value.

Guided Practice – Operations with Scientific Notation – Page No. 54

Add or subtract. Write your answer in scientific notation.

Question 1.
4.2 × 106 + 2.25 × 105 + 2.8 × 106
4.2 × 106 + ? × 10 ? + 2.8 × 106
4.2 + ? + ?
? × 10?
Type below:
______________

Answer:
4.2 × 106 + 0.225 × 10 × 105 + 2.8 × 106
Rewrite 2.25 = 0.225 × 10
(4.2 + 0.225 + 2.8) × 106
7.225 × 106

Question 2.
8.5 × 103 − 5.3 × 103 − 1.0 × 102
8.5 × 103 − 5.3 × 103 − ? × 10?
? − ? − ?
? × 10?
Type below:
______________

Answer:
8.5 × 103 − 5.3 × 103 − 0.1 × 103
(8.5 − 5.3 − 0.1) × 103
(3.1) × 103

Question 3.
1.25 × 102 + 0.50 × 102 + 3.25 × 102
Type below:
______________

Answer:
1.25 × 102 + 0.50 × 102 + 3.25 × 102
(1.25 + 0.50 + 3.25) × 102
5 × 102

Question 4.
6.2 × 105 − 2.6 × 104 − 1.9 × 102
Type below:
______________

Answer:
6.2 × 105 − 2.6 × 104 − 1.9 × 102
6.2 × 105 − 0.26 × 105 − 0.0019 × 105
(6.2 – 0.26 – 0.0019) × 105
5.9381 × 105

Multiply or divide. Write your answer in scientific notation.

Question 5.
(1.8 × 109)(6.7 × 1012)
Type below:
______________

Answer:
12.06 × 1021

Explanation:
(1.8 × 109)(6.7 × 1012)
1.8 × 6.7 = 12.06
109+12 = 1021
12.06 × 1021

Question 6.
\(\frac { { 3.46×10 }^{ 17 } }{ { 2×10 }^{ 9 } } \)
Type below:
______________

Answer:
1.73 × 108

Explanation:
3.46/2 = 1.73
1017/109 = 1017-9 = 108
1.73 × 108

Question 7.
(5 × 1012)(3.38 × 106)
Type below:
______________

Answer:
16.9 × 1018

Explanation:
(5 × 1012)(3.38 × 106)
5 × 3.38 = 16.9
106+12 = 1018
16.9 × 1018

Question 8.
\(\frac { { 8.4×10 }^{ 21 } }{ { 4.2×10 }^{ 14 } } \)
Type below:
______________

Answer:
2 × 107

Explanation:
8.4/4.2 = 2
1021/1014 = 1021-14 = 107
2 × 107

Write each number using calculator notation.

Question 9.
3.6 × 1011
Type below:
______________

Answer:
3.6e11

Question 10.
7.25 × 10-5
Type below:
______________

Answer:
7.25e-5

Question 11.
8 × 10-1
Type below:
______________

Answer:
8e-1

Write each number using scientific notation.

Question 12.
7.6E − 4
Type below:
______________

Answer:
7.6 × 10-4

Question 13.
1.2E16
Type below:
______________

Answer:
1.2 × 1016

Question 14.
9E1
Type below:
______________

Answer:
9 × 101

ESSENTIAL QUESTION CHECK-IN

Question 15.
How do you add, subtract, multiply, and divide numbers written in scientific notation?
Type below:
______________

Answer:
Numbers with exponents can be added and subtracted only when they have the same base and exponent.
To multiply two numbers in scientific notation, multiply their coefficients and add their exponents.
To divide two numbers in scientific notation, divide their coefficients, and subtract their exponents.

Independent Practice – Operations with Scientific Notation – Page No. 55

Question 16.
An adult blue whale can eat 4.0 × 107 krill in a day. At that rate, how many krill can an adult blue whale eat in 3.65 × 102 days?
Type below:
______________

Answer:
14.6 × 109

Explanation:
(4.0 × 107 )(3.65 × 102 )
4.0 × 3.65 = 14.6
107+2  =  109
14.6 × 109

Question 17.
A newborn baby has about 26,000,000,000 cells. An adult has about 4.94 × 1013 cells. How many times as many cells does an adult have than a newborn? Write your answer in scientific notation.
Type below:
______________

Answer:
1.9 × 103

Explanation:
26,000,000,000 = 2.6 × 1010
4.94 × 1013
(4.94 × 1013 )/(2.6 × 1010 )
1.9 × 103

Represent Real-World Problems

The table shows the number of tons of waste generated and recovered (recycled) in 2010.
Go Math Grade 8 Answer Key Chapter 2 Exponents and Scientific Notation Lesson 4: Operations with Scientific Notation img 5

Question 18.
What is the total amount of paper, glass, and plastic waste generated?
Type below:
______________

Answer:
11.388 × 107

Explanation:
7.131 × 107 + 1.153 × 107 + 3.104 × 107
11.388 × 107

Question 19.
What is the total amount of paper, glass, and plastic waste recovered?
Type below:
______________

Answer:
5.025 × 107

Explanation:
4.457 × 107  + 0.313 × 107  + 0.255 × 107
5.025 × 107

Question 20.
What is the total amount of paper, glass, and plastic waste not recovered?
Type below:
______________

Answer:
6.363 × 107

Explanation:
(11.388 × 107 )  – (5.025 × 107)
6.363 × 107

Question 21.
Which type of waste has the lowest recovery ratio?
Type below:
______________

Answer:
Plastics

Explanation:
7.131 × 107  – 4.457 × 107 = 2.674 × 107
1.153 × 107  – 0.313 × 107 = 0.84 × 107
3.104 × 107  – 0.255 × 107 = 2.849 × 107
Plastics has the lowest recovery ratio

Social Studies

The table shows the approximate populations of three countries.
Go Math Grade 8 Answer Key Chapter 2 Exponents and Scientific Notation Lesson 4: Operations with Scientific Notation img 6

Question 22.
How many more people live in France than in Australia?
Type below:
______________

Answer:
4.33 × 107

Explanation:
(6.48 × 107 )  – (2.15× 107)
4.33 × 107

Question 23.
The area of Australia is 2.95 × 106 square miles. What is the approximate average number of people per square mile in Australia?
Type below:
______________

Answer:
About 7 people per square mile

Explanation:
2.95 × 106 square miles = (2.15× 107)
1 square mile = (2.15× 107)/(2.95 × 106) = 7.288

Question 24.
How many times greater is the population of China than the population of France? Write your answer in standard notation.
Type below:
______________

Answer:
20.52; there are about 20 people in china for every 1 person in France.

Question 25.
Mia is 7.01568 × 106 minutes old. Convert her age to more appropriate units using years, months, and days. Assume each month to have 30.5 days.
Type below:
______________

Answer:
13 years 3 months 22.5 days

Explanation:
7.01568 × 106 minutes
(7.01568 × 106 minutes) ÷ (6 × 101)(2.4 × 101)(1.2 × 101)(3.05 × 101)
= (1.331 × 101)
= 13 years 3 months 22.5 days

Operations with Scientific Notation – Page No. 56

Question 26.
Courtney takes 2.4 × 104 steps during her a long-distance run. Each step covers an average of 810 mm. What total distance (in mm) did Courtney cover during her run? Write your answer in scientific notation. Then convert the distance to the more appropriate unit kilometers. Write that answer in standard form.
______ km

Answer:
19.4 km

Explanation:
Courtney takes 2.4 × 104 steps during her a long-distance run. Each step covers an average of 810 mm.
(2.4 × 104 steps) × 810mm
(2.4 × 104 ) × (8.1 × 102 )
The total distance covered = (19.44 × 106 )
Convert to unit kilometers:
(19.44 × 106 ) × (1 × 10-6 )
(1.94 × 101 )
19.4 km

Question 27.
Social Studies
The U.S. public debt as of October 2010 was $9.06 × 1012. What was the average U.S. public debt per American if the population in 2010 was 3.08 × 108 people?
$ _______

Answer:
$29,400 per American

Explanation:
($9.06 × 1012.)/(3.08 × 108 )
($2.94 × 104.) = $29,400 per American

H.O.T.

Focus on Higher Order Thinking

Question 28.
Communicate Mathematical Ideas
How is multiplying and dividing numbers in scientific notation different from adding and subtracting numbers in scientific notation?
Type below:
______________

Answer:
When you multiply or divide in scientific notation, you just add or subtract the exponents. When you add or subtract in scientific notation, you have to make the exponents the same before you can do anything else.

Question 29.
Explain the Error
A student found the product of 8 × 106 and 5 × 109 to be 4 × 1015. What is the error? What is the correct product?
Type below:
______________

Answer:
The error student makes is he multiply the terms instead of addition.

Explanation:
product of 8 × 106 and 5 × 109
40 × 1015
4 × 1016
The student missed the 10 while multiplying the product of 8 × 106 and 5 × 109

Question 30.
Communicate Mathematical Ideas
Describe a procedure that can be used to simplify \(\frac { { (4.87×10 }^{ 12 }) – { (7×10 }^{ 10 }) }{ { (3×10 }^{ 7 })-{ (6.1×10 }^{ 8 }) } \). Write the expression in scientific notation in simplified form.
Type below:
______________

Answer:
\(\frac { { (4.87×10 }^{ 12 }) – { (7×10 }^{ 10 }) }{ { (3×10 }^{ 7 })-{ (6.1×10 }^{ 8 }) } \)
\(\frac { { (487×10 }^{ 10 }) – { (7×10 }^{ 10 }) }{ { (3×10 }^{ 7 })-{ (61×10 }^{ 7 }) } \)
(480 × 1010 )/(64 × 107 )
7.50 × 10³

2.1 Integer Exponents – Model Quiz – Page No. 57

Find the value of each power.

Question 1.
3-4
\(\frac{□}{□}\)

Answer:
\(\frac{1}{81}\)

Explanation:
Base = 3
Exponent = 4
3-4 = (1/3)4 = 1/81

Question 2.
350
______

Answer:
1

Explanation:
350
Base = 35
Exponent = 0
Anything raised to the zeroth power is 1.
350 = 1

Question 3.
44
______

Answer:
256

Explanation:
Base = 4
Exponent = 4
44 = 4 . 4 . 4 . 4 = 2561

Use the properties of exponents to write an equivalent expression.

Question 4.
83 ⋅ 87
Type below:
____________

Answer:
810

Explanation:
83 ⋅ 87
83+7
810

Question 5.
\(\frac { 12^{ 6 } }{ 12^{ 2 } } \)
Type below:
____________

Answer:
124

Explanation:
126 ÷ 122
126-2
124

Question 6.
(103)5
Type below:
____________

Answer:
108

Explanation:
(103)5
(103+5)
(108)

2.2 Scientific Notation with Positive Powers of 10

Convert each number to scientific notation or standard notation.

Question 7.
2,000
Type below:
____________

Answer:
2 × (103)

Explanation:
2 × 1,000
Move the decimal left 3 places
2 × (103)

Question 8.
91,007,500
Type below:
____________

Answer:
9.10075 × (107)

Explanation:
91,007,500
Move the decimal left 7 places
9.10075 × (107)

Question 9.
1.0395 × 109
Type below:
____________

Answer:
1039500000

Explanation:
1.0395 × 109
Move the decimal right 9 places
1039500000

Question 10.
4 × 102
Type below:
____________

Answer:
400

Explanation:
4 × 102
Move the decimal right 2 places
400

2.3 Scientific Notation with Negative Powers of 10

Convert each number to scientific notation or standard notation.

Question 11.
0.02
Type below:
____________

Answer:
2 × 10-2

Explanation:
0.02
Move the decimal right 2 places
2 × 10-2

Question 12.
0.000701
Type below:
____________

Answer:
7.01 × 10-4

Explanation:
0.000701
Move the decimal right 4 places
7.01 × 10-4

Question 13.
8.9 × 10-5
Type below:
____________

Answer:
0.000089

Explanation:
8.9 × 10-5
Move the decimal left 5 places
0.000089

Question 14.
4.41 × 10-2
Type below:
____________

Answer:
0.0441

Explanation:
4.41 × 10-2
Move the decimal left 2 places
0.0441

2.4 Operations with Scientific Notation

Perform the operation. Write your answer in scientific notation.

Question 15.
7 × 106 − 5.3 × 106
Type below:
____________

Answer:
1.7 × 106

Explanation:
7 × 106 − 5.3 × 106
(7 – 5.3) × 106
1.7 × 106

Question 16.
3.4 × 104 + 7.1 × 105
Type below:
____________

Answer:
7.44 × 104

Explanation:
3.4 × 104 + 7.1 × 105
0.34 × 105 + 7.1 × 105
(0.34 + 7.1) × 105
7.44 × 105

Question 17.
(2 × 104)(5.4 × 106)
Type below:
____________

Answer:
10.8 × 1010

Explanation:
(2 × 104)(5.4 × 106)
(2 × 5.4)(104 × 106)
10.8 × 1010

Question 18.
\(\frac { 7.86×10^{ 9 } }{ 3×10^{ 4 } } \)
Type below:
____________

Answer:
2.62 × 105

Explanation:
7.86/3 = 2.62
109/104 = 105
2.62 × 105

Question 19.
Neptune’s average distance from the Sun is 4.503×109 km. Mercury’s average distance from the Sun is 5.791 × 107 km. About how many times farther from the Sun is Neptune than Mercury? Write your answer in scientific notation.
Type below:
____________

Answer:
(0.7776 × 102 km) = 77.76 times

Explanation:
As Neptune’s average distance from the sun is 4.503×109 km and Mercury is 5.791 × 107 km
(4.503×109 km)/(5.791 × 107 km)
(0.7776 × 109-7 km)
(0.7776 × 102 km)
77.76 times

Essential Question

Question 20.
How is scientific notation used in the real world?
Type below:
____________

Answer:
Scientific notation is used to write very large or very small numbers using less digits.

Selected Response – Mixed Review – Page No. 58

Question 1.
Which of the following is equivalent to 6-3?
Options:
a. 216
b. \(\frac{1}{216}\)
c. −\(\frac{1}{216}\)
d. -216

Answer:
b. \(\frac{1}{216}\)

Explanation:
Base = 6
Exponent = 3
63 = (1/6)3 = 1/216

Question 2.
About 786,700,000 passengers traveled by plane in the United States in 2010. What is this number written in scientific notation?
Options:
a. 7,867 × 105 passengers
b. 7.867 × 102 passengers
c. 7.867 × 108 passengers
d. 7.867 × 109 passengers

Answer:
c. 7.867 × 108 passengers

Explanation:
786,700,000
Move the decimal left 8 places
7.867 × 108 passengers

Question 3.
In 2011, the population of Mali was about 1.584 × 107 people. What is this number written in standard notation?
Options:
a. 1.584 people
b. 1,584 people
c. 15,840,000 people
d. 158,400,000 people

Answer:
c. 15,840,000 people

Explanation:
1.584 × 107
Move the decimal right 7 places
15,840,000 people

Question 4.
The square root of a number is between 7 and 8. Which could be the number?
Options:
a. 72
b. 83
c. 51
d. 66

Answer:
c. 51

Explanation:
7²= 49
8²=64
(49+64)/2
56.5

Question 5.
Each entry-level account executive in a large company makes an annual salary of $3.48 × 104. If there are 5.2 × 102 account executives in the company, how much do they make in all?
Options:
a. $6.69 × 101
b. $3.428 × 104
c. $3.532 × 104
d. $1.8096 × 107

Answer:
d. $1.8096 × 107

Explanation:
Each entry-level account executive in a large company makes an annual salary of $3.48 × 104. If there are 5.2 × 102 account executives in the company,
($3.48 × 104)( 5.2 × 102)
$1.8096 × 107

Question 6.
Place the numbers in order from least to greatest.
0.24,4 × 10-2, 0.042, 2 × 10-4, 0.004
Options:
a. 2 × 10-4, 4 × 10-2, 0.004, 0.042, 0.24
b. 0.004, 2 × 10-4, 0.042, 4 × 10-2, 0.24
c. 0.004, 2 × 10-4, 4 × 10-2, 0.042, 0.24
d. 2 × 10-4, 0.004, 4 × 10-2, 0.042, 0.24

Answer:
d. 2 × 10-4, 0.004, 4 × 10-2, 0.042, 0.24

Explanation:
2 × 10-4 = 0.0002
4 × 10-2 = 0.04

Question 7.
Guillermo is 5 \(\frac{5}{6}\) feet tall. What is this number of feet written as a decimal?
Options:
a. 5.7 feet
b. 5.\(\bar{7}\) feet
c. 5.83 feet
d. 5.8\(\bar{3}\) feet

Answer:
c. 5.83 feet

Question 8.
A human hair has a width of about 6.5 × 10-5 meters. What is this width written in standard notation?
Options:
a. 0.00000065 meter
b. 0.0000065 meter
c. 0.000065 meter
d. 0.00065 meter

Answer:
c. 0.000065 meter

Explanation:
6.5 × 10-5 meter = 0.000065

Mini-Task

Question 9.
Consider the following numbers: 7000, 700, 70, 0.7, 0.07, 0.007
a. Write the numbers in scientific notation.
Type below:
_____________

Answer:
7000 = 7 × 10³
700 = 7 × 10²
70 = 7 × 10¹
0.7 = 7 × 10¯¹
0.07 = 7 × 10¯²
0.007 = 7 × 10¯³

Question 9.
b. Look for a pattern in the given list and the list in scientific notation. Which numbers are missing from the lists?
Type below:
_____________

Answer:
In the given list the decimal is moving to the left by one place. From the scientific notation, numbers are decreasing by 10. The number missing is 7

Question 9.
c. Make a conjecture about the missing numbers.
Type below:
_____________

Answer:
The numbers will continue to decrease by 10 in the given list.

Conclusion:

We wish the information provided in the Go Math Grade 8 Answer Key Chapter 2 Exponents and Scientific Notation for all the students. Go through the solved examples to have a complete grip on the subject and also on the way of solving each problem. Go Math Grade 8 Chapter 2 Exponents and Scientific Notation Key will help the students to score the highest marks in the exam.

Go Math Grade 3 Answer Key Chapter 12 Two-Dimensional Shapes Extra Practice

go-math-grade-3-chapter-12-two-dimensional-shapes-extra-practice-answer-key

Go Math Grade 3 Answer Key Chapter 12 Two-Dimensional Shapes Extra Practice makes it easy for you to test your preparation level. Solve all the practice questions on Go Math Grade 3 Answer Key Chapter 12 Two-Dimensional Shapes Extra Practice. We even provided the Step by Step Solutions for all the 3rd Grade Go Math Answer Key Ch 12 Two-Dimensional Shapes by which you can verify your answers. HMH Go Math Grade 3 gives you a new way of problem-solving and makes it easy for you to get a good grip on the concepts underlying.

Go Math Grade 3 Chapter 12 Two-Dimensional Shapes Extra Practice Answer Key

Download Go Math Grade 3 Answer Key Chapter 12 Two-Dimensional Shapes Extra Practice and prepare whenever you want. Grab the required knowledge and solve the problems of Grade 3 Chapter 12 Two-Dimensional Shapes Extra Practice on a regular basis. Check out the Step by Step Solutions provided Go Math Grade 3 Chapter 12 Two-Dimensional Shapes Extra Practice Answer Key and cross-check your answers.

Common Core – Page No. 257000

Chapter 12 Extra Practice

Lessons 12.1–12.3

Name the polygon.

Question 1.
Go Math Grade 3 Answer Key Chapter 12 Two-Dimensional Shapes Extra Practice Common Core img 1
_________

Answer:
quadrilateral

Explanation:
Chapter 12 Common Core image 1 257000
4 sides; 4 angles; quadrilateral

Question 2.
Go Math Grade 3 Answer Key Chapter 12 Two-Dimensional Shapes Extra Practice Common Core img 2
_________

Answer:
decagon

Explanation:
Chapter 12 Common Core image 2 257000
10 sides; 10 angles; decagon

Question 3.
Go Math Grade 3 Answer Key Chapter 12 Two-Dimensional Shapes Extra Practice Common Core img 3
_________

Answer:
hexagon

Explanation:
Chapter 12 Common Core image 3 257000
6 sides; 6 angles; hexagon

Question 4.
Go Math Grade 3 Answer Key Chapter 12 Two-Dimensional Shapes Extra Practice Common Core img 4
_________

Answer:
triangle

Explanation:
Chapter 12 Common Core image 4 257000
3 sides; 3 angles; triangle

Question 5.
Go Math Grade 3 Answer Key Chapter 12 Two-Dimensional Shapes Extra Practice Common Core img 5
_________

Answer:
octagon

Explanation:
Chapter 12 Common Core image 5 257000
8 sides; 8 angles; octagon

Question 6.
Go Math Grade 3 Answer Key Chapter 12 Two-Dimensional Shapes Extra Practice Common Core img 6
_________

Answer:
pentagon

Explanation:
Chapter 12 Common Core image 6 257000
5 sides; 5 angles; pentagon

Lesson 12.4

Look at the dashed sides of the polygon. Tell if they appear to be intersecting, perpendicular, or parallel. Write all the words that describe the sides.

Question 7.
Go Math Grade 3 Answer Key Chapter 12 Two-Dimensional Shapes Extra Practice Common Core img 7
_________
_________

Answer:
perpendicular lines

Explanation:
The dashed sides are meeting to form a right angle. So, they are perpendicular lines.

Question 8.
Go Math Grade 3 Answer Key Chapter 12 Two-Dimensional Shapes Extra Practice Common Core img 8
_________

Answer:
parallel lines

Explanation:
The dashed sides are not intersecting with each other. So, the given lines are parallel lines.

Question 9.
Go Math Grade 3 Answer Key Chapter 12 Two-Dimensional Shapes Extra Practice Common Core img 9
_________

Answer:
intersecting lines

Explanation:
The dashed line segments meet and form an angle. So, they are intersecting lines.

Lesson 12.5

Circle all the words that describe the quadrilateral.

Question 10.
Go Math Grade 3 Answer Key Chapter 12 Two-Dimensional Shapes Extra Practice Common Core img 10
Options:
a. rhombus
b. trapezoid
c. rectangle

Answer:
c. rectangle

Explanation:
The given shape has two pairs opposite with the same length. Also, all the angles are right angles. The given shape is a rectangle.

Question 11.
Go Math Grade 3 Answer Key Chapter 12 Two-Dimensional Shapes Extra Practice Common Core img 11
Options:
a. square
b. rhombus
c. trapezoid

Answer:
a. square
b. rhombus

Explanation:
The given shape has 4 sides with equal lengths. Also, all the angles are right angles. So, a possible answer is a square and rhombus.

Question 12.
Go Math Grade 3 Answer Key Chapter 12 Two-Dimensional Shapes Extra Practice Common Core img 12
Options:
a. trapezoid
b. rectangle
c. rhombus

Answer:
a. trapezoid

Explanation:
Even though the given shape has four sides, they are not equal. Also, it has only two right angles. The given shape is a trapezoid.

Common Core – Page No. 258000

Lesson 12.6

Draw a quadrilateral that does not belong. Then explain why.

Question 1.
Go Math Grade 3 Answer Key Chapter 12 Two-Dimensional Shapes Extra Practice Common Core img 13
Type below:
_________

Answer:
Chapter 12 Common Core image 1 258000

Explanation:
The shape is a trapezoid. Even though the given shape has four sides, they are not equal. Also, the angles are not right angles.

Lesson 12.7

Use the triangles for 1–2. Write A, B, or C.
Then complete the sentences.
Go Math Grade 3 Answer Key Chapter 12 Two-Dimensional Shapes Extra Practice Common Core img 14

Question 2.
Triangle ____ has 1 angle greater than a right angle and appears to have ____ sides of equal length.

Answer:
Triangle C has 1 angle greater than a right angle and appears to have 0 sides of equal length.

Question 3.
Triangle____ has 1 right angle and appears to have ____ sides of equal length.

Answer:
Triangle A has 1 right angle and appears to have 2 sides of equal length.

Lesson 12.8
Go Math Grade 3 Answer Key Chapter 12 Two-Dimensional Shapes Extra Practice Common Core img 15

Question 4.
What label could you use to describe Circle A?
Type below:
_________

Answer:
All sides of Equal Lengths

Question 5.
What label could you use to describe Circle B?
Type below:
_________

Answer:
Right Angle

Lesson 12.9

Draw lines to divide the shape into equal parts that show the fraction given.

Question 6.
Go Math Grade 3 Answer Key Chapter 12 Two-Dimensional Shapes Extra Practice Common Core img 16 \(\frac{1}{4}\)

Answer:
Chapter 12 Common Core image 2 258000

Question 7.
Go Math Grade 3 Answer Key Chapter 12 Two-Dimensional Shapes Extra Practice Common Core img 17 \(\frac{1}{3}\)

Answer:
Chapter 12 Relate Shapes, Fractions, and Area image 2 752

Conclusion

Go Math Grade 3 Answer Key Chapter 12 Two-Dimensional Shapes Extra Practice helps you understand various topics in the Chapter easily. Download Go Math Grade 3 Answer Key PDF free of cost and prepare offline too. Each Problem is clearly explained with images and graphs so that you can better understand the concepts. For more assistance Go Math Grade 3 Answer Key Chapter 12 Two-Dimensional Shapes and achieve valuable knowledge.

Go Math Grade 4 Answer Key Chapter 3 Multiply 2-Digit Numbers

go-math-grade-4-chapter-3-multiply-2-digit-numbers-answer-key

Are you seeing everywhere to find Go Math Grade 4 Answer Key Chapter 3 Multiply 2-Digit Numbers in pdf? If so, you have stepped onto the right page. On this page, we have compiled a pdf formatted HMH Go Math Grade 4 Chapter 3 Multiply 2-Digit Numbers Answer Key. Take the help of the Go Math Grade 4 Solution Key during your practice sessions and clear your queries. Assess your preparation standards and concentrate on the areas you are flagging.

Go Math Grade 4 Solution Key Pdf Chapter 3 Multiply 2-Digit Numbers

With the help of HMH Go Math Grade 4 Answer Key Ch 3, you can learn the easy methods to solve problems. Answer Key for HMH Go Math Grade 4 Chapter 3 Multiply 2-Digit Numbers consists of all the questions from practice tests, exercises, assessments tests. You guys are recommended to solve the 4th grade Go Math Answers of Chapter 3 Multiply 2-Digit Numbers regularly & attempt the exams with confidence. The topics covered in the Go Math HMH grade 4 Solutions are designed perfectly & in an understandable way.

Lesson 1: Multiply by Tens

Lesson 2: Estimate Products

Lesson 3: Investigate • Area Models and Partial Products

Lesson 4: Multiply Using Partial Products

Mid-Chapter Checkpoint

Lesson 5: Multiply with Regrouping

Lesson 6: Choose a Multiplication Method

Lesson 7: Problem Solving • Multiply 2-Digit Numbers

Review/Test

Common Core – Page No. 149

Multiply by Tens

Choose a method. Then find the product.

Question 1.
16 × 60 = 960
Use the halving-and-doubling strategy.
Find half of 16: 16 ÷ 2 = 8.
Multiply this number by 60: 8 × 60 = 480
Double this result: 2 × 480 = 960

Answer:
960

Explanation:
Use the halving-and-doubling strategy.
Find half of 16: 16 ÷ 2 = 8.
Multiply this number by 60: 8 × 60 = 480
Double this result: 2 × 480 = 960

Question 2.
80 × 22 = ______

Answer:
1760

Explanation:
By using the place value method, Multiply 80 x 22
You can think of 80 as 8 tens
80 x 22 = (22 x 8) tens
= 176 tens
= 176 x 10 = 1760
80 x 22 = 1760

Question 3.
30 × 52 = ______

Answer:
1560

Explanation:
Use the Associative Property
You can think of 30 as 3 x 10
30 x 52 = (3 x 10) x 52
= 3 x (10 x 52)
=  3 x 520
= 1560
30 x 52 = 1560

Question 4.
60 × 20 = ______

Answer:
1200

Explanation:
60 x 20
Use the halving and doubling strategy
half of the 60 to make the problem simpler
60/ 2 = 30
Multiply 30 with 20
30 x 20 = 600
Double the 600
2 x 600= 1200
60 x 20 = 1200

Question 5.
40 × 35 = ______

Answer:
1400

Explanation:
By using the Associative Property 40 x 35
You can think of 40 as 4 x 10
40 x 35 = (4 x 10) x 35
= 4  x (10 x 35)
= 4 x 350
= 1400
40 x 35 = 1400

Question 6.
10 × 90 = ______

Answer:
900

Explanation:
By using the place value method, Multiply 10 x 90
You can think of 90 as 9 tens
10 x 90 = (10 x 9) tens
= 90 tens
= 10 x 90 = 900

Question 7.
31 × 50 = ______

Answer:
1,550

Explanation:
Use the place value method to multiply 31 x 50
You can think of 50 as 5 tens
31 x 50 = 31 x 5 tens
= 155 tens
= 1,550
31 x 50 = 1,550

Problem Solving

Question 8.
Kenny bought 20 packs of baseball cards. There are 12 cards in each pack. How many cards did Kenny buy?
______ cards

Answer:
240 cards

Explanation:
From the given data,
Kenny bought 20 packs of basketball cards
There are 12 cards in each pack = 12 x 20 cards
Use the associative property
You can write 20 as 2 x 10
12 x 20 = 12 x (2 x 10)
= (12 x 2) x 10
= (24) x 10
= 240 cards
Kenny bought 240 cards

Question 9.
The Hart family drove 10 hours to their vacation spot. They drove an average of 48 miles each hour. How many miles did they drive in all?
______ miles

Answer:
480 miles

Explanation:
As per the given data,
Hart family drove 10 hours to their vacation spot
Average speed per each hour is = 48 miles
Total miles = 48 x 10
Use the halving and doubling strategy
Half of the 48 to make the problem simpler
48/ 2 = 24
Multiply 24 with 10 = 24x 10 = 240
Double the value = 2 x 240 = 480 miles
Total miles drove by hart family = 480 miles.

Common Core – Page No. 150

Lesson Check

Question 1.
For the school play, 40 rows of chairs are set up. There are 22 chairs in each row. How many chairs are there in all?
Options:
a. 800
b. 840
c. 880
d. 8,800

Answer:
c. 880

Explanation:
As per the given data
For the school play, 40 rows of chairs are available. 22 chairs are available in each row.
Then total chairs in school play are = 22 x 40
By using the place value method
You can think of 40 as 4 tens
22 x 40 = 22 x 4 tens
= 88 tens
= 880
Total chairs in school are = 880

Question 2.
At West School, there are 20 classrooms. Each classroom has 20 students. How many students are at West School?
Options:
a. 40
b. 400
c. 440
d. 4,000

Answer:
b. 400

Explanation:
From the given data,
Total classrooms in west school = 20
Number of students per each classroom = 20
Then, total students at West School = 20 x 20
By using the associative property
You can think of 20 as 2 x 10
20 x 20 = 20 x (2 x 10)
= (20 x 2) x 10
=(40) x 10
=400
Total number of students at West School = 400

Spiral Review

Question 3.
Alex has 48 stickers. This is 6 times the number of stickers Max has. How many stickers does Max have?
Options:
a. 6
b. 7
c. 8
d. 9

Answer:
c. 8

Explanation:
As per the give data,
Alex has 48 stickers
That means, X= 48
This is 6 times the number of stickers max has = Y = 6X = 48
Then, number of stickers with Max = Y = X = 48/6 = 8
Number of stickers with Max = Y = 8 Stickers.

Question 4.
Ali’s dog weighs 8 times as much as her cat. Together, the two pets weigh 54 pounds. How much does Ali’s dog weigh?
Options:
a. 6 pounds
b. 42 pounds
c. 46 pounds
d. 48 pounds

Answer:
d. 48 pounds

Explanation:
From the given data,
Ali’s cat weight = X
Ali’s dog weight = 8 times as much as Ali’s cat = 8X
Together, the two pets weight = (X+8X) = 54 pounds
= 9X = 54 pounds
= X = 54/9 pounds = 6 pounds
Then, Ali’s dog weight = 8X =8 x 6 = 48 pounds.

Question 5.
Allison has 3 containers with 25 crayons in each. She also has 4 boxes of markers with 12 markers in each box. She gives 10 crayons to a friend. How many crayons and markers does Allison have now?
Options:
a. 34
b. 113
c. 123
d. 133

Answer:
b. 113

Explanation:
As per the given data,
Allison has 3 containers with 25 crayons in each = X = 3 x 25 = 75
Allison has 4 boxes of markers with 12 markers in each box = Y = 4 x 12 = 48
Allison gives 10 crayons to a friend = Z = 75-10 = 65
Now, total number of crayons and markers with Allison = Y + Z = 48 + 65 = 113

Question 6.
The state of Utah covers 82,144 square miles. The state of Montana covers 145,552 square miles. What is the total area of the two states?
Options:
a. 63,408 square miles
b. 223,408 square miles
c. 227,696 square miles
d. 966,992 square miles

Answer:
c. 227,696 square miles

Explanation:
From the given data,
The state of Utah covers 82,144 square miles
The state of Montana covers 145,552 square miles
Then, Total area of the two states = 82,144 + 145,552
The total area of two states = 227,696 square miles.

Page No. 153

Question 1.
To estimate the product of 62 and 28 by rounding, how would you round the factors? What would the estimated product be?
about _____

Answer:
1800

Explanation:
By using rounding and mental math
Estimate 62 x 28
Firstly, round each factor
62 x 28
Go Math Grade 4 Answer Key Chapter 3 Multiply 2-Digit Numbers      Go Math Grade 4 Answer Key Chapter 3 Multiply 2-Digit Numbers
60 x 30
Use mental math
6 x 3 = 18
60 x 30 = 1800
So, estimated product of 62 and 28 = 1800

Estimate the product. Choose a method.

Question 2.
96 × 34
Estimate: _____

Answer:
3000

Explanation:
Use mental math and compatible numbers
96 x 34
Go Math Grade 4 Answer Key Chapter 3 Multiply 2-Digit Numbers      Go Math Grade 4 Answer Key Chapter 3 Multiply 2-Digit Numbers
100 x 30
Use mental math
1 x 30 = 30
100 x 30= 3000

Question 3.
47 × $39
Estimate: $ _____

Answer:
2000

Explanation:
Round to the nearest ten
47 x $39
Go Math Grade 4 Answer Key Chapter 3 Multiply 2-Digit Numbers      Go Math Grade 4 Answer Key Chapter 3 Multiply 2-Digit Numbers
50 x $40
50 x $ 4 = $200
50 x $40 = 2000

Question 4.
78 × 72
Estimate: _____

Answer:
5600

Explanation:
Use rounding and mental math
Round each factor
78 x 72
Go Math Grade 4 Answer Key Chapter 3 Multiply 2-Digit Numbers      Go Math Grade 4 Answer Key Chapter 3 Multiply 2-Digit Numbers
80 x 70
Use mental math
8 x 7 = 56
80 x 70 = 5600

Question 5.
41 × 78
Estimate: _____

Answer:
3200

Explanation:
Use compatible numbers and mental math
41 x 78
Go Math Grade 4 Answer Key Chapter 3 Multiply 2-Digit Numbers      Go Math Grade 4 Answer Key Chapter 3 Multiply 2-Digit Numbers
40 x 80
Use mental math
40 x 8 = 320
40 x 80 = 3200

Question 6.
51 × 73
Estimate: _____

Answer:
3500

Explanation:
Round to the nearest ten
51 x 73
Go Math Grade 4 Answer Key Chapter 3 Multiply 2-Digit Numbers      Go Math Grade 4 Answer Key Chapter 3 Multiply 2-Digit Numbers
50 x 70 = 3500

Question 7.
34 × 80
Estimate: _____

Answer:
2400

Explanation:
Round each factor
34 x 80
Go Math Grade 4 Answer Key Chapter 3 Multiply 2-Digit Numbers      Go Math Grade 4 Answer Key Chapter 3 Multiply 2-Digit Numbers
30 x 80
3 x 8 = 240
30 x 80 = 2400

Practice: Copy and Solve Estimate the product. Choose a method.

Question 8.
61 × 31
Estimate: _____

Answer:
1800

Explanation:
Round to the nearest ten
61 x 31
Go Math Grade 4 Answer Key Chapter 3 Multiply 2-Digit Numbers      Go Math Grade 4 Answer Key Chapter 3 Multiply 2-Digit Numbers
60 x 30 = 1800

Question 9.
52 × 68
Estimate: _____

Answer:
3500

Explanation:
Round each factor
52 x 68
Go Math Grade 4 Answer Key Chapter 3 Multiply 2-Digit Numbers      Go Math Grade 4 Answer Key Chapter 3 Multiply 2-Digit Numbers
50 x 70
Use mental math
5 x 7 =35
50 x 70 = 3500

Question 10.
26 × 44
Estimate: _____

Answer:
1200

Explanation:
Round to the nearest tens
26 x 44
Go Math Grade 4 Answer Key Chapter 3 Multiply 2-Digit Numbers      Go Math Grade 4 Answer Key Chapter 3 Multiply 2-Digit Numbers
30 x 40 = 1200

Question 11.
57 × $69
Estimate: $ _____

Answer:
$4200

Explanation:
Round each factor
57 x $69
Go Math Grade 4 Answer Key Chapter 3 Multiply 2-Digit Numbers      Go Math Grade 4 Answer Key Chapter 3 Multiply 2-Digit Numbers
60 x $70
Use mental math
6 x $7 = $42
60 x $70 = $4200

Find two possible factors for the estimated product.

Question 12.
2,800
Type below:
___________

Answer:
2800

Explanation:
Let us consider 7 x 4 = 28
70 x 40 = 2800

Question 13.
8,100
Type below:
___________

Answer:
8,100

Explanation:
Let us take 9 x 9 = 81
90 x 90 = 8,100

Question 14.
5,600
Type below:
___________

Answer:
5,600

Explanation:
Let us consider 7 x 8 = 56
70 x 80 = 5,600

Question 15.
2,400
Type below:
___________

Answer:
2,400

Explanation:
Let us take 4 x 6 = 24
40 x 60 = 2400
Or 3 x 8 = 24
30 x 80 = 2,400

Question 16.
Mr. Parker jogs for 35 minutes each day. He jogs 5 days in week 1, 6 days in week 2, and 7 days in week 3. About how many minutes does he jog?
about _____ minutes

Answer:
about 630 minutes

Explanation:
From the given data,
Mr. Parker jogs per day = 35 minutes
He jogs 5 days in week 1 = 5 x 35 = 175 minutes
6 days in week 2 = 6 x 35 = 210 minutes
7 days in week 3 = 7 x 35 = 245 minutes
Total minutes of jog by Mr. Parker = week 1 + week 2 + week 3
= 175 + 210 + 245
= 630 minutes
So, total minutes of jog by Mr. Parker = 630 minutes

Question 17.
There are 48 beads in a package. Candice bought 4 packages of blue, 9 packages of gold, 6 packages of red, and 2 packages of silver beads. About how many beads did Candice buy?
about _____ beads

Answer:
about 1008 beads

Explanation:
As per the given data,
48 beads are there in a package
Candice bought 4 packages of blue beads = 4 x 48 = 192
9 packages of gold beads = 9 x 48 = 432
6 packages of red beads = 6 x 48 = 288
2 packages of silver beads = 2 x 48 = 96
Total beads bought by Candice = 192 + 432 + 288 + 96
= 1008 beads
So, total beads bought by Candice = 1008.

Page No. 154

Question 18.
On average, a refrigerator door is opened 38 times each day. Len has two refrigerators in his house. Based on this average, about how many times in a 3-week period are the refrigerator doors opened?
about _____ times

Answer:
about 1600 times

Explanation:
From the given data,
On average, a refrigerator door is opened per day = 38 times
3-week period = 7 x 3 = 21
Then, a refrigerator door is opened per 21 days = 21 x 38 = 798 times
Len has 2 refrigerators in his house
Then, two refrigerators door are opened per 21 days = 2 x 798
= 1596 times
So, in a 3 – week period refrigerator door is opened about 1600 times

Question 19.
The cost to run a refrigerator is about $57 each year. About how much will it have cost to run by the time it is 15 years old?
about $ _____

Answer:
1200

Explanation:
As per the data,
The cost to run a refrigerator per each year = $57
Cost to run a refrigerator by the time it is 15 years old = $57 * 15
Round to the nearest tens
$57 x 15
Go Math Grade 4 Answer Key Chapter 3 Multiply 2-Digit Numbers      Go Math Grade 4 Answer Key Chapter 3 Multiply 2-Digit Numbers
$60 x 20
Use mental math
$6 x 2 = 12
$ 60 x 20 = 1200

Question 20.
If Mel opens his refrigerator door 36 times every day, about how many times will it be opened in April? Will the exact answer be more than or less than the estimate? Explain.
Type below:
___________

Answer:
1200

Explanation:
From the given data,
Mel opens his refrigerator door per day = 36 times
Number of days in April month = 30 days
Refrigerator door opened in April month = 36 * 30
Round the factors
36 x 30
Go Math Grade 4 Answer Key Chapter 3 Multiply 2-Digit Numbers      Go Math Grade 4 Answer Key Chapter 3 Multiply 2-Digit Numbers
40 x 30 = 1200

Question 21.
Represent a Problem What question could you write for this answer? The estimated product of two numbers, that are not multiples of ten, is 2,800.
Type below:
___________

Answer:
2800

Explanation:
Let us take
1.
38 × 21
↓        ↓
40 × 20 = 800
2,800 = 42 x 68
↓    ↓
40 x  70 = 2800

Question 22.
Which is a reasonable estimate for the product? Write the estimate. An estimate may be used more than once.
Go Math Grade 4 Answer Key Chapter 3 Multiply 2-Digit Numbers img 1
26 × 48 __________
28 × 21 __________
21 × 22 __________
51 × 26 __________

Answer:
25 x 50 = 1250
30 x 20 = 600
20 x 20 = 400
50 x 25 = 1250

Explanation:
26 x 48 -> 25 x 50 = 1250
28 x 21 -> 30 x 20 = 600
21 x 22 -> 20 x 20 = 400
51 x 26 -> 50 x 25 = 1250

Common Core – Page No. 155

Estimate Products
Estimate the product. Choose a method.

Question 1.
38 × 21
38 × 21
↓       ↓
40 × 20
800

Answer:
800

Explanation:
38 × 21
↓        ↓
40 × 20
800

Question 2.
63 × 19
Estimate: _____

Answer:
1200

Explanation:
63 x 19
Go Math Grade 4 Answer Key Chapter 3 Multiply 2-Digit Numbers      Go Math Grade 4 Answer Key Chapter 3 Multiply 2-Digit Numbers
60 x 20 = 1200
Estimated product of 63 x 19 = 1200

Question 3.
27 × $42
Estimate: $ _____

Answer:
$1000

Explanation:
27 × $42
Go Math Grade 4 Answer Key Chapter 3 Multiply 2-Digit Numbers      Go Math Grade 4 Answer Key Chapter 3 Multiply 2-Digit Numbers
25 x $40 = $1000
Estimated Product of 25 x $ 42 = $1000

Question 4.
73 × 67
Estimate: _____

Answer:
4900

Explanation:
73 × 67
Go Math Grade 4 Answer Key Chapter 3 Multiply 2-Digit Numbers      Go Math Grade 4 Answer Key Chapter 3 Multiply 2-Digit Numbers
70 x 70 = 4900
Estimated Product of 73 x 67 = 4900

Question 5.
37 × $44
Estimate:$ _____

Answer:
$1600

Explanation:
37 × $44
Go Math Grade 4 Answer Key Chapter 3 Multiply 2-Digit Numbers      Go Math Grade 4 Answer Key Chapter 3 Multiply 2-Digit Numbers
40 x $40 = $1600
Estimated Product of 37 x $44 = $1600

Question 6.
85 × 71
Estimate: _____

Answer:
6300

Explanation:
85 × 71
Go Math Grade 4 Answer Key Chapter 3 Multiply 2-Digit Numbers      Go Math Grade 4 Answer Key Chapter 3 Multiply 2-Digit Numbers
90 x 70 = 6300
Estimated Product of 85 x 71 = 6300

Question 7.
88 × 56
Estimate: _____

Answer:
4950

Explanation:
88 × 56
Go Math Grade 4 Answer Key Chapter 3 Multiply 2-Digit Numbers      Go Math Grade 4 Answer Key Chapter 3 Multiply 2-Digit Numbers
90 x 55 = 4950
Estimated Product of 90 x 55 = 4950

Question 8.
97 × 13
Estimate: _____

Answer:
1,000

Explanation:
97 × 13
Go Math Grade 4 Answer Key Chapter 3 Multiply 2-Digit Numbers      Go Math Grade 4 Answer Key Chapter 3 Multiply 2-Digit Numbers
100 x 10 = 1,000

Question 9.
92 × 64
Estimate: _____

Answer:
5850

Explanation:
92 × 64
Go Math Grade 4 Answer Key Chapter 3 Multiply 2-Digit Numbers      Go Math Grade 4 Answer Key Chapter 3 Multiply 2-Digit Numbers
90 x 65 = 5850

Problem Solving

Question 10.
A dime has a diameter of about 18 millimeters. About how many millimeters long would a row of 34 dimes be?
about _____ millimeters

Answer:
about 600 millimeters

Explanation:
From the given data,
A dime has a diameter of about 18 millimeters
Then, 34 dimes diameter = 18 * 34
18 x 34
Go Math Grade 4 Answer Key Chapter 3 Multiply 2-Digit Numbers      Go Math Grade 4 Answer Key Chapter 3 Multiply 2-Digit Numbers
20 x 30 = 600
So, 34 dimes have a diameter of about 600 millimeters long

Question 11.
A half-dollar has a diameter of about 31 millimeters. About how many millimeters long would a row of 56 half-dollars be?
about _____ millimiters

Answer:
1800 millimeters

Explanation:
As per the given data,
A half – dollar has a diameter of about 31 millimeters
Then, 56 half-dollars diameter = 31 * 56
31 * 56
Go Math Grade 4 Answer Key Chapter 3 Multiply 2-Digit Numbers      Go Math Grade 4 Answer Key Chapter 3 Multiply 2-Digit Numbers
30 * 60
So, 56 half-dollars have a diameter of about 1800 millimeters long.

Common Core – Page No. 156

Lesson Check

Question 1.
Which is the best estimate for the product
43 × 68?
Options:
a. 3,500
b. 2,800
c. 2,400
d. 280

Answer:
b. 2,800

Explanation:
Round to the nearest tens
43 x 68
Go Math Grade 4 Answer Key Chapter 3 Multiply 2-Digit Numbers      Go Math Grade 4 Answer Key Chapter 3 Multiply 2-Digit Numbers
40 x 70
Use mental math
4 x 7 = 28
40 x 70 = 2800
Estimated product of 43 x 68 = 2800

Question 2.
Marissa burns 93 calories each time she plays fetch with her dog. She plays fetch with her dog once a day. About how many calories will Marissa burn playing fetch with her dog in 28 days?
Options:
a. 4,000
b. 2,700
c. 2,000
d. 270

Answer:
b. 2,700

Explanation:
From the given data,
Marissa burned calories each time when she plays fetch with her dog= 93 calories
Then, Marissa burned calories in 28 days while playing fetch with her dog = 28 x 93
Round to the nearest tens
28 x 93
Go Math Grade 4 Answer Key Chapter 3 Multiply 2-Digit Numbers      Go Math Grade 4 Answer Key Chapter 3 Multiply 2-Digit Numbers
30 x 90
Then, estimated burned calories in 28 days by Marissa = 2700 calories

Spiral Review

Question 3.
Use the model to find 3 × 126.
Go Math Grade 4 Answer Key Chapter 3 Multiply 2-Digit Numbers Common Core img 2
Options:
a. 368
b. 378
c. 468
d. 478

Answer:
b. 378

Explanation:
From the above Figure,
3 x 126 = 3 x 100 + 3 x 20 + 3 x 6
= 300 + 60 + 18
= 378
3 x 126 = 378

Question 4.
A store sells a certain brand of jeans for $38. One day, the store sold 6 pairs of jeans of that brand. How much money did the store make from selling the 6 pairs of jeans?
Options:
a. $188
b. $228
c. $248
d. $288

Answer:
b. $228

Explanation:
As per the given data,
A store sells a certain brand of jeans for rupees = $38
One day, the store sold 6 pairs of jeans of that brand = 6 x $38
6 x $38 = $228
The total amount of 6 pairs of jeans = $228

Question 5.
The Gateway Arch in St. Louis, Missouri, weighs about 20,000 tons. Which amount could be the exact number of tons the Arch weighs?
Options:
a. 31,093 tons
b. 25,812 tons
c. 17,246 tons
d. 14,096 tons

Answer:
c. 17,246 tons

Explanation:
From the given data,
The Gateway Arch in St.Louis, Missouri weight = about 20,000 tons
From the available options, 17,246 tons is closer to 20,000 tons
Then, the exact number of tons the Arch weighs = 17,246 tons

Question 6.
Which is another name for 23 ten thousands?
Options:
a. 23,000,000
b. 2,300,000
c. 230,000
d. 23,000

Answer:
c. 230,000

Explanation:
As per the data,
Another name for 23 ten thousands = 23 x 10,000
= 230,000
Another name for 23 ten thousand = 2,30,000

Page No. 159

Find the product.

Question 1.
16 × 19
Go Math Grade 4 Answer Key Chapter 3 Multiply 2-Digit Numbers img 3
16 × 19 = _____

Answer:
304

Explanation:
Go Math Grade 4 Answer Key Chapter 3 Multiply 2-Digit Numbers img 3
16 x 19 = 304

Question 2.
18 × 26
Go Math Grade 4 Answer Key Chapter 3 Multiply 2-Digit Numbers img 4
18 × 26 = _____

Answer:
468

Explanation:
Chapter 3 - Common core - Image 1. jpg
200 + 160 + 60 + 48 = 468

Question 3.
27 × 39
Go Math Grade 4 Answer Key Chapter 3 Multiply 2-Digit Numbers img 5
27 × 39 = ______

Answer:
1,053

Explanation:
Chapter 3 - Common core - image 2
600 + 210 + 180 +63 = 1053

Draw a model to represent the product.
Then record the product.

Question 4.
14 × 16 = ______

Answer:
224

Explanation:
Chapter 3 - Common core - Image 3
100 + 40 + 60 + 24 = 224

Question 5.
23 × 25 = ______

Answer:
575

Explanation:
Chapter 3 - Common core - Image 4
400 + 60 + 100 + 15 = 575

Question 6.
Explain how modeling partial products can be used to find the products of greater numbers.
Type below:
__________

Answer:
You can use mental math to find the partial products and then find the sum of the partial products.

Explanation:

Question 7.
Emma bought 16 packages of rolls for a party. There were 12 rolls in a package. After the party there were 8 rolls left over. How many rolls were eaten? Explain.
______ rolls

Answer:
184 rolls were eaten

Explanation:
From the given data,
Emma bought 16 packages of rolls for a party
There were 12 rolls in a package
Then, total rolls = 16 x 12 = 192
Chapter 3 - Common core - Image 5
100 + 60 + 20 + 12 =192
After the party there were 8 rolls left over
Then, total eaten rolls are = 192 – 8 = 184

Page No. 160

Question 8.
Jamal and Kim used different ways to solve 12 × 15 by using partial products. Whose answer makes sense? Whose answer is nonsense? Explain your reasoning.
Jamal’s Work
Go Math Grade 4 Answer Key Chapter 3 Multiply 2-Digit Numbers img 6
100 + 20 + 10 = 130

Kim’s Work
Go Math Grade 4 Answer Key Chapter 3 Multiply 2-Digit Numbers img 7
120 + 60 = 180
a. For the answer that is nonsense, write an answer that makes sense.
Type below:
__________

Answer:
a. Jamal’s work makes nonsense.
100 + 20 + 50 + 10 = 180 it makes sense

Question 8.
b. Look at Kim’s method. Can you think of another way Kim could use the model to find the product? Explain.
Go Math Grade 4 Answer Key Chapter 3 Multiply 2-Digit Numbers img 8
Type below:
__________

Answer:
Other method: 12 x 15
10 x 12 = 120
5 x 12 = 60
120 + 60 = 180.

Explanation:
Kim follows another method to find 12 x 15
That is, 100 + 50 = 150
20 + 10 = 30
Then, 150 + 30 =180
12 x 15 = 180

Question 9.
Look at the model in 8b. How would the partial products change if the product was 22 × 15? Explain why you think the products changed.
Type below:
__________

Answer:
330

Explanation:
Following the 8b method
22 x 15 =330
Chapter 3 - Common core - Image 6
200 + 100 = 300
20 + 10 = 30
Now, 300 + 30 = 330
Finally, 22 x 15 = 330
The factor of 15 is increased in present problem. So, the product also increases for 15 x 22.

Common Core – Page No. 161

Area Models and Partial Products

Draw a model to represent the product.
Then record the product.

Question 1.
13 × 42
Go Math Grade 4 Answer Key Chapter 3 Multiply 2-Digit Numbers Common Core img 9

Answer:
Go Math Grade 4 Answer Key Chapter 3 Multiply 2-Digit Numbers Common Core img 9

Question 2.
18 × 34 = ______

Answer:
chapter 3 - Area models and partial products- image 1. jpg
300 + 40 + 240 + 32 = 612

Question 3.
22 × 26 = ______

Answer:
chapter 3 - Area models and partial products- image 2. jpg
400 + 120 + 40 + 12 = 572

Question 4.
1 5 × 33 = ______

Answer:
chapter 3 - Area models and partial products- image 3. jpg
300 + 30 + 150 + 15 = 495

Question 5.
23 × 29 = ______

Answer:
chapter 3 - Area models and partial products- image 4. jpg
400 + 180 + 60 + 27 = 667

Question 6.
19 × 36 = ______

Answer:
chapter 3 - Area models and partial products- image 5. jpg
300 + 60 + 270 + 54 = 684

Problem Solving

Question 7.
Sebastian made the following model to find the product 17 × 24.
Go Math Grade 4 Answer Key Chapter 3 Multiply 2-Digit Numbers Common Core img 10
Is his model correct? Explain.
a. yes
b. no

Answer:
b. no

Explanation:
chapter 3 - Area models and partial products- image 6. jpg
200 + 40 + 140 + 28 = 408

Question 8.
Each student in Ms. Sike’s kindergarten class has a box of crayons. Each box has 36 crayons. If there are 18 students in Ms. Sike’s class, how many crayons are
there in all?
______ crayons

Answer:
648 crayons

Explanation:
From the given information,
Each student in Ms.Sike’s kindergarten class has a box of crayons
Crayons in each box = 36 Crayons
Number of students in Mr.Sike’s class = 18 students
Total crayons = 18 x 36
chapter 3 - Area models and partial products- image 7. jpg
300 + 60 + 240 + 48 = 648

Common Core – Page No. 162

Lesson Check

Question 1.
Which product does the model below represent?
Go Math Grade 4 Answer Key Chapter 3 Multiply 2-Digit Numbers Common Core img 11
Options:
a. 161
b. 230
c. 340
d. 391

Answer:
d. 391

Explanation:
200 + 30 + 140 + 21 = 391
17 x 23 = 391

Question 2.
Which product does the model below represent?
Go Math Grade 4 Answer Key Chapter 3 Multiply 2-Digit Numbers Go Math Grade 4 Answer Key Chapter 3 Multiply 2-Digit Numbers img 12 img 12
Options:
a. 219
b. 225
c. 244
d. 275

Answer:
b. 225

Explanation:
130 + 20 + 65 + 10 = 225
15 x 15 = 225

Spiral Review

Question 3.
Mariah builds a tabletop using square tiles. There are 12 rows of tiles and 30 tiles in each row. How many tiles in all does Mariah use?
Options:
a. 100
b. 180
c. 360
d. 420

Answer:
c. 360

Explanation:
From the given data,
Mariah builds a tabletop using square tiles
Square contains 12 rows of tiles and 30 tiles in each row = 12 x 30
12 x 30 = 360 tiles
Total tiles used by Mariah = 360 tiles

Question 4.
Trevor bakes 8 batches of biscuits, with 14 biscuits in each batch. He sets aside 4 biscuits from each batch for a bake sale and puts the rest in a jar. How many biscuits does Trevor put in the jar?
Options:
a. 112
b. 80
c. 50
d. 32

Answer:
b. 80

Explanation:
As per the given data,
Number of biscuits baked by Trevor = 8 batches
Number of biscuits in each batch = 14 biscuits
So, total biscuits = 14 x 8 = 112
Trevor sets aside 4 biscuits from each batch for a bake = 8*4 = 32 biscuits are aside for a bake
Trevor kept rest of biscuits in a jar = 112 – 32 = 80
So, 80 biscuits are put in the jar by the Trevor

Question 5.
Li feeds her dog 3 cups of food each day. About how many cups of food does her dog eat in 28 days?
Options:
a. 60 cups
b. 70 cups
c. 80 cups
d. 90 cups

Answer:
c. 80 cups

Explanation:
As per the given data,
Li feeds her dog per day = 3 cups of food
Then, Li feeds her dog for 28 days = 3 x 28
= 84 cups of food
So, Li feeds her dog with 84 cups of food in 28 days

Question 6.
Which symbol makes the number sentence true?
4 ■ 0 = 0
Options:
a. +
b. –
c. ×
d. ÷

Answer:
c. ×

Explanation:
4 x 0 = 0

Page No. 165

Question 1.
Find 24 × 34.
Go Math Grade 4 Answer Key Chapter 3 Multiply 2-Digit Numbers img 13
Go Math Grade 4 Answer Key Chapter 3 Multiply 2-Digit Numbers img 14
_____

Answer:
816

Explanation:
Go Math Grade 4 Answer Key Chapter 3 Multiply 2-Digit Numbers img 13
chapter 3 - Area models and partial products- image 9. jpg

Question 2.
1 2
× 1 2
——–
_____

Answer:
144

Explanation:
chapter 3 - Area models and partial products- image 10. jpg
chapter 3 - Area models and partial products- image 11. jpg

Question 3.
3 1
× 2 4
——-
_____

Answer:
744

Explanation:
chapter 3 - Area models and partial products- image 12. jpg
chapter 3 - Area models and partial products- image 13. jpg

Question 4.
2 5
× 4 3
——-
_____

Answer:
1,075

Explanation:
chapter 3 - Area models and partial products- image 14. jpg
chapter 3 - Area models and partial products- image 15. jpg

Question 5.
3 7
× 2 4
——-
_____

Answer:
888

Explanation:
chapter 3 - Area models and partial products- image 16. jpg
chapter 3 - Area models and partial products- image 17. jpg

Question 6.
5 4
× 1 5
——-
_____

Answer:
810

Explanation:
chapter 3 - Area models and partial products- image 18. jpg
chapter 3 - Area models and partial products- image 19. jpg

Question 7.
8 7
× 1 6
——-
_____

Answer:
1,392

Explanation:
chapter 3 - Area models and partial products- image 20. jpg
chapter 3 - Area models and partial products- image 21. jpg

Question 8.
6 2
× 5 6
——-
_____

Answer:
3,472

Explanation:
chapter 3 - Area models and partial products- image 22. jpg
chapter 3 - Area models and partial products- image 23. jpg

Question 9.
4 9
× 6 3
——-
_____

Answer:
3,087

Explanation:
chapter 3 - Area models and partial products- image 24. jpg
chapter 3 - Area models and partial products- image 25. jpg

Practice: Copy and Solve Record the product.

Question 10.
38 × 47
_____

Answer:
1,786

Explanation:
chapter 3 - Area models and partial products- image 26. jpg
chapter 3 - Area models and partial products- image 27. jpg

Question 11.
46 × 27
_____

Answer:
1,242

Explanation:
chapter 3 - Area models and partial products- image 28. jpg
chapter 3 - Area models and partial products- image 29. jpg

Question 12.
72 × 53
_____

Answer:
3,816

Explanation:
chapter 3 - Area models and partial products- image 30. jpg
chapter 3 - Area models and partial products- image 31. jpg

Question 13.
98 × 69
_____

Answer:
6,762

Explanation:
chapter 3 - Area models and partial products- image 32. jpg
chapter 3 - Area models and partial products- image 33. jpg

Question 14.
53 × 68
_____

Answer:
3,604

Explanation:
chapter 3 - Area models and partial products- image 34. jpg
chapter 3 - Area models and partial products- image 35. jpg

Question 15.
76 × 84
_____

Answer:
6,384

Explanation:
chapter 3 - Area models and partial products- image 36. jpg
chapter 3 - Area models and partial products- image 37. jpg

Question 16.
92 × 48
_____

Answer:
4,416

Explanation:
chapter 3 - Area models and partial products- image 38. jpg
chapter 3 - Area models and partial products- image 39. jpg

Question 17.
37 × 79
_____

Answer:
2,923

Explanation:
chapter 3 - Area models and partial products- image 40. jpg
chapter 3 - Area models and partial products- image 41. jpg

Reason Abstractly Algebra Find the unknown digits. Complete the problem.

Question 18.
Go Math Grade 4 Answer Key Chapter 3 Multiply 2-Digit Numbers img 15
Type below:
___________

Answer:
1,824

Explanation:
chapter 3 - Area models and partial products- image 42. jpg

Question 19.
Go Math Grade 4 Answer Key Chapter 3 Multiply 2-Digit Numbers img 16
Type below:
___________

Answer:
7,954

Explanation:
chapter 3 - Area models and partial products- image 43. jpg

Question 20.
Go Math Grade 4 Answer Key Chapter 3 Multiply 2-Digit Numbers img 17
Type below:
___________

Answer:
1,908

Explanation:
chapter 3 - Area models and partial products- image 44. jpg

Question 21.
Go Math Grade 4 Answer Key Chapter 3 Multiply 2-Digit Numbers img 18
Type below:
___________

Answer:
952

Explanation:
chapter 3 - Area models and partial products- image 45. jpg

Page No. 166

Use the picture graph for 22–24.
Go Math Grade 4 Answer Key Chapter 3 Multiply 2-Digit Numbers img 19

Question 22.
Use Graphs A fruit-packing warehouse is shipping 15 boxes of grapefruit to a store in Santa Rosa, California. What is the total weight of the shipment?
______ pounds

Answer:
1275 pounds

Explanation:
From the given data,
A fruit packing warehouse is shipping 15 boxes of grapefruit to store in Santa Rose, California
Grapefruit weight per box = 85 pounds
Total weight of the shipment = 85 x 15
chapter 3 - Area models and partial products- image 46. jpg
So, the total weight of the shipment = 1275 pounds

Question 23.
How much less do 13 boxes of tangelos weigh than 18 boxes of tangerines?
______ pounds

Answer:
450 pounds

Explanation:
As per the given data,
Tangelos weight per box = 90 pounds
Then, the weight of the 13 boxes of tangelos = 90 x 13
chapter 3 - Area models and partial products- image 47. jpg
And, the weight of the 18 boxes of tangelos = 90 x 18
chapter 3 - Area models and partial products- image 48. jpg
1620 – 1170 = 450
So, 13 boxes of tangelos weight are 450 pounds less than 18 boxes of tangelos weight

Question 24.
What is the weight of 12 boxes of oranges?
______ pounds

Answer:
1,080 pounds

Explanation:
The weight of the oranges per box = 90 pounds
then, weight of 12 boxes oranges = 90 x 12
chapter 3 - Area models and partial products- image 49. jpg
So, weight of 12 boxes oranges = 1,080 pounds

Question 25.
Each person in the United States eats about 65 fresh apples each year. Based on this estimate, how many apples do 3 families of 4 eat each year?
______ apples

Answer:
780 apples

Explanation:
From the given data,
Each person in the united states eats fresh apples per year = 65
3 families of 4 persons = 3 x 4 = 12 persons
Then, the number of apples eat by 12 persons = 65 x 12
chapter 3 - Area models and partial products- image 50. jpg
So, the total number of apples eat by 12 persons per year = 780

Question 26.
The product 26 × 93 is greater than 25 × 93. How much greater? Explain how you know without multiplying.
______

Answer:
The difference is 93
26 x 93 is one more group of 93 than 25 x 93

Question 27.
Margot wants to use partial products to find 22 × 17. Write the numbers in the boxes to show 22 × 17.
Go Math Grade 4 Answer Key Chapter 3 Multiply 2-Digit Numbers img 20
Type below:
__________

Answer:
chapter 3 - Area models and partial products- image 51. jpg

Explanation:
22 x 17
(20 + 2) x 17
20 x 17 + 2 x 17
20 x (10 + 7) + 2 x (10 + 7)
(20 x 10) + (20 x 7) + (2 x 10) + (2 x 7)
chapter 3 - Area models and partial products- image 51. jpg

Common Core – Page No. 167

Multiply Using Partial Products

Record the product.

Question 1.
2 3
× 7 9
———
1, 4 0 0
2 1 0
1 8 0
+ 2 7
——–
1, 8 1 7

Answer:
1, 8 1 7

Explanation:
2 3
× 7 9
———
1, 4 0 0
2 1 0
1 8 0
+ 2 7
——–
1, 8 1 7

Question 2.
5 6
× 3 2
——-
_______

Answer:
1,792

Explanation:
Chapter 3 - Common core - Image 7

Question 3.
8 7
× 6 4
——-
_______

Answer:
5,568

Explanation:
Chapter 3 - Common core - Image 8

Question 4.
3 3
× 2 5
——-
_______

Answer:
825

Explanation:
Chapter 3 - Common core - Image 9

Question 5.
9 4
× 1 2
——-
_______

Answer:
1,128

Explanation:
Chapter 3 - Common core - Image 10

Question 6.
5 1
× 7 7
——-
_______

Answer:
3,927

Explanation:
Chapter 3 - Common core - Image 11

Question 7.
6 9
× 4 9
——-
_______

Answer:
3,381

Explanation:
Chapter 3 - Common core - Image 12

Question 8.
8 6
× 8 4
——-
_______

Answer:
7,224

Explanation:
Chapter 3 - Common core - Image 13

Question 9.
9 8
× 4 2
——-
_______

Answer:
4,116

Explanation:
Chapter 3 - Common core - Image 14

Question 10.
7 3
× 3 7
——-
_______

Answer:
2,701

Explanation:
Chapter 3 - Common core - Image 15

Question 11.
8 5
× 5 1
——-
_______

Answer:
4,335

Explanation:
Chapter 3 - Common core - Image 16

Problem Solving

Question 12.
Evelyn drinks 8 glasses of water a day, which is 56 glasses of water a week. How many glasses of water does she drink in a year? (1 year = 52 weeks)
_______ glasses

Answer:
2,912 glasses

Explanation:
As per the given data,
Evelyn drinks 8 glasses of water a day
Evelyn drinks water per week = 56 glasses
Then, the number of glasses per 52 weeks = 52 x 56
Chapter 3 - Common core - Image 17
Total number of glasses of water drink by Evelyn per year = 2912 glasses of water

Question 13.
Joe wants to use the Hiking Club’s funds to purchase new walking sticks for each of its 19 members. The sticks cost $26 each. The club has $480. Is this enough money to buy each member a new walking stick? If not, how much more money is needed?
Is the money enough? _______
How much more is needed? _______

Answer:
This amount is not enough to buy walking sticks
Still, $14 amount is needed to buy walking sticks

Explanation:
From the given data,
Joe wants to use the Hiking club funds to purchase new walking sticks for each of its 19 members
Cost per each stick = $26
Total walking sticks cost per 19 members = $26 x 19
Chapter 3 - Common core - Image 18
Total cost for walking sticks for 19 members = $494
The club has = $480
This amount is not enough to buy walking sticks
Still, $14 amount is needed to buy walking sticks

Common Core – Page No. 168

Lesson Check

Question 1.
A carnival snack booth made $76 selling popcorn in one day. It made 22 times as much selling cotton candy. How much money did the snack booth make selling
cotton candy?
Options:
a. $284
b. $304
c. $1,562
d. $1,672

Answer:
d. $1,672

Explanation:
As per the given data,
A carnival snack booth made popcorn in one day = $76
It made 22 times as much selling cotton candy
Then, total selling cotton candy made by snack booth = $76 x 22
Chapter 3 - Common core - Image 19
So, $1672 money snack booth will get for selling cotton candy

Question 2.
What are the partial products of
42 × 28?
Options:
a. 800, 80, 40, 16
b. 800, 16
c. 800, 40, 320, 16
d. 80, 16

Answer:
c. 800, 40, 320, 16

Explanation:
Chapter 3 - Common core - Image 20
So, partial products of 42 x 28 are 800, 40, 320, 16

Spiral Review

Question 3.
Last year, the city library collected 117 used books for its shelves. This year, it collected 3 times as many books. How many books did it collect this year?
Options:
a. 832
b. 428
c. 351
d. 72

Answer:
c. 351

Explanation:
From the given data,
Last year, the number of used books collected by city library by its shelves = 117 books
This year, it collected 3 times as many books = 3 x 117 =351 books
Total number of books collected by the city library for this year = 351 books

Question 4.
Washington Elementary has 232 students. Washington High has 6 times as many students. How many students does Washington High have?
Options:
a. 1,392
b. 1,382
c. 1,292
d. 1,281

Answer:
a. 1,392

Explanation:
As per the given data,
The number of students in Washington elementary = 232 students
Washington High has 6 times as many students = 6 x 232 = 1392
Total number of students in Washington High = 1392 students

Question 5.
What are the partial products of 35 × 7?
Options:
a. 10, 12
b. 21, 35
c. 210, 35
d. 350, 21

Answer:
c. 210, 35

Explanation:
Partial products of 35 x 7 are 210, 35

Question 6.
Shelby has ten $5 bills and thirteen $10 bills. How much money does Shelby have in all?
Options:
a. $15
b. $60
c. $63
d. $180

Answer:
d. $180

Explanation:
From the given data,
Shelby has ten $5 bills and thirteen $10 bills = (10 x $5) + (13 x $10)
= ($50) + ($130)
=$180
Total money with Shelby = $180

Page No. 169

Question 1.
Explain how to find 40 × 50 using mental math.
Type below:
__________

Answer:
200

Explanation:
40 x 50
By using mental math
4 x 5 = 20
40 x 50 = 200

Question 2.
What is the first step in estimating 56 × 27?
Type below:
__________

Answer:
18 centimeters

Explanation:
Round to the nearest values.
So, the first step of the estimated 56 x 27 is rounding to the nearest values that is 60 x 30

Choose a method. Then find the product.

Question 3.
35 × 10 = _____

Answer:
350

Explanation:
By using the place value method
You can take 10 as 1 ten
35 x 10 = 35 x 1 ten
= 35 ten
35 x 10 = 350

Question 4.
19 × 20 = _____

Answer:
380

Explanation:
19 x 20
By using the associative property
You can think of 20 as (2 x 10)
19 x 20 = 19 x (2 x 10)
= (19 x 2) x 10
= 38 x 10
19 x 20 = 380

Question 5.
12 × 80 = _____

Answer:
960

Explanation:
Use the halving and doubling strategy
half of the 80 to make the problem simpler
80/ 2 = 40
Multiply 40 with 12
40*12 = 480
Double the 480
2*480= 960
12*80 = 960

Question 6.
70 × 50 = _____

Answer:
3,500

Explanation:
70 x 50
By using the place value method
You can take 50 as 5 tens
70 x 50 = 70 x 5 tens
= 350 tens
70 x 50 = 3,500

Question 7.
58 × 40 = _____

Answer:
2,320

Explanation:
By using the associative property
You can think of 40 as (4 x 10)
58 x 40 = 58 x (4 x 10)
= (58 x 4) x 10
= 232 x 10
58 x 40 = 2,320

Question 8.
30 × 40 = _____

Answer:
1,200

Explanation:
Use the halving and doubling strategy
half of the 40 to make the problem simpler
40/ 2 = 20
Multiply 20 with 30
20*30 = 600
Double the 600
2*600= 1200
30*40 = 1,200

Question 9.
14 × 60 = _____

Answer:
840

Explanation:
By using the place value method
You can take 60 as 6 tens
14 x 60 = 14 x 6 tens
= 84 tens
14 x 60 = 840

Question 10.
20 × 30 = _____

Answer:
600

Explanation:
By using the associative property
You can think of 30 as (3 x 10)
20 x 30 = 20 x (3 x 10)
= (20 x 3) x 10
= 60 x 10
20 x 30 = 600

Question 11.
16 × 90 = _____

Answer:
1,440

Explanation:
Use the halving and doubling strategy
half of the 90 to make the problem simpler
90/ 2 = 45
Multiply 45 with 16
16*45 = 720
Double the 720
2*720= 1440
16*90 = 1,440

Estimate the product. Choose a method.

Question 12.
81 × 38
Estimate: _____

Answer:
3,200

Explanation:
Round to the nearest tens.
81 is close to 80; 38 is close to 40;
80 x 40 = 3,200

Question 13.
16 × $59
Estimate: $ _____

Answer:
$120

Explanation:
Round to the nearest tens.
16 is close to 20; $59 is close to $60;
Use the mental math to find the product of 20 x $60
2 x $6 = $12
20 x $60 = $120
Estimated product of 16 x $59 = $120

Question 14.
43 × 25
Estimate: _____

Answer:
1,000

Explanation:
Round to the nearest tens.
43 is close to 40; 25 is close to 25;
40 x 25 = 1000
Estimated product of 43 x 25 = 1,000

Question 15.
76 × 45
Estimate: _____

Answer:
3,200

Explanation:
Round to the nearest tens.
76 is close to 80; 45 is close to 40;
Use the mental math
8 x 4 = 32
80 x 40 = 3200
So, the estimated product of 76 x 45 = 3,200

Question 16.
65 × $79
Estimate: _____

Answer:
$4,800

Explanation:
Round to the nearest tens.
65 is close to 60; $79 is close to $80;
Use the mental math
6 x $8 = $48
60 x $80 = $4800
So, estimated product of 65 x $79 = $4,800

Question 17.
92 × 38
Estimate: _____

Answer:
3,600

Explanation:
Round to the nearest tens.
92 is close to 90; 38 is close to 40;
Use the mental math, then
9 x 4 = 36
90 x 40 = 3,600
So, estimated product of 92 x 38 = 3,600

Question 18.
37 × 31
Estimate: _____

Answer:
1,200

Explanation:
Round to the nearest tens.
37 is close to 40; 31 is close to 30;
Use the mental math, then
4 x 3 = 12
40 x 30 = 1,200
So, estimated product of 37 x 31 = 1,200

Question 19.
26 × $59
Estimate: _____

Answer:
$1,800

Explanation:
Round to the nearest tens.
26 is close to 30; $59 is close to $60;
Use the mental math, then
3 x $6 = $18
30 x $60 = $1,800
So, estimated product of 26 x $59 = $1,800

Question 20.
54 × 26
Estimate: _____

Answer:
18 centimeters

Explanation:
Round to the nearest tens.
54 is close to 50; 26 is close to 30;
Use the mental math
5 x 3 = 15
50 x 30 = 1,500
So, estimated product of 54 x 26 = 1,500

Question 21.
52 × 87
Estimate: _____

Answer:
4,500

Explanation:
Round to the nearest tens.
52 is close to 50; 87 is close to 90;
Use the mental math
5 x 9 = 45
50 x 90 = 4500
So, estimated product of 52 x 87 = 4,500

Question 22.
39 × 27
Estimate: _____

Answer:
18 centimeters

Explanation:
Round to the nearest tens.
39 is close to 40; 27 is close to 30;
Use the mental math
4 x 3 = 12
40 x 30 = 1,200
So, estimated product of 39 x 27 = 1,200

Question 23.
63 × 58
Estimate: _____

Answer:
3,600

Explanation:
Round to the nearest tens.
63 is close to 60; 58 is close to 60;
Use the mental math
6 x 6 = 36
60 x 60 = 3,600
So, estimated product of 63 x 58 = 3,600

Page No. 170

Question 24.
Ms. Traynor’s class is taking a field trip to the zoo. The trip will cost $26 for each student. There are 22 students in her class. What is a good estimate for the cost of the students’ field trip?
Type below:
__________

Answer:
18 centimeters

Explanation:
As per the given data,
Ms. Traynor’s class is taking a field trip to the zoo
Cost of the trip for each student = $26
Total number of students in her class = 22
The total cost of the trip for students = $26 x 22
Round to the nearest tens.
26 is close to 30; 22 is close to 20;
Use the mental math
$3 x 2 = $6
$30 x 20 = $600
Then, the total estimated cost for the trip for students = $600

Question 25.
Tito wrote the following on the board. What is the unknown number?
Go Math Grade 4 Answer Key Chapter 3 Multiply 2-Digit Numbers img 21
______

Answer:
400

Explanation:
An unknown number is 50 x 8 = 400

Question 26.
What are the partial products that result from multiplying 15 × 32?
Type below:
__________

Answer:
Partial products are 300, 150, 20, 10

Explanation:
Chapter 3 - Common core - Image 36
Partial products are 300, 150, 20, 10

Question 27.
A city bus company sold 39 one-way tickets and 20 round-trip tickets from West Elmwood to East Elmwood. One-way tickets cost $14. Round trip tickets cost $25. How much money did the bus company collect?
$ ______

Answer:
$1,046

Explanation:
As per the given data,
Number of one – way tickets sold by the city bus company = 39
Round trip tickets from west Elmwood to east Elmwood = 20
Cost of one – way tickets = $14
Then, cost of 39 one – way tickets = 39 x $14 =$546
Cost of round trip tickets = $25
Then, cost of 20 round trip tickets = $25 x 20 = $500
Total money collected by the city bus company = $546 + $500 = $1,046

Page No. 173

Question 1.
Look at the problem. Complete the sentences.
Multiply ____ and ____ to get 0.
Multiply ____ and ____ to get 1,620.
Add the partial products.
0 + 1,620 = ____
Go Math Grade 4 Answer Key Chapter 3 Multiply 2-Digit Numbers img 22
_____

Answer:
Multiply 27 and 0 to get 0.
Multiply 27 and 6 to get 1,620.
Add the partial products. 0 + 1,620 = 1,620.

Estimate. Then find the product.

Question 2.
6 8
× 5 3
——-
Estimate: _________
Product: __________

Answer:
Estimate: 3,500
Product: 3,604

Explanation:
68 is closer to 70 and 53 is closer to 50
Estimate: 70 x 50 = 3,500
60 x 53 = 3180
8 x 53 = 424
3180 + 424 = 3604
Product 3,604

Question 3.
6 1
× 5 4
——-
Estimate: _________
Product: __________

Answer:
Estimate: 3,000
Product: 3,294

Explanation:
61 is closer to 60 and 54 is closer to 50
Estimate: 60 x 50 = 3,000
60 x 54 = 3240
1 x 54 = 54
3240 + 54 = 3294
Product 3,294

Question 4.
9 0
× 2 7
——-
Estimate: _________
Product: __________

Answer:
Estimate: 2,700
Product: 2,430

Explanation:
27 is closer to 30
Estimate: 90 x 30 = 2,700
90 x 27 = 2430
Product 2,430

Question 5.
3 0
× 4 7
——-
Estimate: _________
Product: __________

Answer:
Estimate: 1,500
Product: 1,410

Explanation:
47 is closer to 50
Estimate: 30 x 50 = 1,500
30 x 47 = 1410
Product 1,410

Question 6.
7 8
× 5 6
——-
Estimate: _________
Product: __________

Answer:
Estimate: 4,800
Product: 4,368

Explanation:
78 is closer to 80 and 56 is closer to 60
Estimate: 80 x 60 = 4,800
70 x 56 = 3920
8 x 56 = 448
3920 + 448 = 4368
Product 4,368

Question 7.
2 7
× 2 5
——-
Estimate: _________
Product: __________

Answer:
Estimate: 600
Product: 675

Explanation:
27 is closer to 30 and 25 is closer to 20
Estimate: 30 x 20 = 600
20 x 25 = 500
7 x 25 = 175
500 + 175 = 675
Product 675

Practice: Copy and Solve Estimate. Then find the product.

Question 8.
34 × 65
Estimate: _________
Product: __________

Answer:
Estimate: 1,800
Product: 2,210

Explanation:
34 is closer to 30 and 65 is closer to 60
Estimate: 30 x 60 = 1,800
30 x 65 = 1950
4 x 65 = 260
1950 + 260 = 2210
Product 2,210

Question 9.
42 × $13
Estimate: $ _________
Product: $ _________

Answer:
Estimate: $400
Product: $546

Explanation:
42 is closer to 40 and 13 is closer to 10
Estimate: 40 x 10 = 400
40 x $13 = $520
2 x $13= $26
$520 + $26 = $546
Product $546

Question 10.
60 × 17
Estimate: _________
Product: __________

Answer:
Estimate: 1,200
Product: 1,020

Explanation:
17 is closer to 20
Estimate: 60 x 20 = 1,200
60 x 17 = 1020
Product = 1,020

Question 11.
62 × 45
Estimate: _________
Product: __________

Answer:
Estimate: 2,400
Product: 2,790

Explanation:
62 is closer to 60 and 45 is closer to 40
Estimate: 60 x 40 = 2,400
60 x 45 = 2700
2 x 45= 90
2700 + 90 = 2790
Product 2,790

Question 12.
57 × $98
Estimate: $ _________
Product: $ _________

Answer:
Estimate: 6,000
Product: 5,586

Explanation:
57 is closer to 60 and 98 is closer to 100
Estimate: 60 x 100 = 6,000
50 x 98 = 4900
7 x 98= 686
4900 + 686 = 5586
Product 5,586

Look for a Pattern Algebra Write a rule for the pattern.
Use your rule to find the unknown numbers.

Question 13.
Go Math Grade 4 Answer Key Chapter 3 Multiply 2-Digit Numbers img 23
Rule _____________
Type below:
_________

Answer:
Chapter 3 - Common core - Image 37

Explanation:
1 hour = 60 min
Then, 5hr = 5 x 60 = 300 min
10hr = 10 x 60 = 600 min
15hr = 15 x 60 = 900 min
20hr = 20 x 60 = 1200 min
25hr = 25 x 60 = 1500 min

Question 14.
Owners of a summer camp are buying new cots for their cabins. There are 16 cabins. Each cabin needs 6 cots. Each cot costs $92. How much will the new cots cost?
$ _______

Answer:
$8,832

Explanation:
As per the given data,
Owners pf a summer camp are buying new cots for their cabins
Number of cabins = 16
Each cabin needs 6 cots
Then, total cots = 16 x 6 = 96
Each cot cost = $92
Then, cost for total cots = $92 x 96
92 is closer to 90 and 96 is closer to 100
Estimate = 90 x 100 = 9,000
90 x 96 = 8640
2 x 96 = 192
8640 + 192 = 8832
Product = 8,832

Question 15.
A theater has 28 rows of 38 seats downstairs and 14 rows of 26 seats upstairs. How many seats does the theater have?
______ seats

Answer:
1,428 seats

Explanation:
As per the given data,
A theatre has 28 rows of 38 seats downstairs = 28 x 38 = 1064
14 rows of 26 seats upstairs = 14 x 26 = 364
Total number of seats = 1064 + 364 = 1,428 seats

Page No. 174

Question 16.
Machine A can label 11 bottles in 1 minute. Machine B can label 12 bottles in 1 minute. How many bottles can both machines label in 15 minutes?
Go Math Grade 4 Answer Key Chapter 3 Multiply 2-Digit Numbers img 24
a. What do you need to know?
Type below:
__________

Answer:
number of bottles labeled by Machine A and Machine B in 15 minutes

Question 16.
b. What numbers will you use?
Type below:
__________

Answer:
15x 11 and 15 x 12

Question 16.
c. Tell why you might use more than one operation to solve the problem.
Type below:
__________

Answer:
To find out the total number of bottle made by both machines A & B

Question 16.
d. Solve the problem.
So, both machines can label ____ bottles in ____ minutes.
Type below:
__________

Answer:
Machine A can label 11 bottles in 1 minute
Then, the number of bottles labeled by machine A in 15 minutes = 15 x 11 = 165
Machine B can label 12 bottles in 1 minute
Then, number of bottles labelled by Machine B in 15 minutes = 15 x 12 = 180
Total bottles labelled by both the machines in 15 minutes = 165 + 180 = 345

Question 17.
Make Sense of Problems
A toy company makes wooden blocks. A carton holds 85 blocks. How many blocks can 19 cartons hold?
______ blocks

Answer:
1,615 blocks

Explanation:
From the given data,
A toy company makes wooden blocks
A carton holds 85 blocks
Then, number of blocks hold by 19 cartons = 19 x 85 = 1615
Total number of blocks held by 19 cartons = 1,615

Question 18.
A company is packing cartons of candles. Each carton can hold 75 candles. So far, 50 cartons have been packed, but only 30 cartons have been loaded on a truck. How many more candles are left to load on the truck?
______ candles

Answer:
1,500 candles

Explanation:
As per the given data,
A company is packing cartons of candles
Each carton can hold 75 candles
Then, number of candles hold by 50 cartons = 50 x 75 = 3750
Number of candles hold by 30 cartons = 30 x 75 = 2250
50 cartons have been packed, but only 30 cartons have been loaded on a truck
Remaining candles are left to load on truck = 3750 – 2250 = 1,500

Question 19.
Mr. Garcia’s class raised money for a field trip to the zoo. There are 23 students in his class. The cost of the trip will be $17 for each student. What is the cost for all the students? Explain how you found your answer.
$ ______

Answer:
$391

Explanation:
As per the given data,
Mr. Garcia’s class raised money for a field trip to the zoo
Total number of students in his class = 23 students
Cost of the trip for each student = $17
Then, total cost for all the students = $17 x 23 = $391

Common Core – Page No. 175

Multiply with Regrouping
Estimate. Then find the product.

Question 1.
Estimate: 2,700
Think: 87 is close to 90 and 32 is close to 30.
90 × 30 = 2,700
Go Math Grade 4 Answer Key Chapter 3 Multiply 2-Digit Numbers Common Core img 25

Answer:
2,784

Explanation:
Think: 87 is close to 90 and 32 is close to 30.
90 × 30 = 2,700
Go Math Grade 4 Answer Key Chapter 3 Multiply 2-Digit Numbers Common Core img 25

Question 2.
7 3
× 2 8
——–
Estimate: ______
Product: _______

Answer:
Estimate: 2,100
Product: 2,044

Explanation:
Estimate: 73 is close to 70; 28 is close to 30.
So, 70 x 30 = 2,100.
Product: Write 73 as 7 tens and 3 ones. Multiply 28 by 3 ones.
2
28
x 73
——–
84 <– 3 x 28
Multiply 28 by 7 tens
5
28
x 73
——–
1960 <– 70 x 28
Add the partial products.
84 + 1960 = 2,044.
So, 73 x 28 = 2,044.

Question 3.
4 8
× 3 8
——–
Estimate: ______
Product: _______

Answer:
Estimate: 2,000
Product: 1,824

Explanation:
48 is close to 50 and 38 is close to 40.
Estimate: 50 × 40 = 2,000
40 x 38 = 1520
8 x 38 = 304
1520 + 304 = 1824.
Product: 1,824

Question 4.
5 9
× 5 2
——–
Estimate: ______
Product: _______

Answer:
Estimate: 3,000
Product: 3,068

Explanation:
59 is close to 60 and 52 is close to 50.
Estimate: 60 × 50 = 3,000
50 x 52 = 2600
9 x 52 = 468
2600 + 468 = 3068.
Product: 3,068.

Question 5.
8 4
× 4 0
——–
Estimate: ______
Product: _______

Answer:
Estimate: 3,200
Product: 3,360

Explanation:
84 is close to 80 and 40 is close to 40.
Estimate: 80 × 40 = 3,200
80 x 40 = 3,200
4 x 40 = 160
3200 + 160 = 3,360.
Product: 3,360.

Question 6.
8 3
× 7 7
——–
Estimate: ______
Product: _______

Answer:
Estimate: 6,400
Product: 6,391

Explanation:
83 is close to 80 and 77 is close to 80.
Estimate: 80 × 80 = 6,400
80 x 77 = 6,160
3 x 77 = 231
6,160 + 231 = 6,391.
Product: 6,391.

Question 7.
9 1
× 1 9
——–
Estimate: ______
Product: _______

Answer:
Estimate: 1,800
Product: 1,729

Explanation:
91 is close to 90 and 19 is close to 20.
Estimate: 90 × 20 = 1,800
90 x 19 = 1,710
1 x 19 = 19
1,710+ 19 = 1,729.
Product: 1,729.

Problem Solving

Question 8.
Baseballs come in cartons of 84 baseballs. A team orders 18 cartons of baseballs. How many baseballs does the team order?
_______ baseballs

Answer:
1,512 baseballs

Explanation:
To find total baseballs, 84 x 18
80 x 18 = 1,440
4 x 18 = 72
84 x 18 = 1,512

Question 9.
There are 16 tables in the school lunch room. Each table can seat 22 students. How many students can be seated at lunch at one time?
_______ students

Answer:
352 students

Explanation:
Total Students = 16 x 22
10 x 22 = 220
6 x 22 = 132
220 + 132 = 352.
352 students can be seated at lunch at one time

Common Core – Page No. 176

Lesson Check

Question 1.
The art teacher has 48 boxes of crayons. There are 64 crayons in each box. Which is the best estimate of the number of crayons the art teacher has?
Options:
a. 2,400
b. 2,800
c. 3,000
d. 3,500

Answer:
c. 3,000

Explanation:
1. Total number of crayons = 48 x 64
48 is close to 50; 64 is close to 60
50 x 60 = 3,000.
The art teacher has about to 3, 000 crayons.

Question 2.
A basketball team scored an average of 52 points in each of 15 games. How many points did the team score in all?
Options:
a. 500
b. 312
c. 780
d. 1,000

Answer:
c. 780

Explanation:
Total Points = 52 x 15
50 x 15 = 750
2 x 15 = 30
750 + 30 = 780.
The basketball team scored 780 points in total.

Spiral Review

Question 3.
One Saturday, an orchard sold 83 bags of apples. There are 27 apples in each bag. Which expression represents the total number of apples sold?
Options:
a. 16 + 6 + 56 + 21
b. 160 + 60 + 56 + 21
c. 160 + 60 + 560 + 21
d. 1,600 + 60 + 560 + 21

Answer:
d. 1,600 + 60 + 560 + 21

Explanation:
Total number of apples sold = 83 x 27
80 x 27 = 2,160
3 x 27 = 81
2,160 + 81 = 2,241.
The total number of apples sold = 2,241.
16 + 6 + 56 + 21 = 99 not equal to 2,241
160 + 60 + 56 + 21 = 297 not equal to 2,241
160 + 60 + 560 + 21 = 801 not equal to 2,241
1,600 + 60 + 560 + 21 = 2,241 equal to 2,241
1,600 + 60 + 560 + 21 = 2,241 is correct.

Question 4.
Hannah has a grid of squares that has 12 rows with 15 squares in each row. She colors 5 rows of 8 squares in the middle of the grid blue. She colors the rest of
the squares red. How many squares does Hannah color red?
Options:
a. 40
b. 140
c. 180
d. 220

Answer:
b. 140

Explanation:
Hannah has a grid of squares that has 12 rows with 15 squares in each row = 12 x 15 = 180.
The grid of squares in blue = 5 x 8 = 40.
The grid of squares in red = 180 – 40 = 140.

Question 5.
Gabriella has 4 times as many erasers a Leona. Leona has 8 erasers. How many erasers does Gabriella have?
Options:
a. 32
b. 24
c. 12
d. 2

Answer:
a. 32

Explanation:
Gabriella have 4 x 8 = 32 erasers.

Question 6.
Phil has 3 times as many rocks as Peter. Together, they have 48 rocks. How many more rocks does Phil have than Peter?
Options:
a. 36
b. 24
c. 16
d. 12

Answer:
b. 24

Explanation:
Phil has 3 times as many rocks as Peter. Together, they have 48 rocks
If Peter has x rocks, Phil has 3x rocks
3x + x = 48.
4x = 48.
x = 48/4 = 12.
Peter has 12 rocks. Phil has 3 x 12 = 36 rocks.
Phil has 36 – 12 = 24 more rocks than Peter.

Page No. 179

Question 1.
Find the product.
Go Math Grade 4 Answer Key Chapter 3 Multiply 2-Digit Numbers img 26
Estimate: ______
Product: _______

Answer:
Estimate: 1,500
Product: 1,566

Explanation:
54 x 29
Estimate: Think 54 is close to 50; 29 is close to 30.
50 x 30 = 1,500
Product:
20 x 5 tens = 100 tens
20 x 4 ones = 80 ones
9 x 5 tens = 45 tens
9 x 4 ones = 36 ones.
Add partial products.
1000 + 80 + 450 + 36 = 1,566.

Estimate. Then choose a method to find the product.

Question 2.
3 6
× 1 4
——-
Estimate: ______
Product: _______

Answer:
Estimate: 400
Product: 504

Explanation:
36 x 14
Estimate: Think 36 is close to 40; 14 is close to 10.
40 x 10 = 400
Product:
10 x 3 tens = 30 tens
10 x 6 ones = 60 ones
4 x 3 tens = 12 tens
4 x 6 ones = 24 ones.
Add partial products.
300 + 60 + 120 + 24 = 504.

Question 3.
6 3
× 4 2
——-
Estimate: ______
Product: _______

Answer:
Estimate: 2,400
Product: 2646

Explanation:
63 x 42
Estimate: Think 63 is close to 60; 42 is close to 40.
60 x 40 = 2400
Product:
40 x 6 tens = 240 tens
40 x 3 ones = 120 ones
2 x 6 tens = 12 tens
2 x 3 ones = 6 ones.
Add partial products.
2400 + 120 + 120 + 6 = 2646.

Question 4.
8 4
× 5 3
——-
Estimate: ______
Product: _______

Answer:
Estimate: 4,000
Product: 4,452

Explanation:
84 x 53
Estimate: Think 84 is close to 80; 53 is close to 50.
80 x 50 = 4,000
Product:
50 x 8 tens = 400 tens
50 x 4 ones = 200 ones
3 x 8 tens = 24 tens
3 x 4 ones = 12 ones.
Add partial products.
4000 + 200 + 240 + 12 = 4,452.

Question 5.
7 1
× 1 3
——-
Estimate: ______
Product: _______

Answer:
Estimate: 700
Product: 923

Explanation:
71 x 13
Estimate: Think 71 is close to 70; 13 is close to 10.
70 x 10 = 700
Product:
10 x 7 tens = 70 tens
10 x 1 ones = 10 ones
3 x 7 tens = 21 tens
3 x 1 ones = 3 ones.
Add partial products.
700 + 10 + 210 + 3 = 923.

Practice: Copy and Solve Estimate. Find the product.

Question 6.
29 × $82
Estimate: $ _______
Product: $ _______

Answer:
Estimate: $2,400
Product: $2,378

Explanation:
29 x $82
Estimate: Think 29 is close to 30; $82 is close to $80.
30 x $80 = $2,400
Product:
$80 x 2 tens = $160 tens
$80 x 9 ones = $720 ones
$2 x 2 tens = $4 tens
$2 x 9 ones = $18 ones.
Add partial products.
$1600 + $720 + $40 + $18 = $2,378.

Question 7.
57 × 79
Estimate: _______
Product: _______

Answer:
Estimate: 4,800
Product: 4,503

Explanation:
57 x 79
Estimate: Think 57 is close to 60; 79 is close to 80.
60 x 80 = 4,800
Product:
70 x 5 tens = 350 tens
70 x 7 ones = 490 ones
9 x 5 tens = 45 tens
9 x 7 ones = 63 ones.
Add partial products.
3500 + 490 + 450 + 63 = 4,503.

Question 8.
80 × 27
Estimate: _______
Product: _______

Answer:
Estimate: 2,400
Product: 2,160

Explanation:
80 x 27
Estimate: Think 27 is close to 30.
30 x 80 = 2,400
Product:
20 x 8 tens = 160 tens
20 x 0 ones = 0 ones
7 x 8 tens = 56 tens
7 x 0 ones = 0 ones.
Add partial products.
1600 + 0 + 560 + 0 = 2,160.

Question 9.
32 × $75
Estimate: $ _______
Product: $ _______

Answer:
Estimate: $2,100
Product: $2,400

Explanation:
32 × $75
Estimate: Think 32 is close to 30; $75 is close to $70.
30 x $70 = $2,100
Product:
$70 x 3 tens = $210 tens
$70 x 2 ones = $140 ones
$5 x 3 tens = $15 tens
$5 x 2 ones = $10 ones.
Add partial products.
$2100 + $140 + $150 + $10 = $2,400.

Question 10.
55 × 48
Estimate: _______
Product: _______

Answer:
Estimate: 2,750
Product: 2,640

Explanation:
55 × 48
Estimate: Think 48 is close to 50.
55 x 50 = 2,750
Product:
40 x 5 tens = 200 tens
40 x 5 ones = 200 ones
8 x 5 tens = 40 tens
8 x 5 ones = 40 ones.
Add partial products.
2000 + 200 + 400 + 40 = 2,640.

Question 11.
19 × $82
Estimate: $ _______
Product: $ _______

Answer:
Estimate: $1,600
Product: $1,558

Explanation:
19 × $82
Estimate: Think 19 is close to 20; $82 is close to $80.
20 x $80 = $1,600
Product:
$80 x 1 tens = $80 tens
$80 x 9 ones = $720 ones
$2 x 1 tens = $2 tens
$2 x 9 ones = $18 ones.
Add partial products.
$800 + $720 + $20 + $18 = $1,558.

Question 12.
25 × $25
Estimate: $ _______
Product: $ _______

Answer:
Estimate: $625
Product: $625

Explanation:
25 × $25
Estimate:
25 x $25 = $625
Product:
$20 x 2 tens = $40 tens
$20 x 5 ones = $100 ones
$5 x 2 tens = $10 tens
$5 x 5 ones = $25 ones.
Add partial products.
$400 + $100 + $100 + $25 = $625.

Question 13.
41 × 98
Estimate: _______
Product: _______

Answer:
Estimate: 4,000
Product: 4,018

Explanation:
41 × 98
Estimate: Think 41 is close to 40; 98 is close to 100.
40 x 100 = 4,000
Product:
90 x 4 tens = 360 tens
90 x 1 ones = 90 ones
8 x 4 tens = 32 tens
8 x 1 ones = 8 ones.
Add partial products.
3600 + 90 + 320 + 8 = 4,018.

Identify Relationships Algebra Use mental math to find the number.

Question 14.
30 × 14 = 420, so
30 × 15 = ______

Answer:
30 × 15 = 450

Explanation:
30 × 15 = 30 + 420
30 × 15 = 450

Question 15.
25 × 12 = 300, so
25 × ______ = 350

Answer:
25 x 14 = 350

Explanation:
25 × 12 = 300
For every next multiplication, the product value is increased by 25.
25 x 13 = 325.
25 x 14 =350.

Question 16.
The town conservation manager bought 16 maple trees for $26 each. She paid with five $100 bills. How much change will the manager receive? Explain.
$ ______

Answer:
$84

Explanation:
The town conservation manager bought 16 maple trees for $26 each = 16 x $26 = $416.
She paid with five $100 bills = 5 x $100 = $500.
The manager receive = $500 – $416 = $84.

Question 17.
Each of 25 students in Group A read for 45 minutes. Each of 21 students in Group B read for 48 minutes. Which group read for more minutes? Explain.
_________

Answer:
Group A read for more minutes than Group B.

Explanation:
Group A read for 25 x 45 = 1125 minutes.
Group B read for 21 x 48 = 1008 minutes.
Group A read for more minutes than Group B.

Page No. 180

Question 18.
Martin collects stamps. He counted 48 pages in his collector’s album. The first 20 pages each have 35 stamps in 5 rows. The rest of the pages each have 54 stamps. How many stamps does Martin have in his album?
Go Math Grade 4 Answer Key Chapter 3 Multiply 2-Digit Numbers img 27
a. What do you need to know?
Type below:
_________

Answer:
The total stamps in the first 20 pages + The total stamps in the remaining pages.

Question 18.
b. How will you use multiplication to find the number of stamps?
Type below:
_________

Answer:
The first 20 pages each have 35 stamps in 5 rows.
So, 35 x 5 = 175 stamps.

Question 18.
c. Tell why you might use addition and subtraction to help solve the problem.
Type below:
_________

Answer:
As mentioned that the number of stamps available in the first 20 pages and the number of stamps available in the rest of the pages. We need to add all pages to get 48 pages stamps.

Question 18.
d. Show the steps to solve the problem.
Type below:
_________

Answer:
Martin has 48 pages in his collector’s album.
The first 20 pages each have 35 stamps in 5 rows.
So, 35 x 5 = 175 stamps.
The first 20 pages have 175 stamps.
The rest of the pages each have 54 stamps.
So, total stamps = 175 + 54 = 229 stamps.

Question 18.
e. Complete the sentences.
Martin has a total of _____ stamps on the first 20 pages.
There are _____ more pages after the first 20 pages in Martin’s album.
There are _____ stamps on the rest of the pages.
There are _____ stamps in the album.
Type below:
_________

Answer:
Martin has a total of __175___ stamps on the first 20 pages.
There are __24___ more pages after the first 20 pages in Martin’s album.
There are __54___ stamps on the rest of the pages.
There are ___229__ stamps in the album.

Question 19.
Select the expressions that have the same product as 35 × 17. Mark all that apply.
Options:
a. (30 × 10) + (30 × 7) + (5 × 10) + (5 × 7)
b. (30 × 17) + (5 × 17)
c. (35 × 30) + (35 × 5) + (35 × 10) + (35 × 7)
d. (35 × 10) + (35 × 7)
e. (35 × 10) + (30 × 10) + (5 × 10) + (5 × 7)
f. (35 × 30) + (35 × 5)

Answer:
a. (30 × 10) + (30 × 7) + (5 × 10) + (5 × 7)
b. (30 × 17) + (5 × 17)
d. (35 × 10) + (35 × 7)

Explanation:
35 × 17
30 x 10 =300
30 x 7 = 210
5 x 10 = 50
5 x 7 = 35
300 + 210 + 50 + 35 = 595.
a. (30 × 10) + (30 × 7) + (5 × 10) + (5 × 7) = 300 + 210 + 50 + 35 = 595 equal to 595.
b. (30 × 17) + (5 × 17) = 510 + 85 = 595 equal to 595.
c. (35 × 30) + (35 × 5) + (35 × 10) + (35 × 7) = 1050 + 175 + 350 + 245 = 1820 not equal to 595.
d. (35 × 10) + (35 × 7) = 350 + 245 = 595 equal to 595
e. (35 × 10) + (30 × 10) + (5 × 10) + (5 × 7) = 350 + 300 + 50 + 35 = 735 not equal to 595.
f. (35 × 30) + (35 × 5) = 1050 + 175 = 1,225 not equal to 595.

Common Core – Page No. 181

Choose a Multiplication Method

Estimate. Then choose a method to find the product.

Question 1.
Estimate: 1,200
3 1
× 4 3
——-
9 3
+ 1, 2 4 0
————
1, 3 3 3

Answer:
Estimate: 1,200
Product: 1, 3 3 3

Explanation:
Estimate: 1,200
3 1
× 4 3
——-
9 3
+ 1, 2 4 0
————
1, 3 3 3

Question 2.
6 7
× 8 5
——-
Estimate: _____
Product: ______

Answer:
Estimate: 6,300
Product: 5,695

Explanation:
Estimate: 67 is close to 70; 85 is close to 90.
70 x 90 = 6,300.
Product: 67 x 85
80 x 6 tens = 480 tens
80 x 7 ones = 560 ones
5 x 6 tens = 30 tens
5 x 7 ones = 35 ones.
Add partial products.
4800 + 560 + 300 + 35 = 5,695.

Question 3.
6 8
× 3 8
——-
Estimate: _____
Product: ______

Answer:
Estimate: 2,800
Product: 2,584

Explanation:
Estimate: 68 is close to 70; 38 is close to 40.
70 x 40 = 2,800.
Product: 68 x 38
30 x 6 tens = 180 tens
30 x 8 ones = 240 ones
8 x 6 tens = 48 tens
8 x 8 ones = 64 ones.
Add partial products.
1800 + 240 + 480 + 64 = 2,584.

Question 4.
9 5
× 1 7
——-
Estimate: _____
Product: ______

Answer:
Estimate: 1,700
Product: 1,615

Explanation:
Estimate: 95 is close to 100.
100 x 17 = 1,700.
Product: 95 x 17
10 x 9 tens = 90 tens
10 x 5 ones = 50 ones
7 x 9 tens = 63 tens
7 x 5 ones = 35 ones.
Add partial products.
900 + 50 + 630 + 35 = 1,615.

Question 5.
4 9
× 5 4
——-
Estimate: _____
Product: ______

Answer:
Estimate: 2,500
Product: 2,646

Explanation:
Estimate: 49 is close to 50; 54 is close to 50.
50 x 50 = 2,500.
Product: 49 x 54
50 x 4 tens = 200 tens
50 x 9 ones = 450 ones
4 x 4 tens = 16 tens
4 x 9 ones = 36 ones.
Add partial products.
2000 + 450 + 160 + 36 = 2,646.

Question 6.
9 1
× 2 6
——-
Estimate: _____
Product: ______

Answer:
Estimate: 2,700
Product: 2,366

Explanation:
Estimate: 91 is close to 90; 26 is close to 30.
90 x 30 = 2,700.
Product: 49 x 54
20 x 9 tens = 180 tens
20 x 1 ones = 20 ones
6 x 9 tens = 54 tens
6 x 1 ones = 6 ones.
Add partial products.
1800 + 20 + 540 + 6 = 2,366.

Question 7.
8 2
× 1 9
——-
Estimate: _____
Product: ______

Answer:
Estimate: 1,600
Product: 1,558

Explanation:
Estimate: 82 is close to 80; 19 is close to 20.
80 x 20 = 1,600.
Product: 82 x 19
10 x 8 tens = 80 tens
10 x 2 ones = 20 ones
9 x 8 tens = 72 tens
9 x 2 ones = 18 ones.
Add partial products.
800 + 20 + 720 + 18 = 1,558.

Question 8.
4 6
× 2 7
——-
Estimate: _____
Product: ______

Answer:
Estimate: 1,500
Product: 1,242

Explanation:
Estimate: 46 is close to 50; 27 is close to 30.
50 x 30 = 1,500.
Product: 46 x 27
20 x 4 tens = 80 tens
20 x 6 ones = 120 ones
7 x 4 tens = 28 tens
7 x 6 ones = 42 ones.
Add partial products.
800 + 120 + 280 + 42 = 1,242.

Question 9.
4 1
× 3 3
——-
Estimate: _____
Product: ______

Answer:
Estimate: 1,200
Product: 1,353

Explanation:
Estimate: 41 is close to 40; 33 is close to 30.
40 x 30 = 1,200.
Product: 41 x 33
30 x 4 tens = 120 tens
30 x 1 ones = 30 ones
3 x 4 tens = 12 tens
3 x 1 ones = 3 ones.
Add partial products.
1200 + 30 + 120 + 3 = 1,353.

Question 10.
9 7
× 1 3
——-
Estimate: _____
Product: ______

Answer:
Estimate: 1,300
Product: 1,261

Explanation:
Estimate: 97 is close to 100.
100 x 13 = 1,300.
Product: 97 x 13
10 x 9 tens = 90 tens
10 x 7 ones = 70 ones
3 x 9 tens = 27 tens
3 x 7 ones = 21 ones.
Add partial products.
900 + 70 + 270 + 21 = 1,261.

Question 11.
7 5
× 6 9
——-
Estimate: _____
Product: ______

Answer:
Estimate: 5,600
Product: 5,195

Explanation:
Estimate: 75 is close to 80; 69 is close to 70.
80 x 70 = 5,600.
Product: 75 x 69
60 x 7 tens = 420 tens
60 x 5 ones = 300 ones
9 x 7 tens = 63 tens
9 x 5 ones = 45 ones.
Add partial products.
4200 + 300 + 630 + 45 = 5,195.

Problem Solving

Question 12.
A movie theatre has 26 rows of seats. There are 18 seats in each row. How many seats are there in all?
______ seats

Answer:
468 seats

Explanation:
26 x 18 = 468 seats.
20 x 18 = 360
6 x 18 = 108
108+360 = 468.

Question 13.
Each class at Briarwood Elementary collected at least 54 cans of food during the food drive. If there are 29 classes in the school, what was the least number of
cans collected?
______ cans

Answer:
1,566 cans

Explanation:
Each class at Briarwood Elementary collected at least 54 cans of food.
If there are 29 classes in the school,
the least number of cans collected = 54 x 29 = 1,566 cans.

Common Core – Page No. 182

Lesson Check

Question 1.
A choir needs new robes for each of its 46 singers. Each robe costs $32. What will be the total cost for all 46 robes?
Options:
a. $1,472
b. $1,372
c. $1,362
d. $230

Answer:
a. $1,472

Explanation:
46 x $32
40 x $32 = $1,280
6 x $32 = $192
$1,280 + $192 = $1,472

Question 2.
A wall on the side of a building is made up of 52 rows of bricks with 44 bricks in each row. How many bricks make up the wall?
Options:
a. 3,080
b. 2,288
c. 488
d. 416

Answer:
b. 2,288

Explanation:
52 x 44
50 x 44 = 2,200
2 x 44 = 88
2,200 + 88 = 2,288.
2,288 bricks make up the wall.

Spiral Review

Question 3.
Which expression shows how to multiply 4 × 362 by using place value and expanded form?
Options:
a. (4 × 3) + (4 × 6) + (4 × 2)
b. (4 × 300) + (4 × 600) +(4 × 200)
c. (4 × 300) + (4 × 60) + (4 × 20)
d. (4 × 300) + (4 × 60) + (4 × 2)

Answer:
d. (4 × 300) + (4 × 60) + (4 × 2)

Explanation:
4 × 362 = 1,448
a. (4 × 3) + (4 × 6) + (4 × 2) = 12 + 24 + 8 = 44 not equal to 1,448.
b. (4 × 300) + (4 × 600) +(4 × 200) = 1200 + 2400 + 800 = 4,400 not equal to 1,448.
c. (4 × 300) + (4 × 60) + (4 × 20) = 1200 + 240 + 80 = 1,520 not equal to 1,448.
d. (4 × 300) + (4 × 60) + (4 × 2) = 1200 + 240 + 8 = 1,448 equal to 1,448.

Question 4.
Use the model below. What is the product 4 x 492?
Go Math Grade 4 Answer Key Chapter 3 Multiply 2-Digit Numbers Common Core img 28
Options:
a. 16 + 36 + 8 = 60
b. 160 + 36 + 8 = 204
c. 160 + 360 + 8 = 528
d. 1,600 + 360 + 8 = 1,968

Answer:
d. 1,600 + 360 + 8 = 1,968

Explanation:
Grade 4 Chapter 3 Common Core 182
1,600 + 360 + 8 = 1,968

Question 5.
What is the sum 13,094 + 259,728?
Options:
a. 272,832
b. 272,822
c. 262,722
d. 262,712

Answer:
c. 262,722

Explanation:
13,094 + 259,728 = 262,722

Question 6.
During the 2008–2009 season, there were 801,372 people who attended the home hockey games in Philadelphia. There were 609,907 people who attended the home hockey games in Phoenix. How much greater was the home attendance in Philadelphia than in Phoenix that season?
Options:
a. 101,475
b. 191,465
c. 201,465
d. 202,465

Answer:
b. 191,465

Explanation:
801,372 – 609,907 = 191,465
Philadelphia attendance is 191,465 greater than in Phoenix that season.

Page No. 185

Question 1.
An average of 74 reports with bird counts were turned in each day in June. An average of 89 were turned in each day in July. How many reports were turned in for both months? (Hint: There are 30 days in June and 31 days in July.)
First, write the problem for June.
Type below:
__________

Answer:
Given that An average of 74 reports with bird counts was turned in each day in June.
For June Month, there are 30 days = 30 x 74 = 2,220.

Question 1.
Next, write the problem for July.
Type below:
__________

Answer:
An average of 89 reports with bird counts was turned in each day in July.
For July Month, there are 31 days = 31 x 89 = 2,759.

Question 1.
Last, find and add the two products.
____________ reports were turned in for both months.
Type below:
__________

Answer:
Given that An average of 74 reports with bird counts was turned in each day in June.
For June Month, there are 30 days = 30 x 74 = 2,220.
An average of 89 reports with bird counts was turned in each day in July.
For July Month, there are 31 days = 31 x 89 = 2,759.
Add two products to get the total number of reports that were turned in for both months.
2,220 + 2,759 = 4,979.

Question 2.
What if an average of 98 reports were turned in each day for the month of June? How many reports were turned in for June? Describe how your answer for June would be different.
______ reports

Answer:
720 more reports

Explanation:
Given that an average of 98 reports was turned in each day for the month of June.
June has 30 days.
Total number of reports were turned in for June = 30 x 98 = 2, 940.
From the above answer, 98 − 74 = 24. So, there would be 30 × 24, or 720 more reports.

Question 3.
There are 48 crayons in a box. There are 12 boxes in a carton. Mr. Johnson ordered 6 cartons of crayons for the school. How many crayons did he get?
______ crayons

Answer:
3,456 crayons

Explanation:
There are 48 crayons in a box.
There are 12 boxes in a carton.
So, 1 carton = 48 x 12 = 576 crayons.
If Mr. Johnson ordered 6 cartons of crayons for the school, 6 x 576 crayons = 3,456 crayons.
He gets 3,456 crayons.

Question 4.
Make Sense of Problems Each of 5 birdwatchers reported seeing 15 roseate spoonbills in a day. If they each reported seeing the same number of roseate spoonbills over 14 days, how many would be reported?
______ roseate spoonbills

Answer:
1,050 roseate spoonbills

Explanation:
Given that, 1 day –>5 birdwatchers reported 15 roseate spoonbills = 5 x 15 = 75 roseate spoonbills.
So, in 14 days –> 5 birdwatchers reported 75 x 14 = 1,050 roseate spoonbills.

Page No. 186

Question 5.
On each of Maggie’s bird-watching trips, she has seen at least 24 birds. If she has taken 4 of these trips each year over the past 16 years, at least how many birds has Maggie seen?
at least ______ birds

Answer:
Maggie seen 1,536 birds

Explanation:
Given that, 1 trip –> Maggie seen 24 birds.
For 1 year she goes for 4 bird-watching trips.
So, she has seen 4 x 24 = 96 birds for 1 year.
For 16 years, 16 x 96 = 1,536 birds have Maggie seen.

Question 6.
Make Sense of Problems There are 12 inches in a foot. In September, Mrs. Harris orders 32 feet of ribbon for the Crafts Club. In January, she orders 9 feet less. How many inches of ribbon does Mrs. Harris order? Explain how you found your answer.
______ inches

Answer:
660 inches

Explanation:
There are 12 inches in a foot.
In September, Mrs. Harris orders 32 feet of ribbon for the Crafts Club = 32 x 12 = 384.
In January, she orders 9 feet less = 32 – 9 = 23.
So, in January, she orders 23 x 12 = 276.
Mrs. Harris order 276 + 384 = 660 inches of ribbon in total.
(or)
9 less than 32 is 23, so I added 23 + 32 = 55.
Then, I multiplied 55 × 12 = 660.

Question 7.
Lydia is having a party on Saturday. She decides to write a riddle on her invitations to describe her house number on Cypress Street. Use the clues to find Lydia’s address.
Go Math Grade 4 Answer Key Chapter 3 Multiply 2-Digit Numbers img 29
______ Cypress Street

Answer:
14827 Cypress Street

Explanation:
Given that tens digit is 5 less than 7 = 7 – 5 = 2. 2 is the tens digit.
The thousands digit is twice the digit in the tens place = 2 x 2 = 4.
The hundreds digit is the greatest even number that is less than 10 i.e, 8.
The ones digit is the product of 7 and 1 = 7 x 1 = 7.
The ten thousands digit is the difference between the hundreds digit and the ones digit. So, 8 – 7 = 1.
Add the products to get the final answer = 14827.
Lydia’s address ( house number ) is 14827 Cypress Street.

Question 8.
A school is adding 4 rows of seats to the auditorium. There are 7 seats in each row. Each new seat costs $99. What is the total cost for the new seats? Show your work.
$ ______

Answer:
$2,772

Explanation:
Given that A school is adding 4 rows of seats to the auditorium. There are 7 seats in each row.
So, 7 x 4 = 28 seats are available in an auditorium.
Each new seat costs $99.
28 x $99 = $2,772 for total cost of the new seats.

Common Core – Page No. 187

Problem Solving Multiply 2 – Digit numbers

Solve each problem. Use a bar model to help.

Question 1.
Mason counted an average of 18 birds at his bird feeder each day for 20 days. Gloria counted an average of 21 birds at her bird feeder each day for 16 days. How many more birds did Mason count at his feeder than Gloria counted at hers?
Go Math Grade 4 Answer Key Chapter 3 Multiply 2-Digit Numbers Common Core img 30
Birds counted by Mason: 18 × 20 = 360
Birds counted by Gloria: 21 × 16 = 336
Draw a bar model to compare.
Subtract. 360 – 336 = 24
So, Mason counted 24 more birds.

Answer:
Birds counted by Mason: 18 × 20 = 360
Birds counted by Gloria: 21 × 16 = 336
Draw a bar model to compare.
Go Math Grade 4 Answer Key Chapter 3 Multiply 2-Digit Numbers Common Core img 30
Subtract. 360 – 336 = 24
So, Mason counted 24 more birds.

Question 2.
The 24 students in Ms. Lee’s class each collected an average of 18 cans for recycling. The 21 students in Mr. Galvez’s class each collected an average of 25 cans for recycling. How many more cans were collected by Mr. Galvez’s class than Ms. Lee’s class?
______ more cans

Answer:
The number of cans collected by Ms. Lee’s class = 18 x 24 = 432.
The number of cans collected by Mr. Galvez’s class = 25 x 21 = 525.
Use Bar Model
Grade 4 Chapter 3 Common Core 187
Subtract. 525 – 432 = 93 more cans.
So, Mr. Galvez’s class collected 93 more cans than Ms. Lee’s class.

Question 3.
At East School, each of the 45 classrooms has an average of 22 students. At West School, each of the 42 classrooms has an average of 23 students. How many more students are at East School than at West School?
______ more students

Answer:
Students in East school = 45 x 22 = 990.
Students in West School = 42 x 23 = 966.
Use Bar Model
Grade 4 Chapter 3 Common Core image 2 187
Subtract. 990 – 966 = 24.
So, East School has 24 students more than West School.

Question 4.
A zoo gift shop orders 18 boxes of 75 key rings each and 15 boxes of 80 refrigerator magnets each. How many more key rings than refrigerator magnets does the gift shop order?
______ more key rings

Answer:
Number of Key Rings = 75 x 18 = 1,350.
Number of Refrigerator Magnets= 80 x 15 = 1,200.
Use Bar Model
Grade 4 Chapter 3 Common Core image 3 187
Subtract. 1,350 – 1,200 = 150.
So, key rings are 150 more than refrigerator magnets.

Common Core – Page No. 188

Lesson Check

Question 1.
Ace Manufacturing ordered 17 boxes with 85 ball bearings each. They also ordered 15 boxes with 90 springs each. How many more ball bearings than springs did they order?
Options:
a. 5
b. 85
c. 90
d. 95

Answer:
d. 95

Explanation:
Number of ball bearings = 85 x 17 = 1,445.
Number of springs = 90 x 15 = 1,350.
Use Bar Model
Grade 4 Chapter 3 Common Core image 1 188
Subtract. 1,445 – 1,350 = 95.
So, ball bearings are 95 more than springs.

Question 2.
Elton hiked 16 miles each day on a 12-day hiking trip. Lola hiked 14 miles each day on her 16-day hiking trip. In all, how many more miles did Lola hike than Elton hiked?
Options:
a. 2 miles
b. 18 miles
c. 32 miles
d. 118 miles

Answer:
c. 32 miles

Explanation:
Hiking trip by Elton = 12 x 16 = 192.
Hiking trip by Lola = 16 x 14 = 224.
Use Bar Model
Grade 4 Chapter 3 Common Core image 2 188
Subtract. 224 – 192 = 32.
So, the Hiking trip by Lola is 32 times more than the Hiking trip by Elton.

Spiral Review

Question 3.
An orchard has 24 rows of apple trees. There are 35 apple trees in each row. How many apple trees are in the orchard?
Options:
a. 59
b. 192
c. 740
d. 840

Answer:
d. 840

Explanation:
An orchard has 24 rows of apple trees. There are 35 apple trees in each row.
24 x 35 = 840 apple trees are in the orchard.

Question 4.
An amusement park reported 354,605 visitors last summer. What is this number rounded to the nearest thousand?
Options:
a. 354,600
b. 355,000
c. 360,000
d. 400,000

Answer:
b. 355,000

Explanation:
An amusement park reported 354,605 visitors last summer. 4,605 is close to 5,000. So, the answer is 355,000.

Question 5.
Attendance at the football game was 102,653. What is the value of the digit 6?
Options:
a. 6
b. 60
c. 600
d. 6,000

Answer:
c. 600

Explanation:
Digit 6 is at hundreds of positions. So, the answer is 6 x 100 = 600.

Question 6.
Jill’s fish weighs 8 times as much as her parakeet. Together, the pets weigh 63 ounces. How much does the fish weigh?
Options:
a. 7 ounces
b. 49 ounces
c. 55 ounces
d. 56 ounces

Answer:
d. 56 ounces

Explanation:
Let Jill’s parakeet = X.
Jill’s fish weighs 8 times as much as her parakeet = 8X.
Together, the pets weigh 63 ounces.
X + 8X = 63.
9X = 63.
X = 63/9 = 7.
So, Jill’s parakeet =7.
Jill’s fish = 7 x 8 = 56 ounces.

Review/Test – Page No. 189

Question 1.
Explain how to find 40 × 50 using mental math
Type below:
_________

Answer:
200

Explanation:
40 x 50
By using mental math
4 x 5 = 20
40 x 50 = 200

Mrs. Traynor’s class is taking a field trip to the zoo. The trip will cost $26 for each student. There are 22 students in her class.

Question 2.
Part A
Round each factor to estimate the total cost of the students’ field trip.
$ ______

Answer:
$600

Explanation:
Total cost of the students’ field trip = 22 x $26.
22 x $26
20 x $30 = $600
The total cost would be about $600.

Question 2.
Part B
Use compatible numbers to estimate the total cost of the field trip.
$ ______

Answer:
$500

Explanation:
If we use compatible numbers to estimate the total cost of the field trip.
22 x $26
20 × 25 = 500
The total cost would be about $500.

Question 2.
Part C
Which do you think is the better estimate? Explain.
Better estimate: _________

Answer:
Using rounded numbers is a better estimate. When rounded numbers are used, one estimated factor was $4 more than the actual factor and the other estimated factor was $2 that is less than the actual factor. So, the estimate should be close to the actual one. When compatible numbers are used both estimated factors were less than the actual factors. So, the product will be an underestimate.

Review/Test – Page No. 190

For numbers 3a–3e, select Yes or No to show if the answer is correct.

Question 3.
3a. 35 × 10 = 350
i. yes
ii. no

Answer:
i. yes

Explanation:
35 x 10 = 350
30 x 10 = 300.
5 x 10 = 50.
300 + 50 = 350.

Question 3.
3b. 19 × 20 = 380
i. yes
ii. no

Answer:
i. yes

Explanation:
19 × 20 = 380
19 x 20 = 19 x 2 tens.
19 x 20 = 38 tens = 380.

Question 3.
3c. 12 × 100 = 120
i. yes
ii. no

Answer:
ii. no

Explanation:
12 x 100 = 120.
10 x 100 = 1000
2 x 100 = 200.
1000 + 200 = 1200.

Question 3.
3d. 70 × 100 = 7,000
i. yes
ii. no

Answer:
i. yes

Explanation:
70 x 100 = 7,000
100 x 7 tens = 700 tens = 7,000

Question 3.
3e. 28 × 30 = 2,100
i. yes
ii. no

Answer:
ii. no

Explanation:
28 × 30
20 x 30 = 600
8 x 30 = 240
600 + 240 = 840

Question 4.
There are 23 boxes of pencils in Mr. Shaw’s supply cabinet. Each box contains 100 pencils. How many pencils are in the supply cabinet?
_____ penciles

Answer:
2,300 pencils

Explanation:
23 x 100 = 2,300 pencils are in the supply cabinet.

Question 5.
Which would provide a reasonable estimate for each product? Write the estimate beside the product. An estimate may be used more than once
23 × 38 __________
31 × 32 __________
46 × 18 __________
39 × 21 __________

Answer:
23 × 38 –> 25 x 40
31 x 32 –> 30 × 30
46 × 18 –> 50 × 20
39 × 21 –> 25 × 40

Explanation:
23 × 38; 23 is close to 25; 38 is close to 40.
So, the estimated product is 25 x 40
31 x 32; 31 is close to 30; 32is close to 30.
So, the estimated product is 30 × 30
46 × 18; 46 is close to 50; 18 is close to 20.
So, the estimated product is 50 × 20
39 × 21; 39 is close to 40; 21 is close to 25.
So, the estimated product is 25 × 40

Question 6.
There are 26 baseball teams in the league. Each team has 18 players. Write a number sentence that will provide a reasonable estimate for the number of players in the league. Explain how you found your estimate.
Type below:
__________

Answer:
There are 26 baseball teams in the league. Each team has 18 players.
26 x 18
25 x 20
We Rounded each factor to its close factor, then simplified the multiplication.

Question 7.
The model shows 48 × 37. Write the partial products.
Go Math Grade 4 Answer Key Chapter 3 Multiply 2-Digit Numbers Review/Test img 31
Type below:
__________

Answer:
Grade 4 Chapter 3 Common Core image 1 190
Partial Products are 1200, 240, 280, 56

Review/Test – Page No. 191

Question 8.
Jess made this model to find the product 32 × 17. Her modelis incorrect.
Go Math Grade 4 Answer Key Chapter 3 Multiply 2-Digit Numbers Review/Test img 32
Part A
What did Jess do wrong?
Type below:
__________

Answer:
Jess added the numbers in the model instead of multiplying.

Question 8.
Part B
Redraw the model so that it is correct.
Go Math Grade 4 Answer Key Chapter 3 Multiply 2-Digit Numbers Review/Test img 33
Type below:
__________

Answer:
Grade 4 Chapter 3 Common Core image 2 190

Question 8.
Part C
What is the actual product 32 × 17?
______

Answer:
544

Explanation:
32 × 17
10 x 32 = 320
7 x 32 = 224
320 + 224 = 544.

Question 9.
Tatum wants to use partial products to find 15 × 32. Write the numbers in the boxes to show 15 × 32.
Go Math Grade 4 Answer Key Chapter 3 Multiply 2-Digit Numbers Review/Test img 34
Type below:
__________

Answer:
Grade 4 Chapter 3 Common Core image 4 190

Review/Test – Page No. 192

Question 10.
Which product is shown by the model? Write the letter of the product on the line below the model.
Go Math Grade 4 Answer Key Chapter 3 Multiply 2-Digit Numbers Review/Test img 35
Type below:
__________

Answer:
Go Math Grade 4 Answer Key Chapter 3 Multiply 2-Digit Numbers Review/Test img 35
C                                              A                                                  B
10 + 3 = 13
10 + 3 = 13
13 x 13
2. 10 + 7 = 17
30 + 6 = 36
17 x 36
3. 20 + 4 = 24
10 + 4 = 14
24 x 14

Question 11.
Mrs. Jones places 3 orders for school T-shirts. Each order has 16 boxes of shirts and each box holds 17 shirts. How many T-shirts does Mrs. Jones order? Use partial products to help you.
Type below:
__________

Answer:
816 T-shirts

Explanation:
Mrs. Jones places 3 orders for school T-shirts. Each order has 16 boxes of shirts and each box holds 17 shirts.
Each box has 17 shirts.
16 boxes = 16 x 17 = 272.
Each order = 16 boxes = 272 shirts.
3 orders = 3 x 272 = 816 shirts.
Mrs. Jones order 816 T-shirts.

Question 12.
Write the unknown digits. Use each digit exactly once.
Go Math Grade 4 Answer Key Chapter 3 Multiply 2-Digit Numbers Review/Test img 36
Type below:
__________

Answer:
Grade 4 Chapter 3 Common Core image 1 192
90 x 40 = 3,600
90 x 6 = 540
3 x 40 = 120
3 x 6 = 18.
3,600 + 540 + 120 + 8 = 4,278.

Question 13.
Mike has 16 baseball cards. Niko has 17 times as many baseball cards as Mike does. How many baseball cards does Niko have?
________ baseball cards

Answer:
272 baseball cards

Explanation:
Mike has 16 baseball cards. Niko has 17 times as many baseball cards as Mike does.
Niko have 16 x 17 = 272 baseball cards.

Question 14.
Multiply.
36 × 28 = ________

Answer:
1,008

Explanation:
36 x 28
20 x 30 = 600
20 x 6 = 120
8 x 30 = 240
8 x 6 = 48
600 + 120 + 240 + 48 = 1,008

Review/Test – Page No. 193

Question 15.
A farmer planted 42 rows of tomatoes with 13 plants in each row. How many tomato plants did the farmer grow?
42 × 13 = ______ tomato plants

Answer:
420 + 126 = 546 tomato plants

Explanation:
42 × 13
10 x 42 = 420
3 x 42 = 126
420 + 126 = 546 tomato plants

Question 16.
Select another way to show 25 × 18. Mark all that apply.
Options:
a. (20 × 10) + (20 × 8) + (5 × 10) + (5 × 8)
b. (25 × 20) + (25 × 5) + (25 × 10) + (25 × 8)
c. (20 × 18) + (5 × 10) + (5 × 8)
d. (25 × 10) + (25 × 8)
e. (25 × 20) + (25 × 5)

Answer:
a. (20 × 10) + (20 × 8) + (5 × 10) + (5 × 8)
c. (20 × 18) + (5 × 10) + (5 × 8)
d. (25 × 10) + (25 × 8)

Explanation:
25 × 18
10 x 25 = 250
8 x 25 = 200
250 + 200 = 450.
a. (20 × 10) + (20 × 8) + (5 × 10) + (5 × 8) = 200 + 160 + 50 + 40 = 450
b. (25 × 20) + (25 × 5) + (25 × 10) + (25 × 8) = 500 + 125 + 250 + 200 = 1,075
c. (20 × 18) + (5 × 10) + (5 × 8) = 360 + 50 + 40 = 450
d. (25 × 10) + (25 × 8) = 250 + 200 = 450
e. (25 × 20) + (25 × 5) = 500 + 125 = 625

Question 17.
Terrell runs 15 sprints. Each sprint is 65 meters. How many meters does Terrell run? Show your work.
______ meters

Answer:
975 meters

Explanation:
Terrell run 15 x 65 = 975 meters.

Question 18.
There are 3 new seats in each row in a school auditorium. There are 15 rows in the auditorium. Each new seat cost $74. What is the cost for the new seats? Explain how you found your answer.
$ ______

Answer:
$3,330

Explanation:
Given that There are 3 new seats in each row in a school auditorium. There are 15 rows in the auditorium. Each new seat cost $74.
So, 3 x 15 = 45 seats are available in an auditorium.
Each new seat costs $74.
45 x $74 = $3,330 for total cost of the new seats.

Question 19.
Ray and Ella helped move their school library to a new building. Ray packed 27 boxes with 25 books in each box. Ella packed 23 boxes with 30 books in each box. How many more books did Ella pack? Show your work.
______ books

Answer:
15 books

Explanation:
Ray packed 27 x 25 = 675 books.
Ella packed 23 x 30 = 690 books
Ella packed 690 – 675 = 15 books more than Ray.

Review/Test – Page No. 194

Question 20.
Julius and Walt are finding the product of 25 and 16.
Go Math Grade 4 Answer Key Chapter 3 Multiply 2-Digit Numbers Review/Test img 37
Part A
Julius’ answer is incorrect. What did Julius do wrong?
Type below:
__________

Answer:
Julius multiplied 25 by 10 and then multiplied 25 by 6 correctly. He added the two partial products incorrectly.

Question 20.
Part B
What did Walt do wrong?
Type below:
__________

Answer:
Walt multiplied 6 by 5 and got 300 instead of 30

Question 20.
Part C
What is the correct product?
Type below:
__________

Answer:
25 x 16 = 400

Question 21.
A clothing store sells 26 shirts and 22 pairs of jeans. Each item of clothing costs $32.
Part A
What is a reasonable estimate for the total cost of the clothing?
Show or explain how you found your answer.
$ ______

Answer:
$1500

Explanation:
A clothing store sells 26 shirts and 22 pairs of jeans. 26 + 22 = 48 clothes.
Each item of clothing costs $32.
48 x $32
50 x $30 = $1500

Question 21.
Part B
What is the exact answer for the total cost of the clothing? Show or explain how you found your answer.
$ ______

Answer:
$1,536

Explanation:
48 x $32
40 x $32 = $1,280
8 x $32 = $256
$1,280 + $256 = $1,536

Page No. 199

Question 1.
A restaurant has 68 chairs. There are six chairs at each table. About how many tables are in the restaurant?
Estimate. 68 ÷ 6
Think: What number times 6 is about 68?
10 × 6 = ___
11 × 6 = ___
12 × 6 = ___
68 is closest to ______, so the best estimate is about _______ tables are in the restaurant.
Type below:
__________

Answer:
68 is close to 70 and 6 is close to 5.
So, 70/5 = 12.
10 × 6 = __60_
11 × 6 = _66__
12 × 6 = _72__
68 is closest to ___66___, so the best estimate is about 11 x 6 = 66 tables are in the restaurant.

Find two numbers the quotient is between. Then estimate the quotient.

Question 2.
41 ÷ 3
between _______ and _______

Answer:
between 13 and 14
about 14

Explanation:
13 x 3 = 39; 14 x 3 = 42.
The quotient of 41 ÷ 3 is between 13 and 14.

Question 3.
192 ÷ 5
between _______ and _______

Answer:
between 30 and 40
about 40

Explanation:
30 x 5 = 150; 40 x 5 = 200.
The quotient of 192 ÷ 5 is between 30 and 40.

Find two numbers the quotient is between. Then estimate the quotient.

Question 4.
90 ÷ 7
between _______ and _______

Answer:
between 12 and 13
about 13

Explanation:
12 x 7 = 84; 13 x 7 = 91.
The quotient of 90 ÷ 7 is between 12 and 13.

Question 5.
67 ÷ 4
between _______ and _______

Answer:
between 16 and 17
about 17

Explanation:
16 x 4 = 64; 17 x 4 = 68.
The quotient of 67 ÷ 4 is between 16 and 17.

Question 6.
281 ÷ 9
between _______ and _______

Answer:
between 30 and 40
about 30

Explanation:
30 x 9 = 270; 40 x 9 = 360.
The quotient of 281 ÷ 9 is between 30 and 40.

Question 7.
102 ÷ 7
between _______ and _______

Answer:
between 14 and 15
about 15

Explanation:
14 x 7 = 98; 15 x 7 = 105.
The quotient of 102 ÷ 7 is between 14 and 15.

Question 8.
85 ÷ 6
between _______ and _______

Answer:
between 14 and 15
about 14

Explanation:
14 x 6 = 84; 15 x 6 = 90.
The quotient of 85 ÷ 6 is between 14 and 15.

Question 9.
220 ÷ 8
between _______ and _______

Answer:
between 20 and 30
about 30

Explanation:
20 x 8 = 160; 30 x 8 = 240.
The quotient of 220 ÷ 8 is between 20 and 30.

Decide whether the actual quotient is greater than or less than the estimate given. Write < or >.

Question 10.
83 ÷ 8 _______ 10

Answer:
>

Explanation:
83 ÷ 8 = 10.375 > 10

Question 11.
155 ÷ 4 _______ 40

Answer:
<

Explanation:
155 ÷ 4 = 38.75 < 40

Question 12.
70 ÷ 6 _______ 11

Answer:
>

Explanation:
70 ÷ 6 = 11.666 > 11

Question 13.
What’s the Question? A dolphin’s heart beats 688 times in 6 minutes. Answer: about 100 times.
Type below:
__________

Answer:
About how many times does a dolphin’s heart beats in 1 minute?

Question 14.
Analyze A mother bottlenose ate about 278 pounds of food in one week. About how much food did she eat in a day?
about _____ pounds

Answer:
about 40 pounds

Explanation:
278 ÷ 7
The quotient of 278 ÷ 7 is between 39 and 40.

Question 15.
Tanya has $42 to spend at the Dolphin Island store. T-shirts sell for $7 each and a pair of sunglasses sells for $6. Tanya buys 3 T-shirts. How many pairs of sunglasses can she buy with the amount of money she has left?
_____ pairs of sunglasses

Answer:
3 pairs of sunglasses

Explanation:
Given that Tanya has $42 to spend at the Dolphin Island store. T-shirts sell for $7 each and a pair of sunglasses sell for $6.
Tanya buys 3 T-shirts = 3 x $7 = $21.
pair of sunglasses = $42 – $21 = $21.
1 pair of sunglasses sells for $6.
So, $21 ÷ $7 = 3.
3 pairs of sunglasses can Tanya buy with the amount of money she has left.

Page No. 200

Question 16.
If a bottlenose dolphin can eat 175 pounds of fish, squid, and shrimp in a week, about how many pounds of food does it eat in a day? Milo says the answer is about 20 pounds. Leah says the answer is about 30 pounds. Who is correct? Explain.
Go Math Grade 4 Answer Key Chapter 3 Multiply 2-Digit Numbers img 38
________

Answer:
The bottlenose dolphin can eat 25 pounds for 1 day.
Both answers are correct. Because the 25 pounds is in between 20 and 30 pounds.

Explanation:
1 week = 7 days.
The bottlenose dolphin can eat 175 pounds for 7 days.
For 1 day = 175 ÷ 7 = 25 pounds.
The bottlenose dolphin can eat 25 pounds for 1 day.
Both answers are correct. Because the 25 pounds is in between 20 and 30 pounds.

Question 17.
Four families went out for lunch. The total food bill came to $167. The families also left a $30 tip for the waitress. If each family spent the same amount, about how much did each family spend on dinner? Explain how you found your answer.
$ ______

Answer:
$98.5

Explanation:
Four families went out for lunch. The total food bill came to $167. The families also left a $30 tip for the waitress.
So, total amount = $167 + $30 = $197.
If each family spent the same amount = $197 ÷ 2 = $98.5
Each family spent $98.5.

Question 18.
There are 6 showings of a film about Van Gogh at the Art Museum. A total of 459 people saw the film. The same number of people were at each showing. About how many people were at each showing? Circle the numbers the quotient is between. Then explain how you found your answer.
40 50 60 70 80
Type below:
_________

Answer:
40 50 60 70 80
I found multiples of 6 that 459 is between. 70 × 6 = 420 and 80 × 6 = 480. Since 459 is closer to 480, 459 ÷ 6 is about 80.

Conclusion

Hope the data shared about Go Math Grade 4 Answer Key Chapter 3 Multiply 2- Digit Number has helped you in your preparation. If you feel any learning is missing do give us your suggestions and we will consider them if possible. Just keep on visiting our site to get the latest update on Grade 4 Go Math HMH Answer Keys for other chapters as well.

Go Math Grade 5 Answer Key Chapter 1 Place Value, Multiplication, and Expressions

go-math-grade-5-chapter-1-place-value-multiplication-and-expressions-answer-key

Help your kid be on the track and ensure him Math Proficiency with our Go Math Grade 5 Answer Key Chapter 1 Place Value, Multiplication, and Expressions. Go Math Grade 5 Chapter 1 Place Value, Multiplication, and Expressions Answer Key enhances your subject knowledge. Use the Best resource Go Math Grade 5 Chapter 1 Answer Key and ace up your preparation. Try to solve as many problems as possible from the Go Math Grade 5 Answer Key and get to know the concepts behind them.

Go Math Grade 5 Answer Key Chapter 1 Place Value, Multiplication, and Expressions

It’s really difficult to find Solutions for all the Problems in Go Math Grade 5 Chapter 1 all in one place. Now, you will no longer have such difficulties. We are providing Go Math Grade 5 Answer Key Chapter 1 Place Value, Multiplication, and Expressions. Check out Step by Step Solutions provided for various lessons and the topics in it. Practice using the 3rd Grade Go Math Answer Key Ch 1 Place Value, Multiplication, and Expressions and score better grades in the exam.

Lesson 1: Investigate • Place Value and Patterns

Lesson 2: Place Value of Whole Numbers

Lesson 3: Algebra • Properties

Lesson 4: Algebra • Powers of 10 and Exponents

Lesson 5: Algebra • Multiplication Patterns

Mid-Chapter Checkpoint

Lesson 6: Multiply by 1-Digit Numbers

Lesson 7: Multiply by Multi-Digit Numbers

Lesson 8: Relate Multiplication to Division

Lesson 9: Problem Solving • Multiplication and Division

Lesson 10: Algebra • Numerical Expressions

Lesson 11: Algebra • Evaluate Numerical Expressions

Lesson 12: Algebra • Grouping Symbols

Review/Test

Place Value and Patterns – Share and Show – Page No. 7

Complete the sentence.

Question 1.
500 is 10 times as much as ______

Answer:
50

Explanation:
Let the unknown number is S.
500 = 10S
S = 500/10 = 50.
500 is 10 times as much as 50.

Question 2.
20,000 is \(\frac{1}{10}\) of ______

Answer:
2,00,000

Explanation:
Let the unknown number is S.
20,000 = \(\frac{1}{10}\) S
S = 20,000 X 10 = 2,00,000

Question 3.
900 is \(\frac{1}{10}\) of ______

Answer:
9,000

Explanation:
Let the unknown number is S.
900 = \(\frac{1}{10}\) S
S = 900 X 10 = 9,000

Question 4.
600 is 10 times as much as ______

Answer:
60

Explanation:
Let the unknown number is S.
600 = 10S
S = 600/10 = 60.

Use place-value patterns to complete the table

Question 5.

Numbers 10 times as much as \(\frac{1}{10}\) of
10 ______ ______
3,000 ______ ______
800 ______ ______
50 ______ ______

Answer:

Numbers 10 times as much as \(\frac{1}{10}\) of
10 ___1___ ___100___
3,000 ___300___ ___30,000___
800 ___80___ ___8,000___
50 ___5___ ___500___

Explanation:
1. 10 is 10 times as much as ______
Let the unknown number is S.
10 = 10S
S = 10/10 = 1.
10 is 10 times as much as 1.
10 is \(\frac{1}{10}\) of ______
Let the unknown number is S.
10 = \(\frac{1}{10}\) S
S = 10 X 10 = 100.
2. 3,000 is 10 times as much as ______
Let the unknown number is S.
3,000 = 10S
S = 3,000/10 = 300.
3,000 is 10 times as much as 300.
3,000 is \(\frac{1}{10}\) of ______
Let the unknown number is S.
3,000 = \(\frac{1}{10}\) S
S = 3,000 X 10 = 30,000.
3. 800 is 10 times as much as ______
Let the unknown number is S.
800 = 10S
S = 800/10 = 80.
800 is 10 times as much as 80.
800 is \(\frac{1}{10}\) of ______
Let the unknown number is S.
800 = \(\frac{1}{10}\) S
S = 800 X 10 = 8,000.
4. 50 is 10 times as much as ______
Let the unknown number is S.
50 = 10S
S = 50/10 = 5.
50 is 10 times as much as 5.
50 is \(\frac{1}{10}\) of ______
Let the unknown number is S.
50 = \(\frac{1}{10}\) S
S = 50 X 10 = 500.

Question 6.

Numbers 10 times as much as \(\frac{1}{10}\) of
400 ______ ______
90 ______ ______
6,000 ______ ______
200 ______ ______

Answer:

Numbers 10 times as much as \(\frac{1}{10}\) of
400 __40____ ___4,000___
90 ___9___ ___900___
6,000 __600____ __60,000____
200 ___20___ ___2,000___

Explanation:
1. 400 is 10 times as much as ______
Let the unknown number is S.
400 = 10S
S = 400/10 = 40.
400 is 10 times as much as 40.
400 is \(\frac{1}{10}\) of ______
Let the unknown number is S.
400 = \(\frac{1}{10}\) S
S = 400 X 10 = 4,000.
2. 90 is 10 times as much as ______
Let the unknown number is S.
90 = 10S
S = 90/10 = 9.
90 is 10 times as much as 9.
90 is \(\frac{1}{10}\) of ______
Let the unknown number is S.
90 = \(\frac{1}{10}\) S
S = 90 X 10 = 900.
3. 6,000 is 10 times as much as ______
Let the unknown number is S.
6,000 = 10S
S = 6,000/10 = 600.
6,000 is 10 times as much as 600.
6,000 is \(\frac{1}{10}\) of ______
Let the unknown number is S.
6,000 = \(\frac{1}{10}\) S
S = 6,000 X 10 = 60,000.
4. 200 is 10 times as much as ______
Let the unknown number is S.
200 = 10S
S = 200/10 = 20.
200 is 10 times as much as 20.
200 is \(\frac{1}{10}\) of ______
Let the unknown number is S.
200 = \(\frac{1}{10}\) S
S = 200 X 10 = 2,000.

Complete the sentence with 100 or 1,000.

Question 13.
200 is ______ times as much as 2

Answer:
200 is 100 times as much as 2

Explanation:
Let the unknown number is S.
200 = 2S
S = 200/2 = 100

Question 14.
4,000 is ______ times as much as 4

Answer:
4,000 is 1000 times as much as 4

Explanation:
Let the unknown number is S.
4,000 = 2S
S = 4,000/2 = 1,000

Question 15.
700,000 is ______ times as much as 700

Answer:
700,000 is 1,000 times as much as 700

Explanation:
Let the unknown number is S.
700,000 = 700S
S = 700,000/700 = 1,000

Question 16.
600 is ______ times as much as 6

Answer:
600 is 100 times as much as 6

Explanation:
Let the unknown number is S.
600 = 6S
S = 600/6= 100

Question 17.
50,000 is ______ times as much as 500

Answer:
50,000 is 100_ times as much as 500

Explanation:
Let the unknown number is S.
50,000 = 500S
S = 50,000/500= 100

Question 18.
30,000 is ______ times as much as 30

Answer:
30,000 is 1,000 times as much as 30

Explanation:
Let the unknown number is S.
30,000 = 30S
S = 30,000/30 = 1,000

Question 19.
Explain how you can use place-value patterns to describe how 50 and 5,000 compare.
Type below:
__________

Answer:
5,000 is 100 times as much as 50

Explanation:
5,000/50 = 100

Place Value and Patterns – Problem Solving – Page No. 8

Sense or Nonsense?

Question 20.
Mark and Robyn used base-ten blocks to show that 300 is 100 times as much as 3. Whose model makes sense? Whose model is nonsense? Explain your reasoning.
Go Math Grade 5 Answer Key Chapter 1 Place Value, Multiplication, and Expressions Place Value, Multiplication, and Expressions; Place Value and Patterns img 1
Type below:
__________

Answer:
Robyn’s model makes sense. Because the given data 300 is 100 times as much as 3. It clearly states that there are 100 model-blocks and one model blocks should take to solve the problem.

Question 20.
Explain how you would help Mark understand why he should have used small cubes instead of longs.
Type below:
__________

Answer:
Mark’s drew 100 model-blocks and 10 model-blocks which. To get 300 is 100 times as much as 3, he needs to do 300/3 = 100 model blocks.

Place Value of Whole Numbers – Share and Show – Page No. 11

Complete the place-value chart to find the value of each digit.

Question 1.
Go Math Grade 5 Answer Key Chapter 1 Place Value, Multiplication, and Expressions Place Value, Multiplication, and Expressions; Place Value of Whole Numbers img 2
Type below:
__________

Answer:
Grade 5 Chapter 1 Image 1
7,333,820

Explanation:
7 x 1,000,000 = 7,000,000
3 x 100,000 = 300,000
3 x 1,000 = 3000
8 x 100 = 800
2 x 10 = 20

Write the value of the underlined digit.

Question 2.
1,574,833
__________

Answer:
4,000

Explanation:
(1 x 1,000,000) + (5 x 1,00,000) + (7 x 10,000) + (4 x 1,000) + (8 x 100) + (3 x 10) + (3 x 1)
4 x 1,000 = 4 thousands = 4,000

Question 3.
598,102
__________

Answer:
100

Explanation:
(5 x 1,00,000) + (9 x 10,000) + (8 x 1,000) + (1 x 100) + (0 x 10) + (2 x 1)
1 x 100 = 4 hundreds = 100

Question 4.
7,093,455
__________

Answer:
90,000

Explanation:
(7 x 1,000,000) + (0 x 1,00,000) + (9 x 10,000) + (3 x 1,000) + (4 x 100) + (5 x 10) + (5 x 1)
9 x 10,000 = 9 ten-thousands = 90,000

Question 5.
301,256,878
__________

Answer:
3,00,000,000

Explanation:
(3 x 1,00,000,000) + (0 x 10,000,000) + (1 x 1,000,000) + (2 x 1,00,000) + (5 x 10,000) + (6 x 1,000) + (8 x 100) + (7 x 10) + (8 x 1)
3 x 1,00,000,000 = 3 hundred- millions = 3,00,000,000

Write the number in two other forms.

Question 6.
(8 × 100,000) + (4 × 1,000) + (6 × 1) =
__________

Answer:
80,4006
Eight Hundred Four Thousand Six

Explanation:
(8 × 100,000) + (4 × 1,000) + (6 × 1) = 800,000 + 4,000 + 6 = 80,4006

Question 7.
seven million, twenty thousand, thirty-two
__________

Answer:
7,020,032
Seven Million Twenty Thousand Thirty-Two

Explanation:
seven million = 7,000,000
twenty thousand = 20,000
thirty-two = 32

On Your Own

Write the value of the underlined digit.

Question 8.
849,567,043
__________

Answer:
40,000,000

Explanation:
(8 x 1,00,000,000) + (4 x 10,000,000) + (9 x 1,000,000) + (5 x 1,00,000) + (6 x 10,000) + (7 x 1,000) + (0 x 100) + (4 x 10) + (3 x 1)
4 x 10,000,000 = 4 ten- millions = 40,000,000

Question 9.
9,422,850
__________

Answer:
4,00,000

Explanation:
(9 x 1,000,000) + (4 x 1,00,000) + (2 x 10,000) + (2 x 1,000) + (8 x 100) + (5 x 10) + (0 x 1)
4 x 1,00,000 = 4 Hundred Thousand = 4,00,000

Question 10.
96,283
__________

Answer:
90,000

Explanation:
(9 x 10,000) + (6 x 1,000) + (2 x 100) + (8 x 10) + (3 x 1)
9 x 10,000 = 9 ten-thousands = 90,000

Question 11.
498,354,021
__________

Answer:
4,00,000,000

Explanation:
(4 x 1,00,000,000) + (9 x 10,000,000) + (8 x 1,000,000) + (3 x 1,00,000) + (5 x 10,000) + (4 x 1,000) + (0 x 100) + (2 x 10) + (1 x 1)
4 x 1,00,000,000 = Four Hundred Million = 4,00,000,000

Question 12.
791,350
__________

Answer:
300

Explanation:
(7 x 1,00,000) + (9 x 10,000) + (1 x 1,000) + (3 x 100) + (5 x 10) + (0 x 1)
3 x 100 = 3 hundred = 300

Question 13.
27,911,534
__________

Answer:
7,000,000

Explanation:
(2 x 10,000,000) + (7 x 1,000,000) + (9 x 1,00,000) + (1 x 10,000) + (1 x 1,000) + (5 x 100) + (3 x 10) + (4 x 1)
7 x 1,000,000 = Seven Million = 7,000,000

Question 14.
105,980,774
__________

Answer:
80,000

Explanation:
(1 x 1,00,000,000) + (0 x 10,000,000) + (5 x 1,000,000) + (9 x 1,00,000) + (8 x 10,000) + (0 x 1,000) + (7 x 100) + (7 x 10) + (4 x 1)
8 x 10,000 = 8 ten-thousand = 80,000

Question 15.
8,265,178
__________

Answer:
5,000

Explanation:
(8 x 1,000,000) + (2 x 1,00,000) + (6 x 10,000) + (5 x 1,000) + (1 x 100) + (7 x 10) + (8 x 1)
5 x 1,000 = 5 one-thousand = 5,000

Write the number in two other forms.

Question 16.
345,000
Type below:
__________

Answer:
Three Hundred Forty-Five Thousand
(3 x 1,00,000) + (4 x 10,000) + (5 x 1,000) + (0 x 100) + (0 x 10) + (0 x 1)

Question 17.
119,000,003
Type below:
__________

Answer:
One Hundred Nineteen Million Three
(1 x 100,000,000) + (1 x 10,000,000) + (9 x 1,000,000) + (0 x 1,00,000) + (0 x 10,000) + (0 x 1,000) + (0 x 100) + (0 x 10) + (3 x 1)

Place Value of Whole Numbers – Problem Solving – Page No. 12

Use the table for 18–19.

Question 18.
Which planet is about 10 times as far as Earth is from the Sun?
Go Math Grade 5 Answer Key Chapter 1 Place Value, Multiplication, and Expressions Place Value, Multiplication, and Expressions; Place Value of Whole Numbers img 3
__________

Answer:
Saturn

Explanation:
Saturn = 1,427,000/10 = 142,700 which is 10 times as far as Earth

Question 19.
Which planet is about \(\frac{1}{10}\) of the distance Uranus is from the Sun?
__________

Answer:
Mars

Explanation:
Mars = 227,900
\(\frac{1}{10}\) x 2,871,000 = 287,100
Which planet is about \(\frac{1}{10}\) of the distance Uranus is from the Sun

Question 20.
What’s the Error? Matt wrote the number four million, three hundred five thousand, seven hundred sixty-two as 4,350,762. Describe and correct his error.
Type below:
__________

Answer:
Matt switched 2 digits in the thousands period: 4,305,762

Question 21.
Explain how you know that the values of the digit 5 in the numbers 150,000 and 100,500 are not the same.
Type below:
__________

Answer:
In 150,000, the digit 5 is in the ten-thousands place, So, its value is 50,000; in 100,500, the digit 5 is in the hundreds place. So, its value is 500.

Question 22.
Test Prep In the number 869,653,214, which describes how the digit 6 in the ten-millions place compares to the digit 6 in the hundred-thousands place?
Options:
A. 10 times as much as
B. 100 times as much as
C. 1,000 times as much as
D.\(\frac{1}{10}\) of

Answer:
B. 100 times as much as

Explanation:
869,653,214
(8 x 100,000,000) + (6 x 10,000,000) + (9 x 1,000,000) + (6 x 1,00,000) + (5 x 10,000) + (3 x 1,000) + (2 x 100) + (1 x 10) + (4 x 1)
6 x 10,000,000 = 60,000,000
6 x 1,00,000 = 6,00,000
60,000,000/6,00,000 = 100

Properties – Share and Show – Page No. 15

Use properties to find 4 × 23 × 25.

Question 1.
23 × × 25 ________ Property of Multiplication
23 × ( × ) ________ Property of Multiplication
23 ×
__________
____

Answer:
23 x 4 x 25; Commutative Property of Multiplication
23 x (4 x 25); Associative Property of Multiplication
23 x 100
2,300

Use properties to find the sum or product.

Question 2.
89 + 27 + 11 = ____

Answer:
89 + (27 + 11); Associative Property of Addition
89 + 38
127

Question 3.
9 × 52 = ____

Answer:
468

Explanation:
9 x 52
Write 52 = (50 + 2)
9 x (50 + 2)
(9 x 50) + (9 x 2); Distributive Property of Multiplication
450 + 18
468

Question 4.
107 + 0 + 39 + 13 = ____

Answer:
107 + 0 + 39 + 13
(107 + 0) + (39 + 13); Associative Property of Addition
107 + 0 = 107; Identity Property of Addition
107 + 52 = 159

Complete the equation, and tell which property you used.

Question 5.
9 × (30 + 7) = (9 × ____) + (9 × 7)

Answer:
9 × (30 + 7) = (9 ×30) + (9 × 7)
Distributive Property of Multiplication

Explanation:
9 x (30 + 7)
(9 x 30) + (9 x 7); Distributive Property of Multiplication
270 + 63 = 333

Question 6.
0 + ____ = 47

Answer:
47; Identity Property of Addition

Explanation:
0 + 47 = 47; Identity Property of Addition

Question 6.
Describe how you can use properties to solve problems more easily.
Type below:
__________

Answer:
Using Properties of Addition and Properties of Multiplication, we can solve problems more easily. Simplifying problems is easy with the properties.

On Your Own

Practice: Copy and Solve Use properties to find the sum or product.

Question 7.
3 × 78 = ____

Answer:
234, Associative Property of Multiplication

Explanation:
Write 78 as 6 x 13
3 x 6 x 13
(3 x 6) x 13; Associative Property of Multiplication
18 x 13 = 234

Question 8.
4 × 60 × 5 = ____

Answer:
1,200; Associative Property of Multiplication

Explanation:
4 x 60 x 5
4 x (60 x 5); Associative Property of Multiplication
4 x 300 = 1,200

Question 9.
21 + 25 + 39 + 5 = ____

Answer:
90; Associative Property of Addition

Explanation:
(21 + 25) + (39 + 5); Associative Property of Addition
46 + 44 = 90

Complete the equation, and tell which property you used.

Question 10.
11 + (19 + 6) = (11 + ____) + 6

Answer:
11 + (19 + 6) = (11 + 19) + 6; Associative Property of Addition

Question 11.
25 + 14 = ____ + 25

Answer:
25 + 14 = 14 + 25; Commutative Property of Addition

Question 12.
Show how you can use the Distributive Property to rewrite and find (32 × 6) + (32 × 4).
____

Answer:
(32 × 6) + (32 × 4) = 32 x (6 + 4); Distributive Property

Properties – Problem Solving – Page No. 16

Question 13.
Three friends’ meals at a restaurant cost $13, $14, and $11. Use parentheses to write two different expressions to show how much the friends spent in all. Which property does your pair of expressions demonstrate?
$ ____

Answer:
$38; Associative Law of Addition

Explanation:
Three friends’ meals at a restaurant cost $13, $14, and $11.
Friends spent in all = $13 + $14 + $11
$13 + ($14 + $11) = ($13 + $14) + $11 Associative Law of Addition

Question 14.
Jacob is designing an aquarium for a doctor’s office. He plans to buy 6 red blond guppies, 1 blue neon guppy, and 1 yellow guppy. The table shows the price list for the guppies. How much will the guppies for the aquarium cost?
Go Math Grade 5 Answer Key Chapter 1 Place Value, Multiplication, and Expressions Place Value, Multiplication, and Expressions; Properties img 4
$ ____

Answer:
$162

Explanation:
Jacob is designing an aquarium for a doctor’s office. He plans to buy 6 red blond guppies, 1 blue neon guppy, and 1 yellow guppy.
(6 x $22) + (1 x $11) + (1 x $19) = $132 + $11 + $19 = $162

Question 15.
Sylvia bought 8 tickets to a concert. Each ticket costs $18. To find the total cost in dollars, she added the product 8 × 10 to the product 8 × 8, for a total of 144. Which property did Sylvia use?
i. Distributive Property
ii. Associative Property

Answer:
i. Distributive Property

Explanation:
Sylvia bought 8 tickets to a concert. Each ticket costs $18.
To find the total cost in dollars = 8 x $18
Using Distributive Property
(8 × 10) + (8 × 8) = 8 x (10 + 8) = 144.

Question 16.
Sense or Nonsense? Julie wrote (15 – 6) – 3 = 15 – (6 – 3). Is Julie’s equation sense or nonsense? Do you think the Associative Property works for subtraction? Explain.
__________

Answer:
Nonsense;
(15 – 6) – 3 = 9 – 3 = 6.
15 – (6 – 3) = 15 – 3 = 12
6 not equal to 12.
So, Associative Property does not work for subtraction

Question 17.
Test Prep Canoes rent for $29 per day. Which expression can be used to find the cost in dollars of renting 6 canoes for a day?
Options:
A. (6 + 20) + (6 + 9)
B. (6 × 20) + (6 × 9)
C. (6 + 20) × (6 + 9)
D. (6 × 20) × (6 × 9)

Answer:
B. (6 × 20) + (6 × 9)

Explanation:
Canoes rent for $29 per day. For renting 6 canoes for a day, 6 x $29
6 x $(20 + 9) = (6 x 20) + (6 x 9)

Powers of 10 and Exponents – Share and Show – Page No. 18

Write in exponent form and word form.

Question 1.
10 × 10
Exponent form:
Word form:
Type below:
__________

Answer:
Exponent form: 102
Word form: the second power of ten

Explanation:
10 × 10
Base = 10;
Exponent = 2;
Exponent Form: 102
Word Form: the second power of ten

Question 2.
10 × 10 × 10 × 10
Exponent form:
Word form:
Type below:
__________

Answer:
Exponent Form: 104
Word Form: the fourth power of ten

Explanation:
10 × 10 × 10 × 10
Base = 10;
Exponent = 4;
Exponent Form: 104
Word Form: the fourth power of ten

Find the value.

Question 3.
102 = ____

Answer:
100

Explanation:
100 = 1;
101 = 1 x 10 = 10;
102 = 10 x 10 = 100;

Question 4.
4 × 102 = ____

Answer:
400

Explanation:
4 × 102 =
100 = 1;
101 = 1 x 10 = 10;
102 = 10 x 10 = 100;
4 x 100 = 400

Question 5.
7 × 102 = ____

Answer:
700

Explanation:
7 × 102 =
100 = 1;
101 = 1 x 10 = 10;
102 = 10 x 10 = 100;
7 x 100 = 700

Powers of 10 and Exponents – On Your Own – Page No. 19

Write in exponent form and word form.

Question 6.
10 × 10 × 10
exponent form:
word form:
Type below:
__________

Answer:
Exponent form: 103
Word form: the third power of ten

Explanation:
10 × 10 × 10
Base = 10;
Exponent = 3;
Exponent Form: 103
Word Form: the third power of ten

Question 7.
10 × 10 × 10 × 10 × 10
exponent form:
word form:
Type below:
__________

Answer:
Exponent form: 105
Word form: the fifth power of ten

Explanation:
10 × 10 × 10 × 10 × 10
Base = 10;
Exponent = 5;
Exponent Form: 105
Word Form: the fifth power of ten

Find the value.

Question 8.
104 = ____

Answer:
10,000

Explanation:
100 = 1;
101 = 1 x 10 = 10;
102 = 10 x 10 = 100;
103 = 10 x 10 x 10 = 1,000;
104 = 10 x 10 x 10 x 10 = 10,000;

Question 9.
2 × 103 = ____

Answer:
2,000

Explanation:
2 × 103 =
100 = 1;
101 = 1 x 10 = 10;
102 = 10 x 10 = 100;
103 = 10 x 10 x 10 = 1,000;
2 x 1,000 = 2,000

Question 10.
6 × 104 = ____

Answer:
60,000

Explanation:
6 × 104
100 = 1;
101 = 1 x 10 = 10;
102 = 10 x 10 = 100;
103 = 10 x 10 x 10 = 1,000;
104 = 10 x 10 x 10 x 10 = 10,000;
6 x 10,000 = 60,000

Complete the pattern.

Question 11.
7 × 100 = 7 × 1 = _______
7 × 101 = 7 × 10 = _______
7 × 102 = 7 × 10 × 10 = _______
7 × 103 = 7 × 10 × 10 × 10 = _______
7 × 104 = 7 × 10 × 10 × 10 × 10 = _______

Answer:
7 × 100 = 7 × 1 = 7
7 × 101 = 7 × 10 = 70
7 × 102 = 7 × 10 × 10 = 7 x 100 = 700
7 × 103 = 7 × 10 × 10 × 10 = 7 x 1,000 = 7,000
7 × 104 = 7 × 10 × 10 × 10 × 10 = 7 x 10,000 = 70,000

Explanation:
100 = 1;
101 = 1 x 10 = 10;
102 = 10 x 10 = 100;
103 = 10 x 10 x 10 = 1,000;
104 = 10 x 10 x 10 x 10 = 10,000;

Question 12.
9 × 100 = _______ = 9
9 × 101 = _______ = 90
9 × 102 = _______ = 900
9 × 103 = _______ = 9,000
9 × 104 = _______ = 90,000

Answer:
9 × 100 = 9 x 1 = 9
9 × 101 = 9 x 10 = 90
9 × 102 = 9 x 10 x 10 = 900
9 × 103 = 9 x 10 x 10 x 10= 9,000
9 × 104 = 9 x 10 x 10 x 10 x 10 = 90,000

Explanation:
100 = 1;
101 = 1 x 10 = 10;
102 = 10 x 10 = 100;
103 = 10 x 10 x 10 = 1,000;
104 = 10 x 10 x 10 x 10 = 10,000;

Question 13.
12 × 100 = 12 × 1 = _______
12 × 101 = 12 × 10 = _______
12 × 102 = 12 × 10 × 10 = _______
12 × 103 = 12 × 10 × 10 × 10 _______
12 × 104 = 12 × 10 × 10 × 10 × 10 _______

Answer:
12 × 100 = 12 × 1 = 12
12 × 101 = 12 × 10 = 120
12 × 102 = 12 × 10 × 10 = 1,200
12 × 103 = 12 × 10 × 10 × 10 = 12,000
12 × 104 = 12 × 10 × 10 × 10 × 10 = 120,000

Explanation:
100 = 1;
101 = 1 x 10 = 10;
102 = 10 x 10 = 100;
103 = 10 x 10 x 10 = 1,000;
104 = 10 x 10 x 10 x 10 = 10,000;

Question 14.
103 = 10 × 10n What is the value of n?
Think: 103 = 10 × () × (), or 10 × ()
The value of n is ……..
n = ______

Answer:
2

Explanation:
103 = 10 × 10n
103 = 10 x 10 x 10 = 10 x 102
The value of n is 2

Question 15.
Explain how to write 50,000 using exponents.
Type below:
__________

Answer:
5 x 104

Explanation:
5 x 10,000
100 = 1;
101 = 1 x 10 = 10;
102 = 10 x 10 = 100;
103 = 10 x 10 x 10 = 1,000;
104 = 10 x 10 x 10 x 10 = 10,000;
10,000 = 10 x 10 x 10 x 10 = 104
5 x 104

Powers of 10 and Exponents – UNLOCK the Problem – Page No. 20

Question 16.
Lake Superior is the largest of the Great Lakes. It covers a surface area of about 30,000 square miles. How can you show the estimated area of Lake Superior as a whole number multiplied by a power of ten?
Go Math Grade 5 Answer Key Chapter 1 Place Value, Multiplication, and Expressions Place Value, Multiplication, and Expressions; Powers of 10 and Exponents img 5
a. What are you asked to find?
Options:
A. 3 × 102 sq mi
B. 3 × 103 sq mi
C. 3 × 104 sq mi
D. 3 × 105 sq mi

Answer:
C. 3 × 104 sq mi

Explanation:
Lake Superior is the largest of the Great Lakes. It covers a surface area of about 30,000 square miles.
3 x 10,000
100 = 1;
101 = 1 x 10 = 10;
102 = 10 x 10 = 100;
103 = 10 x 10 x 10 = 1,000;
104 = 10 x 10 x 10 x 10 = 10,000;
10,000 = 10 x 10 x 10 x 10 = 104
3 x 104

Question 16.
b. How can you use a pattern to find the answer?
Type below:
__________

Answer:
3 x 10,000
100 = 1;
101 = 1 x 10 = 10;
102 = 10 x 10 = 100;
103 = 10 x 10 x 10 = 1,000;
104 = 10 x 10 x 10 x 10 = 10,000;
10,000 = 10 x 10 x 10 x 10 = 104
3 x 104

Question 16.
c. Write a pattern using the whole number 3 and powers of ten.
3 × 101 = 3 × 10   =
3 × 102 =              =
3 × 103 =              =
3 × 104 =              =
Type below:
__________

Answer:
3 × 101 = 3 × 10   =
3 × 102 = 3 x 10 x 10 = 300
3 × 103 = 3 x 10 x 10 x 10 = 3,000
3 × 104 = 3 x 10 x 10 x 10 x 10 = 30,000

Question 16.
d. Fill in the correct answer choice above.
Type below:
__________

Answer:
3 × 104 = 3 x 10 x 10 x 10 x 10 = 30,000

Question 17.
The Earth’s diameter through the equator is about 8,000 miles. What is the Earth’s estimated diameter written as a whole number multiplied by a power of ten?
Options:
A. 8 × 101 miles
B. 8 × 102 miles
C. 8 × 103 miles
D. 8 × 104 miles

Answer:
C. 8 × 103 miles

Explanation:
The Earth’s diameter through the equator is about 8,000 miles.
8 x 1,000
100 = 1;
101 = 1 x 10 = 10;
102 = 10 x 10 = 100;
103 = 10 x 10 x 10 = 1,000;
1,000 = 10 x 10 x 10
8 x 1,000 = 8 x 103

Question 18.
The Earth’s circumference around the equator is about 25 × 103 miles. What is the Earth’s estimated circumference written as a whole number?
Options:
A. 250,000 miles
B. 25,000 miles
C. 2,500 miles
D. 250 miles

Answer:
B. 25,000 miles

Explanation:
The Earth’s circumference around the equator is about 25 × 103 miles.
25 × 103 miles;
100 = 1;
101 = 1 x 10 = 10;
102 = 10 x 10 = 100;
103 = 10 x 10 x 10 = 1,000;
25 x 1,000 = 25,000 miles

Multiplication Patterns – Share and Show – Page No. 22

Use mental math and a pattern to find the product.

Question 1.
• What basic fact can you use to help you find 30×4,000?
30 × 4,000 = ____

Answer:
3 x 4 = 12

Explanation:
30 × 4,000
The basic fact is 3 x 4 = 12

Use mental math to complete the pattern.

Question 2.
1 × 1 = 1
1 × 101 = _______
1 × 102 = _______
1 × 103 = _______

Answer:
1 × 1 = 1
1 × 101 = 10
1 × 102 = 100
1 × 103 = 1,000

Explanation:
1 × 1 = 1
1 × 101 = 1 x 10 = 10
1 × 102 = 1 x 10 x 10 = 100
1 × 103 = 1 x 10 x 10 x 10 = 1,000

Question 3.
7 × 8 = 56
(7 × 8) × 101 = _______
(7 × 8) × 102 = _______
(7 × 8) × 106 = _______

Answer:
7 × 8 = 56
(7 × 8) × 101 = 560
(7 × 8) × 102 = 5,600
(7 × 8) × 106 = 56,000,000

Explanation:
7 × 8 = 56
(7 × 8) × 101 = 56 x 10 = 560
(7 × 8) × 102 = 56 x 10 x 10 = 5,600
(7 × 8) × 106 = 56 x 10 x 10 x 10 x 10 x 10 x 10 = 56,000,000

Question 4.
6 × 5 = _______
6 × 5 × _______ = 300
6 × 5 × _______ = 3000
6 × 5 × _______ = 30,000

Answer:
6 × 5 = 30
6 × 5 × 101 = 300
6 × 5 × 103 = 3000
6 × 5 × 104 = 30,000

Explanation:
6 × 5 = 30
6 × 5 × 10 = 300
6 × 5 × 10 x 10 x 10 = 3000
6 × 5 × 10 x 10 x 10 x 10 = 30,000

On Your Own

Use mental math to complete the pattern.

Question 5.
9 × 5 = 45
(9 × 5) × 101 = _______
(9 × 5) × 102 = _______
(9 × 5) × 103 = _______

Answer:
9 × 5 = 45
(9 × 5) × 101 = 450
(9 × 5) × 102 = 4,500
(9 × 5) × 103 = 45,000

Explanation:
9 × 5 = 45
(9 × 5) × 101 = 45 x 10 = 450
(9 × 5) × 102 = 45 x 10 x 10 = 4,500
(9 × 5) × 103 = 45 x 10 x 10 x 10 = 45,000

Question 6.
3 × 7 = 21
(3 × 7) × 101 = _______
(3 × 7) × 102 = _______
(3 × 7) × 103 = _______

Answer:
3 × 7 = 21
(3 × 7) × 101 = 210
(3 × 7) × 102 = 2,100
(3 × 7) × 103 = 21,000

Explanation:
3 × 7 = 21
(3 × 7) × 101 = 21 x 10 = 210
(3 × 7) × 102 = 21 x 10 x 10 = 2,100
(3 × 7) × 103 = 21 x 10 x 10 x 10 = 21,000

Question 7.
5 × 4 = _______
(5 × 4) × _______ = 200
(5 × 4) × _______ = 2,000
(5 × 4) × _______ = 20,000

Answer:
5 × 4 = 20
(5 × 4) × 101 = 200
(5 × 4) × 102 = 2,000
(5 × 4) × 103 = 20,000

Explanation:
5 × 4 = 20
(5 × 4) × 10 = 200
(5 × 4) × 10 x 10 = 2,000
(5 × 4) × 10 x 10 x 10 = 20,000

Question 8.
5 × 7 = _______
(5 × 7) × _______ = 350
(5 × 7) × _______ = 3,500
(5 × 7) × _______ = 35,000

Answer:
5 × 7 = 35
(5 × 7) × 101 = 350
(5 × 7) × 102 = 3,500
(5 × 7) × 103 = 35,000

Explanation:
5 × 7 = 35
(5 × 7) × 10 = 350
(5 × 7) × 10 x 10 = 3,500
(5 × 7) × 10 x 10 x 10 = 35,000

Question 9.
4 × 2 = 8
(4 × 2) × 101 = _______
(4 × 2) × 102 = _______
(4 × 2) × 103 = _______

Answer:
4 × 2 = 8
(4 × 2) × 101 = 80
(4 × 2) × 102 = 800
(4 × 2) × 103 = 8,000

Explanation:
4 × 2 = 8
(4 × 2) × 101 = 8 x 10 = 80
(4 × 2) × 102 = 8 x 10 x 10 = 800
(4 × 2) × 103 = 8 x 10 x 10 x 10 = 8,000

Question 10.
6 × 7 = 42
(6 × 7) × 101 = _______
(6 × 7) × 102 = _______
(6 × 7) × 103 = _______

Answer:
6 × 7 = 42
(6 × 7) × 101 = 420
(6 × 7) × 102 = 4,200
(6 × 7) × 103 = 42,000

Explanation:
6 × 7 = 42
(6 × 7) × 101 = 42 x 10 = 420
(6 × 7) × 102 = 42 x 10 x 10 = 4,200
(6 × 7) × 103 = 42 x 10 x 10 x 10 = 42,000

Use mental math and a pattern to find the product.

Question 11.
(6 × 6) × 101 = ____

Answer:
(6 × 6) × 101 =  360

Explanation:
6 x 6 =36
(6 × 6) × 101 = 36 x 10 = 360

Question 12.
(7 × 4) × 103 = ____

Answer:
28,000

Explanation:
7 x 4 = 28
(7 × 4) × 101 = 28 x 10 = 280
(7 × 4) × 102 = 28 x 10 x 10 = 2,800
(7 × 4) × 103 = 28 x 10 x 10 x 10 = 28,000

Question 13.
(9 × 8) × 102 = ____

Answer:
7,200

Explanation:
(9 × 8) = 72
(9 × 8) × 101 = 72 x 10 = 720
(9 × 8) × 102 = 72 x 10 x 10 = 7,200

Question 14.
(4 × 3) × 102 = ____

Answer:
1,200

Explanation:
(4 × 3) = 12
(4 × 3) × 101 = 12 x 10 = 120
(4 × 3) × 102 = 12 x 10 x 10 = 1,200

Question 15.
(2 × 5) × 103 = ____

Answer:
10,000

Explanation:
(2 × 5) = 10
(2 × 5) × 101 = 10 x 10 = 100
(2 × 5) × 102 = 10 x 10 x 10 = 1,000
(2 × 5) × 103 = 10 x 10 x 10 x 10 = 10,000

Question 16.
(2 × 8) × 102 = ____

Answer:
1,600

Explanation:
(2 × 8) = 16
(2 × 8) × 101 = 16 x 10 = 160
(2 × 8) × 102 = 16 x 10 x 10 = 1,600

Question 17.
(6 × 5) × 103 = ____

Answer:
30,000

Explanation:
(6 × 5) = 30
(6 × 5) × 101 = 30 x 10 = 300
(6 × 5) × 102 = 30 x 10 x 10 = 3,000
(6 × 5) × 103 = 30 x 10 x 10 x 10 = 30,000

Question 18.
(8 × 8) × 104 = ____

Answer:
640,000

Explanation:
(8 × 8) = 64
(8 × 8) × 101 = 64 x 10 = 640
(8 × 8) × 102 = 64 x 10 x 10 = 6,400
(8 × 8) × 103 = 64 x 10 x 10 x 10 = 64,000
(8 × 8) × 104 = 64 x 10 x 10 x 10 x 10 = 640,000

Question 19.
(7 × 8) × 104 = ____

Answer:
560,000

Explanation:
(7 × 8) = 56
(7 × 8) × 101 = 56 x 10 = 560
(7 × 8) × 102 = 56 x 10 x 10 = 5,600
(7 × 8) × 103 = 56 x 10 x 10 x 10 = 56,000
(7 × 8) × 104 = 56 x 10 x 10 x 10 x 10 = 560,000

Multiplication Patterns – Share and Show – Page No. 23

Use mental math to complete the table.

Question 20.
1 roll = 50 dimes ; Think:50 dimes per roll × 20 rolls =(5 × 2) × (10 × 10)
Go Math Grade 5 Answer Key Chapter 1 Place Value, Multiplication, and Expressions Place Value, Multiplication, and Expressions; Multiplication Patterns img 6
Type below:
__________

Answer:
Grade 5 Chapter 1 Image 4

Explanation:
1 roll = 50 dimes ;
Think:50 dimes per roll × 20 rolls =(5 × 2) × (10 × 10) = 10 x 102
50 dimes per roll × 30 rolls = (5 x 3) x (10 × 10) = 15 x 102
50 dimes per roll × 40 rolls = (5 x 4) x (20 × 10) = 20 x 102
50 dimes per roll × 50 rolls = (5 x 5) x (10 × 10) = 25 x 102
50 dimes per roll × 60 rolls = (5 x 6) x (10 × 10) = 30 x 102
50 dimes per roll × 70 rolls = (5 x 7) x (10 × 10) = 35 x 102
50 dimes per roll × 80 rolls = (5 x 8) x (10 × 10) = 40 x 102
50 dimes per roll × 90 rolls = (5 x 9) x (10 × 10) = 45 x 102
50 dimes per roll × 100 rolls = (5 x 10) x (10 × 10) = 50 x 102

Question 21.
1 roll = 40 quarters ; Think:40 quarters per roll × 20 rolls =(4 × 2) × (10 × 10)
Go Math Grade 5 Answer Key Chapter 1 Place Value, Multiplication, and Expressions Place Value, Multiplication, and Expressions; Multiplication Patterns img 7
Type below:
__________

Answer:
Grade 5 Chapter 1 Image 5

Explanation:
1 roll = 40 quarters ;
Think:40 quarters per roll × 20 rolls =(4 × 2) × (10 × 10) = 8 x 102
40 quarters per roll × 30 rolls =(4 × 3) × (10 × 10) = 12 x 102
40 quarters per roll × 40 rolls =(4 × 4) × (10 × 10) = 16 x 102
40 quarters per roll × 50 rolls =(4 × 5) × (10 × 10) = 20 x 102
40 quarters per roll × 60 rolls =(4 × 6) × (10 × 10) = 24 x 102
40 quarters per roll × 70 rolls =(4 × 7) × (10 × 10) = 28 x 102
40 quarters per roll × 80 rolls =(4 × 8) × (10 × 10) = 32 x 102
40 quarters per roll × 90 rolls =(4 × 9) × (10 × 10) = 36 x 102
40 quarters per roll × 100 rolls =(4 × 10) × (10 × 10) = 40 x 102

Question 22.
Go Math Grade 5 Answer Key Chapter 1 Place Value, Multiplication, and Expressions Place Value, Multiplication, and Expressions; Multiplication Patterns img 8
Type below:
__________

Answer:
Grade 5 Chapter 1 Image 6

Explanation:
80 x 800 = 64 x 103
80 x 6 = (8 x 6) x 10 = 48 x 101
80 x 70 = (8 x 7) x (10 x 10) = 56 x 102
80 x 9,000 = (8 x 9) x (10 x 10 x 10 x 10) = 64 x 104

Question 23.
Go Math Grade 5 Answer Key Chapter 1 Place Value, Multiplication, and Expressions Place Value, Multiplication, and Expressions; Multiplication Patterns img 9
Type below:
__________

Answer:
Grade 5 Chapter 1 Image 7

Explanation:
Given that
90 x 9,000 = (9 x 9) x 10 x 10 x 10 x 10 = 81 x 104
90 x 6 = (9 x 6) x 10 = 54 x 101
90 x 70 = (9 x 7) x (10 x 10) = 63 x 102
90 x 800 = (9 x 8) x (10 x 10 x 10) = 72 x 103

Problem Solving

Use the table for 24–26.

Question 24.
What if you magnified the image of a cluster fly by 9 × 103 ? What would the length appear to be?
Go Math Grade 5 Answer Key Chapter 1 Place Value, Multiplication, and Expressions Place Value, Multiplication, and Expressions; Multiplication Patterns img 10
____ mm

Answer:
9,000 mm

Explanation:
9 × 103  = 9 x 10 x 10 x 10 = 9,000

Question 25.
If you magnified the image of a fire ant by 4 × 103 and a tree hopper by 3 × 103 , which insect would appear longer? How much longer?
____ mm

Answer:
103 mm

Explanation:
fire ant: 4 × 103  = 4 x 10 x 10 x 10 = 4,000 mm
tree hopper: 3 × 103 = 3 x 10 x 10 x 10 = 3,000 mm
4,000 > 3,000.
So, fire ant appears to be longer.
4,000 – 3,000 = 1,000 = 103

Question 26.
John wants to magnify the image of a fire ant and a crab spider so they appear to be the same length. How many times their actual sizes would he need to magnify each image?
Fire ant by _______ times
Crab spider by ______ times

Answer:
Fire ant by 5 times
Crab spider by 4 times

Explanation:
Given that Fire ant = 4 mm
crab spider = 5 mm
So, to make them have the same lengths, multiply fire ant by 5 mm and multiply Crab spider by 4 mm

Multiplication Patterns – Share and Show – Page No. 24

Question 27.
What does the product of any whole-number factor multiplied by 100 always have? Explain.
Type below:
__________

Answer:
The product of any whole number factor multiplied by 100 has two digits which are 0 in ones and tens place.
Example: 2 x 100 = 200

Question 28.
Test Prep How many zeros are in the product (5 × 4) × 104?
Options:
A. 3
B. 4
C. 5
D. 6

Answer:
C. 5

Explanation:
(5 × 4) × 104 = 20 x 104 = 2 x 105
5 zeroes

Use patterns and mental math to solve.

Question 29.
A human body has about 30 times as many platelets as white blood cells. A small sample of blood has 8×103 white blood cells. About how many platelets are in the sample?
______ platelets

Answer:
24 x 104 platelets

Explanation:
Let the number of platelets = s.
s = 30 x 8×103
s = 30 x 8 x 10 x 10 x 10 = (3 x 8) x (10 x 10 x 10 x 10) = 24 x 104

Question 30.
Basophils and monocytes are types of white blood cells. A blood sample has about 5 times as many monocytes as basophils. If there are 60 basophils in the sample, about how many monocytes are there?
______ monocytes

Answer:
3 x 102 monocytes

Explanation:
Let the number of monocytes = S
S = 5 x 60 = 300 = 3 x 100
S = 3 x 102

Question 31.
Lymphocytes and eosinophils are types of white blood cells. A blood sample has about 10 times as many lymphocytes as eosinophils. If there are 2 × 102 eosinophils in the sample, about how many lymphocytes are there?
______ lymphocytes

Answer:
2 × 103 lymphocytes

Explanation:
Lymphocytes and eosinophils are types of white blood cells. A blood sample has about 10 times as many lymphocytes as eosinophils.
There are 2 × 102 eosinophils in the sample
Then, Lymphocytes = 10 x 2 × 102 eosinophils = 2 × 103

Question 32.
An average person has 6 × 102 times as many red bloods cells as white blood cells. A small sample of blood has 7 × 103 white blood cells. About how many red blood cells are in the sample?
______ red blood cells

Answer:
42 x 10 red blood cells

Explanation:
Let the red blood cells = S
S = 7 × 103 x 6 × 102
S = 42 x 10

Mid-Chapter Checkpoint – Vocabulary – Page No. 25

Choose the best term for the box.

Question 1.
A group of three digits separated by commas in a multidigit number is a __
________

Answer:
Period

Question 2.
An __ is the number that tells how many times a base is used as a factor
________

Answer:
exponent

Concepts and Skills

Complete the sentence.

Question 3.
7 is \(\frac{1}{10}\) of ______

Answer:
70

Explanation:
Let the unknown number is S.
7 = \(\frac{1}{10}\) S
S = 7 X 10 = 70

Question 4.
800 is 10 times as much as ______

Answer:
80

Explanation:
Let the unknown number is S.
800 = 10S
S = 800/10 = 80.

Write the value of the underlined digit.

Question 5.
6,581,678
________

Answer:
80,000

Explanation:
(6 x 1,000,000) + (5 x 1,00,000) + (8 x 10,000) + (1 x 1,000) + (6 x 100) + (7 x 10) + (8 x 1)
8 x 10,000 = 80,000

Question 6.
25,634
________

Answer:
600

Explanation:
(2 x 10,000) + (5 x 1,000) + (6 x 100) + (3 x 10) + (4 x 1)
6 x 100 = 600

Question 7.
34,634,803
________

Answer:
4,000,000

Explanation:
(3 x 10,000,000) + (4 x 1,000,000) + (6 x 1,00,000) + (3 x 10,000) + (4 x 1,000) + (8 x 100) + (0 x 10) + (3 x 1)
4 x 1,000,000 = 4,000,000

Question 8.
2,764,835
________

Answer:
700,000

Explanation:
(2 x 1,000,000) + (7 x 1,00,000) + (6 x 10,000) + (4 x 1,000) + (8 x 100) + (3 x 10) + (5 x 1)
7 x 1,00,000 = 700,000

Complete the equation, and tell which property you used.

Question 9.
8 × (14 + 7) = ________ + (8 × 7)

Answer:
8 × (14 + 7) = (8 x 14) + (8 × 7);
Distributive Property of Multiplication

Explanation:
8 × (14 + 7)
(8 x 14) + (8 × 7); Distributive Property of Multiplication

Question 10.
7 + (8 + 12) = ________ + 12

Answer:
7 + (8 + 12) = (7 + 8) + 12
Associative Property of Addition

Find the value.

Question 11.
103 = ______

Answer:
1,000

Explanation:
100 = 1;
101 = 1 x 10 = 10;
102 = 10 x 10 = 100;
103 = 10 x 10 x 10 = 1,000;

Question 12.
6 × 102 = ______

Answer:
600

Explanation:
6 × 102
100 = 1;
101 = 1 x 10 = 10;
102 = 10 x 10 = 100;
6 x 100 = 600

Question 13.
4 × 104 = ______

Answer:
40,000

Explanation:
4 × 104
100 = 1;
101 = 1 x 10 = 10;
102 = 10 x 10 = 100;
103 = 10 x 10 x 10 = 1,000;
104 = 10 x 10 x 10 x 10 = 10,000;
4 x 10,000 = 40,000

Use mental math and a pattern to find the product.

Question 14.
70 × 300 = ______

Answer:
21,000

Explanation:
70 × 300 = (7 x 3) x (10 x 10 x 10) = 21 x 1,000 = 21,000

Question 15.
(3 × 4) × 103 = ______

Answer:
12,000

Explanation:
(3 × 4) × 103
100 = 1;
101 = 1 x 10 = 10;
102 = 10 x 10 = 100;
103 = 10 x 10 x 10 = 1,000;
12 x 1,000 = 12,000

Mid-Chapter Checkpoint – Page No. 26

Fill in the bubble completely to show your answer.

Question 16.
DVDs are on sale for $24 each. Which expression can be used to find the cost in dollars of buying 4 DVDs?
Options:
A. (4 + 20) + (4 + 4)
B. (4 × 20) + (4 × 4)
C (4 + 20) × (4 + 4)
D. (4 × 20) × (4 × 4)

Answer:
B. (4 × 20) + (4 × 4)

Explanation:
24 can be written as 25 – 1
4 x 24 = 4 x (20 + 4) = (4 x 20) + (4 x 4)

Question 17.
The Muffin Shop chain of bakeries sold 745,305 muffins last year. Which choice shows that number in expanded form?
Options:
A. (7 × 100,000) + (45 × 10,000) + (3 × 100) + (5 × 10)
B. (7 × 100,000) + (4 × 10,000) + (5 × 1,000) + (5 × 10)
C. (7 × 100,000) + (4 × 10,000) + (5 × 1,000) + (3 × 100) + (5 × 1)
D. (7 × 100,000) + (4 × 10,000) + (3 × 100) + (5 × 1)

Answer:
C. (7 × 100,000) + (4 × 10,000) + (5 × 1,000) + (3 × 100) + (5 × 1)

Explanation:
First, we can write 745,305 as:
700,000 + 40, 000 + 5,000 + 300 + 5
(7 x 100,000) + (4 x 10,000) + (5 x 1,000) + (3 x 100) + 5

Question 18.
The soccer field at Mario’s school has an area of 6,000 square meters. How can Mario show the area as a whole number multiplied by a power of ten?
Options:
A. 6 × 104 sq m
B. 6 × 103 sq m
C. 6 × 102 sq m
D. 6 × 101 sq m

Answer:
B. 6 × 103 sq m

Explanation:
6,000 square meters = 6 x 1,000 = 6 x 10 x 10 x 10 = 6 × 103 sq m

Question 19.
Ms. Alonzo ordered 4,000 markers for her store. Only \(\frac{1}{10}\) of them arrived. How many markers did she receive?
Options:
A. 4
B. 40
C. 400
D. 1,400

Answer:
C. 400

Explanation:
Ms. Alonzo ordered 4,000 markers for her store. Only \(\frac{1}{10}\) of them arrived.
4,000 x \(\frac{1}{10}\) = 400

Question 20.
Mark wrote the highest score he made on his new video game as the product of 70 × 6,000. What was his score?
Options:
A. 420
B. 4,200
C. 42,000
D. 420,000

Answer:
D. 420,000

Explanation:
Mark wrote the highest score he made on his new video game as the product of 70 × 6,000.
(7 x 6) x (10 x 10 x 10 x 10) = 42 x 10,000 = 420,000

Multiply by 1-digit numbers – Share and Show – Page No. 29

Complete to find the product.

Question 1.
6 × 796           Estimate: 6 × ___ = ___
Go Math Grade 5 Answer Key Chapter 1 Place Value, Multiplication, and Expressions Place Value, Multiplication, and Expressions; Multiply by 1-digit numbers img 11
______

Answer:
4,776

Explanation:
Go Math Grade 5 Answer Key Chapter 1 Place Value, Multiplication, and Expressions Place Value, Multiplication, and Expressions; Multiply by 1-digit numbers img 11

Estimate. Then find the product.

Question 2.
Estimate: ___
6 0 8
×   8
———-
Estimate: ________
Product: 608 × 8 = ________

Answer:
Estimate: 6,000
Product: 608 × 8 = 4,864

Explanation:
Estimate: 608 is close to 600; 8 is close to 10
600 x 10 = 6,000
608 x 8
Multiply the ones; 8 x 8 = 64. 4 ones and 6 tens. Write the ones and the
regrouped tens.
Multiply the tens; 0 x 8 = 0 + 6 = 6
Multiply the hundreds; 6 x 8 = 48.
So, 4,864 is the product of 608 × 8
Product: 4,864

Question 3.
Estimate: __
5 5 6
×   4
———–
Estimate: ________
Product: 556 × 4 = ________

Answer:
Estimate: 2,780
Product: 556 × 4 = 2,224

Explanation:
Estimate: 556 is close to 550; 4 is close to 5
556 x 5 = 2,780
556 × 4
Multiply the ones; 6 x 4 = 24. 4 ones and 2 tens. Write the ones and the
regrouped tens.
Multiply the tens; 5 x 4 = 20 + 2 = 22; 2 tens and 2 hundreds. Write the tens and regrouped hundreds.
Multiply the hundreds; 5 x 4 = 20; 20 + 2 = 22.
So, 2,224 is the product of 556 × 4
Product: 2,224

Question 4.
Estimate:
1,925
×    7
———–
Estimate: ________
Product: 1,925 × 7 = ________

Answer:
Estimate: 10,000
Product: 1,925 × 7 = 13,475

Explanation:
Estimate: 1,925 is close to 2000; 7 is close to 5
2,000 x 5 = 10,000
1,925 × 7
Multiply the ones; 7 x 5 = 35. 5 ones and 3 tens. Write the ones and the
regrouped tens.
Multiply the tens; 7 x 2 = 14; 14 + 3 = 17; 7 tens and 1 hundreds. Write the tens and regrouped hundreds.
Multiply the hundreds; 7 x 9 = 63; 63 + 1 = 64. 4 hundred and 6 thousand Write the hundreds and regrouped thousands.
Multiply the thousands; 7 x 1 = 7; 7 + 6 = 13
So, 13,475 is the product of 1,925 × 7
Product: 13,475

On Your Own

Estimate. Then find the product.

Question 5.
Estimate:__
7 9 4
×   3
———-
Estimate: ________
Product:794 × 3 = ________

Answer:
Estimate: 800
Product:794 × 3 = 2,382

Explanation:
Estimate: 794 is close to 800 and 3 is close to 1
800 x 1 = 800
794 x 3 = (700 + 90 + 4) x 3 = (700 x 3) + (90 x 3) + (4 x 3) = 2100 + 270 + 12 = 2,382

Question 6.
Estimate:___
8 2 2
×   6
———-
Estimate: ________
822 × 6 = ________

Answer:
Estimate: 4,000
822 × 6 = 4,932

Explanation:
Estimate: 822 is close to 800 and 6 is close to 5
800 x 5 = 4,000
822 × 6 = (800 + 20 + 2) x 6 = (800 x 6) + (20 x 6) + (2 x 6) = 4800 + 120 + 12 = 4,932

Question 7.
Estimate:
3,102
×    5
———–
Estimate: ________
Product: 3,102 × 5 = ________

Answer:
Estimate: 15,500
Product: 3,102 × 5 = 15,510

Explanation:
Estimate: 3,102 is close to 3,100 and 5 is close to 5
3,100 x 5 = 15,500
3,102 x 5 = (3,000 + 100 + 0 + 2) x 5 = (3000 x 5) + (100 x 5) + 0 + (2 x 5) = 15,000 + 500 + 0 + 10 = 15,510

Algebra Solve for the unknown number.

Question 8.
3 9 6
×   6
———
2, 3 6
396 × 6 = 23 ______ 6

Answer:
7

Explanation:
396 x 6 = (300 + 90 + 6) x 6 = (300 x 6) + (90 x 6) + (6 x 6) = 1800 + 540 + 36 = 2376. So, the unknown number is 7

Question 9.
5,1 2
×   8
——–
16
Type below:
__________

Answer:
5127 x 8 = 41,016.
Unknown numbers = 7 and 410

Explanation:
5,127 x 8 = (5000 + 100 + 20 + 7) x 8 = (5000 x 8) + (100 x 8) + (20 x 8) + (7 x 8) = 40000 + 800 + 160 + 56 = 41,016

Question 10.
8, 5 6
×    7
———
60,03
Type below:
__________

Answer:
8,576 x 7 = 60,032

Explanation:
8,576 x 7 = (8000 + 500 + 70 + 6) x 7 = (8000 x 7) + (500 x 7) + (70 x 7) + (6 x 7) = 56000 + 3500 + 490 + 42 = 60,032

Practice: Copy and Solve Estimate. Then find the product.

Question 11.
116 × 3 = _______
Estimate: _______

Answer:
Estimate: 300
116 × 3 = 348

Explanation:
Estimate: 116 is close to 100;
100 x 3 = 300
116 x 3
6 x 3 =18; add ones and regroup tens
3 x 1 = 3; 3 + 1 = 4
3 x 1 = 3
So, 348 is the product

Question 12.
338 × 4 = _______
Estimate: _______

Answer:
338 × 4 = 1,352
Estimate: 1,200

Explanation:
Estimate: 338 is close to 300;
300 x 4 = 1,200
338 × 4
8 x 4 =32; add ones and regroup tens
3 x 4 = 12; 12 + 3 = 15; add tens and regroup hundreds
3 x 4 = 12; 12 + 1 = 13
So, 1352 is the product

Question 13.
6 × 219 = _______
Estimate: _______

Answer:
6 × 219 = 1,314
Estimate: 1200

Explanation:
Estimate: 219 is close to 200
200 x 6 = 1200
6 × 219
6 x 9 = 54; add ones and regroup tens
6 x 1 = 6; 6 + 5 = 11; add tens and regroup hundreds
6 x 2 = 12; 12 + 1 = 13
So, 1,314

Question 14.
7 × 456 = _______
Estimate: _______

Answer:
7 × 456 = 3192
Estimate: 3500

Explanation:
Estimate: 456 is close to 500
500 x 7 = 3500
7 x 456
7 x 6 = 42; add ones and regroup tens
7 x 5 = 35; 35 + 4 = 39; add tens and regroup hundreds
7 x 4 = 28; 28 + 3 = 31
So, 3192

Question 15.
5 × 1,012 = _______
Estimate: _______

Answer:
5 × 1,012 = 5,060
Estimate: 5,000

Explanation:
Estimate: 1,012 is close to 1,000
1,000 x 5 = 5,000
5 × 1,012
5 x 2 = 10; add ones and regroup tens
5 x 1 = 5; 5 + 1 = 6; add tens and regroup hundreds
5 x 0 = 0
5 x 1 = 5
So, 5,060

Question 16.
2,921 × 3 = _______
Estimate: _______

Answer:
2,921 × 3 = 8,763
Estimate: 9,000

Explanation:
Estimate: 2,921 is close to 3,000
3,000 x 3 = 9,000
2,921 × 3
3 x 1 = 3;
3 x 2 = 6;
3 x 9 = 27; add hundreds and regroup thousands
3 x 2 = 6; 6 + 2 = 8
So, 8,763

Question 17.
8,813 × 4 = _______
Estimate: _______

Answer:
8,813 × 4 = 35,252
Estimate: 3,600

Explanation:
Estimate: 8,813 is close to 9,000
9,000 x 4 = 3,600
8,813 × 4
4 x 3 = 12; add ones and regroup tens
4 x 1 = 4; 4 + 1 = 5;
4 x 8 = 32; add hundreds and regroup thousands
4 x 8 = 32; 32 + 3 = 35
So, 35,252

Question 18.
9 × 3,033 = _______
Estimate: _______

Answer:

Explanation:
Estimate: 3,033 is close to 3,000
3,000 x 9 = 27,000
9 × 3,033
9 x 3 = 27; add ones and regroup tens
9 x 3 = 27; 27 +  = 11; add tens and regroup hundreds
6 x 2 = 12; 12 + 1 = 13
So, 1,314

Multiply by 1-digit numbers – Problem Solving – Page No. 30

What’s the Error?

Question 19.
The Plattsville Glee Club is sending 8 of its members to a singing contest in Cincinnati, Ohio. The cost will be $588 per person. How much will it cost for the entire group of 8 students to attend? Both Brian and Jermaine solve the problem. Brian says the answer is $40,074. Jermaine’s answer is $4,604.
Estimate the cost. A reasonable estimate is _
$ ______

Answer:
$4,800

Explanation:
The Plattsville Glee Club is sending 8 of its members to a singing contest in Cincinnati, Ohio. The cost will be $588 per person.
So, for entire group 8 x $588 = $4,704
Jermaine’s answer is correct. Because the $4,604 is close to $4,704
588 is close to 600. So, 600 x 8 = $4,800

Question 19.
Although Jermaine’s answer seems reasonable, neither Brian nor Jermaine solved the problem correctly. Find the errors in Brian’s and Jermaine’s work.
Then, solve the problem correctly.
Go Math Grade 5 Answer Key Chapter 1 Place Value, Multiplication, and Expressions Place Value, Multiplication, and Expressions; Multiply by 1-digit numbers img 12
What error did Brian make? Explain.
Type below:
__________

Answer:
When Brian multiplied the tens, he wrote the total number of tens in the product instead of regrouping, so the place values of his product are incorrect.

Question 19.
What error did Jermaine make? Explain.
Type below:
__________

Answer:
Jermaine regrouped the wrong amount of hundreds. He regrouped the tens as 6 hundred instead of 7 hundred.
$588 x 8 = $4,704

Question 19.
How could you predict that Jermaine’s answer might be incorrect using your estimate?
Type below:
__________

Answer:
I used 600 × 8 to estimate the product; 588 is 12 less than 600. Since 12 × 8 = 96, and 4,604 is almost 200 less than the estimate of 4,800, the answer is probably too low.

Multiply by 2-digit numbers – Share and Show – Page No. 33

Complete to find the product

Question 1.
Go Math Grade 5 Answer Key Chapter 1 Place Value, Multiplication, and Expressions Place Value, Multiplication, and Expressions; Multiply by 2-digit numbers img 13
Type below:
__________

Answer:
2,752

Explanation:
64 x 3 = 192
64 x 40 = 2,560
2,560 + 192 = 2,752

Question 2.
Go Math Grade 5 Answer Key Chapter 1 Place Value, Multiplication, and Expressions Place Value, Multiplication, and Expressions; Multiply by 2-digit numbers img 14
Type below:
__________

Answer:
21,698

Explanation:
571 x 8 = 4,568
571 x 30 = 17,130
17,130 + 4,568 = 21,698

Estimate. Then find the product.

Question 3.
Estimate:____
2 4
× 1 5
———-
Estimate: ________
Product: ________

Answer:
Estimate: 300
Product: 360

Explanation:
2 4 x 15
Estimate: 20 x 15 = 300
24 x 5 = 120
24 x 10 = 240
Product:: 240 + 120 = 360

Question 4.
Estimate:____
3 7
× 6 3
———-
Estimate: ________
Product: ________

Answer:
Estimate: 2,400
Product: 2,331

Explanation:
37 x 63
Estimate: 40 x 60 = 2,400
37 x 3 = 111
37 x 60 = 2220
Product:: 2220 + 111 = 2,331

Question 5.
Estimate:____
3 8 4
× 4 5
———-
Estimate: ________
Product: ________

Answer:
Estimate: 20,000
Product: 17,280

Explanation:
384 x 45
Estimate: 400 x 50 = 20,000
384 x 5 = 1920
384 x 40 = 15,360
Product:: 15,360 + 1920 = 17,280

On Your Own

Estimate. Then find the product.

Question 6.
Estimate:____
2 8
× 2 2
———-
Estimate: ________
Product: ________

Answer:
Estimate: 600
Product: 616

Explanation:
28 x 22
Estimate: 30 x 20 = 600
28 x 2 = 56
28 x 20 = 560
Product:: 56 + 560 = 616

Question 7.
Estimate:____
9 3
× 7 6
———-
Estimate: ________
Product: ________

Answer:
Estimate: 7200
Product: 7,068

Explanation:
93 x 76
Estimate: 90 x 80 = 7200
93 x 6 = 558
93 x 70 = 6,510
Product:: 558 + 6,510 = 7,068

Question 8.
Estimate:____
2 9 5
× 5 1
———-
Estimate: ________
Product: ________

Answer:
Estimate: 15,000
Product: 15,045

Explanation:
295 x 51
Estimate: 300 x 50 = 15,000
295 x 1 = 295
295 x 50 = 14,750
Product:: 295 + 14,750 = 15,045

Practice: Copy and Solve Estimate. Then find the product.

Question 9.
Estimate: ________
54 × 31 = ________

Answer:
Estimate: 1,500
Product: 1,674

Explanation:
54 x 31
Estimate: 50 x 30 = 1,500
54 x 1 = 54
54 x 30 = 1,620
Product:: 54 + 1,620 = 1,674

Question 10.
Estimate: ________
42 × 26 = ________

Answer:
Estimate: 1,200
Product: 1,092

Explanation:
42 x 26
Estimate: 40 x 30 = 1,200
42 x 6 = 252
42 x 20 = 840
Product:: 252 + 840 = 1,092

Question 11.
Estimate: ________
38 × 64 = ________

Answer:
Estimate: 2,400
Product: 2,432

Explanation:
38 × 64
Estimate: 40 x 60 = 2,400
38 x 4 = 152
38 x 60 = 2,280
Product:: 152 + 2,280 = 2,432

Question 12.
Estimate: ________
63 × 16 = ________

Answer:
Estimate: 1,200
Product: 1,008

Explanation:
63 x 16
Estimate: 60 x 20 = 1,200
63 x 6 = 378
63 x 10 = 630
Product:: 378 + 630 = 1,008

Question 13.
Estimate: ________
204 × 41 = ________

Answer:
Estimate: 8,000
Product: 8,364

Explanation:
204 × 41
Estimate: 200 x 40 = 8,000
204 x 1 = 204
204 x 40 = 8,160
Product:: 204 + 8,160 = 8,364

Question 14.
Estimate: ________
534 × 25 = ________

Answer:
Estimate: 15,000
Product: 13,350

Explanation:
534 x 25
Estimate: 500 x 30 = 15,000
534 x 5 = 2,670
534 x 20 = 10,680
Product:: 2,670 + 10,680 = 13,350

Question 15.
Estimate: ________
722 × 39 = ________

Answer:
Estimate: 28,000
Product: 28,158

Explanation:
722 × 39
Estimate: 700 x 40 = 28,000
722 x 9 = 6,498
722 x 30 = 21,660
Product:: 6,498 + 21,660 = 28,158

Question 16.
Estimate: ________
957 × 43 = ________

Answer:
Estimate: 40,000
Product: 44,022

Explanation:
957 × 43
Estimate: 1,000 x 40 = 40,000
957 x 3 = 2,871
957 x 40 = 41,151
Product:: 2,871 + 41,151 = 44,022

Multiply by 2-digit numbers – Problem Solving – Page No. 34

Use the table for 17–20.

Question 17.
How much sleep does a jaguar get in 1 year?
Go Math Grade 5 Answer Key Chapter 1 Place Value, Multiplication, and Expressions Place Value, Multiplication, and Expressions; Multiply by 2-digit numbers img 15
_____ hours

Answer:
4,004 hours

Explanation:
52 weeks x 77 = 4,004 hours

Question 18.
In 1 year, how many more hours of sleep does a giant armadillo get than a platypus?
_____ hours

Answer:
1,508 hours

Explanation:
giant armadillo: 52 x 127 = 6,604
platypus: 52 x 98 = 5,096
6,604 – 5,096 = 1,508

Question 19.
Owl monkeys sleep during the day, waking about 15 minutes after sundown to find food. At midnight, they rest for an hour or two, then continue to feed until sunrise. They live about 27 years. How many hours of sleep does an owl monkey that lives 27 years get in its lifetime?
_____

Answer:
4927.5 days

Explanation:
Given that the time of Owl monkeys sleeps during the day walking about 15 minutes after sundown. Then, at midnight they rest for an hour or two then continue to feed until sunrise. Notice that the above description doesn’t say that they sleep in after sundown. They either eat or rest.
Day time is usually considered from 6 am to 6 pm which is total of 12 hours.
As per the given information, owl monkey sleeps in that period.
Given that owl, monkeys live for 27 years.
So the time for sleeping in their lifetime = 12 hours * 27 years
= (12/24 days) * 27 years
= (1/2 days) * 27 years
= (1/2 days) * (27*365 days)
= (1/2 days) * (9855 days)
= 4927.5 days
Hence final answer is 4927.5 days.

Question 20.
Three-toed sloths move very slowly, using as little energy as possible. They sleep, eat, and even give birth upside down. A baby sloth may cling to its mother for as much as 36 weeks after being born. How much of that time is the sloth asleep?
_____ hours

Answer:
3,636 hours

Explanation:
101 x 36 = 3,636

Question 21.
Test Prep A sloth’s maximum speed on the ground is 15 feet in 1 minute. Even though it would be unlikely for a sloth to stay in motion for more than a few moments, how far would a sloth travel in 45 minutes at that speed?
Options:
A. 60 feet
B. 270 feet
C. 675 feet
D. 6,750 feet

Answer:
C. 675 feet

Explanation:
sloth’s maximum speed on the ground is 15 feet in 1 minute. For 45 minutes, 45 x 15 = 675 feet

Relate Multiplication to Division – Share and Show – Page No. 37

Question 1.
Brad has 72 toy cars that he puts into 4 equal groups. How many cars does Brad have in each group? Use the array to show your answer.
4 × = 72         72 ÷ 4 =
Go Math Grade 5 Answer Key Chapter 1 Place Value, Multiplication, and Expressions Place Value, Multiplication, and Expressions; Relate Multiplication to Division img 16
4 × ________ = 72
72 ÷ 4 = ________

Answer:
Grade 5 Chapter 1 Image 8

Explanation:
Brad has 18 toy cars in each group.
Because 72÷4 =18
4 × 18 = 72
4 x (9 + 9) = (4 x 9) + (4 x 9)

Use multiplication and the Distributive Property to find the quotient.

Question 2.
108 ÷ 6 = _____

Answer:
18

Explanation:
108 ÷ 6
6 x 18 = 6 x (9 + 9) = (6 x 9) + (6 x 9) = 54 + 54 = 108
(36 + 72) ÷ 6 = (36 ÷ 6) + (72÷ 6) = 6 + 12 = 18

Question 3.
84 ÷ 6 = _____

Answer:
14

Explanation:
84 ÷ 6
(42 + 42) ÷ 6 = (42÷ 6) + (42÷ 6) = 7 + 7 = 14
6 x 14 = 6 x (7 + 7) = (6 x 7) + (6 x 7) = 42 + 42 = 84

Question 4.
184 ÷ 8 = _____

Answer:
23

Explanation:
184 ÷ 8
(92 + 92) ÷ 8 = (92÷ 8) + (92÷ 8) = 11.5 + 11.5 = 23
8 x 23 = 8 x (11 + 12) = (8 x 11) + (8 x 12) = 88 + 96 = 184

On Your Own

Use multiplication and the Distributive Property to find the quotient.

Question 5.
60 ÷ 4 = _____

Answer:
15

Explanation:
60 ÷ 4
(20 + 40) ÷ 4 = (20 ÷ 4) + (40 ÷ 4) = 5 + 10 = 15
4 x 15 = 4 x (7+ 8) = (4 x 7) + (4 x 8) = 28 + 32 = 60

Question 6.
144 ÷ 6 = _____

Answer:
24

Explanation:
144 ÷ 6
(72 + 72) ÷ 6 = (72 ÷ 6) + (72 ÷ 6) = 12 + 12 = 24
6 x 24 = 6 x ( 12 + 12) = (6 x 12) + (6 x 12) = 72 + 72 = 144

Question 7.
252 ÷ 9 = _____

Answer:
28

Explanation:
252 ÷ 9
(126 + 126) ÷ 9 = (126 ÷ 9) + (126 ÷ 9) = 14 + 14 = 28
6 x 28 = 6 x ( 14 + 14) = (6 x 14) + (6 x 14) = 126 + 126 = 252

Find each quotient. Then compare. Write <, > or =.

Question 8.
51 ÷ 3 _____ 68 ÷ 4

Answer:
51 ÷ 3 = 68 ÷ 4

Explanation:
51 ÷ 3 = 17
68 ÷ 4 = 17
17 = 17.

Question 9.
252 ÷ 6 _____ 135 ÷ 3

Answer:
252 ÷ 6 < 135 ÷ 3

Explanation:
252 ÷ 6 = 42
135 ÷ 3 = 45
42 < 45

Question 10.
110 ÷ 5 _____ 133 ÷ 7

Answer:
110 ÷ 5 > 133 ÷ 7

Explanation:
110 ÷ 5 = 22
133 ÷ 7 = 19
22 > 19

Relate Multiplication to Division – Problem Solving – Page No. 38

Use the table to solve 11–13.

Question 11.
A group of 6 friends share a bag of the 45-millimeter bouncy balls equally among them. How many does each friend get?
Go Math Grade 5 Answer Key Chapter 1 Place Value, Multiplication, and Expressions Place Value, Multiplication, and Expressions; Relate Multiplication to Division img 17
____ balls

Answer:
7 balls

Explanation:
A group of 6 friends shares a bag of the 45-millimeter bouncy balls equally among them.
45/6 = 7 balls and 3 balls remained

Question 12.
Mr. Henderson has 2 bouncy-ball vending machines. He buys one bag of the 27-millimeter balls and one bag of the 40-millimeter balls. He puts an equal number of each in the 2 machines. How many bouncy balls does he put in each machine?
________ balls of the 27-millimeter .
________ balls of the 40-millimeter .
________ total balls

Answer:
51 balls of the 27-millimeter
29 balls of the 40-millimeter
80 total balls

Explanation:
There’s 51 27 mm, 29 40 mm

Question 13.
Lindsey buys a bag of each size of bouncy ball. She wants to put the same number of each size of bouncy ball into 5 party-favor bags. How many of each size of bouncy ball will she put in a bag?
________ balls of the 27-millimeter
________ balls of the 40-millimeter
________ balls of the 45-millimeter

Answer:
34 balls of the 27-millimeter
12 balls of the 40-millimeter
34 balls of the 45-millimeter

Explanation:
There’s 34 27 mm, 12 40 mm, and 34 45 mm bouncy balls

Question 14.
What’s the Error? Sandy writes (4 × 30) + (4 × 2) and says the quotient for 128 ÷ 4 is 8. Is she correct? Explain.
128 ÷ 4 = ____

Answer:
Sandy’s answer is incorrect.
128 ÷ 4 = 32

Explanation:
(4 × 30) + (4 × 2) = 120 + 8 = 128
128 ÷ 4 = 32.
(4 × 30) + (4 × 2) = 4 x (30 + 2) = 4 x 32 = 128.
128 ÷ 4 = 32

Question 15.
Test Prep Which of the following can be used to find 150 ÷ 6?
Options:
A. (6 × 20) + (6 × 5)
B. (6 × 10) + (6 × 5)
C. (2 × 75) + (2 × 3)
D. (6 × 15) + (6 × 5)

Answer:
A. (6 × 20) + (6 × 5)

Explanation:
150 ÷ 6 = 25
25 x 6 = 150
6 x 25 = 6 x (20 + 5) = (6 × 20) + (6 × 5)

Problem Solving Multiplication and Division – Share and Show – Page No. 41

Question 1.
To make concrete mix, Monica pours 34 pounds of cement, 68 pounds of sand, 14 pounds of small pebbles, and 19 pounds of large pebbles into a large wheelbarrow. If she pours the mixture into 9 equalsize bags, how much will each bag weigh?
First, find the total weight of the mixture.
____ pounds

Answer:
135 pounds

Explanation:
34 + 68 + 14 + 19 = 135

Question 1.
Then, divide the total by the number of bags. Break the total into two simpler numbers to make the division easier, if necessary.
Finally, find the quotient and solve the problem.
So, each bag will weigh _ pounds.
____ pounds

Answer:
15 pounds

Explanation:
now, calculate 135/9 = 15 pounds.

Question 2.
What if Monica pours the mixture into 5 equal-size bags? How much will each bag weigh?
____ pounds

Answer:
27 pounds

Explanation:
135/5 = 27

Question 3.
Taylor is building doghouses to sell. Each doghouse requires 3 full sheets of plywood which Taylor cuts into new shapes. The plywood is shipped in bundles of 14 full sheets. How many doghouses can Taylor make from 12 bundles of plywood?
____

Answer:
56 doghouses

Explanation:
Taylor is building doghouses to sell. Each doghouse requires 3 full sheets of plywood which Taylor cuts into new shapes. The plywood is shipped in bundles of 14 full sheets.
(12 x 14)/3 = 168/3 = 56 doghouses

Question 4.
Eileen is planting a garden. She has seeds for 60 tomato plants, 55 sweet corn plants, and 21 cucumber plants. She plants them in 8 rows, with the same number of plants in each row. How many seeds are planted in each row?
____ seeds / row

Answer:
17 seeds

Explanation:
Eileen is planting a garden. She has seeds for 60 tomato plants, 55 sweet corn plants, and 21 cucumber plants. She plants them in 8 rows, with the same number of plants in each row.
60 + 55 + 21 = 136
136/8 = 17

Problem Solving Multiplication and Division – On Your Own – Page No. 42

Question 5.
Starting on day 1 with 1 jumping jack, Keith doubles the number of jumping jacks he does every day. How many jumping jacks will Keith do on day 10?
____  jumping jack

Answer:
512 jumping jacks

Explanation:
She doubled the number so you’re supposed to multiply by 2.
On day 1, 1 jumping pack;
On day 2, 2 jumping packs;
On day 3, 2 x 2 = 4 jumping packs;
On day 4, 2 x 2 x 2 = 8 jumping packs;
On day 5, 2 x 2 x 2 x 2 = 16 jumping packs;
On day 6, 2 x 2 x 2 x 2 x 2 = 32 jumping packs;
On day 7, 2 x 2 x 2 x 2 x 2 x 2 = 64 jumping packs;
On day 8, 2 x 2 x 2 x 2 x 2 x 2 x 2 = 128 jumping packs;
On day 9, 2 x 2 x 2 x 2 x 2 x 2 x 2 x 2 = 256 jumping packs;
On day 10, 2 x 2 x 2 x 2 x 2 x 2 x 2 x 2 x 2 = 512 jumping packs;
512 jumping jacks

Question 6.
Starting in the blue square, in how many different ways can you draw a line that passes through every square without picking up your pencil or crossing a line you’ve already drawn? Show the ways.
Go Math Grade 5 Answer Key Chapter 1 Place Value, Multiplication, and Expressions Place Value, Multiplication, and Expressions; Problem Solving Multiplication and Division img 18
____ ways

Answer:
8 ways

Explanation:
Grade 5 Chapter 1 Image 9
8 ways

Question 7.
On April 11, Millie bought a lawn mower with a 50-day guarantee. If the guarantee begins on the date of purchase, what is the first day on which the mower will no longer be guaranteed?
__________

Answer:
May 31

Explanation:
The guarantee begins on April 11. April has 30 days. So, we have 20 days of the guarantee in April.
May has 31 days. So, we have 30 days of the guarantee in May.
Therefore, the last day of the guarantee is May 31.

Question 8.
A classroom bulletin board is 7 feet by 4 feet. If there is a picture of a student every 6 inches along the edge, including one in each corner, how many pictures are on the bulletin board?
____ pictures

Answer:
40 pictures

Explanation:
2 pictures per foot, so that’s 14 pics along the top and bottom (counting the corners), and
6 pictures along each side (not counting the corners)
14+14+6+6 = 40

Question 9.
Dave wants to make a stone walkway. The rectangular walkway is 4 feet wide and 12 feet long. Each 2 foot by 2 foot stone covers an area of 4 square feet. How many stones will Dave need to make his walkway?
____ stones

Answer:
12 stones

Explanation:
Area of walkway = 4 x 12 = 48 square feet
Area of 1 stone = 2 x 2 = 4 square feet
48/4 = 12 stones.

Question 10.
Test Prep Dee has 112 minutes of recording time. How many 4-minute songs can she record?
Options:
A. 28
B. 27
C. 18
D. 17

Answer:
A. 28

Explanation:
Dee has 112 minutes of recording time. 112/4 = 28

Numerical Expressions – Share and Show – Page No. 45

Circle the expression that matches the words.

Question 1.
Teri had 18 worms. She gave 4 worms to Susie and 3 worms to Jamie.
(18 – 4) + 3         18 – (4 + 3)
__________

Answer:
18 – (4 + 3)

Explanation:
Teri had 18 worms. She gave 4 worms.
18 – 4 = 14.
3 worms to Jamie
14 – 3 = 11.
(18 – 4) + 3 = 14 + 3 = 17 not equal to 11;
18 – (4 + 3) = 18 – 7 = 11 = 11

Question 2.
Rick had $8. He then worked 4 hours for $5 each hour.
$8 + (4 × $5)       ($8 + 4) × $5
__________

Answer:
$8 + (4 × $5)

Explanation:
Rick had $8. He then worked 4 hours for $5 each hour = 4 x $5 = $20;
$20 +$8 =$28.
$8 + (4 × $5) = 8 + 20 = $28 = $28.

Write an expression to match the words.

Question 3.
Greg drives 26 miles on Monday and 90 miles on Tuesday.
Type below:
__________

Answer:
26 + 90 = 116 miles

Explanation:
Greg drives 26 miles on Monday and 90 miles on Tuesday. 26 + 90 = 116 miles

Question 4.
Lynda has 27 fewer fish than Jack. Jack has 80 fish.
Type below:
__________

Answer:
80 – 27 = 53

Explanation:
Lynda has 27 fewer fish than Jack. Jack has 80 fish. 80 – 27 = 53

Write words to match the expression.

Question 5.
34 – 17
Type below:
__________

Answer:
Anna has 17 fewer apples than Jack. Jack has 34 apples.

Explanation:
34 – 17 = 17.
Anna has 17 fewer apples than Jack. Jack has 34 apples.

Question 6.
6 × (12 – 4)
Type below:
__________

Answer:
Teri had 12 worms. She gave 4 worms to Susie. She sell remaining worms for $6 each.

Explanation:
Teri had 12 worms. She gave 4 worms to Susie. She sell remaining worms for $6 each.

On Your Own

Write an expression to match the words.

Question 7.
José shared 12 party favors equally among 6 friends.
Type below:
__________

Answer:
12/6 = 2

Explanation:
José shared 12 party favors equally among 6 friends. 12/6 = 2

Question 8.
Braden has 14 baseball cards. He finds 5 more baseball cards.
Type below:
__________

Answer:
14 + 5 = 19

Explanation:
Braden has 14 baseball cards. He finds 5 more baseball cards. 14 + 5 = 19

Question 9.
Isabelle bought 12 bottles of water at $2 each.
Type below:
__________

Answer:
12 x $2 = $24

Explanation:
Isabelle bought 12 bottles of water at $2 each. 12 x $2 = $24

Question 10.
Monique had $20. She spent $5 on lunch and $10 at the bookstore.
Type below:
__________

Answer:
20 – (5 + 10)

Explanation:
Monique had $20. She spent $5 on lunch and $10 at the bookstore. 20 – (5 + 10)

Write words to match the expression.

Question 11.
36 ÷ 9
Type below:
__________

Answer:
Anna shared 36 apples to 9 friends.

Question 12.
35 – (16 + 11)
Type below:
__________

Answer:
Monique had 35 balls. She gave 16 to her one frined and 11 to other friend.

Draw a line to match the expression with the words.

Question 13.
Fred catches 25 fish. Then he
releases 10 fish and catches 8 more.     •        •3 × (15 – 6)

Nick has 25 pens. He gives 10 pens to
one friend and 8 pens to another friend.  •     •15 – 6

Jan catches 15 fish and lets 6 fish go.       •     •25 – (10 + 8)

Libby catches 15 fish and lets 6 fish go
for three days in a row.                             •     •(25 – 10) + 8
Type below:
__________

Answer:
Fred catches 25 fish. Then he releases 10 fish and catches 8 more = (25 – 10) + 8
Nick has 25 pens. He gives 10 pens to one friend and 8 pens to another friend = 25 – (10 + 8)
Jan catches 15 fish and lets 6 fish go = 15 – 6
Libby catches 15 fish and lets 6 fish go for three days in a row = 3 × (15 – 6)

Numerical Expressions – Problem Solving – Page No. 46

Use the rule and the table for 14.
Go Math Grade 5 Answer Key Chapter 1 Place Value, Multiplication, and Expressions Place Value, Multiplication, and Expressions; Numerical Expressions img 19

Question 14.
Write a numerical expression to represent the total number of lemon tetras that could be in a 20-gallon aquarium.
_____ lemon tetras

Answer:
2 lemon tetras

Explanation:
From the given data, increase each inch of lenth, 1 gallon of water increases.
2 + 3 + 5 + 3 + 5 = 18 inches. So, 18 gallon of water available in aquarium. Add 1 lemon tetra to get the 20 gallon of water. So, in total there are 2 lemon tetra available.

Question 15.
Write a word problem for an expression that is three times as great as (15 + 7). Then write the expression.
Type below:
__________

Answer:
3 x (15 + 7)

Explanation:
Given that that is three times as great as (15 + 7)

Question 16.
What’s the Question? Lu has 3 swordtails in her aquarium. She buys 2 more swordtails.
Type below:
__________

Answer:
3 + 2 = 5

Explanation:
Lu has 3 swordtails in her aquarium. She buys 2 more swordtails. So, 3 + 2 = 5 swordtails

Question 17.
Tammy gives 45 stamps to her 9 friends. She shares them equally among her friends. Write an expression to match the words. How many stamps does each friend get?
_____ stamps

Answer:
45/9 = 5 stamps

Explanation:
Tammy gives 45 stamps to her 9 friends. 45/9 = 5.

Question 18.
Test Prep Josh has 3 fish in each of 5 buckets. Then he releases 4 fish. Which expression matches the words?
Options:
a. (3 × 4) – 5
b. (5 × 4) – 3
c. (5 × 3) – 4
d. (5 – 3) × 4

Answer:
c. (5 × 3) – 4

Explanation:
Josh has 3 fish in each of 5 buckets. 3 x 5.
Then he releases 4 fish. (3 x 5) – 4

Evaluate Numerical Expressions – Share and Show – Page No. 49

Evaluate the numerical expression.

Question 1.
10 + 36 ÷ 9
_____

Answer:
14

Explanation:
10 + 36 ÷ 9 = 10 + (36 ÷ 9) = 10 + 4 = 14

Question 2.
10 + (25 – 10) ÷ 5
_____

Answer:
13

Explanation:
10 + (25 – 10) ÷ 5 = 10 + 15 ÷ 5 = 10 + (15 ÷ 5) = 10 + 3 = 13

Question 3.
9 – (3 × 2) + 8
_____

Answer:
11

Explanation:
9 – (3 × 2) + 8 = 9 – 6 + 8 = 3 + 8 = 11

On Your Own

Evaluate the numerical expression.

Question 4.
(4 + 49) – 4 × 10
_____

Answer:
13

Explanation:
(4 + 49) – 4 × 10 = 53 – 4 x 10 = 53 – (4 x 10) = 53 – 40 = 13

Question 5.
5 + 17 – 100 ÷ 5
_____

Answer:

Explanation:
5 + 17 – 100 ÷ 5 = (5 + 17) – (100 ÷ 5) = 22 – 20 = 2

Question 6.
36 – (8 + 5)
_____

Answer:
23

Explanation:
36 – (8 + 5) = 36 – 13 = 23

Question 7.
125 – (68 + 7)
_____

Answer:
50

Explanation:
125 – (68 + 7) = 125 – 75 = 50

Question 8.
(4 × 6) – 12
_____

Answer:
12

Explanation:
(4 × 6) – 12 = 24 – 12 = 12

Question 9.
3 × (22 – 2)
_____

Answer:
60

Explanation:
3 × (22 – 2) = 3 x 20 = 60

Question 10.
23 + (16 – 7)
_____

Answer:
32

Explanation:
23 + (16 – 7) = 23 + 9 = 32

Question 11.
(25 – 4) ÷ 3
_____

Answer:
7

Explanation:
(25 – 4) ÷ 3 = 21 ÷ 3 = 7

Rewrite the expression with parentheses to equal the given value.

Question 12.
100 – 30 ÷ 5
value: 14
Type below:
__________

Answer:
(100 – 30) ÷ 5 = 14

Explanation:
100 – 30 ÷ 5 = (100 – 30) ÷ 5 = 70 ÷ 5 = 14

Question 13.
12 + 17 – 3 × 2
value: 23
Type below:
__________

Answer:
23

Explanation:
(12 + 17) – (3 × 2) = 29 – 6 = 23

Question 14.
9 + 5 ÷ 5 + 2
value: 12
Type below:
__________

Answer:
12

Explanation:
9 + (5 ÷ 5) + 2 = 9 + 1 + 2 = 12

Evaluate Numerical Expressions – UNLOCK the Problem – Page No. 50

Question 15.
A movie theater has 4 groups of seats. The largest group of seats, in the middle, has 20 rows, with 20 seats in each row. There are 2 smaller groups of seats on the sides, each with 20 rows and 6 seats in each row. A group of seats in the back has 5 rows, with 30 seats in each row. How many seats are in the movie theater?
Go Math Grade 5 Answer Key Chapter 1 Place Value, Multiplication, and Expressions Place Value, Multiplication, and Expressions; Evaluate Numerical Expressions img 20
a. What do you need to know?
Type below:
__________

Answer:
We have to know the number of seats in each row to calculate the total number of seats.

Question 15.
b. What operation can you use to find the number of seats in the back group of seats? Write the expression.
Type below:
__________

Answer:
A group of seats in the back has 5 rows with 30 seats in each row. So, to calculate the number of seats, we can use multiplication.

Question 15.
c. What operation can you use to find the number of seats in both groups of side seats? Write the expression.
Type below:
__________

Answer:
2 x (20 x 6) = 2 x 120 = 240

Question 15.
d. What operation can you use to find the number of seats in the middle group? Write the expression.
Type below:
__________

Answer:
20 x 20 = (2 x 2) x (10 x 10) = 4 x 100 = 400

Question 15.
e. Write an expression to represent the total number of seats in the theater.
Type below:
__________

Answer:
(20 x 20) + (2 x 20 x 6) + (5 x 30)

Question 15.
f. How many seats are in the theater? Show the steps you use to solve the problem.
_____ seats

Answer:
(20 x 20) + (2 x 20 x 6) + (5 x 30) = 400 + 240 + 150 = 640 + 150 = 790

Question 16.
Test Prep In the wild, an adult giant panda eats about 30 pounds of food each day. Which expression shows how many pounds of food 6 pandas eat in 3 days?
Options:
a. 3 + (30 × 6)
b. 3 × (30 × 6)
c. (30 × 6) ÷ 3
d. (30 × 6) – 3

Answer:
b. 3 × (30 × 6)

Explanation:
1 panda eats 30 pounds of food each day.
for 3 days, 3 x 30 = 90
1 panda eats 90 pounds of food in 3 days.
6 pandas can eat 90 x 6 = 540 pounds.
3 + (180) = 183 not equal to 540 pounds.
3 x (30 x 6) = 3 x (180) = 540

Question 17.
Test Prep Which expression has a value of 6?
Options:
a. (6 ÷ 3) × 4 + 8
b. 27 – 9 ÷ 3 × ( 4 + 1)
c. (18 + 12) × 6 – 4
d. 71 – 5 × (9 + 4)

Answer:
d. 71 – 5 × (9 + 4)

Explanation:
(6 ÷ 3) × 4 + 8 = 2 x 4 + 8 = 8 + 8 = 16
27 – 9 ÷ 3 × ( 4 + 1) = 27 – (9 ÷ 3) × 5 = 27 – 3 x 5 = 27 – 15 = 12
(18 + 12) × 6 – 4 = 180 – 4 = 176
71 – 5 × (9 + 4) = 71 – (5 x 13) = 71 – 65 = 6

Grouping Symbols – Share and Show – Page No. 53

Evaluate the numerical expression.

Question 1.
12 + [(15 – 5) + (9 – 3)]
12 + [10 + ____]
12 +____
____

Answer:
12 + [(15 – 5) + (9 – 3)]
12 + [10 + 6]
12 + 16
28

Question 2.
5 × [(26 – 4) – (4 + 6)]
____

Answer:
5 × [(26 – 4) – (4 + 6)]
5 x [22 – 10]
5 x 12 = 60

Question 3.
36 ÷ [(18 – 10) – (8 – 6)]
____

Answer:
36 ÷ [(18 – 10) – (8 – 6)]
36 ÷ [8 – 2]
36 ÷ 6 = 6

On Your Own

Evaluate the numerical expression.

Question 4.
4 + [(16 – 4) + (12 – 9)]
____

Answer:
4 + [(16 – 4) + (12 – 9)]
4 + [12 + 3]
4 + 15 = 19

Question 5.
24 – [(10 – 7) + (16 – 9)]
____

Answer:
24 – [(10 – 7) + (16 – 9)]
24 – [3 + 7] = 24 – 10 = 14

Question 6.
16 ÷ [(13 + 7) – (12 + 4)]
____

Answer:
16 ÷ [(13 + 7) – (12 + 4)]
16 ÷ [20 – 16]
16 ÷ 4 = 4

Question 7.
5 × [(7 – 2) + (10 – 8)]
____

Answer:
5 × [(7 – 2) + (10 – 8)]
5 x [5 + 2]
5 x 7 = 35

Question 8.
[(17 + 8) + (29 – 12)] ÷ 6
____

Answer:
[(17 + 8) + (29 – 12)] ÷ 6
[25 + 17] ÷ 6
42 ÷ 6 = 7

Question 9.
[(6 × 7) + (3 × 4)] – 28
____

Answer:
[(6 × 7) + (3 × 4)] – 28
[42 + 12] – 28
54 – 28 = 26

Question 10.
3 × {[(12 – 8) × 2] + [(11 – 9) × 3]}
____

Answer:
3 × {[(12 – 8) × 2] + [(11 – 9) × 3]}
3 x {[4 x 2] + [2 x 3]}
3 x {8 + 6}
3 x 14 = 42

Question 11.
{[(3 × 4) + 18] + [(6 × 7) – 27]} ÷ 5
____

Answer:
{[(3 × 4) + 18] + [(6 × 7) – 27]} ÷ 5
{[12 + 18] + [42 – 27]} ÷ 5
{30 + 15} ÷ 5
45 ÷ 5 = 9

Grouping Symbols – UNLOCK the Problem – Page No. 54

Question 12.
Dan has a flower shop. Each day he displays 24 roses. He gives away 10 and sells the rest. Each day he displays 36 carnations. He gives away 12 and sells the
rest. What expression can you use to find out how many roses and carnations Dan sells in a week?
Go Math Grade 5 Answer Key Chapter 1 Place Value, Multiplication, and Expressions Place Value, Multiplication, and Expressions; Grouping Symbols img 21
a. What information are you given?
Type below:
__________

Answer:
Each day, Dan displays 24 roses (gives away 10 and sells the rest), and he displays 36 carnations (gives away 12 and selles the rest).

Question 12.
b. What are you being asked to do?
Type below:
__________

Answer:
We have to find out how many roses and carnations Dan sells in a week.

Question 12.
c. What expression shows how many roses Dan sells in one day?
Type below:
__________

Answer:
24 – 10 = 14 roses

Question 12.
d. What expression shows how many carnations Dan sells in one day?
Type below:
__________

Answer:
36 – 12 = 24 carnation

Question 12.
e. Write an expression to represent the total number of roses and carnations Dan sells in one day.
Type below:
__________

Answer:
(24 – 10) + (36 – 12) = 14 + 24 = 38 roses and carnations

Question 12.
f. Write the expression that shows how many roses and carnations Dan sells in a week.
Type below:
__________

Answer:
7 x ((24 – 10) + (36 – 12)) = 7 x (14 + 24) = 7 x 38 = 266 roses and carnations

Question 13.
Evaluate the expression to find out how many roses and carnations Dan sells in a week.
____ flowers

Answer:
7 x ((24 – 10) + (36 – 12))
7 x (14 + 24)
7 x 38 = 266 roses and carnations

Explanation:

Question 14.
Test Prep Which expression has a value of 4?
Options:
a. [(4 × 5) + (9 + 7)] + 9
b. [(4 × 5) + (9 + 7)] ÷ 9
c. [(4 × 5) – (9 + 7)] × 9
d. [(4 + 5) + (9 + 7)] – 9

Answer:
b. [(4 × 5) + (9 + 7)] ÷ 9

Explanation:
[(4 × 5) + (9 + 7)] + 9 = [20 + 16] + 9 = 36 + 9 = 45
[(4 × 5) + (9 + 7)] ÷ 9 = [20 + 16] ÷ 9 = 36 ÷ 9 = 4

Chapter Review/Test – Vocabulary – Page No. 55

Go Math Grade 5 Answer Key Chapter 1 Place Value, Multiplication, and Expressions Chapter Review/Test img 22

Question 1.
The ____ states that multiplying a sum by a number is the same as multiplying each addend in the sum by the number and then adding the products.
__________

Answer:
Distributive Property

Concepts and Skills

Complete the sentence.

Question 2.
7,000 is 10 times as much as _____

Answer:
700

Explanation:
Let the unknown number is S.
7,000 = 10S
S = 7,000/10 = 700.
7,000 is 10 times as much as 700.

Question 3.
50 is \(\frac{1}{10}\) of _____

Answer:
500

Explanation:
Let the unknown number is S.
50 = \(\frac{1}{10}\) S
S = 50 X 10 = 500

Complete the equation, and tell which property you used.

Question 4.
4 × (12 + 14) =
Type below:
________

Answer:
104; Distributive Property of Multiplication

Explanation:
4 × (12 + 14) = (4 x 12) + (4 x 14) = 48 + 56 = 104; Distributive Property of Multiplication

Question 5.
45 + 16 =__ + 45
Type below:
Type below:
________

Answer:
45 + 16 = 16 + 45; Commutative Property of Addition

Explanation:
45 + 16 = 16 + 45; Commutative Property of Addition

Find the value.

Question 6.
102 = _____

Answer:
100

Explanation:
100 = 1;
101 = 1 x 10 = 10;
102 = 10 x 10 = 100;

Question 7.
3 × 104 = _____

Answer:
30,000

Explanation:
3 × 104
100 = 1;
101 = 1 x 10 = 10;
102 = 10 x 10 = 100;
103 = 10 x 10 x 10 = 1,000;
104 = 10 x 10 x 10 x 10 = 10,000;
3 x 10,000 = 30,000

Question 8.
8 × 103 = _____

Answer:
8,000

Explanation:
8 × 103
100 = 1;
101 = 1 x 10 = 10;
102 = 10 x 10 = 100;
103 = 10 x 10 x 10 = 1,000;
8 x 1,000 = 8,000

Estimate. Then find the product.

Question 9.
Estimate:
5 7 9
×   6
———-
Estimate: _____ Product: _____

Answer:
Estimate: 3,600
Product: 3,474

Explanation:
Estimate: 579 is close to 600;
600 x 6 = 3,600
579 x 6
6 x 9 =54; add ones and regroup tens
6 x 7 = 42; 42 + 5 = 47; add tens and regroup hundreds
6 x 5 = 30; 30 + 4 = 34
So, 3,474 is the product

Question 10.
Estimate:
7,316
×    6
———-
Estimate: _____ Product: _____

Answer:
Estimate: 42,000
Product: 43,986

Explanation:
Estimate: 7,316 is close to 7,000;
7,000 x 6 = 42,000
7,316 x 6
6 x 6 = 36; add ones and regroup tens
6 x 1 = 6; 6 + 3 = 18; add tens and regroup hundreds
6 x 3 = 18; 18 + 1 = 19; add hundreds and regroup thousads
6 x 7 = 42; 42 + 1 = 43
So, 43,986 is the product

Question 11.
Estimate:
4 3 6
× 3 2
———-
Estimate: _____ Product: _____

Answer:
Estimate: 12,000
Product: 13,952

Explanation:
Estimate: 436 is close to 400; 32 is close to 30
400 x 30 = 12,000
436 x 32;
436 x 2 = 872
436 x 30 = 13,080
13,080 + 872 = 13,952

Use multiplication and the Distributive Property to find the quotient.

Question 12.
54 ÷ 3 = _____

Answer:
18

Explanation:
54 ÷ 3
(27 + 27) ÷ 3 = (27 ÷ 3) + (27 ÷ 3) = 9 + 9 = 18
3 x 18 = 3 x (9+ 9) = (3 x 9) + (3 x 9) = 27 + 27 = 54

Question 13.
90 ÷ 5 = _____

Answer:
18

Explanation:
90 ÷ 5
(45 + 45) ÷ 5 = (45 ÷ 5) + (45 ÷ 5) = 9 + 9 = 18
5 x 18 = 5 x (9+ 9) = (5 x 9) + (5 x 9) = 45 + 45 = 90

Question 14.
96 ÷ 6 = _____

Answer:

Explanation:
96 ÷ 6
(48 + 48) ÷ 6 = (48 ÷ 6) + (48 ÷ 6) = 8 + 8 = 16
6 x 16 = 6 x (8 + 8) = (6 x 8) + (6 x 8) = 48 + 48 = 96

Evaluate the numerical expression.

Question 15.
42 − (9 + 6) = _____

Answer:
42 − (9 + 6)
42 – 15
27

Question 16.
15 + (22 − 4) ÷ 6 = _____

Answer:
15 + (22 − 4) ÷ 6
15 + (18 ÷ 6)
15 + 3
18

Question 17.
6 × [(5 × 7) − (7 + 8)] = _____

Answer:
6 × [(5 × 7) − (7 + 8)]
6 x [35 – 15]
6 x [20]
120

Chapter Review/Test – Page No. 56

Fill in the bubble completely to show your answer.

Question 18.
Erica’s high score on her new video game is 30,000 points. Maria’s high score is \(\frac{1}{10}\) of Erica’s. How many points did Maria score?
Options:
A. 30
B. 300
C. 3,000
D. 30,000

Answer:
C. 3,000

Explanation:
Erica’s high score on her new video game is 30,000 points. Maria’s high score is \(\frac{1}{10}\) of Erica’s.
30,000 x \(\frac{1}{10}\) = 3,000

Question 19.
Rich makes $35 a week mowing lawns in his neighborhood. Which expression can be used to show how much money he makes in 8 weeks?
Options:
A. (8 × 30) + (8 × 5)
B. (8 × 30) + (8 × 5)
C. (8 + 30) × (8 + 5)
D. (8 × 30) × (8 × 5)

Answer:
B. (8 × 30) + (8 × 5)

Explanation:
Rich makes $35 a week mowing lawns in his neighborhood.
8 x $35 = 8 x (30 + 5) = (8 x 30) + (8 x 5)

Question 20.
Mr. Rodriguez bought a supply of 20 reams of printer paper. Each ream contains 500 sheets of paper. How many sheets of printer paper are there?
Options:
A. 1,000
B. 5,000
C. 10,000
D. 100,000

Answer:
C. 10,000

Explanation:
Mr. Rodriguez bought a supply of 20 reams of printer paper. Each ream contains 500 sheets of paper.
500 x 20 = 10,000

Question 21.
Harvester ants are common in the southwestern United States. A single harvester ant colony may have as many as 90,000 members. What is that number written as a whole number multiplied by a power of ten?
Options:
A. 9 × 104
B. 9 × 103
C. 9 × 102
D. 9 × 101

Answer:
A. 9 × 104

Explanation:
Harvester ants are common in the southwestern United States. A single harvester ant colony may have as many as 90,000 members.
90,000
100 = 1;
101 = 1 x 10 = 10;
102 = 10 x 10 = 100;
103 = 10 x 10 x 10 = 1,000;
104 = 10 x 10 x 10 x 10 = 10,000;
9 x 10,000 = 90,000 = 9 x 104

Chapter Review/Test – Page No. 57

Fill in the bubble completely to show your answer.

Question 22.
Megan used the following expression to find the quotient of a division problem.
(4 × 12) + (4 × 6)
What was the division problem and the quotient?
Options:
A. 24 ÷ 4 = 6
B. 48 ÷ 4 = 12
C. 64 ÷ 4 = 16
D. 72 ÷ 4 = 18

Answer:
D. 72 ÷ 4 = 18

Explanation:
Megan used (4 × 12) + (4 × 6)
4 x (12 + 6)
4 x 18 = 72
72 ÷ 4 = 18

Question 23.
It is 1,325 feet from Kinsey’s house to her school. Kinsey walks to school each morning and gets a ride home each afternoon. How many feet does Kinsey walk to school in 5 days?
Options:
A. 6,725 feet
B. 6,625 feet
C. 6,525 feet
D. 5,625 feet

Answer:
B. 6,625 feet

Explanation:
It is 1,325 feet from Kinsey’s house to her school. Kinsey walks to school each morning and gets a ride home each afternoon.
1,325 feet x 5 = 6,625 feet

Question 24.
An adult elephant eats about 300 pounds of food each day. Which expression shows about how many pounds of food a herd of 12 elephants eats in 5 days?
Options:
A. 5 + (300 × 12)
B. 5 × (300 × 12)
C. (300 × 12) ÷ 5
D. (300 × 12) − 5

Answer:
B. 5 × (300 × 12)

Explanation:
An adult elephant eats about 300 pounds of food each day.
(300 x 12)
In 5 days, 5 × (300 × 12)

Question 25.
Carla can type 265 characters a minute on her computer keyboard. At that rate, how many characters can she type in 15 minutes?
Options:
A. 2,975
B. 3,875
C. 3,905
D. 3,975

Answer:
D. 3,975

Explanation:
Carla can type 265 characters a minute on her computer keyboard.
In 15 minutes, 265 x 15 = 3,975

Chapter Review/Test – Page No. 58

Constructed Response

Question 26.
Donavan copied the problem below from the board. He missed one of the numbers needed to show his work. What number is missing in his work? Explain how you found the missing number
Go Math Grade 5 Answer Key Chapter 1 Place Value, Multiplication, and Expressions Chapter Review/Test img 23
17 × 5 = ( ______ +7) × 5
= ( ______ × 5) + (7 × 5)

Answer:
Missing number is 10. Using the Distributive Property, the sum of the two number within the () has to be equal to the other factor 17.
10 + 7 = 17
17 × 5 = ( 10 +7) × 5
= ( 10 × 5) + (7 × 5)

Performance Task

Question 27.
Drew’s weekly allowance is $8.00. His friend Jan’s weekly allowance is $10. Drew spends $3 a week and Jan spends $4 a week.
A. Write two expressions to show how much money each person has at the end of the week. Use parentheses.
Drew has __ .
Jan has __ .
Type below:
__________

Answer:
Drew has ($8 – $3) .
Jan has ($10 – $4).

Question 27.
B. Drew and Jan decide that they want to put their money together to buy a video game. Write an expression that shows how much they can save each week. Explain.
Type below:
__________

Answer:
(8 – 3) + (10 – 4); Add the amount of money Drew and Jan have at the end of the week. This equals the amount they can save a week.

Question 27.
C. The video game Drew and Jan want to buy costs $55. Write an expression to show how many weeks it will take them to save enough to buy the video game. Use parentheses and brackets in your expression. Then evaluate the expression.
_____ weeks

Answer:
55 ÷ [(8 – 3) +(10 – 4)]
55 ÷ [5 + 6]
55 ÷ 11
5
It will take them 5 weeks to save the money from the game.

Conclusion:

You need not look everywhere to seek assistance regarding Go Math Grade 5 Ch 1. Simply access Go Math Grade 5 Answer Key Chapter 1 Place Value, Multiplication, and Expressions and know how to approach while solving different problems on the related concepts.

Go Math Grade 5 Answer Key Chapter 5 Divide Decimals

go-math-grade-5-chapter-5-divide-decimals-answer-key

Looking everywhere for Go Math Grade 5 Answer Key Chapter 5 Divide Decimals? Yes, you have come the right way and we have curated the Go Math Grade 5 Answer Key. Access the Step by Step Solutions provided for the Questions from Chapter Test, Practice Test, Cumulative Practice. Refer to the HMH Go math Grade 5 Chapter 5 Divide Decimals Solution Key to cross-check your Answers. Practice all Grade 5 Go Math Chapter 5 Divide Decimals for free of cost and score well.

Go Math Grade 5 Chapter 5 Divide Decimals Answer Key

You can improve math proficiency by practicing with the Go Math Grade 5 Chapter 5 Answer Key on a regular basis. The Best Guide to Prepare Math is to refer to HMH Go Math Grade 5 Answer Key. Solve as many times as possible and try to cover all the topics in it via quick links out there. Simply click on the topics you want to prepare and check all the Problems in it.

Lesson 1: Algebra • Division Patterns with Decimals

Lesson 2: Investigate • Divide Decimals by Whole Numbers

Lesson 3: Estimate Quotients

Lesson 4: Division of Decimals by Whole Numbers

Mid-Chapter Checkpoint

Lesson 5: Investigate • Decimal Divisions

Lesson 6: Divide Decimals

Lesson 7: Write Zeros in the Dividend

Lesson 8: Problem Solving • Decimal Operations

Chapter 5 Review/Test

Share and Show – Page No. 203

Complete the pattern.

Question 1.
456 ÷ 100 = 456
456 ÷ 101 = 45.6
456 ÷ 102 = 4.56
Think: The dividend is being divided by an increasing power of 10, so the decimal point will move to the left one place for each increasing power of 10.
456 ÷ 103 = _____

Answer:
456 ÷ 103 = 0.456

Explanation:
The dividend is being divided by an increasing power of 10, so the decimal point will move to the left one place for each increasing power of 10.
456 ÷ 100 = 456
456 ÷ 101 = 45.6
456 ÷ 102 = 4.56
456 ÷ 103 = 0.456

Complete the pattern.

Question 2.
225 ÷ 100 = _____
225 ÷ 101 = _____
225 ÷ 102 = _____
225 ÷ 103 = _____

Answer:
225 ÷ 100 = 225
225 ÷ 101 = 22.5
225 ÷ 102 = 2.25
225 ÷ 103 = 0.25

Explanation:
The dividend is being divided by an increasing power of 10, so the decimal point will move to the left one place for each increasing power of 10.
225 ÷ 100 = 225/1 = 225
225 ÷ 101 = 225/10 = 22.5
225 ÷ 102 = 225/100 = 2.25
225 ÷ 103 = 225/1,000 = 0.25

Question 3.
605 ÷ 100 = _____
605 ÷ 101 = _____
605 ÷ 102 = _____
605 ÷ 103 = _____

Answer:
605 ÷ 100 = 605
605 ÷ 101 = 60.5
605 ÷ 102 = 6.05
605 ÷ 103 = 0.605

Explanation:
The dividend is being divided by an increasing power of 10, so the decimal point will move to the left one place for each increasing power of 10.
605 ÷ 100 = 605/1 = 605
605 ÷ 101 = 605/10 = 60.5
605 ÷ 102 = 605/100 = 6.05
605 ÷ 103 = 605/1,000 = 0.605

Question 4.
74.3 ÷ 1 = _____
74.3 ÷ 10 = _____
74.3 ÷ 100 = _____

Answer:
74.3 ÷ 1 = 74.3
74.3 ÷ 10 = 7.43
74.3 ÷ 100 = 0.743

Explanation:
The dividend is being divided by an increasing power of 10, so the decimal point will move to the left one place for each increasing power of 10.
74.3 ÷ 100 = 74.3 ÷ 1 = 74.3
74.3 ÷ 101 = 74.3 ÷ 10 = 7.43
74.3 ÷ 102 = 74.3 ÷ 100 = 0.743

On Your Own

Complete the pattern.

Question 5.
156 ÷ 1 = _____
156 ÷ 10 = _____
156 ÷ 100 = _____
156 ÷ 1,000 = _____

Answer:
156 ÷ 1 = 156
156 ÷ 10 = 15.6
156 ÷ 100 = 1.56
156 ÷ 1,000 = 0.156

Explanation:
The dividend is being divided by an increasing power of 10, so the decimal point will move to the left one place for each increasing power of 10.
156 ÷ 1 = 156
156 ÷ 10 = 15.6
156 ÷ 100 = 1.56
156 ÷ 1,000 = 0.156

Question 6.
32 ÷ 1 = _____
32 ÷ 10 = _____
32 ÷ 100 = _____
32 ÷ 1,000 = _____

Answer:
32 ÷ 1 = 32
32 ÷ 10 = 3.2
32 ÷ 100 = 0.32
32 ÷ 1,000 = 0.032

Explanation:
The dividend is being divided by an increasing power of 10, so the decimal point will move to the left one place for each increasing power of 10.
32 ÷ 1 = 32
32 ÷ 10 = 3.2
32 ÷ 100 = 0.32
32 ÷ 1,000 = 0.032

Question 7.
16 ÷ 100 = _____
16 ÷ 101 = _____
16 ÷ 102 = _____
16 ÷ 103 = _____

Answer:
16 ÷ 100 = 16
16 ÷ 101 = 1.6
16 ÷ 102 = 0.16
16 ÷ 103 = 0.016

Explanation:
The dividend is being divided by an increasing power of 10, so the decimal point will move to the left one place for each increasing power of 10.
16 ÷ 100 = 16
16 ÷ 101 = 1.6
16 ÷ 102 = 0.16
16 ÷ 103 = 0.016

Question 8.
12.7 ÷ 1 = _____
12.7 ÷ 10 = _____
12.7 ÷ 100 = _____
12.7 ÷ 1,000 = _____

Answer:
12.7 ÷ 1 = 12.7
12.7 ÷ 10 = 1.27
12.7 ÷ 100 = 0.127
12.7 ÷ 1,000 = 0.0127

Explanation:
The dividend is being divided by an increasing power of 10, so the decimal point will move to the left one place for each increasing power of 10.
12.7 ÷ 1 = 12.7
12.7 ÷ 10 = 1.27
12.7 ÷ 100 = 0.127
12.7 ÷ 1,000 = 0.0127

Question 9.
92.5 ÷ 100 = _____
92.5 ÷ 101 = _____
92.5 ÷ 102 = _____
92.5 ÷ 103 = _____

Answer:
92.5 ÷ 100 = 92.5
92.5 ÷ 101 = 9.25
92.5 ÷ 102 = 0.925
92.5 ÷ 103 = 0.0925

Explanation:
The dividend is being divided by an increasing power of 10, so the decimal point will move to the left one place for each increasing power of 10.
92.5 ÷ 100 = 92.5
92.5 ÷ 101 = 9.25
92.5 ÷ 102 = 0.925
92.5 ÷ 103 = 0.0925

Question 10.
86.3 ÷ 100 = _____
86.3 ÷ 101 = _____
86.3 ÷ 102 = _____
86.3 ÷ 103 = _____

Answer:
86.3 ÷ 100 = 86.3
86.3 ÷ 101 = 8.63
86.3 ÷ 102 = 0.863
86.3 ÷ 103 = 0.0863

Explanation:
The dividend is being divided by an increasing power of 10, so the decimal point will move to the left one place for each increasing power of 10.
86.3 ÷ 100 = 86.3
86.3 ÷ 101 = 8.63
86.3 ÷ 102 = 0.863
86.3 ÷ 103 = 0.0863

Algebra Find the value of n.

Question 11.
268 ÷ n = 0.268
n = _____

Answer:
n = 1000

Explanation:
268 ÷ n = 0.268
268 = n x 0.268
n = 268 ÷ 0.268
n = 1000

Question 12.
n ÷ 102 = 0.123
n = _____

Answer:
n = 12.3

Explanation:
n ÷ 102 = 0.123
n = 0.123 x 102
n = 0.123 x 100
n = 12.3

Question 13.
n ÷ 101 = 4.6
n = _____

Answer:
n = 46

Explanation:
n ÷ 101 = 4.6
n = 4.6 x 101
n = 4.6 x 10
n = 46

Problem Solving – Page No. 204

Use the table to solve 14–16.
Go Math Grade 5 Answer Key Chapter 5 Divide Decimals img 1

Question 14.
If each muffin contains the same amount of cornmeal, how many kilograms of cornmeal are in each corn muffin?
_____ kilograms

Answer:
0.15 kilograms

Explanation:
There are 1,000 muffins. Cornmeal = 150 Kg
If each muffin contains the same amount of cornmeal, 150 ÷ 1000 = 0.15
0.15 kilograms of cornmeal is in each corn muffin

Question 15.
If each muffin contains the same amount of sugar, how many kilograms of sugar, to the nearest thousandth, are in each corn muffin?
_____ kilograms

Answer:
0.07 kilograms

Explanation:
There are 1,000 muffins. Sugar = 66.7 kilograms
If each muffin contains the same amount of sugar, 66.7 ÷ 1000 = 0.0667.
0.0667 kilograms of sugar is in each corn muffin.
The thousandth digit is 6. 6 > 5
So, 0.07

Question 16.
The bakery decides to make only 100 corn muffins on Tuesday. How many kilograms of sugar will be needed?
_____ kilograms

Answer:
0.007 kilograms

Explanation:
The bakery decides to make only 100 corn muffins on Tuesday.
As 0.07 kilograms required for 1,000 muffins,
for 100 muffins, (100 x 0.07) ÷ 1000 = 0.007

Question 17.
Explain how you know that the quotient 47.3 ÷ 101 is equal to the product 47.3 × 0.1.
Type below:
_________

Answer:
Quotient 47.3 ÷ 101 = 47.3 ÷ 10 = 4.73. The power of 101 = 10.
47.3 × 0.1 = 4.73.
By dividing 10 to a number is equal to the multiplying 0.1 to that number.

Question 18.
Test Prep Ella used 37.2 pounds of apples to make applesauce. She used one-tenth as many pounds of sugar as pounds of apples. How many pounds of sugar did Ella use?
Options:
a. 372 pounds
b. 3.72 pounds
c. 0.372 pound
d. 0.0372 pound

Answer:
b. 3.72 pounds

Explanation:
Ella used 37.2 pounds of apples to make applesauce. She used one-tenth as many pounds of sugar as pounds of apples.
37.2 ÷ 10 = 3.72 pounds

Share and Show – Page No. 207

Use the model to complete the number sentence.

Question 1.
1.6 ÷ 4 =
Go Math Grade 5 Answer Key Chapter 5 Divide Decimals img 2
_____

Answer:
1.6 ÷ 4  = 0.4

Explanation:
1.6 ÷ 4
Share your model among 4 equal groups.
Since 1 whole cannot be shared among 4 groups without regrouping, cut your model apart to show the tenths.
1 ones = 10 tenths
10 + 6 = 16 tenths
There are 16 tenths in 1.6.
Share the tenths equally among the 4 groups.
There are 0 ones and 16 tenths in each group.
Decimal for the amount in each group = 0.4
1.6 ÷ 4  = 0.4

Question 2.
3.42 ÷ 3 =
Go Math Grade 5 Answer Key Chapter 5 Divide Decimals img 3
_____

Answer:
3.42 ÷ 3 = 1.14

Explanation:
3.42 ÷ 3
Share your model among 3 equal groups.
1 whole in each group. So, 3 wholes shared equally in 3 groups. 1 ones
3 ÷ 3 = 1 ones
3 tenths shared equally in 3 groups. 1 tenth has remained. 1 tenth
3 ÷ 3 = 1 tenths
1 tenth = 10 hundredths.
10 + 2 = 12 hundredths.
Share 12 hundredths equally among the 3 groups.
12 hundredths ÷ 3 = 4 hundredths.
Decimal for the amount in each group = 1.14
3.42 ÷ 3 = 1.14

Divide. Use base-ten blocks.

Question 3.
1.8 ÷ 3 = _____

Answer:
1.8 ÷ 3 = 0.6

Explanation:
1.8 ÷ 3
Share your model among 3 equal groups.
Since 1 whole cannot be shared among 3 groups without regrouping, cut your model apart to show the tenths. 0 ones
1 ones = 10 tenths
10 + 8 = 18 tenths
There are 18 tenths in 1.8.
Share the 18 tenths equally among the 3 groups.
18 ÷ 3 = 6
There are 0 ones and 18 tenths in each group.
Decimal for the amount in each group = 0.6
1.8 ÷ 3 = 0.6

Question 4.
3.6 ÷ 4 = _____

Answer:
3.6 ÷ 4 = 0.9

Explanation:
3.6 ÷ 4
Share your model among 4 equal groups.
Since 3 whole cannot be shared among 4 groups without regrouping, cut your model apart to show the tenths. 0 ones
1 ones = 10 tenths
30 + 6 = 36 tenths
There are 36 tenths in 3.6.
Share the 36 tenths equally among the 4 groups.
There are 0 ones and 36 tenths in each group.
36 ÷ 4 = 9
Decimal for the amount in each group = 0.9
3.6 ÷ 4 = 0.9

Question 5.
2.5 ÷ 5 = _____

Answer:
2.5 ÷ 5 = 0.5

Explanation:
2.5 ÷ 5
Share your model among 5 equal groups.
Since 2 whole cannot be shared among 5 groups without regrouping, cut your model apart to show the tenths. 0 ones
1 ones = 10 tenths
20 + 5 = 25 tenths
There are 25 tenths in 2.5.
Share the 25 tenths equally among the 5 groups.
There are 0 ones and 25 tenths in each group.
25 ÷ 5 = 5
Decimal for the amount in each group = 0.5
2.5 ÷ 5 = 0.5

Question 6.
2.4 ÷ 8 = _____

Answer:
2.4 ÷ 8 = 0.3

Explanation:
2.4 ÷ 8
Share your model among 8 equal groups.
Since 2 whole cannot be shared among 8 groups without regrouping, cut your model apart to show the tenths. 0 ones
1 ones = 10 tenths
20 + 4 = 24 tenths
There are 24 tenths in 2.4.
Share the 24 tenths equally among the 8 groups.
There are 0 ones and 24 tenths in each group.
24 ÷ 8 = 3
Decimal for the amount in each group = 0.3
2.4 ÷ 8 = 0.3

Question 7.
3.78 ÷ 3 = _____

Answer:
3.78 ÷ 3 = 1.26

Explanation:
3.78 ÷ 3
Share your model among 3 equal groups.
1 whole in each group. So, 3 wholes shared equally in 3 groups.
3 ÷ 3 = 1 ones
6 tenths shared equally in 3 groups. 1 tenth has remained.
6 ÷ 3 = 2 tenths
1 tenth = 10 hundredths.
10 + 8 = 18 hundredths.
Share 18 hundredths equally among the 3 groups.
18 hundredths ÷ 3 = 6 hundredths.
Decimal for the amount in each group = 1.26
3.78 ÷ 3 = 1.26

Question 8.
1.33 ÷ 7 = _____

Answer:
1.33 ÷ 7 = 0.19

Explanation:
1.33 ÷ 7
Share your model among 7 equal groups.
Since 1 whole cannot be shared among 7 groups without regrouping, cut your model apart to show the tenths. 0 ones
1 ones = 10 tenths
10 + 3 = 13 tenths
There are 13 tenths in 1.3.
Share the 13 tenths equally among the 7 groups.
Share 7 tenths equally among the 3 groups. 6 tenths remained.
7 ÷ 7 = 1 tenths
Since 6 tenths cannot be shared among 7 groups without regrouping, cut your model apart to show the tenths.
1 tenths = 10 hundredths
60 + 3 = 63 hundredths
Share 63 hundredths equally among the 7 groups.
63 ÷ 7 = 9 hundredths
Decimal for the amount in each group = 0.19
1.33 ÷ 7 = 0.19

Question 9.
4.72 ÷ 4 = _____

Answer:
4.72 ÷ 4 = 1.18

Explanation:
4.72 ÷ 4
Share your model among 4 equal groups.
1 whole in each group. So, 4 wholes shared equally in 4 groups. 1 ones
4 ÷ 4 = 1 ones
4 tenths shared equally in 4 groups. 3 tenths have remained.
4 ÷ 4 = 1 tenths
1 tenth = 10 hundredths.
30 + 2 = 32 hundredths.
Share 32 hundredths equally among the 4 groups.
32 hundredths ÷ 4 = 8 hundredths.
Decimal for the amount in each group = 1.18
4.72 ÷ 4 = 1.18

Question 10.
2.52 ÷ 9 = _____

Answer:
2.52 ÷ 9 = 0.28

Explanation:
2.52 ÷ 9
Share your model among 9 equal groups.
Since 2 whole cannot be shared among 9 groups without regrouping, cut your model apart to show the tenths.
9 ÷ 9 = 1 ones
1 ones = 10 tenths
20 + 5 = 25 tenths
There are 25 tenths in 2.5.
Share the 18 tenths equally among the 9 groups. 7 tenths remained.
18 ÷ 9 = 2 tenths
1 tenth = 10 hundredths.
70 + 2 hundredths = 72 hundredths
Share the 72 hundredths equally among the 9 groups.
72 ÷ 9 = 8
Decimal for the amount in each group = 0.28
2.52 ÷ 9 = 0.28

Question 11.
6.25 ÷ 5 = _____

Answer:
6.25 ÷ 5 = 1.25

Explanation:
6.25 ÷ 5
Share your model among 5 equal groups.
1 whole in each group. So, 5 wholes shared equally in 5 groups. 1 whole remained.
5 ÷ 5 = 1 ones 
1 ones = 10 tenths
10 + 2 = 12 tenths
10 tenths shared equally in 5 groups. 2 tenths have remained.
10 ÷ 5 = 2 tenths
1 tenth = 10 hundredths.
20 + 5 = 25 hundredths.
Share 25 hundredths equally among the 5 groups.
25 hundredths ÷ 5 = 5 hundredths.
Decimal for the amount in each group = 1.25
6.25 ÷ 5 = 1.25

Problem Solving – Page No. 208

What’s the Error?

Question 12.
Aida is making banners from a roll of paper that is 4.05 meters long. She will cut the paper into 3 equal lengths. How long will each banner be?
Go Math Grade 5 Answer Key Chapter 5 Divide Decimals img 4
Look how Aida solved the problem.      Solve the problem and correct
Find the error.                                            the error.
Go Math Grade 5 Answer Key Chapter 5 Divide Decimals img 5
So, Aida said that each banner would be _________ meters long, but each banner should be _________ meters long.
Type below:
_________

Answer:
So, Aida said that each banner would be 4.05 meters long, but each banner should be 1.35 meters long.
So, 1 ones, 3 tenths, and 5 hundredths are shared among 3 groups.
But Aida draws only one whole and 5 hundredths among 3 groups.

Explanation:
Aida is making banners from a roll of paper that is 4.05 meters long. She will cut the paper into 3 equal lengths.
4.05 ÷ 3
Share your model among 3 equal groups.
1 whole in each group. So, 3 wholes shared equally in 5 groups. 1 whole remained.
3 ÷ 3 = 1 ones 
1 ones = 10 tenths
10 + 0 = 10 tenths
9 tenths shared equally in 3 groups. 1 tenth remained.
9 ÷ 3 = 3 tenths
1 tenth = 10 hundredths.
10 + 5 = 15 hundredths.
Share 6 hundredths equally among the 3 groups.
15 hundredths ÷ 3 = 5 hundredths.
Decimal for the amount in each group = 1.35
4.05 ÷ 3 = 1.35

Question 12.
• Describe Aida’s error.
Type below:
_________

Answer:
So, Aida said that each banner would be 4.05 meters long, but each banner should be 1.35 meters long.
So, 1 ones, 3 tenths, and 5 hundredths are shared among 3 groups.
But Aida draws only one whole and 5 hundredths among 3 groups.

Question 12.
• What if the roll of paper were 4.35 meters long? How long would each banner be?
Type below:
_________

Answer:
1.45 meters long

Explanation:
If the roll paper were 4.35 meters long,
4.35 ÷ 3
Share your model among 3 equal groups.
1 whole in each group. So, 3 wholes shared equally in 5 groups. 1 whole remained.
3 ÷ 3 = 1 ones 
1 ones = 10 tenths
10 + 3 = 13 tenths
12 tenths shared equally in 3 groups. 1 tenth remained.
12 ÷ 3 = 4 tenths
1 tenth = 10 hundredths.
10 + 5 = 15 hundredths.
Share 6 hundredths equally among the 3 groups.
15 hundredths ÷ 3 = 5 hundredths.
Decimal for the amount in each group = 1.45
4.35 ÷ 3 = 1.45

Share and Show – Page No. 211

Use compatible numbers to estimate the quotient.

Question 1.
28.8 ÷ 9
Type below:
_________

Answer:
About to 3

Explanation:
28.8 is closer to 30
9 is closer to 10
30 ÷ 10 = 3

Question 2.
393.5 ÷ 41
_____ ÷ _____ = _____

Answer:
400 ÷ 40 = 10

Explanation:
Use a whole number greater than the dividend.
393.5 is closer to 400
41 is closer to 40
400 ÷ 40 = 10
Use a whole number less than the dividend.
393.5 is closer to 360
41 is closer to 40
360 ÷ 40 = 9
The quotient should be between 9 and 10.
393.5 is closer to 400.
So, the answer is about 10

Estimate the quotient.

Question 3.
161.7 ÷ 7
_____

Answer:
About 23

Explanation:
161.7 is closer to 161
161 ÷ 7 = 23

Question 4.
17.9 ÷ 9
_____

Answer:
About 2

Explanation:
17.9 is closer to 18
18 ÷ 9 = 2

Question 5.
145.4 ÷ 21
_____

Answer:
about 7

Explanation:
Use a whole number greater than the dividend.
145.4 is closer to 160
21 is closer to 20
160 ÷ 20 = 8
Use a whole number less than the dividend.
145.4 is closer to 140
21 is closer to 20
140 ÷ 20 = 7
The quotient should be between 7 and 8.
145.4 is closer to 140.
So, the answer is about 7

On Your Own

Estimate the quotient.

Question 6.
15.5 ÷ 4
_____

Answer:
About 4

Explanation:
15.5 is closer to 16
16 ÷ 4 = 4

Question 7.
394.8 ÷ 7
_____

Answer:
About 60

Explanation:
394.8 is closer to 420
420 ÷ 7 = 60
About 60

Question 8.
410.5 ÷ 18
_____

Answer:
About 22

Explanation:
410.5 is closer to 396
396 ÷ 18 = 22

Question 9.
72.1 ÷ 7
_____

Answer:
About 10

Explanation:
72.1 is closer to 70
70 ÷ 7 = 10

Question 10.
32.4 ÷ 52
_____

Answer:
About 1

Explanation:
32.4 is closer to 52
52 ÷ 52 = 1

Question 11.
$134.42 ÷ 28
$ _____

Answer:
about 4

Explanation:
Use a whole number greater than the dividend.
$134.42 is closer to 150
28 is closer to 30
150 ÷ 30 = 5
Use a whole number less than the dividend.
$134.42 is closer to 120
28 is closer to 30
120 ÷ 30 = 4
The quotient should be between 4 and 5.
$134.42 is closer to $120.
So, the answer is about 4

Question 12.
21.8 ÷ 4
_____

Answer:
About 5

Explanation:
21.8 is closer to 20
20 ÷ 4 = 5

Question 13.
3.4 ÷ 5
_____

Answer:
About 1

Explanation:
3.4 is closer to 5
5 ÷ 5 = 1
About 1

Question 14.
$759.92 ÷ 42
$ _____

Answer:
About $18

Explanation:
$759.92 is closer to $756
$756 ÷ 42 = 18
About 18

Question 15.
157.5 ÷ 38
_____

Answer:
About 4

Explanation:
157.5 is closer to 152
152 ÷ 38 = 4

Question 16.
379.2 ÷ 6
_____

Answer:
About 63

Explanation:
379.2 is closer to 378
378 ÷ 6 = 63

Question 17.
108.4 ÷ 21
_____

Answer:
About 5

Explanation:
108.4 is closer to 100
21 is closer to 20
100 ÷ 20 = 5

Problem Solving – Page No. 212

Use the table to solve 18–20.
Go Math Grade 5 Answer Key Chapter 5 Divide Decimals img 6

Question 18.
Estimate the average daily snowfall for Alaska’s greatest 7-day snowfall.
_____ inches

Answer:
26 inches

Explanation:
Alaska = 186.9 inches snowfall for 7 days.
For 1 day, 186.9 ÷ 7
186.9 is closer to 182
1825 ÷ 7 = 26 inches

Question 19.
How does the estimate of the average daily snowfall for Wyoming’s greatest 7-day snowfall compare to the estimate of the average daily snowfall for South Dakota’s greatest 7-day snowfall?
Type below:
_________

Answer:
South Dakota’s snowfall is greater than Wyoming’s snowfall.

Explanation:
Wyoming’s = 84.5 inches snowfall for 7 days.
For 1 day, 84.5 ÷ 7
84.5 is closer to 84
84 ÷ 7 = 12 inches
South Dakota’s = 112.7 inches snowfall for 7 days.
For 1 day, 112.7 ÷ 7
112.7 is closer to 112
112 ÷ 7 = 16 inches

Question 20.
The greatest monthly snowfall total in Alaska is 297.9 inches. This happened in February, 1953. Compare the daily average snowfall for February, 1953, with the average daily snowfall for Alaska’s greatest 7-day snowfall. Use estimation.
Type below:
_________

Answer:
The average snowfall of Alaska in February, 1953 is greater than snowfall for Alaska’s greatest 7-day snowfall.

Explanation:
Alaska = 186.9 inches snowfall for 7 days.
For 1 day, 186.9 ÷ 7
186.9 is closer to 182
1825 ÷ 7 = 26 inches
Alaska = 297.9 inches snowfall for 7 days.
For 1 day, 297.9 ÷ 7
297.9 is closer to 294
294 ÷ 7 = 42 inches

Question 21.
What’s the Error? During a 3-hour storm, it snowed 2.5 inches. Jacob said that it snowed an average of about 8 inches per hour.
Type below:
_________

Answer:
During a 3-hour storm, it snowed 2.5 inches.
For 1 hour, 2.5 ÷ 3 = 0.833
Jacob said that it snowed an average of about 8 inches per hour. He missed the remained inches.

Question 22.
Test Prep A plant grew 23.8 inches over 8 weeks. Which is the best estimate of the average number of inches the plant grew each week?
Options:
a. 0.2 inch
b. 0.3 inch
c. 2 inch
d. 3 inch

Answer:
d. 3 inch

Explanation:
A plant grew 23.8 inches over 8 weeks.
23.8 is closer to 24
24 ÷ 8 = 3 inch

Share and Show – Page No. 215

Write the quotient with the decimal point placed correctly.

Question 1.
4.92 ÷ 2 = 246
_____

Answer:
2.46

Explanation:
Divide. 4 ones ÷ 2
Multiply. 2 × 2 ones = 4 ones. Subtract. 4 ones − 4 ones = 0
Check. 4 ones shared among 2 groups as 2 wholes.
Divide. 9 tenths ÷ 2
Multiply. 2 × 4 tenths = 8 tenths. Subtract. 9 tenths − 8 tenths = 1 tenths
Check. 8 tenths shared among 2 groups as 4 tenths. 1 tenth remained.
1 tenth = 10 hundredths.
10 + 2 = 12 hundredths
Divide. 12 hundredths ÷ 2
Multiply. 2 × 6 hundredths = 12 hundredths. Subtract. 12 hundredths − 12 hundredths = 0 hundredths
Check. 12 hundredths shared among 2 groups as 6 hundredths.
So, 2.46

Question 2.
50.16 ÷ 38 = 132
_____

Answer:
1.32

Explanation:
Divide. 50 ones ÷ 38
Multiply. 38 × 1 ones = 38 ones. Subtract. 50 ones − 38 ones = 12 ones
Check. 38 ones shared among 38 groups as 1 whole.
12 one(s) cannot be shared among 38 groups without regrouping.
120 + 1 = 121 tenths
Divide. 121 tenths ÷ 38
Multiply. 38 x 3 tenths = 114 tenths. Subtract. 121 tenths − 114 tenths = 7 tenths
Check. 114 tenths shared among 38 groups as 3 tenths. 7 tenths remained.
1 tenth = 10 hundredths.
70 + 6 = 76 hundredths
Divide. 76 hundredths ÷ 38
Multiply. 38 x 2 hundredths = 76 hundredths. Subtract. 76 hundredths − 76 hundredths = 0 hundredths
Check. 76 hundredths shared among 38 groups as 2 hundredths.
So, 1.32

Divide.

Question 3.
5)\(\overline { 8.65 }\)
_____

Answer:
1.73

Explanation:
8.65 ÷ 5
Divide. 8 ones ÷ 5
Multiply. 5 × 1 ones = 5ones. Subtract. 8 ones − 5 ones = 3 ones
Check. 5 ones shared among 5 groups as 1 whole.
3 one(s) cannot be shared among 5 groups without regrouping.
30 + 6 = 36 tenths
Divide. 36 tenths ÷ 5
Multiply. 5 x 7 tenths = 35 tenths. Subtract. 36 tenths − 35 tenths = 7 tenths
Check. 35 tenths shared among 5 groups as 7 tenths.
1 tenths(s) cannot be shared among 5 groups without regrouping.
1 tenth = 10 hundredths.
10 + 5 = 15 hundredths
Divide. 15 hundredths ÷ 5
Multiply. 5 x 3 hundredths = 15 hundredths. Subtract. 15 hundredths − 15 hundredths = 0 hundredths
Check. 15 hundredths shared among 5 groups as 3 hundredths.
So, 1.73

Question 4.
3)\(\overline { 2.52 }\)
_____

Answer:
0.84

Explanation:
2.52 ÷ 3
2 one(s) cannot be shared among 3 groups without regrouping.
20 + 5 = 25 tenths
Divide. 25 tenths ÷ 3
Multiply. 3 x 8 tenths = 24 tenths. Subtract. 25 tenths − 24 tenths = 1 tenths
Check. 24 tenths shared among 3 groups as 8 tenths.
1 tenths(s) cannot be shared among 3 groups without regrouping.
1 tenth = 10 hundredths.
10 + 2 = 12 hundredths
Divide. 12 hundredths ÷ 3
Multiply. 3 x 4 hundredths = 12 hundredths. Subtract. 12 hundredths − 12 hundredths = 0 hundredths
Check. 12 hundredths shared among 3 groups as 4 hundredths.
So, 0.84

Question 5.
27)\(\overline { 97.2 }\)
_____

Answer:
3.6

Explanation:
97.2 ÷ 27
Divide. 97 ones ÷ 27
Multiply. 27 x 3 ones = 81 ones. Subtract. 97 ones − 81 ones = 16 ones
Check. 81 ones shared among 27 groups as 3 whole.
16 one(s) cannot be shared among 27 groups without regrouping.
160 + 2 = 162 tenths
Divide. 162 tenths ÷ 27
Multiply. 27 x 6 tenths = 162 tenths. Subtract. 162 tenths − 162 tenths = 0 tenths
Check. 162 tenths shared among 27 groups as 6 tenths.
So, 3.6

On Your Own

Divide.

Question 6.
6)\(\overline { 8.94 }\)
_____

Answer:
1.49

Explanation:
8.94 ÷ 6
Divide. 8 ones ÷ 6
Multiply. 6 × 1 ones = 6 ones. Subtract. 8 ones − 6 ones = 2 ones
Check. 6 ones shared among 6 groups as 1 whole.
2 one(s) cannot be shared among 6 groups without regrouping.
20 + 9 = 29 tenths
Divide. 29 tenths ÷ 6
Multiply. 6 x 4 tenths = 24 tenths. Subtract. 29 tenths − 24 tenths = 5 tenths
Check. 24 tenths shared among 6 groups as 4 tenths.
5 tenths(s) cannot be shared among 6 groups without regrouping.
1 tenth = 10 hundredths.
50 + 4 = 54 hundredths
Divide. 54 hundredths ÷ 6
Multiply. 6 x 9 hundredths = 54 hundredths. Subtract. 54 hundredths − 54 hundredths = 0 hundredths
Check. 54 hundredths shared among 6 groups as 54 hundredths.
So, 1.49

Question 7.
5)\(\overline { 3.75 }\)
_____

Answer:
0.75

Explanation:
3.75 ÷ 5
3 one(s) cannot be shared among 5 groups without regrouping.
30 + 7 = 37 tenths
Divide. 37 tenths ÷ 5
Multiply. 5 x 7 tenths = 35 tenths. Subtract. 37 tenths − 35 tenths = 2 tenths
Check. 35 tenths shared among 5 groups as 7 tenths.
2 tenths(s) cannot be shared among 5 groups without regrouping.
1 tenth = 10 hundredths.
20 + 5 = 25 hundredths
Divide. 25 hundredths ÷ 5
Multiply. 5 x 5 hundredths = 25 hundredths. Subtract. 25 hundredths − 25 hundredths = 0 hundredths
Check. 25 hundredths shared among 5 groups as 25 hundredths.
So, 0.75

Question 8.
19)\(\overline { 55.1 }\)
_____

Answer:
2.9

Explanation:
55.1 ÷ 19
Divide. 55 ones ÷ 19
Multiply. 19 × 2 ones = 38 ones. Subtract. 55 ones − 38 ones = 17 ones
Check. 38 ones shared among 19 groups as 2 whole.
17 one(s) cannot be shared among 19 groups without regrouping.
170 + 1 = 171 tenths
Divide. 171 tenths ÷ 19
Multiply. 19 x 9 tenths = 171 tenths. Subtract. 171 tenths − 171 tenths = 0 tenths
Check. 171 tenths shared among 19 groups as 9 tenths.
So, 2.9

Question 9.
23)\(\overline { 52.9 }\)
_____

Answer:
2.3

Explanation:
52.9 ÷ 23
Divide. 52 ones ÷ 23
Multiply. 23 × 2 ones = 46 ones. Subtract. 52 ones − 46 ones = 6 ones
Check. 46 ones shared among 23 groups as 2 whole.
6 one(s) cannot be shared among 23 groups without regrouping.
60 + 9 = 69 tenths
Divide. 69 tenths ÷ 23
Multiply. 23 x 3 tenths = 69 tenths. Subtract. 69 tenths − 69 tenths = 0 tenths
Check. 69 tenths shared among 23 groups as 3 tenths.
So, 2.3

Question 10.
8)\(\overline { $8.24 }\)
$ _____

Answer:

Explanation:
8.24 ÷ 8
Divide. 8 ones ÷ 8
Multiply. 8 × 1 ones = 8 ones. Subtract. 8 ones − 8 ones = 0 ones
Check. 8 ones shared among 8 groups as 1 whole.
2 tenths(s) cannot be shared among 8 groups without regrouping.
1 tenth = 10 hundredths.
20 + 4 = 24 hundredths
Divide. 24 hundredths ÷ 8
Multiply. 8 x 3 hundredths = 24 hundredths. Subtract. 24 hundredths − 24 hundredths = 0 hundredths
Check. 24 hundredths shared among 8 groups as 3 hundredths.
So, 1.03

Question 11.
5)\(\overline { 44.5 }\)
_____

Answer:

Explanation:
44.5 ÷ 5
Divide. 44 ones ÷ 5
Multiply. 5 x 8 ones = 40 ones. Subtract. 44 ones − 40 ones = 4 ones
Check. 40 ones shared among 5 groups as 8 wholes.
4 one(s) cannot be shared among 5 groups without regrouping.
40 + 5 = 45 tenths
Divide. 45 tenths ÷ 5
Multiply. 5 x 9 tenths = 45 tenths. Subtract. 45 tenths − 45 tenths = 0 tenths
Check. 45 tenths shared among 5 groups as 9 tenths.
So, 8.9

Practice: Copy and Solve Divide.

Question 12.
3)\(\overline { $7.71 }\)
$_____

Answer:
2.57

Explanation:
7.71 ÷ 3
Divide. 7 ones ÷ 3
Multiply. 3 x 2 ones = 6 ones. Subtract. 7 ones − 6 ones = 1 ones
Check. 6 ones shared among 3 groups as 2 wholes.
1 one(s) cannot be shared among 3 groups without regrouping.
10 + 7 = 17 tenths
Divide. 17 tenths ÷ 3
Multiply. 3 x 5 tenths = 15 tenths. Subtract. 17 tenths − 15 tenths = 2 tenths
Check. 15 tenths shared among 3 groups as 5 tenths.
2 tenths(s) cannot be shared among 3 groups without regrouping.
1 tenth = 10 hundredths.
20 + 1 = 21 hundredths
Divide. 21 hundredths ÷ 3
Multiply. 3 x 7 hundredths = 21 hundredths. Subtract. 21 hundredths − 21 hundredths = 0 hundredths
Check. 21 hundredths shared among 3 groups as 7 hundredths.
So, 2.57

Question 13.
14)\(\overline { 79.8 }\)
_____

Answer:
5.7

Explanation:
79.8 ÷ 14
Divide. 79 ones ÷ 14
Multiply. 14 x 5 ones = 70 ones. Subtract. 79 ones − 70 ones = 9 ones
Check. 70 ones shared among 14 groups as 5 wholes.
9 one(s) cannot be shared among 14 groups without regrouping.
90 + 8 = 98 tenths
Divide. 98 tenths ÷ 14
Multiply. 14 x 7 tenths = 98 tenths. Subtract. 98 tenths − 98 tenths = 0 tenths
Check. 98 tenths shared among 14 groups as 7 tenths.
So, 5.7

Question 14.
33)\(\overline { 25.41 }\)
_____

Answer:
0.77

Explanation:
25.41 ÷ 33
Divide. 25 ones ÷ 33
25 one(s) cannot be shared among 33 groups without regrouping.
250 + 4 = 254 tenths
Divide. 254 tenths ÷ 33
Multiply. 33 x 7 tenths = 231 tenths. Subtract. 254 tenths − 231 tenths = 23 tenths
Check. 231 tenths shared among 33 groups as 7 tenths.
23 tenths(s) cannot be shared among 33 groups without regrouping.
1 tenth = 10 hundredths.
230 + 1 = 231 hundredths
Divide. 231 hundredths ÷ 33
Multiply. 33 x 7 hundredths= 231 hundredths. Subtract. 231 hundredths− 231 hundredths = 0 hundredths
Check. 231 tenths shared among 33 groups as 7 tenths.
So, 0.77

Question 15.
7)\(\overline { 15.61 }\)
_____

Answer:
2.23

Explanation:
15.61 ÷ 7
Divide. 15 ones ÷ 7
Multiply. 7 x 2 ones =  14 ones. Subtract. 15 ones − 14 ones = 1 ones
Check. 14 ones shared among 7 groups as 2 wholes.
1 one(s) cannot be shared among 7 groups without regrouping.
10 + 6 = 16 tenths
Divide. 16 tenths ÷ 7
Multiply. 7 x 2 tenths = 14 tenths. Subtract. 16 tenths − 14 tenths = 2 tenths
Check. 14 tenths shared among 7 groups as 2 tenths.
2 tenths(s) cannot be shared among 7 groups without regrouping.
1 tenth = 10 hundredths.
20 + 1 = 21 hundredths
Divide. 21 hundredths ÷ 7
Multiply. 7 x 3 hundredths = 21 hundredths. Subtract. 21 hundredths − 21 hundredths = 0 hundredths
Check. 21 hundredths shared among 7 groups as 3 hundredths.
So, 2.23

Question 16.
14)\(\overline { 137.2 }\)
_____

Answer:
9.8

Explanation:
137.2 ÷ 14
Divide. 137 ones ÷ 14
Multiply. 14 x 9 ones =  126 ones. Subtract. 137 ones − 126 ones = 11 ones
Check. 126 ones shared among 14 groups as 9 wholes.
11 one(s) cannot be shared among 14 groups without regrouping.
110 + 2 = 112 tenths
Divide. 112 tenths ÷ 14
Multiply. 14 x 8 tenths = 112 tenths. Subtract. 112 tenths − 112 tenths = 0 tenths
Check. 112 tenths shared among 14 groups as 8 tenths.
So, 9.8

Question 17.
34)\(\overline { 523.6 }\)
_____

Answer:
15.4

Explanation:
523.6 ÷ 34
Divide. 523 ones ÷ 34
Multiply. 34 x 15 ones = 510 ones. Subtract. 523 ones − 510 ones = 13 ones
Check. 510 ones shared among 34 groups as 15 wholes.
13 one(s) cannot be shared among 34 groups without regrouping.
130 + 6 = 136 tenths
Divide. 136 tenths ÷ 34
Multiply. 34 x 4 tenths = 136 tenths. Subtract. 136 tenths − 136 tenths = 0 tenths
Check. 136 tenths shared among 34 groups as 4 tenths.
So, 15.4

Algebra Write the unknown number for each ■.

Question 18.
■ ÷ 5 = 1.21
■ = _____

Answer:
6.05

Explanation:
■ ÷ 5 = 1.21
■ = 1.21 x 5
■ = 6.05

Question 19.
46.8 ÷ 1.2 = ■
■ = _____

Answer:
39

Explanation:
46.8 ÷ 1.2 = ■
39 = ■

Question 20.
34.1 ÷ ■ = 22
■ = _____

Answer:
1.55

Explanation:
34.1 ÷ ■ = 22
■ = 34.1 ÷ 22
■ = 1.55

UNLOCK the Problem – Page No. 216

Question 21.
The standard width of 8 lanes in swimming pools used for competitions is 21.92 meters. The standard width of 9 lanes is 21.96 meters. How much wider is each lane when there are 8 lanes than when there are 9 lanes?
Go Math Grade 5 Answer Key Chapter 5 Divide Decimals img 7
a. What are you asked to find?
Type below:
_________

Answer:
We have to find how much wider is each line When there are 8 lanes than when there are 9 lanes.

Question 21.
b. What operations will you use to solve the problem?
Type below:
_________

Answer:
Division
Subtraction

Question 21.
c. Show the steps you used to solve the problem.
Type below:
_________

Answer:
21.92 ÷ 8 = 2.74 meters
21.96 ÷ 9 = 2.44 meters
2.74 – 2.44 = 0.30 meters

Question 21.
d. Complete the sentences.
Each lane is ______ meters wide when there are 8 lanes.
Each lane is ______ meters wide when there are 9 lanes.
Since ______ – ______ = ______ , the lanes are ______ meter(s) wider when there are 8 lanes than when there are 9 lanes.
Type below:
_________

Answer:
Each lane is 2.74 meters wide when there are 8 lanes.
Each lane is 2.44 meters wide when there are 9 lanes.
Since 2.74 – 2.44 = 0.30, the lanes are 0.30 meter(s) wider when there are 8 lanes than when there are 9 lanes.

Question 21.
e. Fill in the bubble for the correct answer choice.
Options:
a. 0.30 meter
b. 2.44 meters
c. 2.74 meters
d. 22.28 meters

Answer:
a. 0.30 meter

Question 22.
Robert pays $32.04 for 6 student tickets to the basketball game. What is the cost of each student ticket?
Options:
a. $192.24
b. $53.40
c. $26.04
d. $5.34

Answer:
d. $5.34

Explanation:
Robert pays $32.04 for 6 student tickets to the basketball game.
$32.04 ÷ 6 = $5.34

Question 23.
Jasmine uses 14.24 pounds of fruit for 16 servings of fruit salad. If each serving contains the same amount of fruit, how much fruit is in each serving?
Options:
a. 0.089 pound
b. 0.89 pound
c. 1.76 pounds
d. 17.6 pounds

Answer:
b. 0.89 pound

Explanation:
Jasmine uses 14.24 pounds of fruit for 16 servings of fruit salad. If each serving contains the same amount of fruit, 14.24 ÷ 16 = 0.89 pounds

Mid-Chapter Checkpoint – Page No. 217

Concepts and Skills

Question 1.
Explain how the position of the decimal point changes in a quotient as you divide by increasing powers of 10.
Type below:
_________

Answer:
Dividing a number by ten shifts the decimal point of the dividend one place to the left, making that into a smaller number, the desired quotient, which is one-tenth of the dividend.

Question 2.
Explain how you can use base-ten blocks to find 2.16 ÷ 3.
Type below:
_________

Answer:
0.72

Explanation:
2.16 ÷ 3
There are 2 ones, 1 tenths, and 6 hundredths.
2.16 ÷ 3 =
Chapter 5 grade 5 Divide Decimals 217 image 1
2 ones cannont be shared for 3 groups.
So, write 2 ones as 20 tenths.
20 + 1 = 21 tenths.
21 tenths ÷ 3 = 7 tenths.
7 tenths shared amount 3 groups = 21 tenths
6 hundredths ÷ 3 = 2 hundredths.
2 hundredths shared amount 3 groups = 6 hundredths.
0.72

Complete the pattern.

Question 3.
223 ÷ 1 = _______
223 ÷ 10 = _______
223 ÷ 100 = _______
223 ÷ 1000 = _______

Answer:
223 ÷ 1 = 223
223 ÷ 10 = 22.3
223 ÷ 100 = 2.23
223 ÷ 1000 = 0.223

Explanation:
223 ÷ 1 = 223
223 ÷ 10 = 22.3
223 ÷ 100 = 2.23
223 ÷ 1000 = 0.223

Question 4.
61 ÷ 1 = 61
61 ÷ 10 = 6.1
61 ÷ 100 = 0.61
61 ÷ 1,000 = 0.061

Answer:
61 ÷ 1 = 61
61 ÷ 10 = 6.1
61 ÷ 100 = 0.61
61 ÷ 1,000 = 0.061

Explanation:

Question 5.
57.4 ÷ 100 = _______
57.4 ÷ 101 = _______
57.4 ÷ 102 = _______

Answer:
57.4 ÷ 100 = 57.4
57.4 ÷ 101 = 5.74
57.4 ÷ 102 = 0.574

Explanation:
57.4 ÷ 100 = 57.4 ÷ 1 = 57.4
57.4 ÷ 101 = 57.4 ÷ 10 = 5.74
57.4 ÷ 102 = 57.4 ÷ 100 = 0.574

Estimate the quotient.

Question 6.
31.9 ÷ 4 = ______

Answer:
About 8

Explanation:
31.9 is closer to 32
32 ÷ 4 = 8

Question 7.
6.1 ÷ 8 = ______

Answer:
About 8

Explanation:
6.1 is closer to 64
64 ÷ 8 = 8

Question 8.
492.6 ÷ 48 = ______

Answer:
About 10

Explanation:
492.6 is closer to 480
480 ÷ 48 = 10

Divide.

Question 9.
5)\(\overline { 4.35 }\)
_____

Answer:
0.87

Explanation:
4.35 ÷ 5
Divide. 8 ones ÷ 5
4 one(s) cannot be shared among 5 groups without regrouping.
40 + 3 = 43 tenths
Divide. 43 tenths ÷ 5
Multiply. 5 x 8 tenths = 40 tenths. Subtract. 43 tenths − 40 tenths = 3 tenths
Check. 40 tenths shared among 5 groups as 8 tenths.
3 tenths(s) cannot be shared among 5 groups without regrouping.
1 tenth = 10 hundredths.
30 + 5 = 35 hundredths
Divide. 35 hundredths ÷ 5
Multiply. 5 x 7 hundredths = 35 hundredths. Subtract. 35 hundredths − 35 hundredths = 0 hundredths
Check. 35 hundredths shared among 5 groups as 7 hundredths.
So, 0.87

Question 10.
8)\(\overline { 9.92 }\)
_____

Answer:
1.24

Explanation:
9.92 ÷ 8
Divide. 9 ones ÷ 8
Multiply. 8 × 1 ones = 8 ones. Subtract. 9 ones − 8 ones = 1 ones
Check. 8 ones shared among 8 groups as 1 whole.
1 one(s) cannot be shared among 8 groups without regrouping.
10 + 9 = 19 tenths
Divide. 19 tenths ÷ 8
Multiply. 8 x 2 tenths = 16 tenths. Subtract. 19 tenths − 16 tenths = 3 tenths
Check. 16 tenths shared among 8 groups as 2 tenths.
3 tenths(s) cannot be shared among 8 groups without regrouping.
1 tenth = 10 hundredths.
30 + 2 = 32 hundredths
Divide. 32 hundredths ÷ 8
Multiply. 8 x 4 hundredths = 32 hundredths. Subtract. 32 hundredths − 32 hundredths = 0 hundredths
Check. 32 hundredths shared among 8 groups as 4 hundredths.
So, 1.24

Question 11.
61)\(\overline { 207.4 }\)
_____

Answer:
3.4

Explanation:
207.4 ÷ 61
Divide. 207 ones ÷ 61
Multiply. 61 x 3 ones = 183 ones. Subtract. 207 ones − 183 ones = 24 ones
Check. 183 ones shared among 61 groups as 3 wholes.
24 one(s) cannot be shared among 61 groups without regrouping.
240 + 4 = 244 tenths
Divide. 244 tenths ÷ 61
Multiply. 61 x 4 tenths = 244 tenths. Subtract. 244 tenths − 244 tenths = 0 tenths
Check. 244 tenths shared among 61 groups as 4 tenths.
So, 3.4

Mid-Chapter Checkpoint – Page No. 218

Question 12.
The Westside Bakery uses 440 pounds of flour to make 1,000 loaves of bread. Each loaf contains the same amount of flour. How many pounds of flour are used in each loaf of bread?
_____ pounds

Answer:
0.440 pounds

Explanation:
The Westside Bakery uses 440 pounds of flour to make 1,000 loaves of bread. Each loaf contains the same amount of flour.
440/1000 = 0.440 pounds

Question 13.
Elise pays $21.75 for 5 student tickets to the fair. What is the cost of each student ticket?
$ _____

Answer:
$4.35

Explanation:
Elise pays $21.75 for 5 student tickets to the fair.
$21.75/5 = $4.35

Question 14.
Jason has a piece of wire that is 62.4 inches long. He cuts the wire into 3 equal pieces. Estimate the length of 1 piece of wire.
_____ in

Answer:
20.8 inches

Explanation:
Jason has a piece of wire that is 62.4 inches long. He cuts the wire into 3 equal pieces.
62.4/3 = 20.8 inches

Question 15.
Elizabeth uses 23.25 ounces of granola and 10.5 ounces of raisins for 15 servings of trail mix. If each serving contains the same amount of trail mix, how much trail mix is in each serving?
_____ ounces

Answer:
2.25 ounces

Explanation:
Elizabeth uses 23.25 ounces of granola and 10.5 ounces of raisins for 15 servings of trail mix.
23.25 + 10.5 = 33.75
33.75/15 = 2.25 ounces

Share and Show – Page No. 221

Use the model to complete the number sentence.

Question 1.
1.2 ÷ 0.3 =
Go Math Grade 5 Answer Key Chapter 5 Divide Decimals img 8
_____

Answer:
1.2 ÷ 0.3 = 4

Explanation:
There are 4 groups of 3 tenths.
So, 1.2 ÷ 0.3 = 4

Question 2.
0.45 ÷ 0.09 =
Go Math Grade 5 Answer Key Chapter 5 Divide Decimals img 9
_____

Answer:
0.45 ÷ 0.09 = 5

Explanation:
There are 5 groups. Each group has 9 hundredths.
0.09 x 5 = 0.45
So, 0.45 ÷ 0.09 = 5

Question 3.
0.96 ÷ 0.24 =
Go Math Grade 5 Answer Key Chapter 5 Divide Decimals img 10
_____

Answer:
0.96 ÷ 0.24 = 4

Explanation:
There are 4 groups. Each group has 2 tenths and 4 hundredths. 24 hundredths
0.24 x 4 = 0.96
0.96 ÷ 0.24 = 4

Question 4.
1 ÷ 0.5 =
Go Math Grade 5 Answer Key Chapter 5 Divide Decimals img 11
_____

Answer:
2

Explanation:
There are 2 groups. Each group has 5 tenths.
0.5 x 2 = 1.0 = 1
1 ÷ 0.5 = 2

Divide. Use decimal models.

Question 5.
1.8 ÷ 0.6 =
_____

Answer:
3

Explanation:
1.8
There are 1 one(s) and 8 tenth(s).
Cut 1.8 to show groups of 0.6.
There are 3 groups of 1.8 tenths.
1.8 ÷ 0.6 = 3

Question 6.
1.2 ÷ 0.3 =
_____

Answer:
4

Explanation:
1.2
There are 1 one(s) and 2 tenth(s).
Cut 1.2 to show groups of 0.3.
There are 4 groups of 1.2 tenths.
1.2 ÷ 0.3 = 4

Question 7.
0.24 ÷ 0.04 =
_____

Answer:
6

Explanation:
0.24
There are 0 one(s) and 2 tenth(s) and 4 hundredth(s).
Cut 0.24 to show groups of 0.04.
There are 6 groups of 0.04 hundredths.
0.24 ÷ 0.04 = 6

Question 8.
1.75 ÷ 0.35 =
_____

Answer:
5

Explanation:
1.75
There are 1 one(s) and 7 tenth(s) and 5 hundredth(s).
Cut 1.75 to show groups of 0.35.
There are 5 groups of 1.75 hundredths.
1.75 ÷ 0.35 = 5

Question 9.
2 ÷ 0.4 =
_____

Answer:
5

Explanation:
2
There are 2 one(s).
Cut 2 to show groups of 0.4.
There are 5 groups of 2 ones.
2 ÷ 0.4 = 5

Question 10.
2.7 ÷ 0.9 =
_____

Answer:
3

Explanation:
2.7
There are 2 one(s) and 7 tenth(s).
Cut 2.7 to show groups of 0.9.
There are 3 groups of 2.7 tenths.
2.7 ÷ 0.9 = 3

Question 11.
1.24 ÷ 0.62 =
_____

Answer:
2

Explanation:
1.24
There are 1 one(s), 2 tenth(s) and 4 hundredth(s).
Cut 1.24 to show groups of 0.62.
There are 2 groups of 1.24 hundredths.
1.24 ÷ 0.62 = 2

Question 12.
0.84 ÷ 0.14 =
_____

Answer:
6

Explanation:
0.84
There are 0 one(s), 8 tenth(s) and 4 hundredth(s).
Cut 0.84 to show groups of 0.14.
There are 6 groups of 0.84 hundredths.
0.84 ÷ 0.14 = 6

Question 13.
1.6 ÷ 0.4 =
_____

Answer:
4

Explanation:
1.6
There are 1 one(s) and 6 tenth(s).
Cut 1.6 to show groups of 0.4.
There are 4 groups of 1.6 tenths.
1.6 ÷ 0.4 = 4

Use the model to find the unknown value.

Question 14.
2.4 ÷ □ = 3
Go Math Grade 5 Answer Key Chapter 5 Divide Decimals img 12
_____

Answer:
0.8

Explanation:
2.4 ÷ □ = 3
There are 3 groups with 8 tenths in each group.
So, 2.4 ÷ 0.8 = 3
The unknown number is 0.8

Question 15.
□ ÷ 0.32 = 4
Go Math Grade 5 Answer Key Chapter 5 Divide Decimals img 13
_____

Answer:
1.28

Explanation:
□ ÷ 0.32 = 4
There are 4 groups with 32 hundredths in each group.
So, 0.32 x 4 = 1.28
The unknown number is 1.28

Question 16.
Make a model to find 0.6 ÷ 0.15. Describe your model.
_____

Answer:
0.6 ÷ 0.15
Divide 6 tenths into 15 hundredths each group.
15 hundredths + 15 hundredths + 15 hundredths + 15 hundredths
4 groups.
0.6 ÷ 0.15 = 4

Question 17.
Explain, using the model, what the equation represents in Exercise 15.
Type below:
_________

Answer:
Using the decimal model, we have to find the value of 0.6 ÷ 0.15
0.6 ÷ 0.15 = 4

Problem Solving – Page No. 222

Pose a Problem
Go Math Grade 5 Answer Key Chapter 5 Divide Decimals img 14

Question 18.
Emilio buys 1.2 kilograms of grapes. He separates the grapes into packages that contain 0.3 kilogram of grapes each. How many packages of grapes does Emilio make?
Go Math Grade 5 Answer Key Chapter 5 Divide Decimals img 15
1.2 ÷ 0.3 = 4
Emilio made 4 packages of grapes.
Write a new problem using a different amount for the weight in each package. The amount should be a decimal with tenths. Use a total amount of 1.5 kilograms of grapes. Then use decimal models to solve your problem.
Pose a problem.                          Solve your problem. Draw a picture of the
model you used to solve your problem.
• Explain why you chose the amount you did for your problem.
Type below:
_________

Answer:
Emilio buys 1.5 kilograms of grapes. He separates the grapes into packages that contain 0.5 kilogram of grapes each.
1.5/0.5 = 3
3 groups

Share and Show – Page No. 225

Copy and complete the pattern.

Question 1.
45 ÷ 9 = _____
4.5 ÷ _____ = 5
_____ ÷ 0.09 = 5

Answer:
45 ÷ 9 = 5
4.5 ÷ 0.9 = 5
0.45 ÷ 0.09 = 5

Explanation:
45 ÷ 9 = 5
4.5 ÷ _____ = 5
45 ÷ 10 = 4.5. So, 9 ÷ 10 = 0.9
4.5 ÷ 0.9 = 5
_____ ÷ 0.09 = 5
0.9 ÷ 10 = 0.09
4.5 ÷ 10 = 0.45
0.45 ÷ 0.09 = 5

Question 2.
175 ÷ 25 = _____
17.5 ÷ _____ = 7
_____ ÷ 0.25 = 7

Answer:
175 ÷ 25 = 7
17.5 ÷ 2.5 = 7
1.75 ÷ 0.25 = 7

Explanation:
175 ÷ 25 = 7
17.5 ÷ _____ = 7
175 ÷ 10 = 17.5. So, 25 ÷ 10 = 2.5
17.5 ÷ 2.5 = 7
_____ ÷ 0.25 = 7
2.5 ÷ 10 = 0.25
17.5 ÷ 10 = 1.75
1.75 ÷ 0.25 = 7

Question 3.
164 ÷ 2 = _____
16.4 ÷ _____ = 82
_____ ÷ 0.02 = 82

Answer:
164 ÷ 2 = 82
16.4 ÷ 0.2 = 82
1.64 ÷ 0.02 = 82

Explanation:
164 ÷ 2 = 82
16.4 ÷ _____ = 82
164 ÷ 10 = 16.4
2 ÷ 10 = 0.2
16.4 ÷ 0.2 = 82
_____ ÷ 0.02 = 82
0.2 ÷ 10 = 0.02
16.4 ÷ 10 = 1.64
1.64 ÷ 0.02 = 82

Divide.

Question 4.
1.6)\(\overline { 9.6 }\) = _____

Answer:
6

Explanation:
9.6 ÷ 1.6
9.6 x 10 = 96
1.6 x 10 = 16
96 ÷ 16 = 6
So, 9.6 ÷ 1.6 = 6

Question 5.
0.3)\(\overline { 0.24 }\) = _____

Answer:
0.8

Explanation:
0.24 ÷ 0.3
0.24 x 10 = 2.4
0.3 x 10 = 3
2.4 ÷ 3 = 0.8
0.24 ÷ 0.3 = 0.8

Question 6.
3.45 ÷ 1.5 = _____

Answer:
2.3

Explanation:
3.45 ÷ 1.5
3.45 x 10 = 34.5
1.5 x 10 = 15
34.5 ÷ 15 = 2.3
3.45 ÷ 1.5 = 2.3

On Your Own

Divide.

Question 7.
0.6)\(\overline { 13.2 }\) = _____

Answer:
22

Explanation:
13.2 ÷ 0.6
13.2 x 10 = 132
0.6 x 10 = 6
132 ÷ 6 = 22
13.2 ÷ 0.6 = 22

Question 8.
0.3)\(\overline { 0.9 }\) = _____

Answer:
3

Explanation:
0.9 ÷ 0.3
0.9 x 10 = 9
0.3 x 10 = 3
9 ÷ 3 = 3
0.9 ÷ 0.3 = 3

Question 9.
0.26)\(\overline { 1.56 }\) = _____

Answer:
6

Explanation:
1.56 ÷ 0.26
1.56 x 100 = 156
0.26 x 100 = 26
156 ÷ 26 = 6
1.56 ÷ 0.26 = 6

Question 10.
0.45)\(\overline { 5.85 }\) = _____

Answer:
13

Explanation:
5.85 ÷ 0.45
5.85 x 100 = 585
0.45 x 100 = 45
585 ÷ 45 = 13

Question 11.
0.3)\(\overline { 0.69 }\) = _____

Answer:
2.3

Explanation:
0.69 ÷ 0.3
0.69 x 10 = 6.9
0.3 x 10 = 3
6.9 ÷ 3 = 2.3

Question 12.
3.6 ÷ 0.4 = _____

Answer:
9

Explanation:
3.6 ÷ 0.4
3.6 x 10 = 36
0.4 x 10 = 4
36 ÷ 4 = 9
3.6 ÷ 0.4 = 9

Question 13.
1.26 ÷ 2.1 = _____

Answer:
0.6

Explanation:
1.26 ÷ 2.1
1.26 x 10 = 12.6
2.1 x 10 = 21
12.6 ÷ 21 = 0.6

Question 14.
7.84 ÷ 0.28 = _____

Answer:
28

Explanation:
7.84 ÷ 0.28
7.84 x 100 = 784
0.28 x 100 = 28
784 ÷ 28 = 28
7.84 ÷ 0.28 = 28

Question 15.
9.28 ÷ 2.9 = _____

Answer:
3.2

Explanation:
9.28 ÷ 2.9
9.28 x 10 = 92.8
2.9 x 10 = 29
92.8 ÷ 29 = 3.2
9.28 ÷ 2.9 = 3.2

Problem Solving – Page No. 226

Use the table to solve 16—19.
Go Math Grade 5 Answer Key Chapter 5 Divide Decimals img 16

Question 16.
Connie paid $1.08 for pencils. How many pencils did she buy?
_____ pencils

Answer:
9 pencils

Explanation:
1 pencil = $0.12
Connie paid $1.08 for pencils.
$1.08 ÷ $0.12 = 9
she buys 9 pencils

Question 17.
Albert has $2.16. How many more pencils can he buy than markers?
_____

Answer:
Albert can buy 12 more pencils than markers

Explanation:
Albert has $2.16.
1 pencil = $0.12
$2.16/$0.12 = 18 pencils
1 marker = $0.36
$2.16/$0.36 = 6 markers
18 – 6 = 12 more pencils

Question 18.
How many erasers can Ayita buy for the same amount that she would pay for one notepad?
_____ erasers

Answer:
Ayita can buy 13 erasers with one notepad money.

Explanation:
1 notepad = $0.65
$0.65/$0.05 = 13
Ayita can buy 13 erasers with one notepad money.

Question 19.
Ramon paid $3.25 for notepads and $1.44 for markers. What is the total number of items he bought?
_____ items

Answer:
9 items

Explanation:
Ramon paid $3.25 for notepads.
$3.25/$0.65 = 5
He paid $1.44 for markers
$1.44/$0.36 = 4
5 + 4 = 9
He bought 9 items

Question 20.
What’s the Error? Katie divided 4.25 by 0.25 and got a quotient of 0.17.
_____

Answer:
Katie divided 4.25 by 0.25
4.25 ÷ 0.25
4.25 x 100 = 425
0.25 x 100 = 25
425 ÷ 25 = 17
4.25 ÷ 0.25 = 17
But he said 0.17

Question 21.
Test Prep Marcus bought apples that cost $0.45 per pound. He paid $1.35 for the apples. How many pounds of apples did he buy?
Options:
a. 0.3 pound
b. 2.8 pounds
c. 3 pounds
d. 30 pounds

Answer:
c. 3 pounds

Explanation:
Marcus bought apples that cost $0.45 per pound.
He paid $1.35 for the apples.
$1.35 ÷ $0.45 = 3 pounds

Share and Show – Page No. 229

Write the quotient with the decimal point placed correctly.

Question 1.
5 ÷ 0.8 = 625
_____

Answer:
5 ÷ 0.8 = 6.25

Explanation:
5 ÷ 0.8 = 5 ÷ 0.800
50 ÷ 8 = 6.25
Divide 5 ÷ 0.8 until you get remainder 0
Divide: 50 ÷ 8 = 6
8 x 6 = 48; Subtract: 50 – 48 = 2
Divide: 20 ÷ 8 = 2;
Subtract: 20 – 16 = 4
Divide: 40 ÷ 8 = 5
8 x 5 = 40; Subtract: 40 – 40 = 0
So, 6.25

Question 2.
26.1 ÷ 6 = 435
_____

Answer:

Explanation:
26.1 ÷ 6 = 26.1 ÷ 6.0
261 ÷ 60.0
Divide 26.1 ÷ 6 until you get remainder 0
Divide: 261 ÷ 60 = 4;
60 x 4 = 240 Subtract: 261 – 240 = 21
Divide: 210 ÷ 60 = 3
60 x 3 = 180; Subtract: 210 – 180 = 30
Divide: 300 ÷ 60 = 5;
60 x 5 = 300; Subtract: 300 – 300 = 0
261 ÷ 60 = 4.35

Question 3.
0.42 ÷ 0.35 = 12
_____

Answer:
1.2

Explanation:
0.42 ÷ 0.35 = 42 ÷ 35
Divide 0.42 ÷ 0.35 until you get remainder 0
Divide: 42 ÷ 35 = 1;
35 x 1 = 35 Subtract: 42 – 35 = 7
Divide: 70 ÷ 35 = 2
35 x 2 = 70; Subtract: 70 – 70 = 0
0.42 ÷ 0.35 = 1.2

Question 4.
80 ÷ 50 = 16
_____

Answer:
1.6

Explanation:
80 ÷ 50
8 ÷ 5
Divide 8 ÷ 5
5 x 1 = 5 : Subtract: 8 – 5 = 3
Divide 30 ÷ 5 = 6
80 ÷ 50 = 1.6

Divide.

Question 5.
4)\(\overline { 32.6 }\) = _____

Answer:
8.15

Explanation:
32.6 ÷ 4
Divide: 32/4 = 8
8 x 4 = 32; Subtract: 32 – 32 = 0
Divide: 6/4 = 1
4 x 1 = 4; Subtract: 6 – 4 = 2
Add zero
20 ÷ 4 = 5
So, 8.15

Question 6.
1.2)\(\overline { 9 }\) = _____

Answer:
0.1333

Explanation:
1.2/9 = 12/90; 12 < 90
12 cannot be divided by 90. So add zero to 12.
120/90 = 1; 120 – 90 = 30; 30 < 90
300/90 = 3; 300 – 270 = 30
So, 0.13333

Question 7.
15)\(\overline { 42 }\) = _____

Answer:
2.8

Explanation:
42/15 = 2
15 x 2 = 30; 42 – 30 = 12
120/15 = 8;
So, 2.8

Question 8.
0.14)\(\overline { 0.91 }\) = _____

Answer:
6.5

Explanation:
0.91/0.14 = 91/14
91/14 = 6
14 x 6 = 84; 91 – 84 = 7; 7 < 14
70/14 = 5
So, 6.5

On Your Own

Divide.

Question 9.
8)\(\overline { 84 }\) = _____

Answer:
10.5

Explanation:
84/8 = 10
8 x 10 = 80; 84 – 80 = 4;4 < 8
40/8 = 5
So, 10.5

Question 10.
2.5)\(\overline { 4 }\) = _____

Answer:

Explanation:
4/2.5 = 40/25
40/25 = 1;
40 – 25 = 15; 15 < 25
150/25 = 6
So, 1.6

Question 11.
5)\(\overline { 16.2 }\) = _____

Answer:
3.24

Explanation:
16.2/5 = 162/50
162/50 = 3
50 x 3 = 150; 162 – 150 = 12; 12 < 50
120/50 = 2
50 x 2 = 100; 120 – 100 = 20; 20 < 50
200/50 = 4
So, 3.24

Question 12.
0.6)\(\overline { 2.7 }\) = _____

Answer:
4.5

Explanation:
2.7/0.6 = 27/6
27/6 = 4
4 x 6 = 24; 27 – 24 = 3; 3 < 6
30/6 = 5
So, 4.5

Question 13.
18 ÷ 7.5 = _____

Answer:
2.4

Explanation:
18/7.5 = 180/75
180/75 = 2
75 x 2 = 150; 180 – 150 = 30; 30 < 75
300/75 = 4
So, 2.4

Question 14.
34.8 ÷ 24 = _____

Answer:
1.45

Explanation:
34.8/24 = 348/240
348/240 = 1;
240 x 1 = 240; 348 – 240 = 108 < 240
1080/240 = 4
240 x 4 = 960; 1080 – 960 = 120; 120 < 240
1200/240 = 5
So, 1.45

Question 15.
5.16 ÷ 0.24 = _____

Answer:
21.5

Explanation:
5.16/0.24 = 516/24
516/24 = 21
24 x 21 = 504: 516 – 504 = 12; 12 < 24
120/24 = 5;
So, 21.5

Question 16.
81 ÷ 18 = _____

Answer:
4.5

Explanation:
81/18 = 4
18 x 4 = 72; 81 – 72 = 9; 9 < 18
90/18 = 5
4.5

Practice: Copy and Solve Divide.

Question 17.
1.6)\(\overline { 20 }\) = _____

Answer:
12.5

Explanation:
20/1.6 = 200/16
200/16 = 12
16 x 12 = 192; 200 – 192 = 8; 8 < 16
80/16 = 5;
12.5

Question 18.
15)\(\overline { 4.8 }\) = _____

Answer:
0.32

Explanation:
4.8/15 = 48/150; 48 < 150
480/150 = 3
150 x 3 = 450; 480 – 450 = 30 < 150
300/150 = 2
0.32

Question 19.
0.54)\(\overline { 2.43 }\) = _____

Answer:
4.5

Explanation:
2.43/0.54 = 243/54
243/54 = 4
54 x 4 = 216; 243 – 216 = 27 < 54
270/54 = 5
4.5

Question 20.
28)\(\overline { 98 }\) = _____

Answer:
3.5

Explanation:
98/28 = 3
28 x 3 = 84; 98 – 84 = 14 < 28
140/28 = 5
3.5

Question 21.
1.8 ÷ 12 = _____

Answer:
0.15

Explanation:
1.8/12 = 18/120; 18 < 120
180/120 = 1;
120 x 1 = 120; 180 – 120 = 60; 60 < 120
600/120 = 5
0.15

Question 22.
3.5 ÷ 2.5 = _____

Answer:
1.4

Explanation:
3.5/2.5 = 35/25
35/25 = 1;
25 x 1 = 25; 35 – 25 = 10; 10 < 25
100/25 = 4
1.4

Question 23.
40 ÷ 16 = _____

Answer:
2.5

Explanation:
40/16 = 2
16 x 2 = 32; 40 – 32 = 8 < 16
80/16 = 5
2.5

Question 24.
2.24 ÷ 0.35 = _____

Answer:
6.4

Explanation:
2.24/0.35 = 224/35
224/35 = 6
35 x 6 = 210; 224 – 210 = 14 < 35
140/35 = 4
6.4

Problem Solving – Page No. 230

Solve.

Question 25.
Jerry takes trail mix on hikes. A package of dried apricots weighs 25.5 ounces. Jerry divides the apricots equally among 6 bags of trail mix. How many ounces of apricots are in each bag?
_____ ounces

Answer:
4.25 ounces

Explanation:
Jerry takes trail mix on hikes. A package of dried apricots weighs 25.5 ounces. Jerry divides the apricots equally among 6 bags of trail mix.
25.5/6 = 4.25 ounces

Question 26.
Amy has 3 pounds of raisins. She divides the raisins equally into 12 bags. How many pounds of raisins are in each bag? Tell how many zeros you had to write at the end of the dividend.
_____ pound

Answer:
4 pound

Explanation:
Amy has 3 pounds of raisins. She divides the raisins equally into 12 bags.
12/3 = 4 pound. There are no zeros required at the end of the dividend.

Question 27.
Find 65 ÷ 4. Write your answer using a remainder, a fraction, and a decimal. Then tell which form of the answer you prefer. Explain your choice.
Type below:
_________

Answer:
65 ÷ 4 = 16 and 1 remainder.
Fraction = 65/4
65/4 = 16.25
The decimal answer is best to choose as there are no mathematical operations required.

Question 28.
Test Prep Todd has a piece of rope that is 1.6 meters long. He cuts the rope into 5 equal pieces. What is the length of each piece?
Options:
a. 0.8 meter
b. 0.32 meter
c. 3.2 meters
d. 8 meters

Answer:
b. 0.32 meter

Explanation:
Todd has a piece of rope that is 1.6 meters long. He cuts the rope into 5 equal pieces.
1.6/5 = 0.32 meter

Connect to Science

Rate of Speed Formula

The formula for velocity, or rate of speed, is r = d ÷ t, where r represents rate of speed, d represents distance, and t represents time. For example, if an object travels 12 feet in 10 seconds, you can find its rate of speed by using the formula.
r = d ÷ t
r = 12 ÷ 10
r = 1.2 feet per second
Use division and the formula for rate of speed to solve.
Go Math Grade 5 Answer Key Chapter 5 Divide Decimals img 17

Question 29.
A car travels 168 miles in 3.2 hours. Find the car’s rate of speed in miles per hour.
_____ miles per hour

Answer:
52.5 miles per hour

Explanation:
168/3.2 = 52.5 miles per hour

Question 30.
A submarine travels 90 kilometers in 4 hours. Find the submarine’s rate of speed in kilometers per hour.
_____ kilometers per hour

Answer:
22.5 kilometers per hour

Explanation:
A submarine travels 90 kilometers in 4 hours.
For an hour, 90/4 = 22.5 kilometers per hour

Share and Show – Page No. 233

Question 1.
Hector spent $36.75 for 2 DVDs with the same cost. The sales tax was $2.15. Hector also used a coupon for $1.00 off his purchase. How much did each DVD cost?
First, make a flowchart to show the information and show how you would work backward.
Type below:
_________

Answer:
grade 5 Chapter 5 Divide Decimals 232 image 1
Question 1.
Then, work backward to find the cost of 2 DVDs.
Type below:
_________

Answer:
Price of the DVD is s
2s + $2.15 – $1.00 = $36.75
2s = $35.60

Question 1.
Finally, find the cost of one DVD.
So, each DVD costs ________ .
$ _____

Answer:
2s = $35.60
s = $35.60/2
s = $17.8

Question 2.
What if Hector spent $40.15 for the DVDs, the sales tax was $2.55, and he didn’t have a coupon? How much would each DVD cost?
$ _____

Answer:

Explanation:
If Hector spent $40.15 for the DVDs. The sales tax was $2.55, and he didn’t have a coupon
Price of the DVD is s
2s + $2.55 = $40.15
2s = $37.60
s = $18.8
Each DVD price =$18.8

Question 3.
Sophia spent $7.30 for school supplies. She spent $3.00 for a notebook and $1.75 for a pen. She also bought 3 large erasers. If each eraser had the same cost, how much did she spend for each eraser?
$ _____

Answer:
$0.85

Explanation:
Sophia spent $7.30 for school supplies. She spent $3.00 for a notebook and $1.75 for a pen.
$3.00 + $1.75 = $4.75
$7.30 – $4.75 = $2.55
If she bought 3 erasers, $2.55/3 = $0.85
Each eraser cost = $0.85

On Your Own – Page No. 234

Question 4.
The change from a gift purchase was $3.90. Each of 6 students donated an equal amount for the gift. How much change should each student receive?
$ _____

Answer:
$0.65

Explanation:
The change from a gift purchase was $3.90. Each of 6 students donated an equal amount for the gift.
$3.90/6 = $0.65

Question 5.
If you divide this mystery number by 4, add 8, and multiply by 3, you get 42. What is the mystery number?
_____

Answer:
24

Explanation:
Let the unknown number = s.
(s/4 + 8) x 3 = 42
s/4 + 8 = 42/3
s/4 + 8 = 14
s/4 = 6
s = 6 x 4 = 24
s = 24

Question 6.
A mail truck picks up two boxes of mail from the post office. The total weight of the boxes is 32 pounds. One box is 8 pounds heavier than the other box. How much does each box weigh?
Type below:
_________

Answer:
20 pounds.
other box = 12 pounds

Explanation:
If one box weight is s, the other box weight is 8 pounds heavier than the other box. s – 8.
Total weight of two boxes =32
s + s – 8 = 32
2s = 40
s = 20

Question 7.
Stacy buys 3 CDs in a set for $29.98. She saved $6.44 by buying the set instead of buying the individual CDs. If each CD costs the same amount, how much does each of the 3 CDs cost when purchased individually?
$ _____

Answer:
$12.14

Explanation:
Stacy buys 3 CDs in a set for $29.98. She saved $6.44 by buying the set instead of buying the individual CDs.
Cost to buy 3 CDs = $29.98 + $6.44 = $36.42
$36.42/3 = $12.14

Question 8.
A school cafeteria sold 1,280 slices of pizza the first week, 640 the second week, and 320 the third week. If this pattern continues, in what week will the cafeteria sell 40 slices? Explain how you got your answer.
_____ th week

Answer:
sixth week

Explanation:
A school cafeteria sold 1,280 slices of pizza the first week, 640 the second week, and 320 the third week.
1280, 640, 320
1280/2 = 640
640/2 = 320
320/2 = 160
160/2 = 80
80/2 = 40
6th term shows 40. So, the answer is the sixth week.

Question 9.
Test Prep While working at the school store, John sold a jacket for $40.00 and notebooks for $1.50 each. If he collected $92.50, how many notebooks did he sell?
Options:
a. 3.5
b. 6.1
c. 35
d. 61

Answer:
c. 35

Explanation:
John sold a jacket for $40.00 and notebooks for $1.50 each.
$40 + $1.50S = $92.50
$1.50S = $52.5
S = $52.5/$1.50 = 35

Chapter Review/Test – Page No. 235

Concepts and Skills

Complete the pattern.

Question 1.
341 ÷ 1 = ________
341 ÷ 10 = ________
341 ÷ 100 = ________
341 ÷ 1,000 = ________

Answer:
341 ÷ 1 = 341
341 ÷ 10 = 34.1
341 ÷ 100 = 3.41
341 ÷ 1,000 = 0.341

Explanation:
341 ÷ 1 = 341
341 ÷ 10 = 34.1
341 ÷ 100 = 3.41
341 ÷ 1,000 = 0.341

Question 2.
15 ÷ 1 = ________
15 ÷ 10 = ________
15 ÷ 100 = ________
15 ÷ 1,000 = ________

Answer:
15 ÷ 1 = 15
15 ÷ 10 = 1.5
15 ÷ 100 = 0.15
15 ÷ 1,000 = 0.015

Explanation:
15 ÷ 1 = 15
15 ÷ 10 = 1.5
15 ÷ 100 = 0.15
15 ÷ 1,000 = 0.015

Question 3.
68.2 ÷ 100 = ________
68.2 ÷ 101 = ________
68.2 ÷ 102 = ________

Answer:
68.2 ÷ 100 = 68.2
68.2 ÷ 101 = 6.82
68.2 ÷ 102 = 0.682

Explanation:
68.2 ÷ 100 = 68.2 ÷ 1 = 68.2
68.2 ÷ 101 = 68.2 ÷ 10 = 6.82
68.2 ÷ 102 = 68.2 ÷ 100 = 0.682

Estimate the quotient.

Question 4.
49.3 ÷ 6 = _____

Answer:
About 8

Explanation:
49.3 is closer to 48
48 ÷ 6 = 8

Question 5.
3.5 ÷ 4 = _____

Answer:
about 1

Explanation:
3.5 is closer to 4
4 ÷ 4 = 1

Question 6.
396.5 ÷ 18 = _____

Answer:
About 22

Explanation:
396.5 is closer to 396
396 ÷ 18 = 22

Divide.

Question 7.
6)\(\overline { 3.24 }\) = _____

Answer:
0.54

Explanation:
3.24 ÷ 6
Divide. 3 ones ÷ 6
3 one(s) cannot be shared among 6 groups without regrouping.
30 + 2 = 32 tenths
Divide. 32 tenths ÷ 6
Multiply. 6 x 5 tenths = 30 tenths. Subtract. 32 tenths − 30 tenths = 2 tenths
Check. 30 tenths shared among 6 groups as 5 tenths.
2 tenths(s) cannot be shared among 6 groups without regrouping.
1 tenth = 10 hundredths.
20 + 4 = 24 hundredths
Divide. 24 hundredths ÷ 6
Multiply. 6 x 4 hundredths = 24 hundredths. Subtract. 24 hundredths − 24 hundredths = 0 hundredths
Check. 24 hundredths shared among 6 groups as 4 hundredths.
So, 0.54

Question 8.
5)\(\overline { 6.55 }\) = _____

Answer:
1.51

Explanation:
6.55 ÷ 5
Divide. 6 ones ÷ 5
Multiply. 5 × 1 ones = 5 ones. Subtract. 6 ones − 5 ones = 1 ones
Check. 5 ones shared among 5 groups as 1 whole.
1 one(s) cannot be shared among 5 groups without regrouping.
10 + 5 = 15 tenths
Divide. 15 tenths ÷ 5
Multiply. 5 x 3 tenths = 15 tenths. Subtract. 15 tenths − 15 tenths = 0 tenths
Check. 35 tenths shared among 5 groups as 3 tenths.
Divide. 5 hundredths ÷ 5
Multiply. 5 x 1 hundredths = 5 hundredths. Subtract. 5 hundredths − 5 hundredths = 0 hundredths
Check. 5 hundredths shared among 5 groups as 1 hundredth.
So, 1.51

Question 9.
26)\(\overline { 96.2 }\) = _____

Answer:
3.7

Explanation:
96.2 ÷ 26
Divide. 96 ones ÷ 26
Multiply. 26 × 3 ones = 78 ones. Subtract. 96 ones − 78 ones = 18 ones
Check. 78 ones shared among 26 groups as 3 wholes.
18 one(s) cannot be shared among 26 groups without regrouping.
180 + 2 = 182 tenths
Divide. 182 tenths ÷ 26
Multiply. 26 x 7 tenths = 182 tenths. Subtract. 182 tenths − 182 tenths = 0 tenths
So, 3.7

Question 10.
1.08 ÷ 0.4 = _____

Answer:
2.7

Explanation:
1.08 ÷ 0.4
1.08 x 10 = 10.8
0.4 x 10 = 4
10.8 ÷ 4 = 2.7

Question 11.
8.84 ÷ 0.68 = _____

Answer:
13

Explanation:
8.84 ÷ 0.68
8.84 x 100 = 884
0.68 x 100 = 68
884 ÷ 68 = 13

Question 12.
7.31 ÷ 1.7 = _____

Answer:
4.3

Explanation:
7.31 ÷ 1.7
7.31 x 10 = 73.1
1.7 x 10 = 17
73.1 ÷ 17 = 4.3

Question 13.
9.18 ÷ 0.9 = _____

Answer:
10.2

Explanation:
9.18 ÷ 0.9
9.18 x 10 = 91.8
0.9 x 10 = 9
91.8 ÷ 9 = 10.2

Question 14.
12.7 ÷ 5 = _____

Answer:
2.54

Explanation:
12.7 ÷ 5
12.7 x 10 = 127
5 x 10 = 50
127 ÷ 50 = 2.54

Question 15.
8.33 ÷ 0.34 = _____

Answer:
24.5

Explanation:
8.33 ÷ 0.34
8.33 x 100 = 833
0.34 x 100 = 34
833 ÷ 34 = 24.5

Chapter Review/Test – Page No. 236

Fill in the bubble completely to show your answer.

Question 16.
The Orchard Pie Company uses 95 pounds of apples to make 100 pies. Each pie contains the same amount of apples. How many pounds of apples are used in each pie?
Options:
A. 0.095 pound
B. 0.95 pound
C. 9.5 pounds
D. 95 pounds

Answer:
B. 0.95 pound

Explanation:
The Orchard Pie Company uses 95 pounds of apples to make 100 pies. Each pie contains the same amount of apples.
95/100 = 0.95 pounds

Question 17.
During a special sale, all CDs have the same price. Mr. Ortiz pays $228.85 for 23 CDs. Which is the best estimate of the price of each CD?
Options:
A. $9
B. $10
C. $12
D. $13

Answer:
B. $10

Explanation:
During a special sale, all CDs have the same price. Mr. Ortiz pays $228.85 for 23 CDs.
$228.85/23 = $9.95

Question 18.
Ryan earns $20.16 working for 3 hours. How much does he earn per hour?
Options:
A. $60.48
B. $6.82
C. $6.72
D. $6.71

Answer:
C. $6.72

Explanation:
Ryan earns $20.16 working for 3 hours.
$20.16/3 = $6.72

Question 19.
Anna hikes 6.4 miles during a 4-day vacation. If she hikes the same distance each day, how many miles does she hike each day?
Options:
A. 1.06 miles
B. 1.1 miles
C. 1.4 miles
D. 1.6 miles

Answer:
D. 1.6 miles

Explanation:
Anna hikes 6.4 miles during a 4-day vacation. If she hikes the same distance each day, 6.4/4 = 1.6 miles

Chapter Review/Test – Page No. 237

Fill in the bubble completely to show your answer.

Question 20.
Karina pays $1.92 for pencil erasers. The erasers cost $0.08 each. How many erasers does she buy?
Options:
A. 2.4
B. 2.5
C. 24
D. 25

Answer:
C. 24

Explanation:
Karina pays $1.92 for pencil erasers. The erasers cost $0.08 each.
$1.92/$0.08 = 24

Question 21.
Wyatt has 25.4 ounces of fruit juice. He divides the juice equally into 4 glasses. How much juice is in each glass?
Options:
A. 6 ounces
B. 6.35 ounces
C. 6.4 ounces
D. 6.45 ounces

Answer:
B. 6.35 ounces

Explanation:
Wyatt has 25.4 ounces of fruit juice. He divides the juice equally into 4 glasses.
25.4/4 = 6.35 ounces

Question 22.
Jacob walks 70.4 feet in 0.2 hour. If he walks at the same rate the whole time, what is his speed in feet per hour?
Options:
A. 352 feet per hour
B. 140.8 feet per hour
C. 35.2 feet per hour
D. 14.08 feet per hour

Answer:
A. 352 feet per hour

Explanation:
Jacob walks 70.4 feet in 0.2 hour. If he walks at the same rate the whole time,
7.4/0.2 = 352 feet per hour

Question 23.
Meghan earns $20.00 by walking dogs. She uses all of her earnings to buy a shirt for $12.85 and some stickers for $0.65 each. How many stickers does she buy?
Options:
A. 4.65
B. 11
C. 46
D. 110

Answer:
B. 11

Explanation:
Meghan earns $20.00 by walking dogs. She uses all of her earnings to buy a shirt for $12.85 and some stickers for $0.65 each.
$20 – $12.85 = $7.15
$7.15/$0.65 = 11

Chapter Review/Test – Page No. 238

Constructed Response

Question 24.
Percy buys tomatoes that cost $0.58 per pound. He pays $2.03 for the tomatoes. How many pounds of tomatoes does he buy? Show your work using words, pictures, or numbers. Explain how you know your answer is reasonable.
_____ pounds

Answer:
3.5 pounds

Explanation:
Percy buys tomatoes that cost $0.58 per pound. He pays $2.03 for the tomatoes.
$2.03/$0.58
200/50 = 4; Using Compitable numbers
Multiply the divisor by a power of 10 to make it a whole number. Then, multiply the dividend by the same power of 10.
0.58 x 100 = 58
2.03 x 100 = 203
203/58
Divide until you have an amour less than the divisor left over.
Insert a decimal point and a zero at the end of the dividend.
Place a decimal point in the quotient above the decimal point in the dividend.
203/58 = 3.5 pounds

Performance Task

Question 25.
Isabella is buying art supplies. The table at the right shows the prices of the items she wants to buy.
A Isabella spends $2.25 on poster boards. How many poster boards does she buy?
Go Math Grade 5 Answer Key Chapter 5 Divide Decimals Chapter Review/Test img 18
_____ posters

Answer:
3 posters

Explanation:
1 Poster Board = $0.75.
Isabella spends $2.25 on poster boards.
$2.25/$0.75 = 3

Question 25.
B Isabella spends $4.87 on paintbrushes and paint. How many of each item does she buy? Explain how you found your answer.
_______ paint brushes
_______ jars of paint

Answer:
2 paint brushes
3 jars of paint

Explanation:
paintbrushes = $0.95
Paint = $0.99
If she buys 2 paint brushes and 2 paints, she spent $1.9 for paintbrushes and $1.98 for 2 paints. The remaining amount is $0.99. So, she can buy one more paint with them. So, she can buy 2 paint brushes and 3 jars of paint.

Question 25.
C. Isabella spends less than $14.00 for glass beads, paintbrushes, poster board, and paint. She spends $1.68 on beads and $3.96 on paint. She buys more than 3 poster boards and more than 3 paintbrushes. Find how many ounces of glass beads and how many jars of paint she buys. Then, suggest the number of poster boards and paintbrushes she might buy for the total spent.
Type below:
_________

Answer:
Isabella spends less than $14.00 for glass beads, paintbrushes, poster board, and paint.
She spends $1.68 on beads and $3.96 on paint.
Each beads = $0.28
$1.68/$0.28 = 6 beads
Each paint = $0.99
$3.96/$0.99 = 4 paints
$14 – ($1.68 + $3.96) = $8.36.
So, $8.36 needs to spend on more than 3 poster boards and more than 3 paintbrushes.
So, Each poster board = $0.75
paintbrushes = $0.95
If she buys more than 3, 3 x $0.75 = $2.25
$0.95 x 3 = $2.85
$2.25 + $2.85 = $5.1
So, $8.36 – $5.1 = $3.26
If $0.75 x 3 = $2.25
$0.95 x 1 = $0.95
$2.25 + $0.95 = 3.2
So, she can buy 3 + 3 = 6 poster boards
and 3 + 1 = 4 paintbrushes.

Conclusion

We have given Go Math Grade 5 Answer Key Chapter 5 Divide Decimals here for free of cost. Learn the Simple Tricks and Tips to Solve Various Models on Divide Decimals. Step by Step Solutions is provided for all the Questions to make it easy for you to understand.

Go Math Grade 8 Answer Key Chapter 9 Transformations and Congruence

go-math-grade-8-chapter-9-transformations-and-congruence-answer-key

There are multiple ways to learn maths. But choosing the best material is also important for the students to score good marks and also to improve their knowledge. This will be possible only with the help of our Go Math Grade 8 Answer Key Chapter 9 Transformations and Congruence. You can get the explanation for all the questions in detail in Go Math Grade 8 Chapter 9 Transformations and Congruence Answer Key here. Refer to Go Math Grade 8 Answer Key for learning the problems in an easy manner.

Go Math Grade 8 Answer Key Chapter 9 Transformations and Congruence

Practice makes you perfect become a master in maths. Without proper knowledge on the subject, you cannot solve the problems in real-time. This will affect your exam. So, keeping this in our mind we have prepared the solutions briefly which will help you to understand the concept in depth. Get free access to Download Go Math Grade 8 Chapter 9 Transformations and Congruence Solution Key pdf. In this article, you will get the solutions according to the topics. Therefore, students who want to score good marks in the exam must practice with Go Math Grade 8 Answer Key Chapter 9 Transformations and Congruence.

Lesson 1: Properties of Translations

Lesson 2: Properties of Reflections

Lesson 3: Algebraic Representations of Transformations

Lesson 4: Congruent Figures

Model Quiz

Mixed Review

Guided Practice – Properties of Translations – Page No. 282

Question 1.
Vocabulary A __________________is a change in the position, size, or shape of a figure.
____________

Answer:
transformation

Explanation:
A transformation is a change in the position, size, or shape of a figure.

Question 2.
Vocabulary When you perform a transformation of a figure on the coordinate plane, the input of the transformation is called the ________________, and the output of the transformation is called the_________________ .
Type below:
____________

Answer:
pre-image
image

Explanation:
When you perform a transformation of a figure on the coordinate plane, the input of the transformation is called the pre-image, and the output of the transformation is called the image.

Question 3.
Joni translates a right triangle 2 units down and 4 units to the right. How does the orientation of the image of the triangle compare with the orientation of the preimage?
Orientation is: _______

Answer:
Orientation is: Same

Explanation:
Since translation does not change the shape and size of a geometric figure, the two triangles are identical in shape and size, so they are congruent and the orientation is the same

Question 4.
Rashid drew rectangle PQRS on a coordinate plane. He then translated the rectangle 3 units up and 3 units to the left and labeled the image P ‘Q ‘R ‘S ‘. How do rectangle PQRS and rectangle P ‘Q ‘R ‘S ‘ compare?
They are: _______

Answer:
congruent

Explanation:
Since translation does not change the shape and size of a geometric figure, the two rectangles are identical in shape and size, so they are congruent.

Question 5.
The figure shows trapezoid WXYZ. Graph the image of the trapezoid after a translation of 4 units up and 2 units to the left.
Go Math Grade 8 Answer Key Chapter 9 Transformations and Congruence Lesson 1: Properties of Translations img 1
Type below:
____________

Answer:
After translation:
W'(-4, 3)
X'(2, 3)
Y'(1, 1)
Z'(-3, 1)

ESSENTIAL QUESTION CHECK-IN

Question 6.
What are the properties of translations?
Type below:
____________

Answer:
-> a translation is a geometric transformation that moves every point of a figure or space by the same amount in a given direction.
-> So the figures are identical and are congruent.

9.1 Independent Practice – Properties of Translations – Page No. 283

Question 7.
The figure shows triangle DEF.
Go Math Grade 8 Answer Key Chapter 9 Transformations and Congruence Lesson 1: Properties of Translations img 2
a. Graph the image of the triangle after the translation that maps point D to point D ‘.
Type below:
____________

Answer:
2 left, and 4 down

Question 7.
b. How would you describe the translation?
Type below:
____________

Answer:
It has the same size, shape. and orientation, but a different location

Question 7.
c. How does the image of triangle DEF compare with the preimage?
____________

Answer:
congruent

Question 8.
a. Graph quadrilateral KLMN with vertices K(-3, 2), L(2, 2), M(0, -3), and N(-4, 0) on the coordinate grid.
Go Math Grade 8 Answer Key Chapter 9 Transformations and Congruence Lesson 1: Properties of Translations img 3
Type below:
____________

Question 8.
b. On the same coordinate grid, graph the image of quadrilateral KLMN after a translation of 3 units to the right and 4 units up.
Type below:
____________

Answer:
grade 8 chapter 9 image 1

Question 8.
c. Which side of the image is congruent to side \(\overline { LM } \)?
___________
Name three other pairs of congruent sides.
___________
Type below:
____________

Answer:
Line LM is congruent to Line L!M!
Line KL is equal to K’L’
Line MN is equal to M’N’
Line KN is equal to K’N’

Draw the image of the figure after each translation.

Question 9.
4 units left and 2 units down
Go Math Grade 8 Answer Key Chapter 9 Transformations and Congruence Lesson 1: Properties of Translations img 4
Type below:
____________

Answer:
After translation
P'(-3, 1)
Q'(0, 2)
R'(0, -1)
S'(-3, -3)

Question 10.
5 units right and 3 units up
Go Math Grade 8 Answer Key Chapter 9 Transformations and Congruence Lesson 1: Properties of Translations img 5
Type below:
____________

Answer:
After translation
A'(0, 4)
B'(3, 5)
C'(3, 1)
D'(0, 0)

Properties of Translations – Page No. 284

Question 11.
The figure shows the ascent of a hot air balloon. How would you describe the translation?
Go Math Grade 8 Answer Key Chapter 9 Transformations and Congruence Lesson 1: Properties of Translations img 6
Type below:
____________

Answer:
4 units along positive X and 5 units along positive Y

Explanation:
Initial coordinate of balloon = ( -2 , -4)
Final coordinates of the balloon = (2,1)
Translation along x axis = 2 – (-2)
= 4 units along positive x direction
Translation along y axis = 1-(-4)
= 5 units along the positive y direction

Question 12.
Critical Thinking Is it possible that the orientation of a figure could change after it is translated? Explain.
_________

Answer:
No, it is not possible to change the orientation just by translation. As translation means, a transformation in which a figure is moved to another location without any change in size or orientation.

FOCUS ON HIGHER ORDER THINKING

Question 13.
a. Multistep Graph triangle XYZ with vertices X(-2, -5), Y(2, -2), and Z(4, -4) on the coordinate grid.
Go Math Grade 8 Answer Key Chapter 9 Transformations and Congruence Lesson 1: Properties of Translations img 7

Question 13.
b. On the same coordinate grid, graph and label triangle X’Y’Z’, the image of triangle XYZ after a translation of 3 units to the left and 6 units up.

Question 13.
c. Now graph and label triangle X”Y”Z”, the image of triangle X’Y’Z’ after a translation of 1 unit to the left and 2 units down.
Type below:
____________

Answer:
grade 8 chapter 9 image 2

Question 13.
d. Analyze Relationships How would you describe the translation that maps triangle XYZ onto triangle X”Y”Z”?
Type below:
____________

Answer:
Triangle XYZ has translated 4 units up and 4 units to the left

Question 14.
Critical Thinking The figure shows rectangle P’Q’R’S’, the image of rectangle PQRS after a translation of 5 units to the right and 7 units up. Graph and label the preimage PQRS.
Go Math Grade 8 Answer Key Chapter 9 Transformations and Congruence Lesson 1: Properties of Translations img 8
Type below:
____________

Answer:
grade 8 chapter 9 image 3

Question 15.
Communicate Mathematical Ideas Explain why the image of a figure after a translation is congruent to its preimage.
Type below:
____________

Answer:
A translation is a geometric transformation that moves every point of a figure or space by the same amount in a given direction. So the 2 figures are identical and the translated figure is congruent to its pre-image.

Guided Practice – Properties of Reflections – Page No. 288

Question 1.
Vocabulary A reflection is a transformation that flips a figure across a line called the __________ .
____________

Answer:
Reflection Axis

Explanation:
A reflection is a transformation that flips a figure across a line called the Reflection Axis.

Question 2.
The figure shows trapezoid ABCD.
Go Math Grade 8 Answer Key Chapter 9 Transformations and Congruence Lesson 2: Properties of Reflections img 9
a. Graph the image of the trapezoid after a reflection across the x-axis. Label the vertices of the image.
Type below:
____________

Answer:
A'(-3, -4)
B'(1, -4)
C'(3, -1)
D'(-3, -1)

Question 2.
b. How do trapezoid ABCD and trapezoid A’B’C’D’ compare?
____________

Answer:
congruent

Explanation:
trapezoid ABCD and trapezoid A’B’C’D’ are congruent

Question 2.
c. What If? Suppose you reflected trapezoid ABCD across the y-axis. How would the orientation of the image of the trapezoid compare with the orientation of the preimage?
Type below:
____________

Answer:
The orientation would be reversed horizontally.

ESSENTIAL QUESTION CHECK-IN

Question 3.
What are the properties of reflections?
Type below:
____________

Answer:
properties of reflections
-> the size stays the same
-> the shape stays the same
-> the orientation does NOT stay the same

9.2 Independent Practice – Properties of Reflections – Page No. 289

The graph shows four right triangles. Use the graph for Exercises 4-7.
Go Math Grade 8 Answer Key Chapter 9 Transformations and Congruence Lesson 2: Properties of Reflections img 10

Question 4.
Which two triangles are reflections of each other across the x-axis?
Type below:
____________

Answer:
Triangles A and C are the reflections of each other across the x-axis.

Question 5.
For which two triangles is the line of reflection the y-axis?
Type below:
____________

Answer:
For triangle C & D the line of reflection is y-axis.

Question 6.
Which triangle is a translation of triangle C? How would you describe the translation?
Type below:
____________

Answer:
Triangle B is the translation of triangle C.
Lets take any one point of the triangle = (-2, -6)
Lets take the corresponding side of triangle B = (4,2)
Translation across x axis = 4 -(-2) = 6 units
Translation across y axis = 2 -(-6) = 8 units

Question 7.
Which triangles are congruent? How do you know?
Type below:
____________

Answer:
All the 4 triangles A, B, C, D are congruent.
The length of base and height of all the four triangles are 3 units, 4 units respectively.

Explanation:
All the 4 triangles A, B, C, D are congruent.
If base and height are equal then the hypotenuse should also be equal. Thus all three sides of the triangles A,B,C,D are equal. Thus these triangles are congruent,
The length of base and height of all the four triangles are 3 units, 4 units respectively.

Question 8.
a. Graph quadrilateral WXYZ with vertices W(-2, -2), X(3, 1), Y(5, -1), and Z(4, -6) on the coordinate grid.
Type below:
____________

Question 8.
b. On the same coordinate grid, graph quadrilateral W’X’Y’Z’, the image of quadrilateral WXYZ after a reflection across the x-axis.
Type below:
____________

Answer:
grade 8 chapter 9 image 4

Question 8.
c. Which side of the image is congruent to side \(\overline { YZ } \)?
_______________
Name three other pairs of congruent sides.
_______________
Type below:
____________

Answer:
Line YZ = Line Y’Z’
Line WX = Line W’X’
Line XY = Line X’Y’
Line WZ = Line W’Z’

Question 8.
d. Which angle of the image is congruent to ∠X?
_______________
Name three other pairs of congruent angles.
_______________
Type below:
____________

Answer:
Angle X’
Angle W and Angle W’
Angle Y and Angle Y’
Angle Z and Angle Z’

Properties of Reflections – Page No. 290

Question 9.
Critical Thinking Is it possible that the image of a point after a reflection could be the same point as the preimage? Explain.
________

Answer:
Yes

Explanation:
It is possible that the image of a point after a reflection could be the same point as the preimage

FOCUS ON HIGHER ORDER THINKING

Question 10.
a. Graph the image of the figure shown after a reflection across the y-axis.
Go Math Grade 8 Answer Key Chapter 9 Transformations and Congruence Lesson 2: Properties of Reflections img 11
Type below:
____________

Answer:
grade 8 chapter 9 image 5

Question 10.
b. On the same coordinate grid, graph the image of the figure you drew in part a after a reflection across the x-axis.
Type below:
____________

Answer:
grade 8 chapter 9 image 6

Question 10.
c. Make a Conjecture What other sequence of transformations would produce the same final image from the original preimage? Check your answer by performing the transformations. Then make a conjecture that generalizes your findings.
Type below:
____________

Answer:
The same image can be obtained by reflecting first across the x-axis and then across the y-axis.
Reflecting a figure first across the y-axis and then across the x-axis has the same outcome,. reflecting first across the x-axis and then across the y-axis.

Question 11.
a. Graph triangle DEF with vertices D(2, 6), E(5, 6), and F(5, 1) on the coordinate grid.

Question 11.
b. Next graph triangle D ′E ′F ′, the image of triangle DEF after a reflection across the y-axis.
Type below:
____________

Question 11.
c. On the same coordinate grid, graph triangle D′′ E′′ F′′, the image of triangle D ′E ′F ′ after a translation of 7 units down and 2 units to the right.
Type below:
____________

Answer:
grade 8 chapter 9 image 7

Question 11.
d. Analyze Relationships Find a different sequence of transformations that will transform triangle DEF to triangle D ′′E ′′F ′′.
Type below:
____________

Answer:
Translate triangle DEF 7 units down and 2 units to the left. Then reflect the image across the y-axis.

Guided Practice – Properties of Reflections – Page No. 294

Question 1.
Vocabulary A rotation is a transformation that turns a figure around a given _____ called the center of rotation.
____________

Answer:
point

Explanation:
A rotation is a transformation that turns a figure around a given point called the center of rotation.

Siobhan rotates a right triangle 90° counterclockwise about the origin.

Question 2.
How does the orientation of the image of the triangle compare with the orientation of the preimage?
Type below:
____________

Answer:
Each leg in the preimage is perpendicular to its corresponding leg in the image.

Question 3.
Is the image of the triangle congruent to the preimage?
______

Answer:
Yes

Explanation:
The image of the triangle is congruent to the preimage

Draw the image of the figure after the given rotation about the origin.

Question 4.
90° counterclockwise
Go Math Grade 8 Answer Key Chapter 9 Transformations and Congruence Lesson 3: Properties of Rotation img 12
Type below:
____________

Answer:
grade 8 chapter 9 image 8

Question 5.
180°
Go Math Grade 8 Answer Key Chapter 9 Transformations and Congruence Lesson 3: Properties of Rotation img 13
Type below:
____________

Answer:
After 180° rotation
A'(-2, -3)
B'(-4, -1)
C'(-2, 0)
D'(0, -1)

ESSENTIAL QUESTION CHECK-IN

Question 6.
What are the properties of rotations?
Type below:
____________

Answer:
Rotations preserve size and shape but change orientation.

9.3 Independent Practice – Properties of Reflections – Page No. 295

Question 7.
The figure shows triangle ABC and a rotation of the triangle about the origin.
Go Math Grade 8 Answer Key Chapter 9 Transformations and Congruence Lesson 3: Properties of Rotation img 14
a. How would you describe the rotation?
____________

Answer:
ABC was rotated 90º counterclockwise about the origin

Question 7.
b. What are the coordinates of the image?
Type below:
____________

Answer:
A'(3, 1)
B'(2, 3)
C'(-1, 4)

Question 8.
The graph shows a figure and its image after a transformation.
Go Math Grade 8 Answer Key Chapter 9 Transformations and Congruence Lesson 3: Properties of Rotation img 15
a. How would you describe this as a rotation?
____________

Answer:
The figure was rotated 180º about the origin.

Question 8.
b. Can you describe this as a transformation other than a rotation? Explain.
____________

Answer:
Yes

Explanation:
This can also be described as a reflection across the y-axis.

Question 9.
What type of rotation will preserve the orientation of the H-shaped figure in the grid?
Go Math Grade 8 Answer Key Chapter 9 Transformations and Congruence Lesson 3: Properties of Rotation img 16
____________

Answer:
A 180º rotation about the origin will preserve the orientation of the H-shaped figure in the grid.

Question 10.
A point with coordinates (-2, -3) is rotated 90° clockwise about the origin. What are the coordinates of its image?
(_______ , _______)

Answer:
(-3, 2)

Explanation:
The new coordinates are (-3, 2)

Complete the table with rotations of 180° or 90°. Include the direction of rotation for rotations of 90°.

Question 11.
Go Math Grade 8 Answer Key Chapter 9 Transformations and Congruence Lesson 3: Properties of Rotation img 17
Type below:
____________

Answer:
grade 8 chapter 9 image 10

Properties of Reflections – Page No. 296

Draw the image of the figure after the given rotation about the origin.

Question 14.
180°
Go Math Grade 8 Answer Key Chapter 9 Transformations and Congruence Lesson 3: Properties of Rotation img 18
Type below:
____________

Answer:
After 180°
A'(4, 0)
B'(2, -1)
C'(0, 0)
D'(2, 1)

Question 15.
270° counterclockwise
Go Math Grade 8 Answer Key Chapter 9 Transformations and Congruence Lesson 3: Properties of Rotation img 19
Type below:
____________

Answer:
After 270º counterclockwise rotation
A'(1, 2)
B'(2, -1)
C'(4, 2)

Question 16.
Is there a rotation for which the orientation of the image is always the same as that of the preimage? If so, what?
______

Answer:
Yes

Explanation:
A 360º rotation will always be the same as the original image

FOCUS ON HIGHER ORDER THINKING

Question 17.
Problem Solving Lucas is playing a game where he has to rotate a figure for it to fit in an open space. Every time he clicks a button, the figure rotates 90 degrees clockwise. How many times does he need to click the button so that each figure returns to its original orientation?
Figure A ____________
Figure B ____________
Figure C ____________
Go Math Grade 8 Answer Key Chapter 9 Transformations and Congruence Lesson 3: Properties of Rotation img 20
Figure A: _________ time(s)
Figure B: _________ time(s)
Figure C: _________ time(s)

Answer:
Figure A: 2 time(s)
Figure B: 1 time(s)
Figure C: 4 time(s)

Explanation:
2 times to return to original orientation
1 time to return to original orientation
4 times to return to original orientation

Question 18.
Make a Conjecture Triangle ABC is reflected across the y-axis to form the image A′B′C′. Triangle A′B′C′ is then reflected across the x-axis to form the image A″B″C″. What type of rotation can be used to describe the relationship between triangle A″B″C″ and triangle ABC?
Type below:
____________

Answer:
Triangle A’B’C’ is a 90º rotation of triangle ABC
Triangle A”B”C” is a 90º rotation of triangle A’B’C’
Therefore, Triangle A”B”C” is a 180º rotation of triangle ABC

Question 19.
Communicate Mathematical Ideas Point A is on the y-axis. Describe all possible locations of image A′ for rotations of 90°, 180°, and 270°. Include the origin as a possible location for A.
Type below:
____________

Answer:
If Point A is on the y-axis, Point A’ will be on the x-axis for 190° and 270° rotations and on the y-axis for 180° rotation
If point A is at the origin,
A’ is at the origin for any rotation about the origin.

Guided Practice – Algebraic Representations of Transformations – Page No. 300

Question 1.
Triangle XYZ has vertices X(-3, -2), Y(-1, 0), and Z(1, -6). Find the vertices of triangle X′Y′Z′ after a translation of 6 units to the right. Then graph the triangle and its image.
Go Math Grade 8 Answer Key Chapter 9 Transformations and Congruence Lesson 4: Algebraic Representations of Transformations img 21
Type below:
____________

Answer:
After a translation of 6 units to the right:
X'(3, -2)
Y'(5, 0)
Z'(7, -6)

Question 2.
Describe what happens to the x- and y-coordinates after a point is reflected across the x-axis.
Type below:
____________

Answer:
The x-coordinate remains the same, while the sign of the y-coordinate changes

Question 3.
Use the rule (x, y) → (y, -x) to graph the image of the triangle at right. Then describe the transformation.
Go Math Grade 8 Answer Key Chapter 9 Transformations and Congruence Lesson 4: Algebraic Representations of Transformations img 22
Type below:
____________

Answer:
The triangle is rotated 90º clockwise about the origin

ESSENTIAL QUESTION CHECK-IN

Question 4.
How do the x- and y-coordinates change when a figure is translated right a units and down b units?
Type below:
____________

Answer:
The x-coordinates increase by a, and the y-coordinates decrease by b

9.4 Independent Practice – Algebraic Representations of Transformations – Page No. 301

Write an algebraic rule to describe each transformation.Then describe the transformation.

Question 5.
Go Math Grade 8 Answer Key Chapter 9 Transformations and Congruence Lesson 4: Algebraic Representations of Transformations img 23
Type below:
____________

Answer:
algebraic rule
(x, y) -> (x-2, y-5)
transformation
translation of 2 units to the left and 5 units down
new coordinates
M'(-4, -2)
N'(-2, -2)
O'(-1, -4)
P'(-4, -4)

Question 6.
Go Math Grade 8 Answer Key Chapter 9 Transformations and Congruence Lesson 4: Algebraic Representations of Transformations img 24
Type below:
____________

Answer:
algebraic rule
(x, y) -> (-x, -y)
transformation
rotation of 180º
new coordinates
A'(0, 0)
B'(0, -3)
C'(2, -3)
D'(2, 0)

Question 7.
Triangle XYZ has vertices X(6, -2.3), Y(7.5, 5), and Z(8, 4). When translated, X′ has coordinates (2.8, -1.3). Write a rule to describe this transformation. Then find the coordinates of Y′ and Z′.
Type below:
____________

Answer:
algebraic rule
(x, y) -> (x-3.2, y+1)
new coordinates
Y'(4.3, 6)
Z'(4.8, 5)

Question 8.
Point L has coordinates (3, -5). The coordinates of point L′ after a reflection are (-3, -5). Without graphing, tell which axis point L was reflected across. Explain your answer.
____________

Answer:
Point L was reflected on the y-axis.
When you reflect a point across the y-axis, the sign of the x-coordinate changes, and the sign of the y-coordinate remains the same

Question 9.
Use the rule (x, y) → (x – 2, y – 4) to graph the image of the rectangle. Then describe the transformation.
Go Math Grade 8 Answer Key Chapter 9 Transformations and Congruence Lesson 4: Algebraic Representations of Transformations img 25
Type below:
____________

Answer:
The rectangle is translated 2 units to the left and 4 units down

Question 10.
Parallelogram ABCD has vertices A(−2, −5\(\frac{1}{2}\)), B(−4, −5\(\frac{1}{2}\)),C(-3, -2), and D(-1, -2). Find the vertices of parallelogram A′B′C′D′ after a translation of 2 \(\frac{1}{2}\) units down.
Type below:
__________

Answer:
after a translation of 2 \(\frac{1}{2}\) units
A'(-2, -8)
B'(-4, -8)
C'(-3, -4 \(\frac{1}{2}\))
D'(-1, -4 \(\frac{1}{2}\))

Algebraic Representations of Transformations – Page No. 302

Question 11.
Alexandra drew the logo shown on half-inch graph paper. Write a rule that describes the translation Alexandra used to create the shadow on the letter A.
Go Math Grade 8 Answer Key Chapter 9 Transformations and Congruence Lesson 4: Algebraic Representations of Transformations img 26
Type below:
__________

Answer:
(x,y) –> (x+1,y-0.5)
(x+1,y-0.5) –> (x+0.5,y-0.25)

Explanation:
translation in units
(x,y) –> (x+1,y-0.5)
This step converts translation rule in units to translation rule in inches. (Divide by 2 since graph paper is half inch paper.
(x+1,y-0.5) –> (x+0.5,y-0.25)

Question 12.
Kite KLMN has vertices at K(1, 3), L(2, 4), M(3, 3), and N(2, 0). After the kite is rotated, K′ has coordinates (-3, 1). Describe the rotation, and include a rule in your description. Then find the coordinates of L′, M′, and N′.
Type below:
__________

Answer:
rotation
90 counterclockwise
rule
(x, y) -> (-y, x)
new coordinates
L'(-4, 2)
M'(-3, 3)
N'(0, 2)

FOCUS ON HIGHER ORDER THINKING

Question 13.
Make a Conjecture Graph the triangle with vertices (-3, 4), (3, 4), and (-5, -5). Use the transformation (y, x) to graph its image.
a. Which vertex of the image has the same coordinates as a vertex of the original figure? Explain why this is true.
Go Math Grade 8 Answer Key Chapter 9 Transformations and Congruence Lesson 4: Algebraic Representations of Transformations img 27
Type below:
__________

Answer:
(-5, 5) has the same coordinates

Question 13.
b. What is the equation of a line through the origin and this point?
Type below:
__________

Answer:
x and y are equal so switching x and y has no effect on the coordinates

Question 13.
c. Describe the transformation of the triangle.
Type below:
__________

Answer:
x and y are equal so switching x and y has no effect on the coordinates

Question 14.
Critical Thinking Mitchell says the point (0, 0) does not change when reflected across the x- or y-axis or when rotated about the origin. Do you agree with Mitchell? Explain why or why not.
_______

Answer:
Yes, I agree with Mitchell

Explanation:
Reflecting across the x-axis or y-axis changes the sign of the y or x coordinate 0 cannot change signs.
Rotating about the origin does not change the origin (0, 0)

Question 15.
Analyze Relationships Triangle ABC with vertices A(-2, -2), B(-3, 1), and C(1, 1) is translated by (x, y) → (x – 1, y + 3). Then the image, triangle A′B′C′, is translated by (x, y) → (x + 4, y – 1), resulting in A″B″C″.
a. Find the coordinates for the vertices of triangle A″B″C″.
Type below:
__________

Answer:
A”(-2-1+4, -2+3-1) = A”(1, 0)
B”(-3-1+4, 1+3-1) = B”(0, 3)
C”(1-1+4, 1+3-1) = C”(4, 3)

Question 15.
b. Write a rule for one translation that maps triangle ABC to triangle A″B″C″.
Type below:
__________

Answer:
(x, y) -> (x-1+4, y+3-1)
(x, y) -> (x+3, y+2)

Guided Practice – Congruent Figures – Page No. 306

Question 1.
Apply the indicated series of transformations to the rectangle. Each transformation is applied to the image of the previous transformation, not the original figure. Label each image with the letter of the transformation applied.
Go Math Grade 8 Answer Key Chapter 9 Transformations and Congruence Lesson 5: Congruent Figures img 28
A. Reflection across the y-axis
B. Rotation 90° clockwise around the origin
C. (x, y) → (x – 2, y)
D. Rotation 90° counterclockwise around the origin
E. (x, y) → (x – 7, y – 2)
Type below:
__________

Answer:
A. After transformation
(1, 3)
(1, 4)
(4, 4)
(4, 3)
B. After transformation
(3, -1)
(4, -1)
(4, -4)
(3, -4)
C. After transformation
(1, -1)
(2, -1)
(2, -4)
(1, -4)
D. After transformation
(1, 1)
(1, 2)
(4, 2)
(4, 1)
E. After transformation
(-6, -1)
(-6, 0)
(-3, 0)
(-3, -1)

Identify a sequence of transformations that will transform figure A into figure C.
Go Math Grade 8 Answer Key Chapter 9 Transformations and Congruence Lesson 5: Congruent Figures img 29

Question 2.
What transformation is used to transform figure A into figure B?
Type below:
__________

Answer:
Reflection across the y-axis

Explanation:
Reflection across the y-axis is used to transform figure A into figure B

Question 3.
What transformation is used to transform figure B into figure C?
Type below:
__________

Answer:
Translation 3 units right and 4 units down

Explanation:
Translation 3 units right and 4 units down is used to transform figure B into figure C

Question 4.
What sequence of transformations is used to transform figure A into figure C? Express the transformations algebraically.
Type below:
__________

Answer:
Reflection across the y-axis is used to transform figure A into figure B
Translation 3 units right and 4 units down is used to transform figure B into figure C
Algebraically:
(x, y) -> (-x, y)
(x, y) -> (x +3, y-4)

Question 5.
Vocabulary What does it mean for two figures to be congruent?
Type below:
__________

Answer:
Two figures are congruent when the figures have the same size and the same shape.

ESSENTIAL QUESTION CHECK-IN

Question 6.
After a sequence of translations, reflections, and rotations, what is true about the first figure and the final figure?
Type below:
__________

Answer:
After a sequence of translations, reflections, and rotations, the first and final figures have the same size and shape. (They are congruent)

9.5 Independent Practice – Congruent Figures – Page No. 307

For each given figure A, graph figures B and C using the given sequence of transformations. State whether figures A and C have the same or different orientation.

Question 7.
Go Math Grade 8 Answer Key Chapter 9 Transformations and Congruence Lesson 5: Congruent Figures img 30
Figure B: a translation of 1 unit to the right and 3 units up
Figure C: a 90° clockwise rotation around the origin
Type below:
__________

Answer:
Different orientation

Explanation:
grade 8 chapter 9 image 11
Different orientation

Question 8.
Go Math Grade 8 Answer Key Chapter 9 Transformations and Congruence Lesson 5: Congruent Figures img 31
Figure B: a reflection across the y-axis
Figure C: a 180° rotation around the origin
Type below:
__________

Answer:
Different orientation

Explanation:
grade 8 chapter 9 image 12
Different orientation

Question 9.
Go Math Grade 8 Answer Key Chapter 9 Transformations and Congruence Lesson 5: Congruent Figures img 32
Figure B: a reflection across the y-axis
Figure C: a translation 2 units down
Type below:
__________

Answer:
Different orientation

Explanation:
grade 8 chapter 9 image 13
Different orientation

Question 10.
Go Math Grade 8 Answer Key Chapter 9 Transformations and Congruence Lesson 5: Congruent Figures img 33
Figure B: a translation 2 units up
Figure C: a rotation of 180° around the origin
Type below:
__________

Answer:
Different orientation

Explanation:
grade 8 chapter 9 image 14
Different orientation

Congruent Figures – Page No. 308

Question 11.
Represent Real-World Problems A city planner wanted to place the new town library at site A. The mayor thought that it would be better at site B. What transformations were applied to the building at site A to relocate the building to site B? Did the mayor change the size or orientation of the library?
Go Math Grade 8 Answer Key Chapter 9 Transformations and Congruence Lesson 5: Congruent Figures img 34
Type below:
__________

Answer:
From Site A to Site B: Translation 2 units right and 4 units down
The size did NOT change
The orientation changed

Question 12.
Persevere in Problem Solving Find a sequence of three transformations that can be used to obtain figure D from figure A. Graph the figures B and C that are created by the transformations.
Go Math Grade 8 Answer Key Chapter 9 Transformations and Congruence Lesson 5: Congruent Figures img 35
Type below:
__________

Answer:
From figure A to D:
Reflection across the x-axis (-1, -5) (-1, -6) (2, -5) (4, -6)
90º clockwise rotation (4, -1) (5, -1) (5, -4) (4, -6)
translation 6 units left (4, -1) (5, -1) (5, -4) (4, -6)

FOCUS ON HIGHER ORDER THINKING

Question 13.
Counterexamples The Commutative Properties for Addition and Multiplication state that the order of two numbers being added or multiplied does not change the sum or product. Are translations and rotations commutative? If not, give a counterexample.
________

Answer:
No, Translation and rotations are not commutative

Explanation:
The point (2, 2) becomes (2, -4) when translated 2 units to the right then rotated 90 around the origin.
The point (2, 2) becomes (4, -2) when rotated 90 around the origin then translated 2 units to the right.
The above two points are not the same.

Question 14.
Multiple Representations For each representation, describe a possible sequence of transformations.
a. (x, y) → (-x – 2, y + 1)
Type below:
____________

Answer:
translation 2 units right and 1 unit up
reflection across y-axis

Question 14.
b. (x, y) → (y, -x – 3)
Type below:
____________

Answer:
rotation 90º clockwise around the origin
translation 3 units down

Ready to Go On? – Model Quiz – Page No. 309

9.1–9.3 Properties of Translations, Reflections, and Rotations

Question 1.
Graph the image of triangle ABC after a translation of 6 units to the right and 4 units down. Label the vertices of the image A’, B’, and C’.
Go Math Grade 8 Answer Key Chapter 9 Transformations and Congruence Model Quiz img 36
Type below:
____________

Answer:
After translation:
A'(2, 1)
B'(2, -1)
C'(5, -1)

Question 2.
On the same coordinate grid, graph the image of triangle ABC after a reflection across the x-axis. Label the vertices of the image A”, B”, and C”.
Type below:
____________

Answer:
After reflection:
A”(-4, -5)
B”(-4, -3)
C”(-1, -3)

Question 3.
Graph the image of HIJK after it is rotated 180° about the origin. Label the vertices of the image H’I’J’K’.
Go Math Grade 8 Answer Key Chapter 9 Transformations and Congruence Model Quiz img 37
Type below:
____________

Answer:
After rotation:
H'(0, -4)
I'(0, -1)
J'(2, -2)
K'(2, -3)

9.4 Algebraic Representations of Transformations

Question 4.
A triangle has vertices at (2, 3), (−2, 2), and (−3, 5). What are the coordinates of the vertices of the image after the translation (x, y) → (x + 4, y − 3)?
Type below:
____________

Answer:
After translation:
(6, 0)
(2, -1)
(1, 2)

9.5 Congruent Figures

Question 5.
Vocabulary Translations, reflections, and rotations produce a figure that is _____ to the original figure.
Type below:
____________

Answer:
congruent

Explanation:
Vocabulary Translations, reflections, and rotations produce a figure that is congruent to the original figure.

Question 6.
Use the coordinate grid for Exercise 3. Reflect H’I’J’K’ over the y-axis, then rotate it 180° about the origin. Label the new figure H″I″J″K″.
Type below:
____________

Answer:
after reflection
H'(0, -4)
I'(0, -1)
J'(-2, -2)
K'(-2, -3)
after rotation
H”(0, 4)
I”(0, 1)
J”(2, 2)
K”(2, 3)

ESSENTIAL QUESTION

Question 7.
How can you use transformations to solve real-world problems?
Type below:
____________

Answer:
Transformational properties allow the systematic movement of congruent figures while maintaining or adjusting their orientation.

Selected Response – Mixed Review – Page No. 310

Question 1.
What would be the orientation of the figure L after a translation of 8 units to the right and 3 units up?
Go Math Grade 8 Answer Key Chapter 9 Transformations and Congruence Mixed Review img 38
Options:
a. A
b. B
c. C
d. D

Answer:
c. C

Explanation:
After a translation of 8 units right and 3 units up, the orientation of figure L stays the same.

Question 2.
Figure A is reflected over the y-axis and then lowered 6 units. Which sequence describes these transformations?
Options:
a. (x, y) -> (x, -y) and (x, y) -> (x, y – 6)
b. (x, y) -> (-x, y) and (x, y) -> (x, y – 6)
c. (x, y) -> (x, -y) and (x, y) -> (x – 6, y)
d. (x, y) -> (-x, y) and (x, y) -> (x – 6, y)

Answer:
b. (x, y) -> (-x, y) and (x, y) -> (x, y – 6)

Explanation:
reflection over y-axis:
(x, y) -> (-x, y)
Translation 6 units down
(x, y) -> (x, y-6)

Question 3.
What quadrant would the triangle be in after a rotation of 90° counterclockwise about the origin?
Go Math Grade 8 Answer Key Chapter 9 Transformations and Congruence Mixed Review img 39
Options:
a. I
b. II
c. III
d. IV

Answer:
d. IV

Explanation:
After a rotation of 90° counterclockwise about the origin, the triangle will be in QIV

Question 4.
Which rational number is greater than −3 \(\frac{1}{3}\) but less than −\(\frac{4}{5}\)?
Options:
a. −0.4
b. −\(\frac{9}{7}\)
c. −0.19
d. −\(\frac{22}{5}\)

Answer:
b. −\(\frac{9}{7}\)

Question 5.
Which of the following is not true of a trapezoid that has been reflected across the x-axis?
Options:
a. The new trapezoid is the same size as the original trapezoid.
b. The new trapezoid is the same shape as the original trapezoid.
c. The new trapezoid is in the same orientation as the original trapezoid.
d. The x-coordinates of the new trapezoid are the same as the x-coordinates of the original trapezoid.

Answer:
d. The x-coordinates of the new trapezoid are the same as the x-coordinates of the original trapezoid.

Explanation:
The x-coordinates of the new trapezoid are the same as the x-coordinates of the original trapezoid.

Question 6.
A triangle with coordinates (6, 4), (2, −1), and (−3, 5) is translated 4 units left and rotated 180° about the origin. What are the coordinates of its image?
Options:
a. (2, 4), (-2, -1), (-7, 5)
b. (4, 6), (-1, 2), (5, -3)
c. (4, -2), (-1, 2), (5, 7)
d. (-2, -4), (2, 1), (7, -5)

Answer:
d. (-2, -4), (2, 1), (7, -5)

Question 7.
A rectangle with vertices (3, -2), (3, -4), (7, -2), (7, -4) is reflected across the x-axis and then rotated 90° counterclockwise.
a. In what quadrant does the image lie?
____________

Answer:
After reflection and rotation, the image lies in QII

Question 7.
b. What are the vertices of the image?
Type below:
____________

Answer:
image vertices
(-2, 3)
(-4, 3)
(-2, 7)
(-4, 7)

Question 7.
c. What other transformations produce the same image?
Type below:
____________

Answer:
A reflection across the y-axis and 90º clockwise rotation will produce the same result.

Conclusion:

The Go Math Grade 8 Answer Key Chapter 9 Transformations and Congruence pdf are available both online and offline. We have provided in pdf format so that students can practice the problems offline. We know that maths is the scoring and also typical among all the subjects. But you can make it easy if you understand the concept of the chapter. Students can refer to the Go Math Grade 8 Answer Key in their convenient way. Practice well and make maths your favorite subject. Best of Luck!!!

Go Math Grade 6 Answer Key Chapter 2 Fractions and Decimals

go-math-grade-6-chapter-2-fractions-and-decimals-answer-key

Are you looking for the best material to score top in the exams? Then, you are in the right place. HMH Go Math Grade 6 Answer Key Chapter 2 Fractions and Decimals is the best material for the 6th standard students. Here you can find the explanations for each and every question in different methods. Refer to Go Math Grade 6 Chapter 2 Fractions and Decimals Solution Key to learn the concepts of the chapter. So, Download HMH Go Math Grade 6 Answer Key Chapter 2 Fractions and Decimals for free.

Download Go Math Grade 6 Chapter 2 Fractions and Decimals Answer Key PDF

The Go Math Grade 6 Chapter 2 Fractions and Decimals Solution Key consists of various topics like compare and order fractions and decimals, multiply fractions, Divide Fractions, Model Mixed Number Division, etc. We have provided detailed explanations in simple methods here. All the solutions are prepared according to the topics in the Fractions and Decimals Chapter. So, access the links and start your preparation for the exams.

Lesson 1: Fractions and Decimals

Lesson 2: Compare and Order Fractions and Decimals

Lesson 3: Multiply Fractions

Lesson 4: Simplify Factors

Mid-Chapter Checkpoint

Lesson 5: Investigate • Model Fraction Division

Lesson 6: Estimate Quotients

Lesson 7: Divide Fractions

Lesson 8: Investigate • Model Mixed Number Division

Lesson 9: Divide Mixed Numbers

Lesson 10: Problem Solving • Fraction Operations

Chapter 2 Review/Test

Share and Show – Page No. 71

Write as a fraction or as a mixed number in simplest form.

Question 1.
95.5
_____ \(\frac{□}{□}\)

Answer:
\(\frac{1}{2}\)

Explanation:
95.5
95.5 is 95 ones and 5 tenths.
5 tenths = \(\frac{5}{10}\)
Simplify using the GCF.
The GCF of 5 and 10 is 10.
Divide the numerator and the denominator by 10
\(\frac{5 ÷ 10}{10 ÷ 10}\) = \(\frac{1}{2}\)

Question 2.
0.6
\(\frac{□}{□}\)

Answer:
\(\frac{3}{5}\)

Explanation:
0.6
6 tenths = \(\frac{6}{10}\)
Simplify using the GCF.
The GCF of 6 and 10 is 2.
Divide the numerator and the denominator by 10
\(\frac{6 ÷ 2}{10 ÷ 2}\) = \(\frac{3}{5}\)

Question 3.
5.75
_____ \(\frac{□}{□}\)

Answer:
5\(\frac{3}{4}\)

Explanation:
5.75 is 5 ones and 75 hundredths.
75 hundredths = \(\frac{75}{100}\)
Simplify using the GCF.
The GCF of 75 and 100 is 25.
Divide the numerator and the denominator by 25
5\(\frac{75 ÷ 25}{100 ÷ 25}\) = 5\(\frac{3}{4}\)

Write as a decimal.

Question 4.
\(\frac{7}{8}\)
_____

Answer:
0.875

Explanation:
Use division to rename the fraction part as a decimal.
7/8 = 0.875
The quotient has 3 decimal places.
Add the whole number to the decimal.
0 + 0.875 = 0.875.
So, \(\frac{7}{8}\) = 0.875

Question 5.
\(\frac{13}{20}\)
_____

Answer:
0.65

Explanation:
Use division to rename the fraction part as a decimal.
\(\frac{13}{20}\) = 0.65
The quotient has 2 decimal places.
Add the whole number to the decimal.
0 + 0.65 = 0.65.
So, \(\frac{13}{20}\) = 0.65

Question 6.
\(\frac{3}{25}\)
_____

Answer:
0.12

Explanation:
Use division to rename the fraction part as a decimal.
\(\frac{3}{25}\) = 0.12
The quotient has 2 decimal places.
Add the whole number to the decimal.
0 + 0.12 = 0.12.
So, \(\frac{3}{25}\)= 0.12

On Your Own

Write as a fraction or as a mixed number in simplest form.

Question 7.
0.27
\(\frac{□}{□}\)

Answer:
\(\frac{27}{100}\)

Explanation:
0.27 is 0 ones and 27 hundredths.
27 hundredths = \(\frac{27}{100}\)
Simplify using the GCF.
The GCF of 27 and 100 is 1.
Divide the numerator and the denominator by 1
\(\frac{27 ÷ 1}{100 ÷ 1}\) = \(\frac{27}{100}\)

Question 8.
0.055
\(\frac{□}{□}\)

Answer:
\(\frac{11}{200}\)

Explanation:
0.055 is 0 ones and 55 thousandths.
55 thousandths = \(\frac{55}{1000}\)
Simplify using the GCF.
The GCF of 55 and 1000 is 5.
Divide the numerator and the denominator by 5
\(\frac{55 ÷ 5}{1000 ÷ 5}\) = \(\frac{11}{200}\)

Question 9.
2.45
_____ \(\frac{□}{□}\)

Answer:
\(\frac{9}{20}\)

Explanation:
2.45 is 2 ones and 45 hundredths.
45 hundredths = \(\frac{45}{100}\)
Simplify using the GCF.
The GCF of 45 and 100 is 5.
Divide the numerator and the denominator by 1
\(\frac{45 ÷ 5}{100 ÷ 5}\) = \(\frac{9}{20}\)

Write as a decimal.

Question 10.
\(\frac{3}{8}\)
_____

Answer:
0.375

Explanation:
Use division to rename the fraction part as a decimal.
\(\frac{3}{8}\) = 0.375
The quotient has 3 decimal places.
Add the whole number to the decimal.
0 + 0.375 = 0.375.
So, \(\frac{3}{8}\) = 0.375

Question 11.
3 \(\frac{1}{5}\)
_____

Answer:
3.2

Explanation:
Use division to rename the fraction part as a decimal.
\(\frac{1}{5}\) = 0.2
The quotient has 1 decimal place.
Add the whole number to the decimal.
3 + 0.2 = 3.2.
So, 3 \(\frac{1}{5}\) = 3.2

Question 12.
2 \(\frac{11}{20}\)
_____

Answer:
2.55

Explanation:
Use division to rename the fraction part as a decimal.
\(\frac{11}{20}\) = 0.55
The quotient has 2 decimal places.
Add the whole number to the decimal.
2 + 0.55 = 2.55.
So, 2 \(\frac{11}{20}\) = 2.55

Identify a decimal and a fraction in simplest form for the point.
Go Math Grade 6 Answer Key Chapter 2 Fractions and Decimals 1

Question 13.
Point A
Type below:
__________

Answer:
0.2

Question 14.
Point B
Type below:
__________

Answer:
0.9

Explanation:
Point B is between 0.8 and 1.0. Every point is separated by 0.1. So, Point B is at 0.9

Question 15.
Point C
Type below:
__________

Answer:
0.5

Explanation:
Point C is between 0.4 and 0.6. Every point is separated by 0.1. So, Point C is at 0.5

Question 16.
Point D
Type below:
__________

Answer:
0.1

Explanation:
Point D is between 0 and 0.2. Every point is separated by 0.1. So, Point D is at 0.1

Problem Solving + Applications – Page No. 72

Use the table for 17 and 18.
Go Math Grade 6 Answer Key Chapter 2 Fractions and Decimals 2

Question 17.
Members of the Ozark Trail Hiking Club hiked a steep section of the trail in June and July. The table shows the distances club members hiked in miles. Write Maria’s July distance as a decimal.
_____ miles

Answer:
2.625 miles

Explanation:
Maria’s July distance = 2 \(\frac{5}{8}\)
Use division to rename the fraction part as a decimal.
\(\frac{5}{8}\) = 0.625
The quotient has 3 decimal places.
Add the whole number to the decimal.
2 + 0.625 = 2.625.
2 \(\frac{5}{8}\) = 2.625

Question 18.
How much farther did Zoey hike in June and July than Maria hiked in June and July? Explain how you found your answer.
_____ miles

Answer:
0.7 miles

Explanation:
Maria: June = 2.95, July = 2 \(\frac{5}{8}\) = 2.58
Zoey: June = 2.85, July = 3 \(\frac{3}{8}\) = 3.38
[2.85 + 3.38] – [2.95 + 2.58] = 0.7 miles

Question 19.
What’s the Error? Tabitha’s hiking distance in July was 2 \(\frac{1}{5}\) miles. She wrote the distance as 2.02 miles. What error did she make?
Type below:
__________

Answer:
Tabitha’s hiking distance in July was 2 \(\frac{1}{5}\) miles.
2 \(\frac{1}{5}\)
Use division to rename the fraction part as a decimal.
\(\frac{1}{5}\)  = 0.2
The quotient has 1 decimal place.
Add the whole number to the decimal.
2 + 0.2 = 2.2.
2 \(\frac{1}{5}\) = 2.2
She wrote the distance as 2.02 miles in mistake.

Question 20.
Use Patterns Write \(\frac{3}{8}, \frac{4}{8}, \text { and } \frac{5}{8}\) as decimals. What pattern do you see? Use the pattern to predict the decimal form of \(\frac{6}{8}\) and \(\frac{7}{8}\).
Type below:
__________

Answer:
\(\frac{3}{8}, \frac{4}{8}, \text { and } \frac{5}{8}\) as decimals.
0.375, 0.5, 0.625
Each decimal is separated by 0.125.
So, 6/8 = 0.625 + 0.125 = 0.75
7/8 = 0.75 + 0.125 = 0.875

Question 21.
Identify a decimal and a fraction in simplest form for the point.
Go Math Grade 6 Answer Key Chapter 2 Fractions and Decimals 3
Type below:
__________

Answer:
Point A: 0.5
Point B: 0.7
Point C: 0.3
Point D: 0.8

Explanation:
Every point is differentiated by 0.1 distance.
The A is between 0.4 and 0.6 which is 0.5
The B is between 0.6 and 0.8 which is 0.7
The C is between 0.1 and 0.6 which is 0.53

Fractions and Decimals – Page No. 73

Write as a fraction or as a mixed number in simplest form.

Question 1.
0.52
\(\frac{□}{□}\)

Answer:
\(\frac{13}{25}\)

Explanation:
0.52
0.52 is 52 hundredths.
52 hundredths = \(\frac{52}{100}\)
Simplify using the GCF.
The GCF of 52 and 100 is 4.
Divide the numerator and the denominator by 4
\(\frac{52 ÷ 4}{100 ÷ 4}\) = \(\frac{13}{25}\)

Question 2.
0.02
\(\frac{□}{□}\)

Answer:
\(\frac{1}{50}\)

Explanation:
0.02
0.02 is 2 hundredths.
2 hundredths = \(\frac{2}{100}\)
Simplify using the GCF.
The GCF of 2 and 100 is 2.
Divide the numerator and the denominator by 2
\(\frac{2 ÷ 2}{100 ÷ 2}\) = \(\frac{1}{50}\)

Question 3.
4.8
______ \(\frac{□}{□}\)

Answer:
\(\frac{4}{5}\)

Explanation:
4.8
4.8 is 4 ones and 8 tenths.
8 tenths = \(\frac{8}{10}\)
Simplify using the GCF.
The GCF of 8 and 10 is 2.
Divide the numerator and the denominator by 2
\(\frac{8 ÷ 2}{10 ÷ 2}\) = \(\frac{4}{5}\)

Question 4.
6.025
______ \(\frac{□}{□}\)

Answer:
\(\frac{1}{40}\)

Explanation:
6.025 is 6 ones and 25 thousandths.
25 thousandths = \(\frac{25}{1000}\)
Simplify using the GCF.
The GCF of 25 and 1000 is 25.
Divide the numerator and the denominator by 25
\(\frac{25 ÷ 25}{1000 ÷ 25}\) = \(\frac{1}{40}\)

Write as a decimal.

Question 5.
\(\frac{17}{25}\)
______

Answer:
0.68

Explanation:
Use division to rename the fraction part as a decimal.
17/25 = 0.68
The quotient has 2 decimal places.
Add the whole number to the decimal.
0 + 0.68 = 0.68.
So, \(\frac{17}{25}\) = 0.68

Question 6.
\(\frac{11}{20}\)
______

Answer:
0.55

Explanation:
Use division to rename the fraction part as a decimal.
11/20 = 0.55
The quotient has 2 decimal places.
Add the whole number to the decimal.
0 + 0.55 = 0.55.
So, \(\frac{11}{20}\) = 0.55

Question 7.
4 \(\frac{13}{20}\)
______

Answer:
4.65

Explanation:
Use division to rename the fraction part as a decimal.
\(\frac{13}{20}\) = 0.65
The quotient has 2 decimal places.
Add the whole number to the decimal.
4 + 0.65 = 4.65.
So, 4 \(\frac{13}{20}\) = 4.65

Question 8.
7 \(\frac{3}{8}\)
______

Answer:
7.375

Explanation:
Use division to rename the fraction part as a decimal.
\(\frac{3}{8}\) = 0.375
The quotient has 3 decimal places.
Add the whole number to the decimal.
7 + 0.375 = 7.375.
So, 7 \(\frac{3}{8}\) = 7.375

Identify a decimal and a fraction or mixed number in simplest form for each point.
Go Math Grade 6 Answer Key Chapter 2 Fractions and Decimals 4

Question 9.
Point A
Type below:
__________

Answer:
0.4

Explanation:

Point A is between 0 and 0.5. Every point is separated by 0.1. So, Point A is at 0.4

Question 10.
Point D
Type below:
__________

Answer:
1.9

Explanation:
Point D is between 1.5 and 2. Every point is separated by 0.1. So, Point D is at 1.9

Question 11.
Point C
Type below:
__________

Answer:
1.2

Explanation:
Point C is between 1 and 1.5. Every point is separated by 0.1. So, Point C is at 1.2

Question 12.
Point B
Type below:
__________

Answer:
0.6

Explanation:
Point C is between 0.5 and 1. Every point is separated by 0.1. So, Point C is at 0.6

Problem Solving

Question 13.
Grace sold \(\frac{5}{8}\) of her stamp collection. What is this amount as a decimal?
______

Answer:
0.625

Explanation:
Grace sold \(\frac{5}{8}\) of her stamp collection.
Use division to rename the fraction part as a decimal.
\(\frac{5}{8}\)  = 0.625
The quotient has 3 decimal places.
Add the whole number to the decimal.
0 + 0.625 = 0.625.
So, \(\frac{5}{8}\) = 0.625

Question 14.
What if you scored an 0.80 on a test? What fraction of the test, in simplest form, did you answer correctly?
\(\frac{□}{□}\)

Answer:
\(\frac{4}{5}\)

Explanation:
0.80 is 0 ones and 8 tenths.
8 tenths = \(\frac{8}{10}\)
Simplify using the GCF.
The GCF of 8 and 10 is 2.
Divide the numerator and the denominator by 2
\(\frac{8 ÷ 2}{10 ÷ 2}\) = \(\frac{4}{5}\)

Question 15.
What fraction in simplest form is equivalent to 0.45? What decimal is equivalent to \(\frac{17}{20}\)? Explain how you found your answers.
Type below:
__________

Answer:
0.45 is 0 ones and 45 hundredths.
45 hundredths = \(\frac{45}{100}\)
Simplify using the GCF.
The GCF of 45 and 100 is 5.
Divide the numerator and the denominator by 5
\(\frac{45 ÷ 5}{100 ÷ 5}\) = \(\frac{9}{20}\)
\(\frac{17}{20}\)
Use division to rename the fraction part as a decimal.
\(\frac{17}{20}\) = 0.85
The quotient has 2 decimal places.
Add the whole number to the decimal.
0 + 0.85 = 0.85.
So, \(\frac{17}{20}\) = 0.85

Lesson Check – Page No. 74

Question 1.
After a storm, Michael measured 6 \(\frac{7}{8}\) inches of snow. What is this amount as a decimal?
______ inches

Answer:
6.875 inches

Explanation:
Michael measured 6 \(\frac{7}{8}\) inches of snow.
Use division to rename the fraction part as a decimal.
\(\frac{7}{8}\) = 0.875
The quotient has 3 decimal places.
Add the whole number to the decimal.
6 + 0.875 = 6.875.
So, 6 \(\frac{7}{8}\) = 6.875.

Question 2.
A recipe calls for 3.75 cups of flour. What is this amount as a mixed number in simplest form?
______ \(\frac{□}{□}\) cups

Answer:
3 \(\frac{3}{4}\) cups

Explanation:
A recipe calls for 3.75 cups of flour.
3 + 0.75
0.75 is 0 ones and 75 hundredths.
75 hundredths = \(\frac{75}{100}\)
Simplify using the GCF.
The GCF of 75 and 100 is 25.
Divide the numerator and the denominator by 25
\(\frac{75 ÷ 25}{100 ÷ 25}\) = \(\frac{3}{4}\)
3 \(\frac{3}{4}\)

Spiral Review

Question 3.
Gina bought 2.3 pounds of red apples and 2.42 pounds of green apples. They were on sale for $0.75 a pound. How much did the apples cost altogether?
$ ______

Answer:
$3.54

Explanation:
Gina bought 2.3 pounds of red apples and 2.42 pounds of green apples. They were on sale for $0.75 a pound.
$0.75 x 2.3 = 1.725
$0.75 x 2.42 = 1.815
1.725 + 1.815 = 3.54
So the apples cost $3.54

Question 4.
Ken has 4.66 pounds of walnuts, 2.1 pounds of cashews, and 8 pounds of peanuts. He mixes them together and divides them equally among 18 bags. How many pounds of nuts are in each bag?
______ pounds

Answer:
0.82 pounds

Explanation:
Ken has 4.66 pounds of walnuts, 2.1 pounds of cashews, and 8 pounds of peanuts.
4.66 + 2.1 + 8 = 14.76
He mixes them together and divides them equally among 18 bags.
14.76/18 = 0.82

Question 5.
Mia needs to separate 270 blue pens and 180 red pens into packs. Each pack will have the same number of blue pens and the same number of red pens. What is the greatest number of packs she can make? How many red pens and how many blue pens will be in each pack?
Type below:
__________

Answer:
There are 2 red pens and 3 blue pens in each pack.

Explanation:
Mia needs to separate 270 blue pens and 180 red pens into packs.
The GCF of 270 and 180 is 90
The greatest number of packs she can make is 90.
Divide the total number of red pens by the total number of packs.
180/90 = 2
Divide the total number of blue pens by the total number of packs.
270/90 = 3
There are 2 red pens and 3 blue pens in each pack.

Question 6.
Evan buys 19 tubes of watercolor paint for $50.35. What is the cost of each tube of paint?
$ ______

Answer:
$2.65

Explanation:
Evan buys 19 tubes of watercolor paint for $50.35.
$50.35/19 = $2.65

Share and Show – Page No. 77

Order from least to greatest.

Question 1.
\(3 \frac{3}{6}, 3 \frac{5}{8}, 2 \frac{9}{10}\)
Type below:
__________

Answer:
2 \(\frac{9}{10}\) < 3 \(\frac{3}{6}\) < 3 \(\frac{5}{8}\)

Explanation:
\(3 \frac{3}{6}, 3 \frac{5}{8}, 2 \frac{9}{10}\)
Compare the whole numbers first.
2 < 3
If the whole numbers are the same, compare the fractions.
3 \(\frac{3}{6}\), 3 \(\frac{5}{8}\)
6 and 8 are multiples of 48.
So, 48 is a common denominator.
3 \(\frac{3 x 8}{6 x 8}\) = 3 \(\frac{24}{48}\), 3 \(\frac{5 x 6}{8 x 6}\) = 3 \(\frac{30}{48}\)
3 \(\frac{24}{48}\) < 3 \(\frac{30}{48}\)
So, 3 \(\frac{3}{6}\) < 3 \(\frac{5}{8}\)
Order the fractions from least to greatest.
2 \(\frac{9}{10}\) < 3 \(\frac{3}{6}\) < 3 \(\frac{5}{8}\)

Write <, >, or =.

Question 2.
0.8 _____ \(\frac{4}{12}\)

Answer:
0.8 < latex]\frac{4}{12}[/latex]

Explanation:
Write the decimal form of \(\frac{4}{12}\) = 0.3333
0.8 > 0.333
So, 0.8 < latex]\frac{4}{12}[/latex]

Question 3.
0.22 _____ \(\frac{1}{4}\)

Answer:
0.22 < \(\frac{1}{4}\)

Explanation:
Write the decimal form of \(\frac{1}{4}\) = 0.25
0.22 < 0.25
So, 0.22 < \(\frac{1}{4}\)

Question 4.
\(\frac{1}{20}\) _____ 0.06

Answer:
\(\frac{1}{20}\) < 0.06

Explanation:
Write the decimal form of \(\frac{1}{20}\) = 0.05
0.05 < 0.06
So, \(\frac{1}{20}\) < 0.06

Use a number line to order from least to greatest.

Question 5.
\(1 \frac{4}{5}, 1.25, 1 \frac{1}{10}\)
Type below:
__________

Answer:
1\(\frac{1}{10}\), 1.25, 1\(\frac{4}{5}\)

Explanation:
Write the decimal form of 1\(\frac{4}{5}\) = 1.8
Write the decimal form of 1\(\frac{1}{10}\) = 1.1
1.8, 1.25, 1.1
Locate each decimal on a number line.
So, from least to greatest, the order is 1.1, 1.25, 1.8
1\(\frac{1}{10}\), 1.25, 1\(\frac{4}{5}\)

On Your Own

Order from least to greatest.

Question 6.
0.6, \(\frac{4}{5}\), 0.75
Type below:
__________

Answer:
0.6, 0.75, \(\frac{4}{5}\)

Explanation:
Write the decimal form of \(\frac{4}{5}\) = 0.8
0.6, 0.8, 0.75
Compare decimals.
All ones are equal.
Compare tenths: 6 < 7 < 8
So, from least to greatest, the order is 0.6, 0.75, 0.8
So, 0.6, 0.75, \(\frac{4}{5}\)

Question 7.
\(\frac{1}{2}\), \(\frac{2}{5}\), \(\frac{7}{15}\)
Type below:
__________

Answer:
\(\frac{2}{5}\), \(\frac{7}{15}\), \(\frac{1}{2}\)

Explanation:
Write the decimal form of \(\frac{1}{2}\) = 0.5
Write the decimal form of \(\frac{2}{5}\) = 0.4
Write the decimal form of \(\frac{7}{15}\) = 0.466
0.5, 0.4, 0.466
Compare decimals.
All ones are equal.
Compare tenths: 4 < 5
Compare hundredths of 0.4 and 0.466; 0 < 6
So, from least to greatest, the order is 0.4 < 0.466 < 0.5
So, \(\frac{2}{5}\), \(\frac{7}{15}\), \(\frac{1}{2}\)

Question 8.
5 \(\frac{1}{2}\), 5.05, 5 \(\frac{5}{9}\)
Type below:
__________

Answer:
5.05, 5 \(\frac{1}{2}\), 5 \(\frac{5}{9}\)

Explanation:
Write the decimal form of 5 \(\frac{1}{2}\) = 5.5
Write the decimal form of 5 \(\frac{5}{9}\) = 5.555
5.5, 5.05, 5.5555
Compare decimals.
All ones are equal.
Compare tenths: 0 < 5
Compare hundredths of 5.5 and 5.55; 0 < 5
So, from least to greatest, the order is 5.05 < 5.5 < 5.55
So, 5.05, 5 \(\frac{1}{2}\), 5 \(\frac{5}{9}\)

Question 9.
\(\frac{5}{7}\), \(\frac{5}{6}\), \(\frac{5}{12}\)
Type below:
__________

Answer:
\(\frac{5}{12}\), \(\frac{5}{7}\), \(\frac{5}{6}\)

Explanation:
\(\frac{5}{7}\), \(\frac{5}{6}\), \(\frac{5}{12}\)
To compare fractions with the same numerators, compare the denominators.
So, from least to greatest, the order is \(\frac{5}{12}\), \(\frac{5}{7}\), \(\frac{5}{6}\)

Question 10.
\(\frac{7}{15}\) _____ \(\frac{7}{10}\)

Answer:
\(\frac{7}{15}\) < \(\frac{7}{10}\)

Explanation:
\(\frac{7}{15}\) and \(\frac{7}{10}\)
To compare fractions with the same numerators, compare the denominators.
So, \(\frac{7}{15}\) < \(\frac{7}{10}\)

Question 11.
\(\frac{1}{8}\) _____ 0.125

Answer:
\(\frac{1}{8}\) = 0.125

Explanation:
Write the decimal form of \(\frac{1}{8}\) = 0.125
0.125 = 0.125

Question 12.
7 \(\frac{1}{3}\) _____ 6 \(\frac{2}{3}\)

Answer:
7 \(\frac{1}{3}\) > 6 \(\frac{2}{3}\)

Explanation:
Compare the whole numbers first.
7 > 6.
So, 7 \(\frac{1}{3}\) > 6 \(\frac{2}{3}\)

Question 13.
1 \(\frac{2}{5}\) _____ 1 \(\frac{7}{15}\)

Answer:
1 \(\frac{2}{5}\) < 1 \(\frac{7}{15}\)

Explanation:
1 \(\frac{2}{5}\) _____ 1 \(\frac{7}{15}\)
If the whole numbers are the same, compare the fractions.
Compare \(\frac{2}{5}\) and \(\frac{7}{15}\)
5 and 15 are multiples of 15.
So, \(\frac{2 x 3}{5 x 3}\) = \(\frac{6}{15}\)
\(\frac{6}{15}\) < \(\frac{7}{15}\)
Use common denominators to write equivalent fractions.
1 \(\frac{2}{5}\) < 1 \(\frac{7}{15}\)

Question 14.
Darrell spent 3 \(\frac{2}{5}\) hours on a project for school. Jan spent 3 \(\frac{1}{4}\) hours and Maeve spent 3.7 hours on the project. Who spent the least amount of time? Show how you found your answer. Then describe another possible method.
Type below:
__________

Answer:
Jan spent the least amount of time.

Explanation:
Darrell spent 3 \(\frac{2}{5}\) hours on a project for school. Jan spent 3 \(\frac{1}{4}\) hours and Maeve spent 3.7 hours on the project.
Write the decimal form of 3 \(\frac{2}{5}\) = 3.4
Write the decimal form of 3 \(\frac{1}{4}\) = 3.25
3.4, 3.25, 3.7
3.25 is the least one.
So, Jan spent the least amount of time.

Problem Solving + Applications – Page No. 78

Use the table for 15–18.
Go Math Grade 6 Answer Key Chapter 2 Fractions and Decimals 5

Question 15.
In one week, Altoona, PA, and Bethlehem, PA, received snowfall every day, Monday through Friday. On which days did Altoona receive over 0.1 inch more snow than Bethlehem?
Type below:
__________

Answer:
Altoona received over 1 inch more snow than Bethlehem on Friday

Explanation:
Altoona (converted to decimal form): 2.25, 3.25, 2.625, 4.6, 4.75
Bethlehem: 2.6, 3.2, 2.5, 4.8, 2.7
Altoona received over 1 inch more snow than Bethlehem on Friday

Question 16.
What if Altoona received an additional 0.3 inch of snow on Thursday? How would the total amount of snow in Altoona compare to the amount received in Bethlehem that day?
Type below:
__________

Answer:
Altoona received 0.1 inches more snow than Bethlehem on Thursday

Explanation:
Altoona received an additional 0.3 inch of snow on Thursday = 4.6 + 0.3 = 4.9
Bethlehem received on Thursday = 4.8
Altoona received 0.1 inches more snow than Bethlehem on Thursday

Question 17.
Explain two ways you could compare the snowfall amounts in Altoona and Bethlehem on Monday.
Type below:
__________

Answer:

Explanation:
Altoona received on Monday = 2.25
Bethlehem received on Monday = 2.6
Bethlehem received 0.35 inches more snow than Altoona on Monday.
As the whole numbers are equal compare 1/4 and 0.6.
0.25 < 0.6
So, Altoona received less snow compared to Bethlehem on Monday.

Question 18.
Explain how you could compare the snowfall amounts in Altoona on Thursday and Friday.
Type below:
__________

Answer:
Altoona received on Thursday = 4.6
Altoona received on Friday = 4.75
4.6 < 4.75
Altoona received less snow on Thursday compared to Friday.

Question 19.
Write the values in order from least to greatest.
Go Math Grade 6 Answer Key Chapter 2 Fractions and Decimals 6
Type below:
__________

Answer:
1/3, 0.39, 2/5, 0.45

Explanation:
1/3 = 0.333
0.45
0.39
2/5 = 0.4
Compare tenths: 3 < 4
Compare hundredths:
0.33 < 0.39
0.4 < 0.45
So, 1/3, 0.39, 2/5, 0.45

Compare and Order Fractions and Decimals – Page No. 79

Write <, >, =.

Question 1.
0.64 _____ \(\frac{7}{10}\)

Answer:
0.64 < \(\frac{7}{10}\)

Explanation:
Write the decimal form of \(\frac{7}{10}\) = 0.7
Compare tenths: 6 < 7
So, 0.64 < 0.7
0.64 < \(\frac{7}{10}\)

Question 2.
0.48 _____ \(\frac{6}{15}\)

Answer:
0.48 > \(\frac{6}{15}\)

Explanation:
Write the decimal form of \(\frac{6}{15}\) = 0.4
Compare hundredths:
0.48 > 0.4
0.48 > \(\frac{6}{15}\)

Question 3.
0.75 _____ \(\frac{7}{8}\)

Answer:
0.75 < \(\frac{7}{8}\)

Explanation:
Write the decimal form of \(\frac{7}{8}\) = 0.875
Compare tenths:
7 < 8
0.75 < \(\frac{7}{8}\)

Question 4.
7 \(\frac{1}{8}\) _____ 7.025

Answer:
7 \(\frac{1}{8}\) > 7.025

Explanation:
Write the decimal form of 7 \(\frac{1}{8}\) = 7.125
Compare tenths:
1 > 0
7 \(\frac{1}{8}\) > 7.025

Order from least to greatest.

Question 5.
\(\frac{7}{15}\), 0.75, \(\frac{5}{6}\)
Type below:
__________

Answer:
\(\frac{7}{15}\), 0.75, \(\frac{5}{6}\)

Explanation:
Write the decimal form of \(\frac{7}{15}\) = 0.466
0.75
Write the decimal form of \(\frac{5}{6}\) = 0.833
Order from least to greatest: \(\frac{7}{15}\), 0.75, \(\frac{5}{6}\)

Question 6.
0.5, 0.41, \(\frac{3}{5}\)
Type below:
__________

Answer:
0.41, 0.5, \(\frac{3}{5}\)

Explanation:
Write the decimal form of \(\frac{3}{5}\) = 0.6
Compare tenths:
0.41, 0.5, 0.6
Order from least to greatest: 0.41, 0.5, \(\frac{3}{5}\)

Question 7.
3.25, 3 \(\frac{2}{5}\), 3 \(\frac{3}{8}\)
Type below:
__________

Answer:
3.25, 3 \(\frac{2}{5}\), 3 \(\frac{3}{8}\)

Explanation:
Write the decimal form of 3 \(\frac{2}{5}\) = 3.4
Write the decimal form of 3 \(\frac{3}{8}\) = 3.375
Compare tenths:
Order from least to greatest: 3.25, 3 \(\frac{2}{5}\), 3 \(\frac{3}{8}\)

Question 8.
0.9, \(\frac{8}{9}\), 0.86
Type below:
__________

Answer:
0.86, \(\frac{8}{9}\), 0.9

Explanation:
Write the decimal form of \(\frac{8}{9}\) = 0.88
Compare tenths:
0.86, 0.88, 0.9
Order from least to greatest: 0.86, \(\frac{8}{9}\), 0.9

Order from greatest to least.

Question 9.
0.7, \(\frac{7}{9}\), \(\frac{7}{8}\)
Type below:
__________

Answer:
\(\frac{7}{8}\), \(\frac{7}{9}\), 0.7

Explanation:
0.7 = 7/10
To compare fractions with the same numerators, compare the denominators.
7/10, 7/9, 7/8
Order from greatest to least: 7/8, 7/9, 7/10

Question 10.
0.2, 0.19, \(\frac{3}{5}\)
Type below:
__________

Answer:
\(\frac{3}{5}\), 0.2, 0.19

Explanation:
Write the decimal form of \(\frac{3}{5}\) = 0.6
Compare tenths:
0.6, 0.2, 0.19
Order from greatest to least: \(\frac{3}{5}\), 0.2, 0.19

Question 11.
6\(\frac{1}{20}\), 6.1, 6.07
Type below:
__________

Answer:

Explanation:
Write the decimal form of 6\(\frac{1}{20}\) = 121/20 = 6.05
Compare tenths:
6.1, 6.07, 6.05
Order from greatest to least: 6.1, 6.07, 6\(\frac{1}{20}\)

Question 12.
2 \(\frac{1}{2}\), 2.4, 2.35, 2 \(\frac{1}{8}\)
Type below:
__________

Answer:
2 \(\frac{1}{2}\), 2.4, 2.35, 2 \(\frac{1}{8}\)

Explanation:
Write the decimal form of 2 \(\frac{1}{2}\) = 2.5
Write the decimal form of 2 \(\frac{1}{8}\) = 2.125
Compare tenths: 2.5, 2.4, 2.35, 2.125
Order from greatest to least: 2 \(\frac{1}{2}\), 2.4, 2.35, 2 \(\frac{1}{8}\)

Question 13.
One day it snowed 3 \(\frac{3}{8}\) inches in Altoona and 3.45 inches in Bethlehem. Which city received less snow that day?
__________

Answer:
Altoona

Explanation:
One day it snowed 3 \(\frac{3}{8}\) inches in Altoona and 3.45 inches in Bethlehem.
Write the decimal form of 3 \(\frac{3}{8}\) = 27/8 = 3.375
3.375 < 3.45.
Altoona received less snow that day

Question 14.
Malia and John each bought 2 pounds of sunflower seeds. Each ate some seeds. Malia has 1 \(\frac{1}{3}\) pounds left, and John has 1 \(\frac{2}{5}\) pounds left. Who ate more sunflower seeds?
__________

Answer:
Malia

Explanation:
Malia and John each bought 2 pounds of sunflower seeds. Each ate some seeds. Malia has 1 \(\frac{1}{3}\) pounds left, and John has 1 \(\frac{2}{5}\) pounds left.
2 – 1 \(\frac{1}{3}\) = 0.667
2 – 1 \(\frac{2}{5}\) = 0.6
0.667 > 0.6
So, Malia ate more sunflower seeds

Question 15.
Explain how you would compare the numbers 0.4 and \(\frac{3}{8}\).
Type below:
__________

Answer:
Write the decimal form of \(\frac{3}{8}\) = 0.375
Compare tenths:
0.4 > 0.375

Lesson Check – Page No. 80

Question 1.
Andrea has 3 \(\frac{7}{8}\) yards of purple ribbon, 3.7 yards of pink ribbon, and 3 \(\frac{4}{5}\) yards of blue ribbon. List the numbers in order from least to greatest.
Type below:
__________

Answer:
Andrea has 3 \(\frac{7}{8}\) yards of purple ribbon, 3.7 yards of pink ribbon, and 3 \(\frac{4}{5}\) yards of blue ribbon.
Write the decimal form of 3 \(\frac{7}{8}\) = 3.875
3.7
Write the decimal form of 3 \(\frac{4}{5}\) = 3.8
Least to greatest : 3.7, 3 \(\frac{4}{5}\), 3 \(\frac{7}{8}\)

Question 2.
Nassim completed \(\frac{18}{25}\) of the math homework. Kara completed 0.7 of it. Debbie completed \(\frac{5}{8}\) of it. List the numbers in order from greatest to least.
Type below:
__________

Answer:
$1.39, $0.70, $0.63

Explanation:
Nassim completed \(\frac{18}{25}\) of the math homework. Kara completed 0.7 of it. Debbie completed \(\frac{5}{8}\) of it.
Write the decimal form of 18/25 = 1.39
0.7
Write the decimal form of 5/8 = 0.63
They are now in order from greatest to least.
Think of the amounts as money:
$1.39, $0.70, $0.63

Spiral Review

Question 3.
Tyler bought 3 \(\frac{2}{5}\) pounds of oranges. Graph 3 \(\frac{2}{5}\) on a number line and write this amount using a decimal.
Type below:
__________

Answer:
grade 6 chapter 2 image 1
Tyler bought 3 \(\frac{2}{5}\) pounds of oranges.
Decimal Form: 17/5 = 3.4

Question 4.
At the factory, a baseball card is placed in every 9th package of cereal. A football card is placed in every 25th package of the cereal. What is the first package that gets both a baseball card and a football card?
Type below:
__________

Answer:
225th package

Explanation:
Look for the first number where both 25 and 9 are a factor of.
25 x 1 = 25 which isn’t a factor of 9, so it won’t be 25.
25 x 2 = 50, which isn’t a factor of 9.
75 is not a factor of 9. (you know because you don’t get a whole number when you divide 75 into 9.)
100 is not a factor of 9, nor is 125, 150, 175, or 200.
However, 225 is a factor of both 25 and 9. This makes sense because 25 x 9 is 225.
This means that the first package with both will be the 225th package.

Question 5.
$15.30 is divided among 15 students. How much does each student receive?
$ _____

Answer:
$1.02

Explanation:
$15.30 is divided among 15 students.
$15.30/15 = $1.02
each student receive $1.02

Question 6.
Carrie buys 4.16 pounds of apples for $5.20. How much does 1 pound cost?
$ _____

Answer:
$1.25

Explanation:
Carrie buys 4.16 pounds of apples for $5.20.
$5.20/4.16 = $1.25
1 pound cost = $1.25

Share and Show – Page No. 83

Find the product. Write it in simplest form.

Question 1.
6 × \(\frac{3}{8}\)
\(\frac{□}{□}\)

Answer:
\(\frac{9}{4}\)

Explanation:
\(\frac{6 × 3}{1 × 8}\)
\(\frac{18}{8}\)
Simplify using the GCF.
The GCF of 18 and 8 is 2.
Divide the numerator and the denominator by 2.
\(\frac{18 ÷ 2}{8 ÷ 2}\) = \(\frac{9}{4}\)

Question 2.
\(\frac{3}{8}\) × \(\frac{8}{9}\)
\(\frac{□}{□}\)

Answer:
\(\frac{1}{3}\)

Explanation:
Multiply the numerators and Multiply the denominators.
\(\frac{3 × 8}{8 × 9}\) = \(\frac{24}{72}\)
Simplify using the GCF.
The GCF of 24 and 72 is 24.
Divide the numerator and the denominator by 24.
\(\frac{24 ÷ 24}{72 ÷ 24}\) = \(\frac{1}{3}\)

Question 3.
Sam and his friends ate 3 \(\frac{3}{4}\) bags of fruit snacks. If each bag contained 2 \(\frac{1}{2}\) ounces, how many ounces of fruit snacks did Sam and his friends eat?
\(\frac{□}{□}\)

Answer:
\(\frac{75}{8}\) ounces

Explanation:
Sam and his friends ate 3 \(\frac{3}{4}\) bags of fruit snacks. If each bag contained 2 \(\frac{1}{2}\) ounces
3 \(\frac{3}{4}\) x 2 \(\frac{1}{2}\)
\(\frac{15}{4}\) x \(\frac{5}{2}\)
\(\frac{15 x 5}{4 x 2}\) = \(\frac{75}{8}\)

Attend to Precision Algebra Evaluate using the order of operations.

Write the answer in simplest form.

Question 4.
\(\left(\frac{3}{4}-\frac{1}{2}\right) \times \frac{3}{5}\)
\(\frac{□}{□}\)

Answer:
\(\frac{3}{20}\)

Explanation:
\(\left(\frac{3}{4}-\frac{1}{2}\right) \times \frac{3}{5}\)
Perform operations in parentheses.
\(\frac{3}{4}\) – \(\frac{1}{2}\) = \(\frac{1}{4}\)
\(\frac{1}{4}\) x \(\frac{3}{5}\) = \(\frac{1 x 3}{4 x 5}\) = \(\frac{3}{20}\)

Question 5.
\(\frac{1}{3}+\frac{4}{9} \times 12\)
\(\frac{□}{□}\)

Answer:
\(\frac{28}{3}\)

Explanation:
\(\frac{1}{3}\) + \(\frac{4}{9}\) = \(\frac{7}{9}\)
\(\frac{7 x 12}{9 x 1}\) = \(\frac{84}{9}\)
Simplify using the GCF.
The GCF of 84 and 9 is 3.
Divide the numerator and the denominator by 3.
\(\frac{84 ÷ 3}{9 ÷ 3}\) = \(\frac{28}{3}\)

Question 6.
\(\frac{5}{8} \times \frac{7}{10}-\frac{1}{4}\)
\(\frac{□}{□}\)

Answer:
\(\frac{11}{16}\)

Explanation:
\(\frac{5 x 7}{8 x 10}\) = \(\frac{35}{80}\)
\(\frac{35}{80}\) – \(\frac{1}{4}\) = \(\frac{11}{16}\)

Question 7.
3 × (\(\frac{5}{18}\) + \(\frac{1}{6}\)) + \(\frac{2}{5}\)
\(\frac{□}{□}\)

Answer:
\(\frac{38}{15}\)

Explanation:
3 x \(\frac{4}{9}\) + \(\frac{2}{5}\)
3 x \(\frac{38}{45}\) = \(\frac{38}{15}\)

On Your Own

Practice: Copy and Solve Find the product. Write it in simplest form.

Question 8.
\(1 \frac{2}{3} \times 2 \frac{5}{8}\)
\(\frac{□}{□}\)

Answer:
\(\frac{35}{8}\)

Explanation:
1 \(\frac{2}{3}\) = \(\frac{5}{3}\)
2 \(\frac{5}{8}\) = \(\frac{21}{8}\)
\(\frac{5 × 21}{3 × 8}\) = \(\frac{105}{24}\)
Simplify using the GCF
The GCF of 105 and 24 is 3.
Divide the numerator and the denominator by 3.
\(\frac{105 ÷ 3}{24 ÷ 3}\) = \(\frac{35}{8}\)

Question 9.
\(\frac{4}{9} \times \frac{4}{5}\)
\(\frac{□}{□}\)

Answer:
\(\frac{16}{45}\)

Explanation:
\(\frac{4 × 4}{9 × 5}\) = \(\frac{16}{45}\)

Question 10.
\(\frac{1}{6} \times \frac{2}{3}\)
\(\frac{□}{□}\)

Answer:
\(\frac{1}{9}\)

Explanation:
\(\frac{1 × 2}{6 × 3}\) = \(\frac{2}{18}\)
Simplify using the GCF
The GCF of 2 and 18 is 2.
Divide the numerator and the denominator by 2.
\(\frac{2 ÷ 2}{18 ÷ 2}\) = \(\frac{1}{9}\)

Question 11.
\(4 \frac{1}{7} \times 3 \frac{1}{9}\)
\(\frac{□}{□}\)

Answer:
\(\frac{116}{7}\)

Explanation:
4\(\frac{1}{7}\) = \(\frac{29}{7}\)
3\(\frac{1}{9}\) = \(\frac{28}{9}\)
\(\frac{29 × 28}{7 × 9}\) = \(\frac{812}{63}\)
Simplify using the GCF
The GCF of 812 and 63 is 7.
Divide the numerator and the denominator by 7.
\(\frac{812 ÷ 7}{63 ÷ 7}\) = \(\frac{116}{7}\)

Question 12.
\(\frac{5}{6}\) of the 90 pets in the pet show are cats. \(\frac{4}{5}\) of the cats are calico cats. What fraction of the pets are calico cats? How many of the pets are calico cats?
Type below:
__________

Answer:
60 calico cats

Explanation:
5/6 x 90 = 450/6 = 150/2
150/2 x 4/5 = 60

Question 13.
Five cats each ate \(\frac{1}{4}\) cup of cat food. Four other cats each ate \(\frac{1}{3}\) cup of cat food. How much food did the nine cats eat?
Type below:
__________

Answer:
\(\frac{31}{12}\)

Explanation:
5 x 1/4 = 5/4
4 x 1/3 = 4/3
5/4 + 4/3 = 31/12

Attend to Precision Algebra Evaluate using the order of operations.

Write the answer in simplest form.

Question 14.
\(\frac{1}{4} \times\left(\frac{3}{9}+5\right)\)
\(\frac{□}{□}\)

Answer:
\(\frac{4}{3}\)

Explanation:
3/9 + 5 = 16/3
1/4 x 16/3
1 x 16 = 16
4 x 3 = 12
16/12
Simplify using the GCF
The GCF of 16 and 12 is 4.
Divide the numerator and the denominator by 4.
\(\frac{16 ÷ 4}{12÷ 4}\) = \(\frac{4}{3}\)

Question 15.
\(\frac{9}{10}-\frac{3}{5} \times \frac{1}{2}\)
\(\frac{□}{□}\)

Answer:
\(\frac{3}{5}\)

Explanation:
3/5 x 1/2 = 3/10
9/10 – 3/10 = 6/10
Simplify using the GCF
The GCF of 6 and 10 is 2.
Divide the numerator and the denominator by 2.
\(\frac{6 ÷ 2}{10 ÷ 2}\) = \(\frac{3}{5}\)

Question 16.
\(\frac{4}{5}+\left(\frac{1}{2}-\frac{3}{7}\right) \times 2\)
\(\frac{□}{□}\)

Answer:
\(\frac{33}{35}\)

Explanation:
1/2 – 3/7 = 1/14
1/14 x 2 = 1/7
4/5 + 1/7 = 33/35

Question 17.
\(15 \times \frac{3}{10}+\frac{7}{8}\)
\(\frac{□}{□}\)

Answer:
\(\frac{141}{8}\)

Explanation:
3/10 + 7/8 = 47/40
15 x 47/40 = 141/8
\(\frac{141}{8}\)

Page No. 84

Question 18.
Write and solve a word problem for the expression \(\frac{1}{4} \times \frac{2}{3}\). Show your work.
Type below:
__________

Answer:
\(\frac{1}{6}\)

Explanation:
\(\frac{1}{4} \times \frac{2}{3}\) = \(\frac{1 X 2}{4 X 3}\) = \(\frac{2}{12}\)
Simplify using the GCF
The GCF of 2 and 12 is 2.
Divide the numerator and the denominator by 2.
\(\frac{2 ÷ 2}{12 ÷ 2}\) = \(\frac{1}{6}\)

Question 19.
Michelle has a recipe that asks for 2 \(\frac{1}{2}\) cups of vegetable oil. She wants to use \(\frac{2}{3}\) that amount of oil and use applesauce to replace the rest. How much applesauce will she use?
Type below:
__________

Answer:
\(\frac{10}{6}\)

Explanation:
2 1/2 * 2/3 = 5/2 * 2/3 = 10/6 She will use 10/6 or 1 2/3 cups of vegetable oil

Question 20.
Cara’s muffin recipe asks for 1 \(\frac{1}{2}\) cups of flour for the muffins and \(\frac{1}{4}\) cup of flour for the topping. If she makes \(\frac{1}{2}\) of the original recipe, how much flour will she use for the muffins and topping?
Type below:
__________

Answer:
Cara will use 1\(\frac{1}{8}\) cups of flour.

Explanation:
For first we will find how many cups of flours need to makes the original recipe. Cara uses 1 1/2 cups of flour for the muffins and 1/4 cup off flour for the topping.
So, 1 1/2 + 1/4 cups of flour to make the original recipe.
1 1/2 = 3/2
3/2 + 1/4 = 7/4
To make the original recipe Cara needs 7/4 cups of flour.
If she makes \(\frac{1}{2}\) of the original recipe, then
7/4 x 1/2 = 7/8 = 1 1/8
Cara will use 1 1/8 cups of flour.

Multiply Fractions – Page No. 85

Find the product. Write it in simplest form.

Question 1.
\(\frac{4}{5} \times \frac{7}{8}\)
\(\frac{□}{□}\)

Answer:
\(\frac{7}{10}\)

Explanation:
Multiply the numerators and Multiply the denominators.
\(\frac{4 × 7}{5 × 8}\) = \(\frac{28}{40}\)
Simplify using the GCF.
The GCF of 28 and 40 is 4.
Divide the numerator and the denominator by 4.
\(\frac{28 ÷ 4}{40 ÷ 4}\) = \(\frac{7}{10}\)

Question 2.
\(\frac{1}{8} \times 20\)
\(\frac{□}{□}\)

Answer:
\(\frac{5}{2}\)

Explanation:
\(\frac{1 × 20}{1 × 8}\)
\(\frac{20}{8}\)
Simplify using the GCF.
The GCF of 20 and 8 is 4.
Divide the numerator and the denominator by 4.
\(\frac{20 ÷ 4}{8 ÷ 4}\) = \(\frac{5}{2}\)

Question 3.
\(\frac{4}{5} \times \frac{3}{8}\)
\(\frac{□}{□}\)

Answer:
\(\frac{3}{10}\)

Explanation:
Multiply the numerators and Multiply the denominators.
\(\frac{4 × 3}{5 × 8}\) = \(\frac{12}{40}\)
Simplify using the GCF.
The GCF of 12 and 40 is 4.
Divide the numerator and the denominator by 4.
\(\frac{12 ÷ 4}{40 ÷ 4}\) = \(\frac{3}{10}\)

Question 4.
\(1 \frac{1}{8} \times \frac{1}{9}\)
\(\frac{□}{□}\)

Answer:
\(\frac{1}{8}\)

Explanation:
1\(\frac{1}{8}\) = \(\frac{9}{8}\)
Multiply the numerators and Multiply the denominators.
\(\frac{9 × 1}{8 × 9}\) = \(\frac{9}{72}\)
Simplify using the GCF.
The GCF of 9 and 72 is 9.
Divide the numerator and the denominator by 9.
\(\frac{9 ÷ 9}{72 ÷ 9}\) = \(\frac{1}{8}\)

Question 5.
\(\frac{3}{4} \times \frac{1}{3} \times \frac{2}{5}\)
\(\frac{□}{□}\)

Answer:
\(\frac{1}{10}\)

Explanation:
Multiply the numerators and Multiply the denominators.
\(\frac{3 × 1 × 2}{4 × 3 × 5}\) = \(\frac{6}{60}\)
Simplify using the GCF.
The GCF of 6 and 60 is 6.
Divide the numerator and the denominator by 6.
\(\frac{6 ÷ 6}{60 ÷ 6}\) = \(\frac{1}{10}\)

Question 6.
Karen raked \(\frac{3}{5}\) of the yard. Minni raked \(\frac{1}{3}\) of the amount Karen raked. How much of the yard did Minni rake?
\(\frac{□}{□}\)

Answer:
\(\frac{1}{3}\)

Explanation:
Minni raked 1/5 of the yard.
So, minni raked 3/5 of 1/3 means 3/5 x 1/3
Multiply the numerators and Multiply the denominators.
\(\frac{3 × 1}{5 × 3}\) = \(\frac{3}{15}\)
Simplify using the GCF.
The GCF of 3 and 15 is 3.
Divide the numerator and the denominator by 3.
\(\frac{3 ÷ 3}{15 ÷ 3}\) = \(\frac{1}{3}\)

Question 7.
\(\frac{3}{8}\) of the pets in the pet show are dogs. \(\frac{2}{3}\) of the dogs have long hair. What fraction of the pets are dogs with long hair?
\(\frac{□}{□}\)

Answer:
\(\frac{1}{4}\) are dogs with long hair

Explanation:
\(\frac{3}{8}\) of the pets in the pet show are dogs. \(\frac{2}{3}\) of the dogs have long hair.
\(\frac{3}{8}\) of \(\frac{2}{3}\) = \(\frac{3 × 2}{8 × 3}\) = \(\frac{6}{24}\)
The GCF of 6 and 24 is 6.
Divide the numerator and the denominator by 6.
\(\frac{6 ÷ 6}{24 ÷ 6}\) = \(\frac{1}{4}\)
\(\frac{1}{4}\) are dogs with long hair

Evaluate using the order of operations.

Question 8.
\(\left(\frac{1}{2}+\frac{3}{8}\right) \times 8\)
______

Answer:
7

Explanation:
1/2 + 3/8 = 7/8
7/8 × 8 = 7

Question 9.
\(\frac{3}{4} \times\left(1-\frac{1}{9}\right)\)
\(\frac{□}{□}\)

Answer:
\(\frac{2}{3}\)

Explanation:
1 – 1/9 = 8/9
3/4 × 8/9 = 24/36
The GCF of 24 and 36 is 12.
Divide the numerator and the denominator by 12.
\(\frac{24 ÷ 12}{36 ÷ 12}\) = \(\frac{2}{3}\)

Question 10.
\(4 \times \frac{1}{8} \times \frac{3}{10}\)
\(\frac{□}{□}\)

Answer:
\(\frac{3}{20}\)

Explanation:
Multiply the numerators and Multiply the denominators.
\(\frac{4 × 1 × 3}{1 × 8 × 10}\) = \(\frac{12}{80}\)
Simplify using the GCF.
The GCF of 12 and 80 is 4.
Divide the numerator and the denominator by 4.
\(\frac{12 ÷ 4}{80 ÷ 4}\) = \(\frac{3}{20}\)

Question 11.
\(6 \times\left(\frac{4}{5}+\frac{2}{10}\right) \times \frac{2}{3}\)
______

Answer:
4

Explanation:
4/5 + 2/10 = 1
6 × 1 × 2/3 = 12/3
The GCF of 12 and 3 is 4.
Divide the numerator and the denominator by 3.
\(\frac{12 ÷ 3}{3 ÷ 3}\) = \(\frac{4}{1}\) = 4

Problem Solving

Question 12.
Jason ran \(\frac{5}{7}\) of the distance around the school track. Sara ran \(\frac{4}{5}\) of Jason’s distance. What fraction of the total distance around the track did Sara run?
\(\frac{□}{□}\)

Answer:
\(\frac{4}{7}\)

Explanation:
Jason ran \(\frac{5}{7}\) of the distance around the school track. Sara ran \(\frac{4}{5}\) of Jason’s distance.
\(\frac{5}{7}\) × \(\frac{4}{5}\) = 20/35
The GCF of 20 and 35 is 5.
Divide the numerator and the denominator by 5.
\(\frac{20 ÷ 5}{35 ÷ 5}\) = \(\frac{4}{7}\)

Question 13.
A group of students attend a math club. Half of the students are boys and \(\frac{4}{9}\) of the boys have brown eyes. What fraction of the group are boys with brown eyes?
\(\frac{□}{□}\)

Answer:
\(\frac{2}{9}\) group are boys with brown eyes

Explanation:
A group of students attend a math club. Half of the students are boys and \(\frac{4}{9}\) of the boys have brown eyes.
\(\frac{4}{9}\) × \(\frac{1}{2}\) = 4/18 = 2/9
2/9 group are boys with brown eyes

Question 14.
Write and solve a word problem that involves multiplying by a fraction.
Type below:
__________

Answer:
A group of students attends a math club. Half of the students are boys and \(\frac{6}{9}\) of the boys have brown eyes. What fraction of the group are boys with brown eyes?
\(\frac{□}{□}\)
Answer:
A group of students attends a math club. Half of the students are boys and \(\frac{6}{9}\) of the boys have brown eyes.
\(\frac{6}{9}\) × \(\frac{1}{2}\) = 6/18 = 1/3
1/3 group are boys with brown eyes.

Lesson Check – Page No. 86

Question 1.
Veronica’s mom left \(\frac{3}{4}\) of a cake on the table. Her brothers ate \(\frac{1}{2}\) of it. What fraction of the cake did they eat?
\(\frac{□}{□}\)

Answer:
\(\frac{2}{4}\)

Explanation:
Veronica’s mom left \(\frac{3}{4}\) of a cake on the table. Her brothers ate \(\frac{1}{2}\) of it.
Since the fraction of the eaten cake is 1/2, you can multiply the numerator and denominator by and get an equivalent fraction, which is 2/4.

Question 2.
One lap around the school track is \(\frac{5}{8}\) mile. Carin ran 3 \(\frac{1}{2}\) laps. How far did she run?
_____ \(\frac{□}{□}\)

Answer:
2\(\frac{3}{16}\)

Explanation:
One lap around the school track is \(\frac{5}{8}\) mile. Carin ran 3 \(\frac{1}{2}\) laps.
3 \(\frac{1}{2}\) = \(\frac{7}{2}\)
Therefore, the total distance covered = 7/2 × 5/8 = 35/16 = 2 3/16

Spiral Review

Question 3.
Tom bought 2 \(\frac{5}{16}\) pounds of peanuts and 2.45 pounds of cashews. Which did he buy more of? Explain.
Type below:
__________

Answer:

Explanation:
Tom bought 2 \(\frac{5}{16}\) pounds of peanuts and 2.45 pounds of cashews.
2 \(\frac{5}{16}\) = 2.3125
2.3125 < 2.45
He buys more cashews.

Question 4.
Eve has 24 stamps each valued at $24.75. What is the total value of her stamps?
$ _____

Answer:
$594

Explanation:
Eve has 24 stamps each valued at $24.75.
24 x $24.75 = $594

Question 5.
Naomi went on a 6.5-mile hike. In the morning, she hiked 1.75 miles, rested, and then hiked 2.4 more miles. She completed the hike in the afternoon. How much farther did she hike in the morning than in the afternoon?
_____ miles

Answer:
Naomi went on a 6.5-mile hike. In the morning, she hiked 1.75 miles, rested, and then hiked 2.4 more miles. She completed the hike in the afternoon.
To find how many miles she walked in the afternoon you just subtract the morning miles 4.15 from the total miles 6.5.
6.5 – 4.15  = 2.35
To find how many more miles she walked in the morning you just subtract the morning from the afternoon 4.15 – 2.35=1.8 miles.
She hiked 1.8 more miles in the morning

Question 6.
A bookstore owner has 48 science fiction books and 30 mysteries he wants to sell quickly. He will make discount packages with one type of book in each. He wants the most books possible in each package, but all packages must contain the same number of books. How many packages can he make? How many packages of each type of book does he have?
Type below:
__________

Answer:
18 packages

Explanation:
The bookstore owner can make 18 possible packages
48 – 30 = 18 packages

Share and Show – Page No. 89

Find the product. Simplify before multiplying.

Question 1.
\(\frac{5}{6} \times \frac{3}{10}\)
\(\frac{□}{□}\)

Answer:
\(\frac{1}{4}\)

Explanation:
\(\frac{5}{6} \times \frac{3}{10}\)
Multiply the numerators and Multiply the denominators.
\(\frac{5 × 3}{6 × 10}\) = \(\frac{15}{60}\)
Simplify using the GCF.
The GCF of 15 and 60 is 15.
Divide the numerator and the denominator by 15.
\(\frac{15 ÷ 15}{60 ÷ 15}\) = \(\frac{1}{4}\)

Question 2.
\(\frac{3}{4} \times \frac{5}{9}\)
\(\frac{□}{□}\)

Answer:
\(\frac{5}{12}\)

Explanation:
\(\frac{3}{4} \times \frac{5}{9}\)
Multiply the numerators and Multiply the denominators.
\(\frac{3 × 5}{4 × 9}\) = \(\frac{15}{36}\)
Simplify using the GCF.
The GCF of 15 and 36 is 3.
Divide the numerator and the denominator by 3.
\(\frac{15 ÷ 3}{36 ÷ 3}\) = \(\frac{5}{12}\)

Question 3.
\(\frac{2}{3} \times \frac{9}{10}\)
\(\frac{□}{□}\)

Answer:
\(\frac{3}{5}\)

Explanation:
\(\frac{2}{3} \times \frac{9}{10}\)
Multiply the numerators and Multiply the denominators.
\(\frac{2 × 9}{3 × 10}\) = \(\frac{18}{30}\)
Simplify using the GCF.
The GCF of 18 and 30 is 6.
Divide the numerator and the denominator by 6.
\(\frac{18 ÷ 6}{30 ÷ 6}\) = \(\frac{3}{5}\)

Question 4.
After a picnic, \(\frac{5}{12}\) of the cornbread is left over. Val eats \(\frac{3}{5}\) of the leftover cornbread. What fraction of the cornbread does Val eat?
\(\frac{□}{□}\)

Answer:
\(\frac{1}{4}\)

Explanation:
After a picnic, \(\frac{5}{12}\) of the cornbread is left over. Val eats \(\frac{3}{5}\) of the leftover cornbread.
\(\frac{5}{12} \times \frac{3}{5}\)
Multiply the numerators and Multiply the denominators.
\(\frac{5 × 3}{12 × 5}\) = \(\frac{15}{60}\)
Simplify using the GCF.
The GCF of 15 and 60 is 15.
Divide the numerator and the denominator by 15.
\(\frac{15 ÷ 15}{60 ÷ 15}\) = \(\frac{1}{4}\)

Question 5.
The reptile house at the zoo has an iguana that is \(\frac{5}{6}\) yd long. It has a Gila monster that is \(\frac{4}{5}\) of the length of the iguana. How long is the Gila monster?
\(\frac{□}{□}\)

Answer:
\(\frac{2}{3}\)

Explanation:
The reptile house at the zoo has an iguana that is \(\frac{5}{6}\) yd long. It has a Gila monster that is \(\frac{4}{5}\) of the length of the iguana.
\(\frac{5}{6} \times \frac{4}{5}\)
Multiply the numerators and Multiply the denominators.
\(\frac{5 × 4}{6× 5}\) = \(\frac{20}{30}\)
Simplify using the GCF.
The GCF of 20 and 30 is 10.
Divide the numerator and the denominator by 10.
\(\frac{20 ÷ 10}{30 ÷ 10}\) = \(\frac{2}{3}\)

On Your Own

Find the product. Simplify before multiplying.

Question 6.
\(\frac{3}{4} \times \frac{1}{6}\)
\(\frac{□}{□}\)

Answer:

Explanation:
\(\frac{3}{4} \times \frac{1}{6}\)
Multiply the numerators and Multiply the denominators.
\(\frac{3 × 1}{4 × 6}\) = \(\frac{3}{24}\)
Simplify using the GCF.
The GCF of 3 and 24 is 3.
Divide the numerator and the denominator by 3.
\(\frac{3 ÷ 3}{24 ÷ 3}\) = \(\frac{1}{8}\)

Question 7.
\(\frac{7}{10} \times \frac{2}{3}\)
\(\frac{□}{□}\)

Answer:
\(\frac{7}{15}\)

Explanation:
\(\frac{7}{10} \times \frac{2}{3}\)
Multiply the numerators and Multiply the denominators.
\(\frac{7 × 2}{10 × 3}\) = \(\frac{14}{30}\)
Simplify using the GCF.
The GCF of 14 and 30 is 2.
Divide the numerator and the denominator by 2.
\(\frac{14 ÷ 2}{30 ÷ 2}\) = \(\frac{7}{15}\)

Question 8.
\(\frac{5}{8} \times \frac{2}{5}\)
\(\frac{□}{□}\)

Answer:
\(\frac{1}{4}\)

Explanation:
\(\frac{5}{8} \times \frac{2}{5}\)
Multiply the numerators and Multiply the denominators.
\(\frac{5 × 2}{8 × 5}\) = \(\frac{10}{40}\)
Simplify using the GCF.
The GCF of 10 and 40 is 10.
Divide the numerator and the denominator by 10.
\(\frac{10 ÷ 10}{40 ÷ 10}\) = \(\frac{1}{4}\)

Question 9.
\(\frac{9}{10} \times \frac{5}{6}\)
\(\frac{□}{□}\)

Answer:
\(\frac{3}{4}\)

Explanation:
\(\frac{9}{10} \times \frac{5}{6}\)
Multiply the numerators and Multiply the denominators.
\(\frac{9 × 5}{10 × 6}\) = \(\frac{45}{60}\)
Simplify using the GCF.
The GCF of 45 and 60 is 15.
Divide the numerator and the denominator by 15.
\(\frac{45 ÷ 15}{60 ÷ 15}\) = \(\frac{3}{4}\)

Question 10.
\(\frac{11}{12} \times \frac{3}{7}\)
\(\frac{□}{□}\)

Answer:
\(\frac{11}{28}\)

Explanation:
\(\frac{11}{12} \times \frac{3}{7}\)
Multiply the numerators and Multiply the denominators.
\(\frac{11 × 3}{12 × 7}\) = \(\frac{33}{84}\)
Simplify using the GCF.
The GCF of 33 and 84 is 3.
Divide the numerator and the denominator by 3.
\(\frac{33 ÷ 3}{84 ÷ 3}\) = \(\frac{11}{28}\)

Question 11.
Shelley’s basketball team won \(\frac{3}{4}\) of their games last season. In \(\frac{1}{6}\) of the games they won, they outscored their opponents by more than 10 points. What fraction of their games did Shelley’s team win by more than 10 points?
\(\frac{□}{□}\)

Answer:
\(\frac{1}{8}\)

Explanation:
Let the total number of games be x.
Number of games Shelley’s team won = 3/4x
Number of games they outscored their opponents by more than 10 points = 1/6 X 3/4x = 1/8x
Hence, 1/8 of the total games, Shelley’s team won by 10 points.

Question 12.
Mr. Ortiz has \(\frac{3}{4}\) pound of oatmeal. He uses \(\frac{2}{3}\) of the oatmeal to bake muffins. How much oatmeal does Mr. Ortiz have left?
\(\frac{□}{□}\)

Answer:
\(\frac{1}{2}\)

Explanation:
Mr. Ortiz has \(\frac{3}{4}\) pound of oatmeal. He uses \(\frac{2}{3}\) of the oatmeal to bake muffins.
\(\frac{3}{4} \times \frac{2}{3}\)
Multiply the numerators and Multiply the denominators.
\(\frac{3 × 2}{4 × 3}\) = \(\frac{6}{12}\)
Simplify using the GCF.
The GCF of 6 and 12 is 6.
Divide the numerator and the denominator by 6.
\(\frac{6 ÷ 6}{12 ÷ 6}\) = \(\frac{1}{2}\)

Question 13.
Compare Strategies To find \(\frac{16}{27}\) × \(\frac{3}{4}\), you can multiply the fractions and then simplify the product or you can simplify the fractions and then multiply. Which method do you prefer? Explain.
Type below:
__________

Answer:
\(\frac{16}{27}\) × \(\frac{3}{4}\)
\(\frac{16 × 3}{27 × 4}\) = \(\frac{16 × 3}{4 × 27}\)
\(\frac{48}{96}\)
Simplify using the GCF.
The GCF of 48 and 96 is 48.
Divide the numerator and the denominator by 48.
\(\frac{48 ÷ 48}{96 ÷ 48}\) = \(\frac{1}{2}\)

Problem Solving + Applications – Page No. 90

Go Math Grade 6 Answer Key Chapter 2 Fractions and Decimals 7

Question 14.
Three students each popped \(\frac{3}{4}\) cup of popcorn kernels. The table shows the fraction of each student’s kernels that did not pop. Which student had \(\frac{1}{16}\) cup unpopped kernels?
__________

Answer:
Mirza

Explanation:
Three students each popped \(\frac{3}{4}\) cup of popcorn kernels. The table shows the fraction of each student’s kernels that did not pop.
Katie = 3/4 x 1/10 = 3/40
Mirza = 3/4 x 1/12 = 1/16

Question 15.
The jogging track at Francine’s school is \(\frac{3}{4}\) mile long. Yesterday Francine completed two laps on the track. If she ran \(\frac{1}{3}\) of the distance and walked the remainder of the way, how far did she walk?
____ mile

Answer:
1 mile

Explanation:
Length of jogging track at Francine’s school = 3/4 mile
Let the distance covered by running be = x
Let the distance covered by walking be = y
Total number of laps completed by Francine = 2
Total distance covered by Francine = number of laps X distance covered in one lap
2 x 3/4 = 3/25 mile
Now,
distance covered by running = 1/3 of the total distance
x = 1/3 x 3/2
distance covered by walking y = total distance – distance covered by running
3/2 – x = 3/2 – 1/2 = 1 mile
Hence, Francine walked for 1 mile.

Question 16.
At a snack store, \(\frac{7}{12}\) of the customers bought pretzels and \(\frac{3}{10}\) of those customers bought low-salt pretzels. Bill states that \(\frac{7}{30}\) of the customers bought low-salt pretzels. Does Bill’s statement make sense? Explain.
Type below:
__________

Answer:
Bill’s statement does not make sense because it is incorrect:
7/12 customers bought pretzels.
3/10 Of those customers bought low salt pretzels (x)
3/10 of 7/12 = x
21/120 = x
Simplify: 7/40
To be correct, Bill would have to say that 7/40 of the customers bought low salt pretzels, but instead, he had said 7/30.

Question 17.
The table shows Tonya’s homework assignment. Tonya’s teacher instructed the class to simplify each expression by dividing the numerator and denominator by the GCF. Complete the table by simplifying each expression and then finding the value.
Go Math Grade 6 Answer Key Chapter 2 Fractions and Decimals 8
Type below:
__________

Answer:
Garde 6 chapter 2 image 1

Simplify Factors – Page No. 91

Find the product. Simplify before multiplying.

Question 1.
\(\frac{8}{9} \times \frac{5}{12}\)
\(\frac{□}{□}\)

Answer:
\(\frac{10}{27}\)

Explanation:
\(\frac{8}{9} \times \frac{5}{12}\)
Multiply the numerators and Multiply the denominators.
\(\frac{8 × 5}{9 × 12}\) = \(\frac{40}{108}\)
Simplify using the GCF.
The GCF of 40 and 108 is 4.
Divide the numerator and the denominator by 4.
\(\frac{40 ÷ 4}{108 ÷ 4}\) = \(\frac{10}{27}\)

Question 2.
\(\frac{3}{4} \times \frac{16}{21}\)
\(\frac{□}{□}\)

Answer:
\(\frac{4}{7}\)

Explanation:
\(\frac{3}{4} \times \frac{16}{21}\)
Multiply the numerators and Multiply the denominators.
\(\frac{3 × 16}{4 × 21}\) = \(\frac{48}{84}\)
Simplify using the GCF.
The GCF of 48 and 84 is 12.
Divide the numerator and the denominator by 12.
\(\frac{48 ÷ 12}{84 ÷ 12}\) = \(\frac{4}{7}\)

Question 3.
\(\frac{15}{20} \times \frac{2}{5}\)
\(\frac{□}{□}\)

Answer:
\(\frac{3}{10}\)

Explanation:
\(\frac{15}{20} \times \frac{2}{5}\)
Multiply the numerators and Multiply the denominators.
\(\frac{15 × 2}{20 × 5}\) = \(\frac{30}{100}\)
Simplify using the GCF.
The GCF of 30 and 100 is 10.
Divide the numerator and the denominator by 10.
\(\frac{30 ÷ 10}{100 ÷ 10}\) = \(\frac{3}{10}\)

Question 4.
\(\frac{9}{18} \times \frac{2}{3}\)
\(\frac{□}{□}\)

Answer:
\(\frac{1}{3}\)

Explanation:
\(\frac{9}{18} \times \frac{2}{3}\)
Multiply the numerators and Multiply the denominators.
\(\frac{9 × 2}{18 × 3}\) = \(\frac{18}{54}\)
Simplify using the GCF.
The GCF of 18 and 54 is 18.
Divide the numerator and the denominator by 18.
\(\frac{18 ÷ 18}{54 ÷ 18}\) = \(\frac{1}{3}\)

Question 5.
\(\frac{3}{4} \times \frac{7}{30}\)
\(\frac{□}{□}\)

Answer:
\(\frac{7}{40}\)

Explanation:
\(\frac{3}{4} \times \frac{7}{30}\)
Multiply the numerators and Multiply the denominators.
\(\frac{3 × 7}{4 × 30}\) = \(\frac{21}{120}\)
Simplify using the GCF.
The GCF of 21 and 120 is 3.
Divide the numerator and the denominator by 3.
\(\frac{21 ÷ 3}{120 ÷ 3}\) = \(\frac{7}{40}\)

Question 6.
\(\frac{8}{15} \times \frac{15}{32}\)
\(\frac{□}{□}\)

Answer:
\(\frac{1}{4}\)

Explanation:
\(\frac{8}{15} \times \frac{15}{32}\)
Multiply the numerators and Multiply the denominators.
\(\frac{8 × 15}{15 × 32}\) = \(\frac{120}{480}\)
Simplify using the GCF.
The GCF of 120 and 480 is 120.
Divide the numerator and the denominator by 120.
\(\frac{120 ÷ 120}{480 ÷ 120}\) = \(\frac{1}{4}\)

Question 7.
\(\frac{12}{21} \times \frac{7}{9}\)
\(\frac{□}{□}\)

Answer:
\(\frac{4}{9}\)

Explanation:
\(\frac{12}{21} \times \frac{7}{9}\)
Multiply the numerators and Multiply the denominators.
\(\frac{12 × 7}{21 × 9}\) = \(\frac{84}{189}\)
Simplify using the GCF.
The GCF of 84 and 189 is 21.
Divide the numerator and the denominator by 21.
\(\frac{84 ÷ 21}{189 ÷ 21}\) = \(\frac{4}{9}\)

Question 8.
\(\frac{18}{22} \times \frac{8}{9}\)
\(\frac{□}{□}\)

Answer:
\(\frac{8}{11}\)

Explanation:
\(\frac{18}{22} \times \frac{8}{9}\)
Multiply the numerators and Multiply the denominators.
\(\frac{18 × 8}{22 × 9}\) = \(\frac{144}{198}\)
Simplify using the GCF.
The GCF of 144 and 198 is 18.
Divide the numerator and the denominator by 18.
\(\frac{144 ÷ 18}{198 ÷ 18}\) = \(\frac{8}{11}\)

Problem Solving

Question 9.
Amber has a \(\frac{4}{5}\)-pound bag of colored sand. She uses \(\frac{1}{2}\) of the bag for an art project. How much sand does she use for the project?
\(\frac{□}{□}\) pounds

Answer:
\(\frac{2}{5}\) pounds

Explanation:
Amber has a \(\frac{4}{5}\)-pound bag of colored sand. She uses \(\frac{1}{2}\) of the bag for an art project.
4/5 X 1/2 = 2/5

Question 10.
Tyler has \(\frac{3}{4}\) month to write a book report. He finished the report in \(\frac{2}{3}\) that time. How much time did it take Tyler to write the report?
\(\frac{□}{□}\) month

Answer:
\(\frac{1}{2}\) month

Explanation:
Tyler has \(\frac{3}{4}\) month to write a book report. He finished the report in \(\frac{2}{3}\) that time.
3/4 X 2/3 = 1/2

Question 11.
Show two ways to multiply \(\frac{2}{15} \times \frac{3}{20}\). Then tell which way is easier and justify your choice.
Type below:
__________

Answer:
\(\frac{2}{15} \times \frac{3}{20}\)
2/15 X 3/20 = 2/20 X 3/15 = 1/10 X 1/5 = 1/50

Lesson Check – Page No. 92

Find each product. Simplify before multiplying.

Question 1.
At Susie’s school, \(\frac{5}{8}\) of all students play sports. Of the students who play sports, \(\frac{2}{5}\) play soccer. What fraction of the students in Susie’s school play soccer?
\(\frac{□}{□}\)

Answer:
\(\frac{1}{4}\)

Explanation:
At Susie’s school, \(\frac{5}{8}\) of all students play sports. Of the students who play sports, \(\frac{2}{5}\) play soccer.
Multiply 5/8 X 2/5, and the answer is 0.25, which converts to 25/100 or 1/4

Question 2.
A box of popcorn weighs \(\frac{15}{16}\) pounds. The box contains \(\frac{1}{3}\) buttered popcorn and \(\frac{2}{3}\) cheesy popcorn. How much does the cheesy popcorn weigh?
\(\frac{□}{□}\)

Answer:
\(\frac{5}{8}\)

Explanation:
Total weight of a box of popcorn =15/16 pounds.
We are given two types of popcorns are there, butter popcorns and cheesy popcorns.
Butter popcorn is the one-third of the total weight = 1/3 of the Total weight
Plugging the value of the total weight, we get
= 1/3 * 15/16 = 5/16 pounds.
Cheesy popcorn = 2/3 of Total weight
Plugging the value of total weight, we get
= 2/3 * 15/16 = 10/16 or 5/8 pounds.
Therefore, cheesy popcorn weighs is 5/8 pounds.

Spiral Review

Question 3.
Ramòn bought a dozen ears of corn for $1.80. What was the cost of each ear of corn?
$ ______

Answer:
$0.15

Explanation:
Ramòn bought a dozen ears of corn for $1.80.
So, for the cost of each ear of corn, $1.80/12 = $0.15

Question 4.
A 1.8-ounce jar of cinnamon costs $4.05. What is the cost per ounce?
$ ______

Answer:
$2.25 per ounce

Explanation:
If a 1.8-ounce jar costs $4.05, do $4.05 divided by 1.8.
$4.05 / 1.8 = $2.25 per ounce.

Question 5.
Rose bought \(\frac{7}{20}\) kilogram of ginger candy and 0.4 kilogram of cinnamon candy. Which did she buy more of? Explain how you know.
Type below:
__________

Answer:
Rose bought ginger candy = 7/20 kilogram = 0.35 Kilogram
She bought cinnamon candy = 0.4 kilogram
0.4 > 0.35
Therefore, She bought cinnamon candy more.

Question 6.
Don walked 3 \(\frac{3}{5}\) miles on Friday, 3.7 miles on Saturday, and 3 \(\frac{5}{8}\) miles on Sunday. List the distances from least to greatest.
Type below:
__________

Answer:
3 \(\frac{3}{5}\), 3 \(\frac{5}{8}\), 3.7

Explanation:
3 \(\frac{3}{5}\) = 18/5 = 3.6
3 \(\frac{5}{8}\) = 29/8 = 3.625
3.6 < 3.625 < 3.7
3 \(\frac{3}{5}\), 3 \(\frac{5}{8}\), 3.7

Mid-Chapter Checkpoint – Vocabulary – Page No. 93

Choose the best term from the box to complete the sentence.
Go Math Grade 6 Answer Key Chapter 2 Fractions and Decimals 9

Question 1.
The fractions \(\frac{1}{2}\) and \(\frac{5}{10}\) are _____.
Type below:
__________

Answer:
Equivalent fractions

Question 2.
A _____ is a denominator that is the same in two or more fractions.
Type below:
__________

Answer:
Common Denominator

Concepts and Skills

Write as a decimal. Tell whether you used division, a number line, or some other method.

Question 3.
\(\frac{7}{20}\)
_____

Answer:
0.35

Explanation:
By using Division,
\(\frac{7}{20}\) = 0.35

Question 4.
8 \(\frac{39}{40}\)
_____

Answer:
8.975

Explanation:
By using Division,
8 \(\frac{39}{40}\) = 359/40 = 8.975

Question 5.
1 \(\frac{5}{8}\)
_____

Answer:
1.625

Explanation:
By using Division,
1 \(\frac{5}{8}\) = 13/8 = 1.625

Question 6.
\(\frac{19}{25}\)
_____

Answer:
0.76

Explanation:
By using Division,
\(\frac{19}{25}\) = 0.76

Order from least to greatest.

Question 7.
\(\frac{4}{5}, \frac{3}{4}, 0.88\)
Type below:
__________

Answer:
\(\frac{3}{4}\), \(\frac{4}{5}\),0.88

Explanation:
Write the decimal form of 4/5 = 0.8
Write the decimal form of 3/4 = 0.75
0.88
0.75 < 0.8 < 0.88

Question 8.
0.65, 0.59, \(\frac{3}{5}\)
Type below:
__________

Answer:
0.59, \(\frac{3}{5}\), 0.65

Explanation:
Write the decimal form of 3/5 = 0.6
0.59 < 0.6 < 0.65

Question 9.
\(1 \frac{1}{4}, 1 \frac{2}{3}, \frac{11}{12}\)
Type below:
__________

Answer:
\(\frac{11}{12}\), 1\(\frac{1}{4}\), 1\(\frac{2}{3}\)

Explanation:
Write the decimal form of 1 1/4 = 5/4 = 1.25
Write the decimal form of 1 2/3 = 5/3 = 1.66
Write the decimal form of 11/12 = 0.916
0.916 < 1.25 < 1.66

Question 10.
0.9, \(\frac{7}{8}\), 0.86
Type below:
__________

Answer:
0.86, \(\frac{7}{8}\), 0.9

Explanation:
Write the decimal form of \(\frac{7}{8}\) = 0.875
0.86 < 0.875 < 0.9

Find the product. Write it in simplest form.

Question 11.
\(\frac{2}{3} \times \frac{1}{8}\)
\(\frac{□}{□}\)

Answer:
\(\frac{1}{12}\)

Explanation:
\(\frac{2}{3} \times \frac{1}{8}\)
Multiply the numerators and Multiply the denominators.
\(\frac{2 × 1}{3 × 8}\) = \(\frac{2}{24}\)
Simplify using the GCF.
The GCF of 2 and 24 is 2.
Divide the numerator and the denominator by 2.
\(\frac{2 ÷ 2}{24 ÷ 2}\) = \(\frac{1}{12}\)

Question 12.
\(\frac{4}{5} \times \frac{2}{5}\)
\(\frac{□}{□}\)

Answer:
\(\frac{8}{25}\)

Explanation:
\(\frac{4}{5} \times \frac{2}{5}\)
Multiply the numerators and Multiply the denominators.
\(\frac{4 × 2}{5 × 5}\) = \(\frac{8}{25}\)

Question 13.
12 × \(\frac{3}{4}\)
_____

Answer:
9

Explanation:
12 × \(\frac{3}{4}\)
Multiply the numerators and Multiply the denominators.
\(\frac{12 × 3}{1 × 4}\) = \(\frac{36}{4}\) = 9

Question 14.
Mia climbs \(\frac{5}{8}\) of the height of the rock wall. Lee climbs \(\frac{4}{5}\) of Mia’s distance. What fraction of the wall does Lee climb?
\(\frac{□}{□}\)

Answer:
\(\frac{7}{40}\)

Explanation:
find the LCM (least common denominator) for 5/8 and 4/5.
5/8= 25/40 and 4/5= 32/40.
Subtract and you get 7/40.

Page No. 94

Question 15.
In Zoe’s class, \(\frac{4}{5}\) of the students have pets. Of the students who have pets, \(\frac{1}{8}\) have rodents. What fraction of the students in Zoe’s class have pets that are rodents? What fraction of the students in Zoe’s class have pets that are not rodents?
Type below:
__________

Answer:
\(\frac{1}{10}\) of the students in Zoe’s class have pets that are rodents
\(\frac{7}{10}\) of the students in Zoe’s class have pets that are not rodents

Explanation:
In Zoe’s class, \(\frac{4}{5}\) of the students have pets. Of the students who have pets, \(\frac{1}{8}\) have rodents.
4/5 X 1/8 = 1/10
4/5 – 1/10 = 7/10

Question 16.
A recipe calls for 2 \(\frac{2}{3}\) cups of flour. Terell wants to make \(\frac{3}{4}\) of the recipe. How much flour should he use?
_____ cups

Answer:
2 cups

Explanation:
2 \(\frac{2}{3}\) = 8/3
8/3 * 3/4 = 2

Question 17.
Following the Baltimore Running Festival in 2009, volunteers collected and recycled 3.75 tons of trash. Graph 3.75 on a number line and write the weight as a mixed number.
Type below:
__________

Answer:
Volunteers collected and recycled 3.75 tons of trash.
We need to convert 3.75 as a mixed number.
The mixed number consists of a whole number and a proper fraction.
In the given number 3.75, 3 as the whole number and convert 0.75 to a fraction.
3.75 = 3 + 0.75 = 3 + 75/100
We can reduce the fraction 75/ 100 = 3+ 3/4 = 3 3/4

Question 18.
Four students took an exam. The fraction of the total possible points that each received is given. Which student had the highest score? If students receive a whole number of points on every exam item, can the exam be worth a total of 80 points? Explain.
Go Math Grade 6 Answer Key Chapter 2 Fractions and Decimals 10
Type below:
__________

Answer:
22/25 = 0.88
17/20 = 0.85
4/5 = 0.8
3/4 = 0.75
Monica had the highest score
Let x be the total number of points:
(22/25 + 17/20 + 4/5 + 3/4)x = 80
x = 24.39
That is not a whole number of points.

Share and Show – Page No. 97

Use the model to find the quotient.

Question 1.
\(\frac{1}{2}\) ÷ 3
Go Math Grade 6 Answer Key Chapter 2 Fractions and Decimals 11
\(\frac{□}{□}\)

Answer:
\(\frac{1}{6}\)

Explanation:
1/2 groups of 3
\(\frac{1}{2}\) ÷ 3
1/2 × 1/3 = 1/6

Question 2.
\(\frac{3}{4} \div \frac{3}{8}\)
Go Math Grade 6 Answer Key Chapter 2 Fractions and Decimals 12
______

Answer:
2

Explanation:
3/4 groups of 3/8
3/4 × 8/3 = 2

Use fraction strips to find the quotient. Then draw the model.

Question 3.
\(\frac{1}{3}\) ÷ 4
\(\frac{□}{□}\)

Answer:
\(\frac{1}{12}\)
Garde 6 chapter 2 image 2

Explanation:
\(\frac{1}{3}\) ÷ 4
\(\frac{1}{3}\) × \(\frac{1}{4}\)
\(\frac{1}{12}\)

Question 4.
\(\frac{3}{5} \div \frac{3}{10}\)
______

Answer:
2

Explanation:
\(\frac{3}{5} \div \frac{3}{10}\)
\(\frac{3}{5}\) × \(\frac{10}{3}\)
2

Draw a model to solve. Then write an equation for the model. Interpret the result.

Question 5.
How many \(\frac{1}{4}\) cup servings of raisins are in \(\frac{3}{8}\) cup of raisins?
Type below:
__________

Answer:
1.5

Explanation:
3/8 × 1/4 = 1.5

Question 6.
How many \(\frac{1}{3}\) lb bags of trail mix can Josh make from \(\frac{5}{6}\) lb of trail mix?
Type below:
__________

Answer:
2

Explanation:
Multiply 1/3 with 2
1/3 × 2 = 2/6. 2/6 can go into 5/6 twice so the answer is two bags.

Question 7.
Pose a Problem Write and solve a problem for \(\frac{3}{4}\) ÷ 3 that represents how much in each of 3 groups.
Type below:
__________

Answer:
\(\frac{1}{4}\)

Explanation:
\(\frac{3}{4}\) ÷ 3
\(\frac{3}{4}\) × \(\frac{1}{3}\) = 1/4

Problem Solving + Applications – Page No. 98

The table shows the amount of each material that students in a sewing class need for one purse.

Use the table for 8–10. Use models to solve.
Go Math Grade 6 Answer Key Chapter 2 Fractions and Decimals 13

Question 8.
Mrs. Brown has \(\frac{1}{3}\) yd of blue denim and \(\frac{1}{2}\) yd of black denim. How many purses can be made using denim as the main fabric?
_____ purses

Answer:
5 purses

Explanation:
Mrs. Brown has \(\frac{1}{3}\) yd of blue denim and \(\frac{1}{2}\) yd of black denim.
3 + 2 = 5

Question 9.
One student brings \(\frac{1}{2}\) yd of ribbon. If 3 students receive an equal length of the ribbon, how much ribbon will each student receive? Will each of them have enough ribbon for a purse? Explain.
Type below:
__________

Answer:
One student brings \(\frac{1}{2}\) yd of ribbon. If 3 students receive an equal length of the ribbon,
\(\frac{1}{2}\) ÷ 3
1/2 × 1/3 = 1/6
They don’t have enough ribbon for a purse

Question 10.
Make Arguments There was \(\frac{1}{2}\) yd of purple and pink striped fabric. Jessie said she could only make \(\frac{1}{24}\) of a purse using that fabric as the trim. Is she correct? Use what you know about the meanings of multiplication and division to defend your answer.
Type below:
__________

Answer:
There was \(\frac{1}{2}\) yd of purple and pink striped fabric. Jessie said she could only make \(\frac{1}{24}\) of a purse using that fabric as the trim.
1/2 × 12 = 1/24
So, 12 is the answer

Question 11.
Draw a model to find the quotient.
\(\frac{1}{2}\) ÷ 4 =
Type below:
__________

Answer:
Garde 6 chapter 2 image 3

Explanation:
1/2 × 1/4 = 1/8

Model Fraction Division – Page No. 99

Use the model to find the quotient

Question 1.
\(\frac{1}{4}\) ÷ 3 =
Go Math Grade 6 Answer Key Chapter 2 Fractions and Decimals 14
\(\frac{□}{□}\)

Answer:
\(\frac{1}{12}\)

Explanation:
\(\frac{1}{4}\) ÷ 3
\(\frac{1}{4}\) × \(\frac{1}{3}\) = \(\frac{1}{12}\)

Question 2.
\(\frac{1}{2} \div \frac{2}{12}=\)
Go Math Grade 6 Answer Key Chapter 2 Fractions and Decimals 15
______

Answer:
3

Explanation:
\(\frac{1}{2} \div \frac{2}{12}=\)
\(\frac{1}{2}\) × \(\frac{12}{2}\) = \(\frac{12}{4}\) = 3

Use fraction strips to find the quotient.

Question 3.
\(\frac{5}{6} \div \frac{1}{2}=\)
______ \(\frac{□}{□}\)

Answer:
\(\frac{5}{3}\)

Explanation:
\(\frac{5}{6} \div \frac{1}{2}=\)
\(\frac{5}{6}\) × \(\frac{2}{1}\) = \(\frac{5}{3}\)

Question 4.
\(\frac{2}{3}\) ÷ 4 =
\(\frac{□}{□}\)

Answer:
\(\frac{1}{6}\)

Explanation:
\(\frac{2}{3}\) ÷ 4
\(\frac{2}{3}\) × \(\frac{1}{4}\) = \(\frac{2}{12}\) = 1/6

Question 5.
\(\frac{1}{2}\) ÷ 6 =
\(\frac{□}{□}\)

Answer:
\(\frac{1}{12}\)

Explanation:
\(\frac{1}{2}\) ÷ 6
\(\frac{1}{2}\) × \(\frac{1}{6}\) = \(\frac{1}{12}\)

Question 6.
\(\frac{1}{3} \div \frac{1}{12}\)
______

Answer:
4

Explanation:
\(\frac{1}{3} \div \frac{1}{12}\)
\(\frac{1}{3}\) × \(\frac{12}{1}\) = \(\frac{12}{3}\) = 4

Draw a model to solve. Then write an equation for the model. Interpret the result.

Question 7.
If Jerry runs \(\frac{1}{10}\) mile each day, how many days will it take for him to run \(\frac{4}{5}\) mile?
______ days

Answer:
8 days

Explanation:
If Jerry runs \(\frac{1}{10}\) mile each day,
\(\frac{4}{5}\) ÷ \(\frac{1}{10}\)
\(\frac{4}{5}\) × \(\frac{10}{1}\) = \(\frac{40}{5}\) = 8

Problem Solving

Question 8.
Mrs. Jennings has \(\frac{3}{4}\) gallon of paint for an art project. She plans to divide the paint equally into jars. If she puts \(\frac{1}{8}\) gallon of paint into each jar, how many jars will she use?
______ jars

Answer:
6 jars

Explanation:
Mrs. Jennings has 3/4 Gallons of paint for an art project.
In 1 jar she puts 1/8 gallon of paint.
The number of jars in which she plans to to divide the paint equally is given by,
n= 3/4 ÷ 1/8
n = \(\frac{3}{4}\) × \(\frac{8}{1}\) = \(\frac{24}{4}\) = 6

Question 9.
If one jar of glue weighs \(\frac{1}{12}\) pound, how many jars can Rickie get from \(\frac{2}{3}\) pound of glue?
______ jars

Answer:
8 jars

Explanation:
The weight of glue in one jar = 1/12 pound
To get 2/3 pound of glue Rickie can get the number of jars
2/3 ÷ 1/12
2/3 × 12/1 = 24/3 = 8

Question 10.
Explain how to use a model to show \(\frac{2}{6} \div \frac{1}{12}\) and \(\frac{2}{6}\) ÷ 4.
Type below:
__________

Answer:
Garde 6 chapter 2 image 4
Garde 6 chapter 2 image 2

Explanation:
\(\frac{2}{6} \div \frac{1}{12}\)
2/6 = 1/3
1/3 x 12/1 = 4
\(\frac{2}{6}\) ÷ 4
1/3 x 1/4 = 1/12

Lesson Check – Page No. 100

Question 1.
Darcy needs \(\frac{1}{4}\) yard of fabric to make a banner. She has 2 yards of fabric. How many banners can she make?
______ banners

Answer:
8 banners

Explanation:
Darcy needs \(\frac{1}{4}\) yard of fabric to make a banner. She has 2 yards of fabric.
2 ÷ \(\frac{1}{4}\) = 2 x 4 = 8

Question 2.
Lorenzo bought \(\frac{15}{16}\) pounds of ground beef. He wants to make hamburgers that weigh \(\frac{3}{16}\) pound each. How many hamburgers can he make?
______ hamburgers

Answer:
5 hamburgers

Explanation:
Lorenzo bought \(\frac{15}{16}\) pounds of ground beef. He wants to make hamburgers that weigh \(\frac{3}{16}\) pound each.
\(\frac{15}{16}\) ÷ \(\frac{3}{16}\)
15/3 = 5

Spiral Review

Question 3.
Letisha wants to read 22 pages a night. At that rate, how long will it take her to read a book with 300 pages?
______ nights

Answer:
14 nights

Explanation:
Letisha wants to read 22 pages a night. It takes her to read a book with 300 pages
300/22 = 13.6
13.6 is near to 14
So, it is for 2 weeks.

Question 4.
A principal wants to order enough notebooks for 624 students. The notebooks come in boxes of 28. How many boxes should he order?
______ boxes

Answer:
22 boxes

Explanation:
A principal wants to order enough notebooks for 624 students. The notebooks come in boxes of 28.
624/28 = 22.2857
22.2857 is closer to 22
22 boxes.

Question 5.
Each block in Ton’s neighborhood is \(\frac{2}{3}\) mile long. If he walks 4 \(\frac{1}{2}\) blocks, how far does he walk?
______ miles

Answer:
3 miles

Explanation:
If each block is 2/3 miles long, and he walks 4 1/2 blocks, we can simply multiply to two. It looks like this:
(2/3)(4 1/2)
to multiply, make 4 1/2 into an improper fraction and multiply normally
(2/3)(9/4)
Ton walks 3 miles total.

Question 6.
In Cathy’s garden, \(\frac{5}{6}\) of the area is planted with flowers. Of the flowers, \(\frac{3}{10}\) of them are red. What fraction of Cathy’s garden is planted with red flowers?
\(\frac{□}{□}\)

Answer:
\(\frac{1}{4}\)

Explanation:
In Cathy’s garden, \(\frac{5}{6}\) of the area is planted with flowers. Of the flowers, \(\frac{3}{10}\) of them are red.
5/6 x 3/10 = 1/4

Share and Show – Page No. 103

Estimate using compatible numbers.

Question 1.
\(22 \frac{4}{5} \div 6 \frac{1}{4}\)
_______

Answer:
4

Explanation:
22 \(\frac{4}{5}\) = 114/5 = 22.8
6 \(\frac{1}{4}\) = 25/4 = 6.25
22.8 is closer to 24
6.25 is closer to 6
24/6 = 4

Question 2.
\(12 \div 3 \frac{3}{4}\)
_______

Answer:
3

Explanation:
3 \(\frac{3}{4}\) = 15/4 = 3.75
3.75 is closer to 4
12/4 = 3

Question 3.
\(33 \frac{7}{8} \div 5 \frac{1}{3}\)
_______

Answer:
7

Explanation:
33 \(\frac{7}{8}\) = 271/8 = 33.875
5 \(\frac{1}{3}\) = 16/3 = 5.333
33.875 is closer to 35
5.333 is closer to 5
35/5 = 7

Question 4.
\(3 \frac{7}{8} \div \frac{5}{9}\)
_______

Answer:
4

Explanation:
3 \(\frac{7}{8}\) = 31/8 = 3.875
\(\frac{5}{9}\) = 0.555
3.875 is closer to 4
0.555 is closer to 1
4/1 = 4

Question 5.
\(34 \frac{7}{12} \div 7 \frac{3}{8}\)
_______

Answer:
5

Explanation:
34 \(\frac{7}{12}\) = 415/12 = 34.583
7 \(\frac{3}{8}\) = 59/8 = 7.375
34.583 is closer to 35
7.375 is closer to 7
35/7 = 5

Question 6.
\(1 \frac{2}{9} \div \frac{1}{6}\)
_______

Answer:
5

Explanation:
1 \(\frac{2}{9}\) = 11/9 = 1.222
\(\frac{1}{6}\) = 0.1666
1.222 is closer to 1
0.1666 is closer to 0.2
1/0.2 = 5

On Your Own

Estimate using compatible numbers.

Question 7.
\(44 \frac{1}{4} \div 11 \frac{7}{9}\)
_______

Answer:
4

Explanation:
44 \(\frac{1}{4}\) = 177/4 = 44.25
11 \(\frac{7}{9}\) = 106/9 = 11.77
44.25 is closer to 44
11.77 is closer to 11
44/11 = 4

Question 8.
\(71 \frac{11}{12} \div 8 \frac{3}{4}\)
_______

Answer:
8

Explanation:
71 \(\frac{11}{12}\) = 863/12 = 71.916
8 \(\frac{3}{4}\) = 35/4 = 8.75
71.916 is closer to 72
8.75 is closer to 9
72/9 = 8

Question 9.
\(1 \frac{1}{6} \div \frac{1}{8}\)
_______

Answer:
12

Explanation:
1 \(\frac{1}{6}\) = 7/6 = 1.166
\(\frac{1}{8}\) = 0.125
1.166 is closer to 1.2
0.125 is closer to 0.1
1.2/0.1 = 12

Estimate to compare. Write <, >, or =.

Question 10.
\(21 \frac{3}{10} \div 2 \frac{5}{6}\) _______ \(35 \frac{7}{9} \div 3 \frac{2}{3}\)

Answer:
\(21 \frac{3}{10} \div 2 \frac{5}{6}\) < \(35 \frac{7}{9} \div 3 \frac{2}{3}\)

Explanation:
21 \(\frac{3}{10}\) = 213/10 = 21.3
2 \(\frac{5}{6}\) = 17/6 = 2.833
21.3 is closer to 21
2.833 is closer to 3
21/3 = 7
35 \(\frac{7}{9}\) = 322/9 = 35.777
3 \(\frac{2}{3}\) = 11/3 = 3.666
35.777 is closer to 36
3.666 is closer to 4
36/4 = 9
7 < 9
So, \(21 \frac{3}{10} \div 2 \frac{5}{6}\) < \(35 \frac{7}{9} \div 3 \frac{2}{3}\)

Question 11.
\(29 \frac{4}{5} \div 5 \frac{1}{6}\) _______ \(27 \frac{8}{9} \div 6 \frac{5}{8}\)

Answer:
\(29 \frac{4}{5} \div 5 \frac{1}{6}\) > \(27 \frac{8}{9} \div 6 \frac{5}{8}\)

Explanation:
29 \(\frac{4}{5}\) = 149/5 = 29.8
5 \(\frac{1}{6}\) = 31/6 = 5.1666
29.8 is closer to 30
5.1666 is closer to 5
30/5 = 6
27 \(\frac{8}{9}\) = 251/9 = 27.888
6 \(\frac{5}{8}\) = 53/8 = 6.625
27.888 is closer to 30
6.625 is closer 7
30/7 = 5
6 > 5
\(29 \frac{4}{5} \div 5 \frac{1}{6}\) > \(27 \frac{8}{9} \div 6 \frac{5}{8}\)

Question 12.
\(55 \frac{5}{6} \div 6 \frac{7}{10}\) _______ \(11 \frac{5}{7} \div \frac{5}{8}\)

Answer:
\(55 \frac{5}{6} \div 6 \frac{7}{10}\) < \(11 \frac{5}{7} \div \frac{5}{8}\)

Explanation:
55 \(\frac{5}{6}\) = 335/6 = 55.833
6 \(\frac{7}{10}\) = 67/10 = 6.7
55.833 is closer to 56
6.7 is closer to 7
56/7 = 8
11 \(\frac{5}{7}\) = 82/7 = 11.714
\(\frac{5}{8}\) = 0.625
11.714 is closer to 12
0.625 is closer to 1
12/1 = 12
8 < 12

Question 13.
Marion is making school flags. Each flag uses 2 \(\frac{3}{4}\) yards of felt. Marion has 24 \(\frac{1}{8}\) yards of felt. About how many flags can he make?
About _______ flags

Answer:
About 8 flags

Explanation:
Marion is making school flags. Each flag uses 2 \(\frac{3}{4}\) yards of felt. Marion has 24 \(\frac{1}{8}\) yards of felt.
2 \(\frac{3}{4}\) = 11/4
24 \(\frac{1}{8}\) = 193/8
193/8 ÷ 11/4
193/8 x 4/11 = 8.77
About 8 flags

Question 14.
A garden snail travels about 2 \(\frac{3}{5}\) feet in 1 minute. At that speed, about how many hours would it take the snail to travel 350 feet?
About _______ hours

Answer:
About 2 hours

Explanation:
2 \(\frac{3}{5}\) = 2.6
That’s how long he travels in one minute. There are 60 minutes in an hour so multiply it by 60 and see if that gets you close to 350.
60 x 2.6 = 156
Now let’s add one more hour.
156 + 156 = 312
14 x 2.6 = 36.4
312 + 36.4 = 348.4
348.4 + 2.6 = 351
So two hours and fourteen minutes

Problem Solving + Applications – Page No. 104

What’s the Error?

Question 15.
Megan is making pennants from a piece of butcher paper that is 10 \(\frac{3}{8}\) yards long. Each pennant requires \(\frac{3}{8}\) yard of paper. To estimate the number of pennants she could make, Megan estimated the quotient 10 \(\frac{3}{8}\) ÷ \(\frac{3}{8}\).
Look at how Megan solved the problem. Find her error
Estimate:
10 \(\frac{3}{8}\) ÷ \(\frac{3}{8}\)
10 ÷ \(\frac{1}{2}\) = 5
Correct the error. Estimate the quotient.
So, Megan can make about _____ pennants.
Describe the error that Megan made
Explain Tell which compatible numbers you used to estimate 10 \(\frac{3}{8}\) ÷ \(\frac{3}{8}\). Explain why you chose those numbers.
Type below:
__________

Answer:
10 \(\frac{3}{8}\) ÷ \(\frac{3}{8}\)
10 \(\frac{3}{8}\) = 83/8 = 10.375
\(\frac{3}{8}\) = 0.375
She had written 10 ÷ \(\frac{1}{2}\) = 5
10.375 is closer to 10
0.375 is closer to 0.5
10/0.5 = 20
But she has written 5 instead of 20.
Megan can make about 20 pennants.

For numbers 16a–16c, estimate to compare. Choose <, >, or =.

Question 16.
16a. 18 \(\frac{3}{10} \div 2 \frac{5}{6}\) ? \(30 \frac{7}{9} \div 3 \frac{1}{3}\)
_____

Answer:
16a. 18 \(\frac{3}{10} \div 2 \frac{5}{6}\) < \(30 \frac{7}{9} \div 3 \frac{1}{3}\)

Explanation:
18 \(\frac{3}{10}\) = 183/10 = 18.3
2 \(\frac{5}{6}\) = 17/6 = 2.833
18.3 is closer to 18
2.833 is closer to 3
18/3 = 6
30 \(\frac{7}{9}\) = 277/9 = 30.777
3 \(\frac{1}{3}\) = 10/3 = 3.333
30.777 is closer to 30
3.333 is closer to 3
30/3 = 10
6 < 10

Question 16.
16b. 17 \(\frac{4}{5} \div 6 \frac{1}{6}\) ? \(19 \frac{8}{9} \div 4 \frac{5}{8}\)
_____

Answer:
17 \(\frac{4}{5} \div 6 \frac{1}{6}\) < \(19 \frac{8}{9} \div 4 \frac{5}{8}\)

Explanation:
17 \(\frac{4}{5}\) = 89/5 = 17.8
6 \(\frac{1}{6}\) = 37/6 = 6.1666
17.8 is closer to 18
6.1666 is closer to 6
18/6 = 3
19 \(\frac{8}{9}\) = 179/9 = 19.888
4 \(\frac{5}{8}\) = 37/8 = 4.625
19.888 is closer to 20
4.625 is closer to 5
20/5 = 4
3 < 4
17 \(\frac{4}{5} \div 6 \frac{1}{6}\) < \(19 \frac{8}{9} \div 4 \frac{5}{8}\)

Question 16.
16c. 17 \(\frac{5}{6} \div 6 \frac{1}{4}\) ? \(11 \frac{5}{7} \div 2 \frac{3}{4}\)
_____

Answer:
17 \(\frac{5}{6} \div 6 \frac{1}{4}\) < \(11 \frac{5}{7} \div 2 \frac{3}{4}\)

Explanation:
17 \(\frac{5}{6}\) = 107/6 = 17.833
6 \(\frac{1}{4}\) = 25/4 = 6.25
17.833 is closer to 18
6.25 is closer to 6
18/6 = 3
11 \(\frac{5}{7}\) = 82/7 = 11.714
2 \(\frac{3}{4}\) = 11/4 = 2.75
11.714 is closer to 12
2.75 is closer to 3
12/3 = 4
3 < 4
17 \(\frac{5}{6} \div 6 \frac{1}{4}\) < \(11 \frac{5}{7} \div 2 \frac{3}{4}\)

Estimate Quotients – Page No. 105

Estimate using compatible numbers.

Question 1.
\(12 \frac{3}{16} \div 3 \frac{9}{10}\)
______

Answer:
3

Explanation:
12 \(\frac{3}{16}\) = 195/16 = 12.1875
3 \(\frac{9}{10}\) = 39/10 = 3.9
12.1875 is closer to 12
3.9 is closer to 4
12/4 = 3

Question 2.
\(15 \frac{3}{8} \div \frac{1}{2}\)
______

Answer:
30

Explanation:
15 \(\frac{3}{8}\) = 123/8 = 15.375
\(\frac{1}{2}\) = 0.5
15.375 is closer to 15
0.5 is closer to 0.5
15/0.5 = 30

Question 3.
\(22 \frac{1}{5} \div 1 \frac{5}{6}\)
______

Answer:
11

Explanation:
22 \(\frac{1}{5}\) = 111/5 = 22.2
1 \(\frac{5}{6}\) = 11/6 = 1.8333
22.2 is closer to 22
1.8333 is closer to 2
22/2 = 11

Question 4.
\(7 \frac{7}{9} \div \frac{4}{7}\)
______

Answer:
16

Explanation:
7 \(\frac{7}{9}\) = 70/9 = 7.777
\(\frac{4}{7}\) = 0.571
7.777 is closer to 8
0.571 is closer to 0.5
8/0.5 = 16

Question 5.
\(18 \frac{1}{4} \div 2 \frac{4}{5}\)
______

Answer:
6

Explanation:
18 \(\frac{1}{4}\) = 73/4 = 18.25
2 \(\frac{4}{5}\) = 14/5 = 2.8
18.25 is closer to 18
2.8 is closer to 3
18/3 = 6

Question 6.
\(\frac{15}{16} \div \frac{1}{7}\)
______

Answer:
10

Explanation:
\(\frac{15}{16}\) = 0.9375
\(\frac{1}{7}\) = 0.1428
0.9375 is closer to 1
0.1428 is closer to 0.1
1/0.1 = 10

Question 7.
\(14 \frac{7}{8} \div \frac{5}{11}\)
______

Answer:
30

Explanation:
14 \(\frac{7}{8}\) = 119/8 = 14.875
\(\frac{5}{11}\) = 0.4545
14.875 is closer to 15
0.4545 is closer to 0.5
15/0.5 = 30

Question 8.
\(53 \frac{7}{12} \div 8 \frac{11}{12}\)
______

Answer:
6

Explanation:
53 \(\frac{7}{12}\) = 643/12 = 53.58
8 \(\frac{11}{12}\) = 107/12 = 8.916
53.58 is closer to 54
8.916 is closer to 9
54/9 = 6

Question 9.
\(1 \frac{1}{6} \div \frac{1}{9}\)
______

Answer:
10

Explanation:
1 \(\frac{1}{6}\) = 7/6 = 1.166
\(\frac{1}{9}\) = 0.111
1.166 is closer to 1
0.111 is closer to 0.1
1/0.1 = 10

Problem Solving

Question 10.
Estimate the number of pieces Sharon will have if she divides 15 \(\frac{1}{3}\) yards of fabric into 4 \(\frac{4}{5}\) yard lengths.
About ______ pieces

Answer:
About 3 pieces

Explanation:
Sharon will have if she divides 15 \(\frac{1}{3}\) yards of fabric into 4 \(\frac{4}{5}\) yard lengths.
3 7/36 is the answer.
So, about 3 pieces

Question 11.
Estimate the number of \(\frac{1}{2}\) quart containers Ethan can fill from a container with 8 \(\frac{7}{8}\) quarts of water.
About ______ containers

Answer:
About 18 containers

Question 12.
How is estimating quotients different from estimating products?
Type below:
__________

Answer:
To estimate products and quotients, you need to first round the numbers. To round to the nearest whole number, look at the digit in the tenths place. If it is less than 5, round down. If it is 5 or greater, round up. Remember that an estimate is an answer that is not exact, but is approximate and reasonable.
Let’s look at an example of estimating a product.
Estimate the product: 11.256×6.81
First, round the first number. Since there is a 2 in the tenths place, 11.256 rounds down to 11.
Next, round the second number. Since there is an 8 in the tenths place, 6.81 rounds up to 7.
Then, multiply the rounded numbers. 11×7=77
The answer is 77.
Let’s look at an example of estimating a quotient.
Estimate the quotient: 91.93÷4.39
First, round the first number. Since there is a 9 in the tenths place, 91.93 rounds up to 92.
Next, round the second number. Since there is a 3 in the tenths place, 4.39 rounds down to 4.
Then, divide the rounded numbers.
92÷4=23
The answer is 23.

Lesson Check – Page No. 106

Question 1.
Each loaf of pumpkin bread calls for 1 \(\frac{3}{4}\) cups of raisins. About how many loaves can be made from 10 cups of raisins?
About ______ loaves

Answer:
About 5 loaves

Explanation:
Divide 10 by 1 3/4.
The answer is 5.714285
So you can make about 5 loaves of bread with 10 cups of raisins if each loaf needs 1 3/4 cups of raisins.

Question 2.
Perry’s goal is to run 2 \(\frac{1}{4}\) miles each day. One lap around the school track is \(\frac{1}{3}\) mile. About how many laps must he run to reach his goal?
About ______ laps

Answer:
About 9 laps

Explanation:
Perry’s goal is to run 2 \(\frac{1}{4}\) miles each day. One lap around the school track is \(\frac{1}{3}\) mile.
2 \(\frac{1}{4}\) = 9/4 = 2.25
\(\frac{1}{3}\) = 0.333
Perry will have to run 9 laps to reach his goal.

Spiral Review

Question 3.
A recipe calls for \(\frac{3}{4}\) teaspoon of red pepper. Uri wants to use \(\frac{1}{3}\) of that amount. How much red pepper should he use?
\(\frac{□}{□}\) teaspoon

Answer:
\(\frac{1}{4}\) teaspoon

Explanation:
A recipe calls for \(\frac{3}{4}\) teaspoon of red pepper. Uri wants to use \(\frac{1}{3}\) of that amount.
\(\frac{1}{3}\) of \(\frac{3}{4}\) = \(\frac{1}{4}\)

Question 4.
A recipe calls for 2 \(\frac{2}{3}\) cups of apple slices. Zoe wants to use 1 \(\frac{1}{2}\) times this amount. How many cups of apples should Zoe use?
______ cups

Answer:
4 cups

Explanation:
A recipe calls for 2 2/3 cups of apple slices.
Zoe wants to use 1 1/2 times this amount.
We will multiply the number of apple slices to 1 1/2
2 2/3 X 1 1/2
8/3 X3/2 = 24/6 = 4 cups
Zoe will use 4 cups of apple slices.

Question 5.
Edgar has 2.8 meters of rope. If he cuts it into 7 equal parts, how long will each piece be?
______ meters

Answer:
0.4 meters

Explanation:
2.8/7 = 0.4 meters

Question 6.
Kami has 7 liters of water to fill water bottles that each hold 2.8 liters. How many bottles can she fill?
______ bottles

Answer:
2 bottles

Explanation:
7/2.8 = 2.5
she can only fill 2 because anything over that would 8.4 liters of water

Share and Show – Page No. 109

Estimate. Then find the quotient.

Question 1.
\(\frac{5}{6}\) ÷ 3
\(\frac{□}{□}\)

Answer:
\(\frac{3}{10}\)

Explanation:
5/6 = 0.8333 is closer to 0.9
0.9/3 = 0.3 = 3/10

Use a number line to find the quotient.

Question 2.
\(\frac{3}{4} \div \frac{1}{8}\)
_______

Answer:
grade 6 chapter 2 image 7

Explanation:
3/4 x 8 = 3 x 2 = 6

Question 3.
\(\frac{3}{5} \div \frac{3}{10}\)
_______

Answer:

Explanation:
3/5 x 10/3 = 2

Estimate. Then write the quotient in simplest form.

Question 4.
\(\frac{3}{4} \div \frac{5}{6}\)
\(\frac{□}{□}\)

Answer:
\(\frac{1}{1}\)

Explanation:
3/4 = 0.75 is closer to 0.8
5/6 = 0.8333 is closer to 0.8
0.8/0.8 = 1

Question 5.
\(3 \div \frac{3}{4}\)
_______

Answer:
4

Explanation:
3/4 = 0.75
3/0.75 = 4

Question 6.
\(\frac{1}{2} \div \frac{3}{4}\)
\(\frac{□}{□}\)

Answer:
\(\frac{625}{1000}\)

Explanation:
1/2 = 0.5
3/4 = 0.75 is closer to 0.8
0.5/0.8 = 0.625 = 625/1000

Question 7.
\(\frac{5}{12} \div 3\)
\(\frac{□}{□}\)

Answer:
\(\frac{2}{10}\)

Explanation:
5/12 = 0.4166 is closer to 0.6
0.6/3 = 0.2 = 2/10

On Your Own

Practice: Copy and Solve Estimate. Then write the quotient in simplest form

Question 8.
\(2 \div \frac{1}{8}\)
_______

Answer:
20

Explanation:
1/8 = 0.125 is closer to 0.1
2/0.1 = 20

Question 9.
\(\frac{3}{4} \div \frac{3}{5}\)
\(\frac{□}{□}\)

Answer:
\(\frac{1}{1}\)

Explanation:
3/4 = 0.75 is closer to 0.8
3/5 = is 0.6 closer to 0.8
0.8/0.8 = 1

Question 10.
\(\frac{2}{5} \div 5\)
\(\frac{□}{□}\)

Answer:
\(\frac{1}{10}\)

Explanation:
2/5 = 0.4 is closer to 0.5
0.5/5 = 0.1 = 1/10

Question 11.
\(4 \div \frac{1}{7}\)
_______

Answer:
40

Explanation:
1/7 = 0.1428 is closer to 0.1
4/0.1 = 40

Practice: Copy and Solve Evaluate using the order of operations.

Write the answer in simplest form.

Question 12.
\(\left(\frac{3}{5}+\frac{1}{10}\right) \div 2\)
\(\frac{□}{□}\)

Answer:
\(\frac{7}{20}\)

Explanation:
3/5 + 1/10 = 7/10 = 0.7
0.7/2 = 7/20

Question 13.
\(\frac{3}{5}+\frac{1}{10} \div 2\)
\(\frac{□}{□}\)

Answer:
\(\frac{13}{20}\)

Explanation:
\(\frac{3}{5}+\frac{1}{10} \div 2\)
(1/10)/2 = 1/20
3/5 + 1/20 = 0.65 = 13/20

Question 14.
\(\frac{3}{5}+2 \div \frac{1}{10}\)
_______ \(\frac{□}{□}\)

Answer:

Explanation:
2/(1/10) = 1/5
3/5 + 1/5 = 4/5

Question 15.
Generalize Suppose the divisor and the dividend of a division problem are both fractions between 0 and 1, and the divisor is greater than the dividend. Is the quotient less than, equal to, or greater than 1?
Type below:
__________

Answer:
Divisor and Dividend are fractions lying between 0 and 1
Also, Divisor > Dividend
A smaller number is being divided by a larger number
Whenever a smaller number is divided by a larger number, the quotient is less than 1
Example:
0,5/0,6 Here, they are both numbers between 0 and 1, and the divisor is greater than the dividend.
The result is 0,8333, LESS THAN 1
Hence, the answer is that the quotient will be less than 1

Problem Solving + Applications – Page No. 110

Use the table for 16–19.
Go Math Grade 6 Answer Key Chapter 2 Fractions and Decimals 16

Question 16.
Kristen wants to cut ladder rungs from a 6 ft board. How many ladder rungs can she cut?
_______ ladder rungs

Answer:
8 ladder rungs

Explanation:
Kristen wants to cut ladder rungs from a 6 ft board.
ladder rungs = 3/4 ft
6/(3/4) = 8 rungs

Question 17.
Pose a Problem Look back at Problem 16. Write and solve a new problem by changing the length of the board Kristen is cutting for ladder rungs.
Type below:
__________

Answer:
Kristen wants to cut ladder rungs from a 9 ft board. How many ladder rungs can she cut?
Kristen wants to cut ladder rungs from a 9 ft board.
ladder rungs = 3/4 ft
9/(3/4) = 12 rungs

Question 18.
Dan paints a design that has 8 equal parts along the entire length of the windowsill. How long is each part of the design?
\(\frac{□}{□}\) yards

Answer:
\(\frac{1}{16}\) yards

Explanation:
Dan paints a design that has 8 equal parts along the entire length of the windowsill.
(1/2)/8 = 1/2 x 1/8 = 1/16 yards

Question 19.
Dan has a board that is \(\frac{15}{16}\) yd. How many “Keep Out” signs can he make if the length of the sign is changed to half of the original length?
_______ signs

Answer:
3 signs

Explanation:
Dan has a board that is \(\frac{15}{16}\) yd.
If the length of the sign is changed to half of the original length, (5/8)/2 = 5/16
(15/16) ÷ 5/16 = 15/16 x 16/5 = 3

Question 20.
Lauren has \(\frac{3}{4}\) cup of dried fruit. She puts the dried fruit into bags, each holding \(\frac{1}{8}\) cup. How many bags will Lauren use? Explain your answer using words and numbers.
Type below:
__________

Answer:
6

Explanation:
Lauren has \(\frac{3}{4}\) cup of dried fruit. She puts the dried fruit into bags, each holding \(\frac{1}{8}\) cup.
3/4 ÷ 1/8 = 3/4 x 8 = 6
Lauren has 3/4 and in 1/4 there are 2 1/8s. That 3 fourths times two = 6 so 6 one eights

Divide Fractions – Page No. 111

Estimate. Then write the quotient in simplest form.

Question 1.
\(5 \div \frac{1}{6}\)
_____

Answer:
25

Explanation:
1/6 = 0.166 is closer to 0.2
5/0.2 = 25

Question 2.
\(\frac{1}{2} \div \frac{1}{4}\)
_____

Answer:
5

Explanation:
1/2 = 0.5 is closer to 1
1/4 = 0.25 is closer to 0.2
1/0.2 = 5

Question 3.
\(\frac{4}{5} \div \frac{2}{3}\)
_____ \(\frac{□}{□}\)

Answer:
1 \(\frac{1}{5}\)

Explanation:
4/5 = 0.8 is closer to 0.8
2/3 = 0.66 is closer to 0.6
0.8/0.6 = 1 1/5

Question 4.
\(\frac{14}{15} \div 7\)
\(\frac{□}{□}\)

Answer:
\(\frac{2}{15}\)

Explanation:
14/15 = 0.9333
0.9/7 = 2/15

Question 5.
\(8 \div \frac{1}{3}\)
_____

Answer:
20

Explanation:
1/3 = 0.33 is closer to 0.4
8/0.4 = 20

Question 6.
\(\frac{12}{21} \div \frac{2}{3}\)
\(\frac{□}{□}\)

Answer:
\(\frac{1}{1}\)

Explanation:
12/21 = 0.571 is closer to 0.6
2/3 = 0.666 is closer to 0.6
0.6/0.6 = 1

Question 7.
\(\frac{5}{6} \div \frac{5}{12}\)
_____

Answer:
2

Explanation:
5/6 = 0.833 is closer to 0.8
5/12 = 0.416 is closer to 0.4
0.8/0.4 = 2

Question 8.
\(\frac{5}{8} \div \frac{1}{2}\)
_____ \(\frac{□}{□}\)

Answer:
1 \(\frac{2}{10}\)

Explanation:
5/8 = 0.625 is closer to 0.6
1/2 = 0.5 is closer to 0.5
0.6/0.5 = 1.2 = 1 2/10

Question 9.
Joy ate \(\frac{1}{4}\) of a pizza. If she divides the rest of the pizza into pieces equal to \(\frac{1}{8}\) pizza for her family, how many pieces will her family get?
_____ pieces

Answer:
6 pieces

Explanation:
The pizza is divided into 4 pieces, Joy ate 1/4.
So, the left pices are 1 – 1/4 = 3/4
now, 3/4 of a pizza and Joy will divide this rest of the pizza in pieces equal to 1/8, so we need to make a division
(3/4) ÷ (1/8) = 24/4 = 6 pieces.

Question 10.
Hideko has \(\frac{3}{5}\) yard of ribbon to tie on balloons for the festival. Each balloon will need \(\frac{3}{10}\) yard of ribbon. How many balloons can Hideko tie with ribbon?
_____ balloons

Answer:
2 balloons

Explanation:
3/10 yard of ribbon required to tie = 1 balloon
3/5 yard of ribber can tie = (3/5) ÷ (3/10) = 2 ballons
With 3/5 yard, Hideko can tie 2 balloons

Problem Solving

Question 11.
Rick knows that 1 cup of glue weighs \(\frac{1}{18}\) pound. He has \(\frac{2}{3}\) pound of glue. How many cups of glue does he have?
_____ cups

Answer:
12 cups

Explanation:
For 1/18lb, 1 cup
For 2/3lb, x cups.
1/8x = 1 x 2/3
1/8x = 2/3
x = 2/3 x 18
x = 2 x 6 = 12 cups

Question 12.
Mrs. Jennings had \(\frac{5}{7}\) gallon of paint. She gave \(\frac{1}{7}\) gallon each to some students. How many students received paint if Mrs. Jennings gave away all the paint?
_____ students

Answer:
4 students

Explanation:
Mrs. Jennings had \(\frac{5}{7}\) gallon of paint. She gave \(\frac{1}{7}\) gallon each to some students.
\(\frac{5}{7}\) ÷ \(\frac{1}{7}\) = 25/7 = 3.571 is closer to 4

Question 13.
Write a word problem that involves two fractions. Include the solution.
Type below:
__________

Answer:
Mrs. Jennings had \(\frac{5}{7}\) gallon of paint. She gave \(\frac{1}{7}\) gallon each to some students. How many students received paint if Mrs. Jennings gave away all the paint?
Answer:
Mrs. Jennings had \(\frac{5}{7}\) gallon of paint. She gave \(\frac{1}{7}\) gallon each to some students.
\(\frac{5}{7}\) ÷ \(\frac{1}{7}\) = 25/7 = 3.571 is closer to 4

Lesson Check – Page No. 112

Question 1.
There was \(\frac{2}{3}\) of a pizza for 6 friends to share equally. What fraction of the pizza did each person get?
\(\frac{□}{□}\)

Answer:
\(\frac{1}{9}\)

Explanation:
There was \(\frac{2}{3}\) of a pizza for 6 friends to share equally.
\(\frac{2}{3}\) ÷ 6 = 2/3 x 1/6 = 2/18 = 1/9

Question 2.
Rashad needs \(\frac{2}{3}\) pound of wax to make a candle. How many candles can he make with 6 pounds of wax?
_____ candles

Answer:
9 candles

Explanation:
Rashad needs 2/3 pound a wax to make candles.
1 Candle = 2/3 pounds.
So, for 2 pounds,
3 x 2/3 = 3 candles
2 pounds = 3 candles
1 pound = 3/2 candles
So, for 6 pounds,
6 x 3/2 = 9 candles

Spiral Review

Question 3.
Jeremy had \(\frac{3}{4}\) of a submarine sandwich and gave his friend \(\frac{1}{3}\) of it. What fraction of the sandwich did the friend receive?
\(\frac{□}{□}\)

Answer:
\(\frac{1}{4}\)

Explanation:
Jeremy had \(\frac{3}{4}\) of a submarine sandwich and gave his friend \(\frac{1}{3}\) of it.
1/3 x 3/4 = 1/4

Question 4.
Ebony walked at a rate of 3 \(\frac{1}{2}\) miles per hour for 1 \(\frac{1}{3}\) hours. How far did she walk?
_____ \(\frac{□}{□}\)

Answer:
4 \(\frac{2}{3}\)

Explanation:
Ebony walked at a rate of 3 \(\frac{1}{2}\) miles per hour for 1 \(\frac{1}{3}\) hours.
3 1/2 miles = 7/2 miles … 1 hour
x miles = ? … 1 1/3 hours = 4/3 hours
7/2 x 4/3 = 1 x x
x = 7/2 x 4/3
x = 14/3 = 4 2/3 miles
The correct result would be 4 2/3 miles.

Question 5.
Penny uses \(\frac{3}{4}\) yard of fabric for each pillow she makes. How many pillows can she make using 6 yards of fabric?
_____ pillows

Answer:
8 pillows

Explanation:
Penny uses \(\frac{3}{4}\) yard of fabric for each pillow she makes.
Using 6 yards of fabric 6/(3/4) = 24/3 = 8

Question 6.
During track practice, Chris ran 2.5 laps in 81 seconds. What was his average time per lap?
_____ seconds

Answer:
32.4 seconds

Explanation:
During track practice, Chris ran 2.5 laps in 81 seconds.
81/2.5 = 32.4 seconds

Share and Show – Page No. 115

Use the model to find the quotient.

Question 1.
\(3 \frac{1}{3} \div \frac{1}{3}\)
Go Math Grade 6 Answer Key Chapter 2 Fractions and Decimals 17
_____

Answer:
21

Explanation:
Model 3 with 3 hexagonal blocks.
Model 1/2 with 1 trapezoid block.
For 1/6,
6 triangle blocks are equal to 1 hexagon.
So, a triangle block shows 1/6.
Count the triangles.
There are 21 triangle blocks.
So, 3 1/2 ÷ 1/6 = 21.

Question 2.
\(2 \frac{1}{2} \div \frac{1}{6}\)
Go Math Grade 6 Answer Key Chapter 2 Fractions and Decimals 18
_____

Answer:
15

Explanation:
Model 2 with 2 hexagonal blocks.
Model 1/2 with 1 trapezoid block.
For 1/6,
6 triangle blocks are equal to 1 hexagon.
So, a triangle block shows 1/6.
Count the triangles.
There are 15 triangle blocks.
So, \(2 \frac{1}{2} \div \frac{1}{6}\) = 15.

Use pattern blocks to find the quotient. Then draw the model.

Question 3.
\(2 \frac{2}{3} \div \frac{1}{6}\)
_____

Answer:
grade 6 chapter 2 image 1

Explanation:
2 2/3 = 8/3
8/3 ÷ 1/6 = 16

Question 4.
\(3 \frac{1}{2} \div \frac{1}{2}\)
_____

Answer:
grade 6 chapter 2 image 2

Explanation:
3 1/2 = 7/2
7/2 ÷ 1/2 = 7

Draw a model to find the quotient.

Question 5.
\(3 \frac{1}{2} \div 3\)
_____ \(\frac{□}{□}\)

Answer:
grade 6 chapter 2 image 3

Explanation:
3 1/2 = 7/2
7/2 ÷ 3 = 21/2

Question 6.
\(1 \frac{1}{4} \div 2\)
\(\frac{□}{□}\)

Answer:
grade 6 chapter 2 image 4

Explanation:
1/4 ÷ 2 = 1/2

Question 7.
Use Appropriate Tools Explain how models can be used to divide mixed numbers by fractions or whole numbers
Type below:
__________

Answer:
Multiply the whole number part by the fraction’s denominator. Add that to the numerator. Then write the result on top of the denominator.

Problem Solving + Applications – Page No. 116

Use a model to solve. Then write an equation for the model.

Question 8.
Use Models Eliza opens a box of bead kits. The box weighs 2 \(\frac{2}{3}\) lb. Each bead kit weighs \(\frac{1}{6}\) lb. How many kits are in the box? What does the answer mean?
Type below:
__________

Answer:
grade 6 chapter 2 image 6
16 kits are in the box

Explanation:
Eliza opens a box of bead kits. The box weighs 2 \(\frac{2}{3}\) lb. Each bead kit weighs \(\frac{1}{6}\) lb, 2 \(\frac{2}{3}\) ÷ \(\frac{1}{6}\) = 8/3 ÷ 1/6 = 16.
16 kits are in the box

Question 9.
Hassan has two boxes of trail mix. Each box holds 1 \(\frac{2}{3}\) lb of trail mix. He eats \(\frac{1}{3}\) lb of trail mix each day. How many days can Hassan eat trail mix before he runs out?
_____ days

Answer:
10 days

Explanation:
Hassan has two boxes of trail mix. Each box holds 1 \(\frac{2}{3}\) lb of trail mix.
1 \(\frac{2}{3}\) = 5/3
2 x (5/3) = 10/3
He eats \(\frac{1}{3}\) lb of trail mix each day.
10/3 ÷ 1/3 = 10
Hassan eats trail mix for 10 days before he runs out.

Question 10.
Sense or Nonsense? Steve made this model to show \(2 \frac{1}{3} \div \frac{1}{6}\). He says that the quotient is 7. Is his answer sense or nonsense? Explain your reasoning
Go Math Grade 6 Answer Key Chapter 2 Fractions and Decimals 19
Type below:
__________

Answer:
\(2 \frac{1}{3} \div \frac{1}{6}\) = 7/3 ÷ 1/6 = 14.
He said the quotient is 7.
His answer is Nonsense.

Question 11.
Eva is making muffins to sell at a fundraiser. She has 2 \(\frac{1}{4}\) cups of flour, and the recipe calls for \(\frac{3}{4}\) cup of flour for each batch of muffins. Explain how to use a model to find the number of batches of muffins Eva can make.
Type below:
__________

Answer:
3

Explanation:
Eva is making muffins to sell at a fundraiser. She has 2 \(\frac{1}{4}\) cups of flour, and the recipe calls for \(\frac{3}{4}\) cup of flour for each batch of muffins.
2 \(\frac{1}{4}\) ÷ \(\frac{3}{4}\) = 9/4 ÷ 3/4 = 3

Model Mixed Number Division – Page No. 117

Use the model to find the quotient.

Question 1.
\(4 \frac{1}{2} \div \frac{1}{2}\)
Go Math Grade 6 Answer Key Chapter 2 Fractions and Decimals 20
_____

Answer:
9

Explanation:
Count the number of trapezoids to find the answer.

Question 2.
\(3 \frac{1}{3} \div \frac{1}{6}\)
Go Math Grade 6 Answer Key Chapter 2 Fractions and Decimals 21
_____

Answer:
20

Use pattern blocks or another model to find the quotient. Then draw the model.

Question 3.
\(2 \frac{1}{2} \div \frac{1}{6}\)
_____

Answer:
grade 6 chapter 2 image 5

Explanation:
Model 2 with 2 hexagonal blocks.
Model 1/2 with 1 trapezoid block.
For 1/6,
6 triangle blocks are equal to 1 hexagon.
So, a triangle block shows 1/6.
Count the triangles.
There are 15 triangle blocks.
So, 212÷16 = 15.

Question 4.
\(2 \frac{3}{4} \div 2\)
_____

Answer:
grade 6 chapter 2 image 6

Explanation:
2 3/4 ÷ 2 = 11/2

Problem Solving

Question 5.
Marty has 2 \(\frac{4}{5}\) quarts of juice. He pours the same amount of juice into 2 bottles. How much does he pour into each bottle?
_____ \(\frac{□}{□}\) quarts

Answer:
1\(\frac{2}{5}\) quarts

Explanation:
Marty has 2 \(\frac{4}{5}\) quarts of juice. He pours the same amount of juice into 2 bottles.
2 \(\frac{4}{5}\) = 14/5 = 2.8
2.8/2 = 1.4 = 1 2/5

Question 6.
How many \(\frac{1}{3}\) pound servings are in 4 \(\frac{2}{3}\) pounds of cheese?
_____ pounds

Answer:
14 pounds

Explanation:
4 2/3 = 14/3
(14/3)/(1/3) = 14

Question 7.
Write a word problem that involves dividing a mixed number by a whole number. Solve the problem and describe how you found the answer.
Type below:
__________

Answer:
How many \(\frac{1}{3}\) pound servings are in 4 \(\frac{2}{3}\) pounds of cheese?
Explanation:
4 2/3 = 14/3
(14/3)/(1/3) = 14

Lesson Check – Page No. 118

Sketch a model to find the quotient.

Question 1.
Emma has 4 \(\frac{1}{2}\) pounds of birdseed. She wants to divide it evenly among 3 bird feeders. How much birdseed should she put in each?
_____ \(\frac{□}{□}\) pounds

Answer:
1\(\frac{1}{2}\) pounds

Explanation:
Emma has 4 1/2 pounds of birdseed.
Convert this to an improper fraction.
4 1/2 = 9/2
Emma wants to divide it evenly among 3 bird feeders.
So, she should put (9/2)/3 = 3/2 = 1 1/2

Question 2.
A box of crackers weighs 11 \(\frac{1}{4}\) ounces. Kaden estimates that one serving is \(\frac{3}{4}\) ounce. How many servings are in the box?
_____ servings

Answer:
15 servings

Explanation:
11 1/4 by 3/4
11 1/4 = 45/4
45/4 / 3/4 = 45/4 × 4/3 = 180/12 = 15
there are 15 servings

Spiral Review

Question 3.
The Ecology Club has volunteered to clean up 4.8 kilometers of highway. The members are organized into 16 teams. Each team will clean the same amount of highway. How much highway will each team clean?
_____ kilometers

Answer:
0.3 kilometers

Explanation:
The Ecology Club has volunteered to clean up 4.8 kilometers of highway. The members are organized into 16 teams.
The total length of the highway is given to clean = 4.8 kilometers
If the members are organized into 16 teams.
4.8/16 = 0.3
Hence, each team will clean 0.3 kilometers of the highway.

Question 4.
Tyrone has $8.06. How many bagels can he buy if each bagel costs $0.65?
_____ bagels

Answer:
12 bagels

Explanation:
$8.06/$0.65 = 12.4
12 bagels

Question 5.
A nail is 0.1875 inch thick. What is its thickness as a fraction? Is 0.1875 inch closer to \(\frac{1}{8}\) inch or \(\frac{1}{4}\) inch on a number line?
Type below:
__________

Answer:
0.1875 = 3/16 which is at the same distance to 1/4 and 1/8
It is the same distance apart.

Question 6.
Maria wants to find the product of 5 \(\frac{3}{20}\) × 3 \(\frac{4}{25}\) using decimals instead of fractions. How can she rewrite the problem using decimals?
Type below:
__________

Answer:
16.274

Explanation:
The decimal for 5 3/20 is 5.15
The decimal for 3 4/25 is 3.16
5.15 × 3.16 = 16.274

Share and Show – Page No. 121

Estimate. Then write the quotient in simplest form.

Question 1.
\(4 \frac{1}{3} \div \frac{3}{4}\)
______ \(\frac{□}{□}\)

Answer:
5\(\frac{375}{1000}\)

Explanation:
4 1/3 = 13/3 = 4.333 is closer to 4.3
3/4 = 0.75 is closer to 0.8
4.3/0.8 = 5.375 = 5 375/1000

Question 2.
Six hikers shared 4 \(\frac{1}{2}\) lb of trail mix. How much trail mix did each hiker receive?
\(\frac{□}{□}\)

Answer:
\(\frac{75}{100}\)

Explanation:
6 hikers = 4.5 lbs of trail mix
4.5/6= .75 lbs each hiker.

Question 3.
\(5 \frac{2}{3} \div 3\)
______ \(\frac{□}{□}\)

Answer:
2\(\frac{947}{1000}\)

Explanation:
5 2/3 = 17/3 = 5.666 is closer to 5.6
5.6/3 = 1.866 is closer to 1.9
5.6/1.9 = 2.947 = 2 947/1000

Question 4.
\(7 \frac{1}{2} \div 2 \frac{1}{2}\)
______

Answer:
3

Explanation:
7 1/2 = 15/2 = 7.5
2 1/2 = 5/2 = 2.5
7.5/2.5 = 3

On Your Own

Estimate. Then write the quotient in simplest form.

Question 5.
\(5 \frac{3}{4} \div 4 \frac{1}{2}\)
______ \(\frac{□}{□}\)

Answer:
1\(\frac{27}{100}\)

Explanation:
5 3/4 = 23/4 = 5.75
4 1/2 = 9/2 = 4.5
5.75/4.5 = 1.27 = 1 27/100

Question 6.
\(5 \div 1 \frac{1}{3}\)
______ \(\frac{□}{□}\)

Answer:
3\(\frac{84}{100}\)

Explanation:
1 1/3 = 4/3 = 1.33 is closer to 1.3
5/1.3 = 3.84 = 3 84/100

Question 7.
\(6 \frac{3}{4} \div 2\)
______ \(\frac{□}{□}\)

Answer:
3\(\frac{2}{5}\)

Explanation:
6 3/4 = 27/4 = 6.75 is closer to 6.8
6.8/2 = 3.4 = 3 2/5

Question 8.
\(2 \frac{2}{9} \div 1 \frac{3}{7}\)
______ \(\frac{□}{□}\)

Answer:
1\(\frac{571}{1000}\)

Explanation:
2 2/9 = 20/9 = 2.22 is closer to 2.2
1 3/7 = 10/7 = 1.428 is closer to 1.4
2.2/1.4 = 1.571 = 1 571/1000

Question 9.
How many 3 \(\frac{1}{3}\) yd pieces can Amanda get from a 3 \(\frac{1}{3}\) yd ribbon?
______

Answer:
1

Explanation:
(3 1/3) ÷ (3 1/3) = 1

Question 10.
Samantha cut 6 \(\frac{3}{4}\) yd of yarn into 3 equal pieces. Explain how she could use mental math to find the length of each piece
Type below:
__________

Answer:
27/12

Explanation:
Samantha cut 6 \(\frac{3}{4}\) yd of yarn into 3 equal pieces.
6 3/4 = 27/4
(27/4)/3
(27/4)(1/3) = 27/12

Evaluate Algebra Evaluate using the order of operations. Write the answer in simplest form.

Question 11.
\(1 \frac{1}{2} \times 2 \div 1 \frac{1}{3}\)
_____ \(\frac{□}{□}\)

Answer:
2\(\frac{1}{4}\)

Explanation:
(1 1/2) × 2 = 3/2 × 2 = 3
1 1/3 = 4/3
3/(4/3) = 9/4 = 2.25 = 2 1/4

Question 12.
\(1 \frac{2}{5} \div 1 \frac{13}{15}+\frac{5}{8}\)
_____ \(\frac{□}{□}\)

Answer:
1\(\frac{3}{8}\)

Explanation:
(1 2/5)/(1 13/15) = (7/5)/(28/15) = 3/4 = 0.75
0.75 + 0.625 = 1.375 = 1 3/8

Question 13.
\(3 \frac{1}{2}-1 \frac{5}{6} \div 1 \frac{2}{9}\)
_____

Answer:
2

Explanation:
(1 5/6)/(1 2/9) = (11/6)/11/9 = 3/2 = 1 1/2 = 1.5
3 1/2 = 7/2 = 3.5
3.5 – 1.5 = 2

Question 14.
Look for a Pattern Find these quotients: \(20 \div 4 \frac{4}{5}\), \(10 \div 4 \frac{4}{5}\), \(5 \div 4 \frac{4}{5}\). Describe a pattern you see.
Type below:
__________

Answer:
20 ÷ 4 4/5 = 20 ÷ 24/5 = 20/4.8 = 4.1666
10 ÷ 4 4/5 = 10 ÷ 24/5 = 10/4.8 = 2.08333
5 ÷ 4 4/5 = 5 ÷ 24/5 = 5/4.8 = 1.04166
The pattern is multiplied by 2 every time.

Page No. 122

Question 15.
Dina hikes \(\frac{1}{2}\) of the easy trail and stops for a break every 3 \(\frac{1}{4}\) miles. How many breaks will she take?
Go Math Grade 6 Answer Key Chapter 2 Fractions and Decimals 22
a. What problem are you asked to solve?
Type below:
__________

Answer:
How many breaks Dina will take when hikes \(\frac{1}{2}\) of the easy trail and stops for a break every 3 \(\frac{1}{4}\) mile.

Question 15.
b. How will you use the information in the table to solve the problem?
Type below:
__________

Answer:
Dina easy trail length, break time

Question 15.
c. How can you find the distance Dina hikes? How far does she hike?
______ \(\frac{□}{□}\) miles

Answer:
9\(\frac{3}{4}\) miles

Explanation:
19 1/2 × 1/2 = 39/2 × 1/2 = 39/4 = 9 3/4

Question 15.
d. What operation will you use to find how many breaks Dina takes?
Type below:
__________

Answer:
Division

Question 15.
e. How many breaks will Dina take?
______ breaks

Answer:
3 breaks

Explanation:
39/4 ÷ 13/4 = 3

Question 16.
Carlo packs 15 \(\frac{3}{4}\) lb of books in 2 boxes. Each book weighs 1 \(\frac{1}{8}\) lb. There are 4 more books in Box A than in Box B. How many books are in Box A? Explain your work.
______ books

Answer:
Carlo packs 15 \(\frac{3}{4}\) lb of books in 2 boxes. Each book weighs 1 \(\frac{1}{8}\) lb.
15 \(\frac{3}{4}\) ÷ 1 \(\frac{1}{8}\) = 63/4 ÷ 9/8 = 14
14 books available in 2 boxes.
There are 4 more books in Box A than in Box B.
Box A contains 5 + 4 = 9 books
Box B contains 5 books

Question 17.
Rex’s goal is to run 13 \(\frac{3}{4}\) miles over 5 days. He wants to run the same distance each day. Jordan said that Rex would have to run 3 \(\frac{3}{4}\) miles each day to reach his goal. Do you agree with Jordan? Explain your answer using words and numbers.
Type below:
__________

Answer:
Rex’s goal is to run 13 \(\frac{3}{4}\) miles over 5 days. He wants to run the same distance each day.
13 \(\frac{3}{4}\) ÷ 5 = 55/4 ÷ 5 = 11/4 or 2 3/4.
Jordan answer is wrong

Divide Mixed Numbers – Page No. 123

Estimate. Then write the quotient in simplest form.

Question 1.
\(2 \frac{1}{2} \div 2 \frac{1}{3}\)
______ \(\frac{□}{□}\)

Answer:
1\(\frac{1}{2}\)

Explanation:
2 1/2 = 5/2 = 2.5 is closer to 3
2 1/3 = 7/3 = 2.333 is closer to 2
3/2 = 1.5 = 1 1/2

Question 2.
\(2 \frac{2}{3} \div 1 \frac{1}{3}\)
______

Answer:
2

Explanation:
2 2/3 = 8/3 = 2.666 is closer to 2.6
1 1/3 = 4/3 = 1.333 is closer to 1.3
2.6/1.3 = 2

Question 3.
\(2 \div 3 \frac{5}{8}\)
\(\frac{□}{□}\)

Answer:
\(\frac{1}{2}\)

Explanation:
3 5/8 = 29/8 = 3.625 is closer to 3.6
2/3.6 = 0.5 = 1/2

Question 4.
\(1 \frac{13}{15} \div 1 \frac{2}{5}\)
\(\frac{□}{□}\)

Answer:
\(\frac{126}{100}\)

Explanation:
1 13/15 = 28/15 = 1.8666 is closer to 1.9
1 2/5 = 7/5 = 1.4 is closer to 1.5
1.9/1.5 = 1.266
126/100

Question 5.
\(10 \div 6 \frac{2}{3}\)
______ \(\frac{□}{□}\)

Answer:
1\(\frac{1}{2}\)

Explanation:
6 2/3 = 20/3 = 6.666 is closer to 6.7
10/6.7 = 3/2 = 1 1/2

Question 6.
\(2 \frac{3}{5} \div 1 \frac{1}{25}\)
______ \(\frac{□}{□}\)

Answer:
2\(\frac{3}{5}\)

Explanation:
2 3/5 = 13/5 = 2.6
1 1/25 = 26/25 = 1.04 is closer to 1
2.6/1 = 13/5 or 2 3/5

Question 7.
\(2 \frac{1}{5} \div 2\)
______ \(\frac{□}{□}\)

Answer:
1\(\frac{1}{10}\)

Explanation:
2 1/5 = 11/5 = 2.2 is closer to 2.2
2.2/2 = 1.1 = 11/10 = 1 1/10

Question 8.
Sid and Jill hiked 4 \(\frac{1}{8}\) miles in the morning and 1 \(\frac{7}{8}\) miles in the afternoon. How many times as far did they hike in the morning as in the afternoon?
______ \(\frac{□}{□}\) times

Answer:
2\(\frac{1}{5}\) times

Explanation:
Sid and Jill hiked 4 \(\frac{1}{8}\) miles in the morning and 1 \(\frac{7}{8}\) miles in the afternoon.
4 \(\frac{1}{8}\) = 33/8
1 \(\frac{7}{8}\) = 15/8
(33/8) ÷ (15/8) = 33/15 = 11/5 or 2 1/5

Problem Solving

Question 9.
It takes Nim 2 \(\frac{2}{3}\) hours to weave a basket. He worked Monday through Friday, 8 hours a day. How many baskets did he make?
______ baskets

Answer:
15 baskets

Explanation:
he worked (Mon – Fri) 5 days at 8 hrs per day = 5 × 8= 40 hrs
40/ (2 2/3) = 40 / (8/3) = 40 × 3/8 = 120/8 = 15 baskets

Question 10.
A tree grows 1 \(\frac{3}{4}\) feet per year. How long will it take the tree to grow from a height of 21 \(\frac{1}{4}\) feet to a height of 37 feet?
______ years

Answer:
9 years

Explanation:
A tree grows 1 3/4 = 7/4 feet per year.
If you would like to know how long will it take the tree to grow from a height of 21 1/4 = 85/4 feet to a height of 37 feet,
37 – 21 1/4 = 37 – 85/4 = 148/4 – 85/4 = 63/4 = 15 3/4
15 3/4 / 1 3/4 = 63/4 / 7/4 = 63/4 × 4/7 = 9 years

Question 11.
Explain how you would find how many 1 \(\frac{1}{2}\) cup servings there are in a pot that contains 22 \(\frac{1}{2}\) cups of soup.
Type below:
__________

Answer:
Given that, Total number of cups = 22 1/2
The number of cups required for each serving = 1 1/2
The number of servings = 22 1/2 ÷ 1 1/2
= 45/2 ÷ 3/2 = 45/3 = 15

Lesson Check – Page No. 124

Question 1.
Tom has a can of paint that covers 37 \(\frac{1}{2}\) square meters. Each board on the fence has an area of \(\frac{3}{16}\) square meters. How many boards can he paint?
______ boards

Answer:
200 boards

Explanation:
Tom has a can of paint that covers 37 \(\frac{1}{2}\) square meters. Each board on the fence has an area of \(\frac{3}{16}\) square meters.
37 \(\frac{1}{2}\) ÷ \(\frac{3}{16}\) = 200 square meters

Question 2.
A baker wants to put 3 \(\frac{3}{4}\) pounds of apples in each pie she makes. She purchased 52 \(\frac{1}{2}\) pounds of apples. How many pies can she make?
______ pies

Answer:
14 pies

Explanation:
A baker wants to put 3 \(\frac{3}{4}\) pounds of apples in each pie she makes. She purchased 52 \(\frac{1}{2}\) pounds of apples.
52 \(\frac{1}{2}\) ÷ 3 \(\frac{3}{4}\) = 14 pies

Spiral Review

Question 3.
The three sides of a triangle measure 9.97 meters, 10.1 meters, and 0.53 meter. What is the distance around the triangle?
______ meters

Answer:
20.6 meters

Explanation:
The distance around the triangle is call perimeter, to get it we must add the 3 sides.
So, 9.97 + 10.1 + 0.53 = 20.6 meters

Question 4.
Selena bought 3.75 pounds of meat for $4.64 per pound. What was the total cost of the meat?
$ ______

Answer:
$17.40

Explanation:
Selena bought 3.75 pounds of meat.
The cost of meat of one pound = $4.64
The total cost of the meat = 4.64 × 3.75 = $17.40
The total cost of 3.75 lb of meat was $17.40.

Question 5.
Melanie prepared 7 \(\frac{1}{2}\) tablespoons of a spice mixture. She uses \(\frac{1}{4}\) tablespoon to make a batch of barbecue sauce. Estimate the number of batches of barbecue sauce she can make using the spice mixture.
Type below:
__________

Answer:
30 batches of sauce

Explanation:
Melanie prepared 7 \(\frac{1}{2}\) tablespoons of a spice mixture. She uses \(\frac{1}{4}\) tablespoon to make a batch of barbecue sauce.
4 X 1/4 tbsp = 1 tbsp.
4 X 7 1/2 = 30.
she can make 30 batches of sauce

Question 6.
Arturo mixed together 1.24 pounds of pretzels, 0.78 pounds of nuts, 0.3 pounds of candy, and 2 pounds of popcorn. He then packaged it in bags that each contained 0.27 pounds. How many bags could he fill?
______ bags

Answer:
16 bags

Explanation:
Arturo mixed together 1.24 pounds of pretzels, 0.78 pounds of nuts, 0.3 pounds of candy, and 2 pounds of popcorn.
1.24 + 0.78 + 0.3 + 2 = 4.32
4.32/0.27 = 16

Page No. 127

Question 1.
There is \(\frac{4}{5}\) lb of sand in the class science supplies. If one scoop of sand weighs \(\frac{1}{20}\) lb, how many scoops of sand can Maria get from the class supplies and still leave \(\frac{1}{2}\) lb in the supplies?
Type below:
__________

Answer:
16 scoops

Explanation:
There is \(\frac{4}{5}\) lb of sand in the class science supplies. If one scoop of sand weighs \(\frac{1}{20}\) lb,
\(\frac{4}{5}\) ÷ \(\frac{1}{20}\) = 4/5 × 1/20 = 16 scoops

Question 2.
What if Maria leaves \(\frac{2}{5}\) lb of sand in the supplies? How many scoops of sand can she get?
______ scoops

Answer:
8 scoops

Explanation:
There is \(\frac{2}{5}\) lb of sand in the class science supplies. If one scoop of sand weighs \(\frac{1}{20}\) lb,
\(\frac{2}{5}\) ÷ \(\frac{1}{20}\) = 2/5 × 20 = 8

Question 3.
There are 6 gallons of distilled water in the science supplies. If 10 students each use an equal amount of the distilled water and there is 1 gal left in the supplies, how much will each student get?
\(\frac{□}{□}\) gallon

Answer:
\(\frac{1}{2}\) gallon

Explanation:
There are 6 gallons of distilled water in the science supplies.
There is 1 gal left in the supplies, 6 – 1 = 5
10 students each use an equal amount of the distilled water = 5/10 = 1/2
.5 gal for each student

On Your Own – Page No. 128

Question 4.
The total weight of the fish in a tank of tropical fish at Fish ‘n’ Fur was \(\frac{7}{8}\) lb. Each fish weighed \(\frac{1}{64}\) lb. After Eric bought some fish, the total weight of the fish remaining in the tank was \(\frac{1}{2}\) lb. How many fish did Eric buy?
______ fish

Answer:
386 fish

Explanation:
The total weight of the fish in a tank of tropical fish at Fish ‘n’ Fur was \(\frac{7}{8}\) lb. Each fish weighed \(\frac{1}{64}\) lb. After Eric bought some fish, the total weight of the fish remaining in the tank was \(\frac{1}{2}\) lb.
386 is the answer

Question 5.
Fish ‘n’ Fur had a bin containing 2 \(\frac{1}{2}\) lb of gerbil food. After selling bags of gerbil food that each held \(\frac{3}{4}\) lb, \(\frac{1}{4}\) lb of food was left in the bin. If each bag of gerbil food sold for $3.25, how much did the store earn?
$ ______

Answer:
$9.75

Explanation:
The store would earn 9.75$ because 3 bags of gerbil food is sold. Then you would multiply 3 by 3.25.

Question 6.
Describe Niko bought 2 lb of dog treats. He gave his dog \(\frac{3}{5}\) lb of treats one week and \(\frac{7}{10}\) lb of treats the next week. Describe how Niko can find how much is left.
Type below:
__________

Answer:
Niko bought 2 lb of dog treats. He gave his dog \(\frac{3}{5}\) lb of treats one week and \(\frac{7}{10}\) lb of treats the next week.
Let us find the amount of dog-food eaten by dogs in two months.
3/5 + 7/10 = 13/10
Now we will subtract the amount of food eaten by the dog from the amount of food initially to find the remaining amount of dog food.
2 – 13/10 = 7/10
Therefore, 7/10 pounds of food was remaining in the bag at the end of the two months.

Question 7.
There were 14 \(\frac{1}{4}\) cups of apple juice in a container. Each day, Elise drank 1 \(\frac{1}{2}\) cups of apple juice. Today, there is \(\frac{3}{4}\) cup of apple juice left. Derek said that Elise drank apple juice on nine days. Do you agree with Derek? Use words and numbers to explain your answer.
Type below:
__________

Answer:
Derek is correct.

Explanation:
An apple juice the container had 14 1/2 =14.25
She drank per day 1 1/2= 1.5
The left part in the container 3/4= .75
14.25 cups – .75 cup = 13.5 cups
13.5 cups ÷ 1.5 cups per day= 9 days

Problem Solving Fraction Operations – Page No. 129

Read each problem and solve.

Question 1.
\(\frac{2}{3}\) of a pizza was left over. A group of friends divided the leftover pizza into pieces each equal to \(\frac{1}{18}\) of the original pizza. After each friend took one piece, \(\frac{1}{6}\) of the original pizza remained. How many friends were in the group?
Go Math Grade 6 Answer Key Chapter 2 Fractions and Decimals 23
______ friends

Answer:
9 friends

Explanation:
Let us say that there are x friends.
Each one gets 1/18 of the original pizza: but this in turn leaves 1/6 of the 2/3 leftover.
1x/18 = 2/3 – 1/6
x = 12 – 3 = 9

Question 2.
Sarah’s craft project uses pieces of yarn that are \(\frac{1}{8}\) yard long. She has a piece of yarn that is 3 yards long. How many \(\frac{1}{8}\) -yard pieces can she cut and still have 1 \(\frac{1}{4}\) yards left?
______ pieces

Answer:
14 pieces

Explanation:
Sarah’s craft project uses pieces of yarn that are \(\frac{1}{8}\) yard long. She has a piece of yarn that is 3 yards long.
If she left 1 \(\frac{1}{4}\) yards left, 3 – 1 \(\frac{1}{4}\) = 7/4
7/4 ÷ \(\frac{1}{8}\) = 14

Question 3.
Alex opens a 1-pint container of orange butter. He spreads \(\frac{1}{16}\) of the butter on his bread. Then he divides the rest of the butter into \(\frac{3}{4}\) -pint containers. How many \(\frac{3}{4}\) -pint containers is he able to fill?
______ \(\frac{□}{□}\) containers

Answer:
1\(\frac{1}{4}\) containers

Explanation:
Alex opens a 1-pint container of orange butter. He spreads \(\frac{1}{16}\) of the butter on his bread.
1 – 1/16 = 15/16
Then he divides the rest of the butter into \(\frac{3}{4}\) -pint containers.
(15/16) ÷ (3/4) = 5/4 = 1 1/4

Question 4.
Kaitlin buys \(\frac{9}{10}\) a pound of orange slices. She eats \(\frac{1}{3}\) of them and divides the rest equally into 3 bags. How much is in each bag?
______ lb

Answer:
17/90 lb

Explanation:
Kaitlin buys \(\frac{9}{10}\) a pound of orange slices. She eats \(\frac{1}{3}\) of them and divides the rest equally into 3 bags.
If she starts with 9/10 pounds and has eaten 1/3 of them, 9/10 – 1/3 = 17/30
This is the amount she has left. Let’s divide this value by 3 to see how many pounds are in one bag.
(17/30)/3 = 17/90
There are 17/90 pounds in one bag.

Question 5.
Explain how to draw a model that represents \(\left(1 \frac{1}{4}-\frac{1}{2}\right) \div \frac{1}{8}\).
Type below:
__________

Answer:
Divide 2 bars into 8 quarters.
Below that draw 1 1/4 or 5 quarters.
Remove 1/2 or 2 quarters
Divide each of the 3 quarters left into 2 eighths

Explanation:
\(\left(1 \frac{1}{4}-\frac{1}{2}\right) \div \frac{1}{8}\)
1 1/4 -1/2 = 5/4 – 1/2 = 3/4
3/4 ÷ 1/8 = 6

Lesson Check – Page No. 130

Question 1.
Eva wanted to fill bags with \(\frac{3}{4}\) pounds of trail mix. She started with 11 \(\frac{3}{8}\) pounds but ate \(\frac{1}{8}\) pound before she started filling the bags. How many bags could she fill?
______ bags

Answer:
15 bags

Explanation:
11 and 3/8-1/8=11 and 2/8=11 and 1/4
3/4 times x bags=11 and 1/4
convert 11 and 1/4 to improper fraction
11 and 1/4 = 11 + 1/4 = 44/4 + 1/4 = 45/4
3/4 times x bags=45/4
x bags = 45/4 × 4/3 = 15 bags
she could fill 15 bags

Question 2.
John has a roll containing 24 \(\frac{2}{3}\) feet of wrapping paper. He wants to divide it into 11 pieces. First, though, he must cut off \(\frac{5}{6}\) foot because it was torn. How long will each piece be?
______ \(\frac{□}{□}\) feet

Answer:
2\(\frac{4}{25}\) feet

Explanation:
John had a roll containing wrapping paper = 24 2/3 = 74/3
First, he must cut off 5/6 foot because it was torn.
He wants to divide it into 11 pieces.
74/3 – 5/6
Taking the L.C.M of 3 and 6 is 6
(148-5)/6 = 143/6 = 23.83 feet
He wants to divide it into 11 pieces. length of the each piece = 23.83/11 = 2.16 feet

Spiral Review

Question 3.
Alexis has 32 \(\frac{2}{5}\) ounces of beads. How many necklaces can she make if each uses 2 \(\frac{7}{10}\) ounces of beads?
______ necklaces

Answer:
12 necklaces

Explanation:
Alexis has 32 \(\frac{2}{5}\) ounces of beads.
If each uses 2 \(\frac{7}{10}\) ounces of beads, 32 \(\frac{2}{5}\) × 2 \(\frac{7}{10}\)
32 \(\frac{2}{5}\) = 162/5
2 \(\frac{7}{10}\) = 27/10
162/5 × 27/10 = 12 necklaces

Question 4.
Joseph has $32.40. He wants to buy several comic books that each cost $2.70. How many comic books can he buy?
______ comic books

Answer:
12 comic books

Explanation:
Joseph has $32.40. He wants to buy several comic books that each cost $2.70.
$32.40/$2.70 = 12 comic books

Question 5.
A rectangle is 2 \(\frac{4}{5}\) meters wide and 3 \(\frac{1}{2}\) meters long. What is its area?
______ \(\frac{□}{□}\) m2

Answer:
9\(\frac{4}{5}\) m2

Explanation:
2 \(\frac{4}{5}\) = 14/5
3 \(\frac{1}{2}\) = 7/2
14/5 × 7/2 = 9 4/5

Question 6.
A rectangle is 2.8 meters wide and 3.5 meters long. What is its area?
______ m2

Answer:
9.8 m2

Explanation:
A rectangle is 2.8 meters wide and 3.5 meters long.
2.8 × 3.5 = 9.8

Chapter 2 Review/Test – Page No. 131

Question 1.
Write the values in order from least to greatest.
Go Math Grade 6 Answer Key Chapter 2 Fractions and Decimals 24
Type below:
__________

Answer:
0.45, 0.5, 5/8, 3/4

Explanation:
3/4 = 0.75
5/8 = 0.625
0.45, 0.5
0.45 < 0.5 < 0.625 < 0.75

Question 2.
For numbers 2a–2d, compare. Choose <, >, or =.
2a. 0.75 _____ \(\frac{3}{4}\)
2b. \(\frac{4}{5}\) _____ 0.325
2c. 1 \(\frac{3}{5}\) _____ 1.9
2d. 7.4 _____ 7 \(\frac{2}{5}\)

Answer:
2a. 0.75 = \(\frac{3}{4}\)
2b. \(\frac{4}{5}\) > 0.325
2c. 1 \(\frac{3}{5}\) < 1.9
2d. 7.4 = 7 \(\frac{2}{5}\)

Explanation:
2a. 3/4 = 0.75
0.75 = 0.75
2b. \(\frac{4}{5}\) = 0.8
0.8 > 0.325
2c. 1 \(\frac{3}{5}\) = 8/5 = 1.6
1.6 < 1.9
2d. 7 \(\frac{2}{5}\) = 37/5 = 7.4
7.4 = 7.4

Question 3.
The table lists the heights of 4 trees.
Go Math Grade 6 Answer Key Chapter 2 Fractions and Decimals 25
For numbers 3a–3d, select True or False for each statement.
3a. The oak tree is the shortest. True False
3b. The birch tree is the tallest. True False
3c. Two of the trees are the same height. True False
3d. The sycamore tree is taller than the maple tree. True False
Type below:
__________

Answer:
3a. The oak tree is the shortest. True
3b. The birch tree is the tallest. False
3c. Two of the trees are the same height. False
3d. The sycamore tree is taller than the maple tree. False

Explanation:
Sycamore = 15 2/3 = 47/3 = 15.666
Oak = 14 3/4 = 59/4 = 14.75
Maple = 15 3/4 = 63/4 = 15.75
Birch = 15.72

Page No. 132

Question 4.
For numbers 4a–4d, choose Yes or No to indicate whether the statement is correct.
Go Math Grade 6 Answer Key Chapter 2 Fractions and Decimals 26
4a. Point A represents 1.0. Yes No
4b. Point B represents \(\frac{3}{10}\). Yes No
4c. Point C represents 6.5. Yes No
4d. Point D represents \(\frac{4}{5}\). Yes No
Type below:
__________

Answer:
4a. Point A represents 1.0. Yes
4b. Point B represents \(\frac{3}{10}\). Yes
4c. Point C represents 6.5. No
4d. Point D represents \(\frac{4}{5}\). Yes

Question 5.
Select the values that are equivalent to one twenty-fifth. Mark all that apply.
Options:
a. 125
b. 25
c. 0.04
d. 0.025

Answer:
c. 0.04

Explanation:
one twenty-fifth = 1/25 = 0.04

Question 6.
The table shows Lily’s homework assignment. Lily’s teacher instructed the class to simplify each expression by dividing the numerator and denominator by the GCF. Complete the table by simplifying each expression and then finding the product.
Go Math Grade 6 Answer Key Chapter 2 Fractions and Decimals 27
Type below:
___________

Answer:
a. Simplified Expression: 1/10
Product: 0.1
b. Simplified Expression: 1/2
Product: 0.5
c. Simplified Expression: 15/56
Product: 0.267
d. Simplified Expression: 1/12
Product: 0.083

Explanation:
a. 2/5 × 1/4 = 2/20
Simplify using the GCF.
The GCF of 2 and 20 is 2.
Divide the numerator and the denominator by 2.
So, 1/10 is the answer.
Product: 0.1
b. 4/5 × 5/8 = 1/2
Product: 0.5
c. 3/7 × 5/8 = 15/ 56
Product: 0.267
d. 4/9 × 3/16 = 1/12
Product: 0.083

Page No. 133

Question 7.
Two-fifths of the fish in Gary’s fish tank are guppies. One fourth of the guppies are red. What fraction of the fish in Gary’s tank are red guppies? What fraction of the fish in Gary’s tank are not red guppies? Show your work.
Type below:
___________

Answer:
1/10 of the fish are red guppies.
and 9/10 of the fish are not red guppies.

Explanation:
two-fifths of the fish in Gary’s fish tank are guppies.
One-fourth of the guppies are red.
Let the total number of fish in Gary’s fish tank be x.
It is given that two-fifths of the fish in Gary’s fish tank are guppies.
So, the number of guppies in Gary’s fish tank is 2/5 × x
Given that One-fourth of the guppies are red.
number of red guppies = 1/4 × 2x/5 = x/10
So, 1/10 of the fish are red guppies.
1 – 1/10 = 9/10 of the fish are not red guppies.

Question 8.
One-third of the students at Finley High School play sports. Two-fifths of the students who play sports are girls. What fraction of all students are girls who play sports? Use numbers and words to explain your answer.
Type below:
___________

Answer:
One-third of the students at Finley High School play sports. Two-fifths of the students who play sports are girls.
1/3 × 2/5 = 2/15 of the girls in the school play sports.

Question 9.
Draw a model to find the quotient.
\(\frac{3}{4}\) ÷ 2 =
\(\frac{3}{4}\) ÷ \(\frac{3}{8}\) =
How are your models alike? How are they different?
Type below:
___________

Answer:
grade 6 chapter 2 image 5

Explanation:
\(\frac{3}{4}\) ÷ 2 = 3/4 × 1/2 = 3/8
\(\frac{3}{4}\) ÷ \(\frac{3}{8}\) = 3/4 × 8/3 = 2
Both models are multiplying with the 3/4.
The number line model shows how many groups of 3/8 are in 3/4.

Question 10.
Explain how to use a model to find the quotient.
2 \(\frac{1}{2}\) ÷ 2 =
Type below:
___________

Answer:
5/4

Explanation:
2 1/2 = 5/2
5/2 groups of 2
5/2 ÷ 2 = 5/2 × 1/2 = 5/4

Page No. 134

Divide. Show your work.

Question 11.
\(\frac{7}{8}\) ÷ \(\frac{3}{5}\) =
_______ \(\frac{□}{□}\)

Answer:
1 \(\frac{11}{24}\)

Explanation:
\(\frac{7}{8}\) ÷ \(\frac{3}{5}\)
\(\frac{7}{8}\) × \(\frac{5}{3}\) = 35/24 = 1 \(\frac{11}{24}\)

Question 12.
\(2 \frac{1}{10} \div 1 \frac{1}{5}=\) =
_______ \(\frac{□}{□}\)

Answer:
1 \(\frac{3}{4}\)

Explanation:
2 \(\frac{1}{10}\) = 21/10
1 \(\frac{1}{5}\) = 6/5
(21/10) ÷ (6/5) = 7/4 or 1 3/4

Question 13.
Sophie has \(\frac{3}{4}\) quart of lemonade. If she divides the lemonade into glasses that hold \(\frac{1}{16}\) quart, how many glasses can Sophie fill? Show your work
_______ glasses

Answer:
12 glasses

Explanation:
Let x be the number of glasses
1/16x = 3/4
x = 3/4 × 16 = 3 × 4 = 12 glasses

Question 14.
Ink cartridges weigh \(\frac{1}{8}\) pound. The total weight of the cartridges in a box is 4 \(\frac{1}{2}\) pounds. How many cartridges does the box contain? Show your work and explain why you chose the operation you did.
_______ cartridges

Answer:
36 cartridges

Explanation:
Weight of ink cartridges = 1/8 pounds
Total weight of the cartridges in a box = 4 1/2 = 9/2 pounds
so, the Number of cartridges that box contain is given by
9/2 ÷ 1/8 = 36
Hence, there are 36 cartridges that box contain.

Question 15.
Beth had 1 yard of ribbon. She used \(\frac{1}{3}\) yard for a project. She wants to divide the rest of the ribbon into pieces \(\frac{1}{6}\) yard long. How many \(\frac{1}{6}\) yard pieces of ribbon can she make? Explain your solution.
_______ pieces

Answer:
4 pieces

Explanation:
Beth had 1 yard of ribbon. She used \(\frac{1}{3}\) yard for a project.
1 – \(\frac{1}{3}\) = 2/3 yard left
She wants to divide the rest of the ribbon into pieces \(\frac{1}{6}\) yard long.
2/3 ÷ 1/6 = 4

Page No. 135

Question 16.
Complete the table by finding the products. Then answer the questions in Part A and Part B.
Go Math Grade 6 Answer Key Chapter 2 Fractions and Decimals 28
Part A
Explain how each pair of division and multiplication problems are the same, and how they are different.
Type below:
___________

Answer:
1/5 ÷ 3/4 = 4/15; 1/5 × 4/3 = 4/15
2/13 ÷ 1/5 = 10/13; 2/13 × 5/1 = 10/13
4/5 ÷ 3/5 = 4/3; 4/5 × 5/3 = 4/3
the product of the each pair of division and multiplication problems are the same.
They are different from the operation performed.

Question 16.
Part B
Explain how to use the pattern in the table to rewrite a division problem involving fractions as a multiplication problem.
Type below:
___________

Answer:
First, since it’s the division you have to change the second fraction which is called the reciprocal. That means the second fraction has to be flipped before you can multiple the fractions.

Page No. 136

Question 17.
Margie hiked a 17 \(\frac{7}{8}\) mile trail. She stopped every 3 \(\frac{2}{5}\) miles to take a picture. Martin and Tina estimated how many times Margie stopped.
Go Math Grade 6 Answer Key Chapter 2 Fractions and Decimals 29
Who made the better estimate? Use numbers and words to explain your answer.
Type below:
___________

Answer:
Margie hiked a 17 7/8 mile trail.
Distance hiked by Margie = 17 7/8 = 143/8 mile.
She stopped every 3 2/5 miles to take a picture = 17/5 mile
Number of pictures = (143/8) ÷ (17/5) = 715/136 = 5.28
So she can take a maximum of 6 pictures and a minimum of 5 pictures.
B is the correct answer.

Question 18.
Brad and Wes are building a tree house. They cut a 12 \(\frac{1}{2}\) foot piece of wood into 5 of the same length pieces. How long is each piece of wood? Show your work.
_______ \(\frac{□}{□}\) foot

Answer:
2 \(\frac{1}{2}\) foot

Explanation:
Brad and Wes cut a 12 1/2foot piece of wood into 5 of the same length.
Let the length of 1 piece be x
So, Length of 5 pieces = 5x
The total length of wood = 25/2
5x = 25/2
x = 5/2 = 2 1/2

Free Grade 6 HMH Go Math Answer Key PDF Download

In addition, to the exercise problems students of grade 6 can get the solutions for mid-chapter checkpoint and review test also. So, start solving the problems which are at the end of the chapter and check the solutions from here. Hope the information regarding Go Math 6th Grade Answer Key Chapter 2 Fractions and Decimals is helpful for you to overcome the issues in maths. Check out the links and start solving all the questions.

Go Math Grade 8 Answer Key Chapter 5 Writing Linear Equations

go-math-grade-8-chapter-5-writing-linear-equations-answer-key

It is essential for the students to have the best material to study for the exams. Go Math Grade 8 Answer Key Chapter 5 Writing Linear Equations recommend to every student who wants to succeed in math. Fall in love with math by practicing the problems from Go Math 8th Grade Chapter 5 Writing Linear Equations Solution Key. We have provided the step by step explanation for each and every question in CCSS Math Answers. Quick & smart learning is possible with the Go Math Grade 8 Answer Key Chapter 5 Writing Linear EquationsPDF.

Go Math Grade 8 Chapter 5 Writing Linear Equations Answer Key

The students who are willing to become masters in maths are suggested to follow our Go Math 8th Grade Chapter 5 Writing Linear Equations Solution Key. You can get free access for Go Math Grade 8 Answer Key Chapter 5 Writing Linear equations topicwise here. Check out the topics of Writing Linear equations from the below section.

Lesson 1: Writing Linear Equations from Situations and Graphs 

Lesson 2: Writing Linear Equations from a Table

Lesson 3: Linear Relationships and Bivariate Data

Model Quiz

Mixed Review

Guided Practice – Writing Linear Equations from Situations and Graphs – Page No. 130

Question 1.
Li is making beaded necklaces. For each necklace, she uses 27 spacers, plus 5 beads per inch of necklace length. Write an equation to find how many beads Li needs for each necklace.
a. input variable:
____________

Answer:
The length of the necklace in inches

Question 1.
b. output variable:
____________

Answer:
The total number of beads in the necklace

Question 1.
c. equation:
Type below:
____________

Answer:
y = 5x

Question 2.
Kate is planning a trip to the beach. She estimates her average speed to graph her expected progress on the trip. Write an equation in slope-intercept form that represents the situation.
Go Math Grade 8 Answer Key Chapter 5 Writing Linear Equations Lesson 1: Writing Linear Equations from Situations and Graphs img 1
Type below:
____________

Answer:
y = -60x + 300

Explanation:
Choose two points on the graph to find the slope.
Find the slope
m = (y2 -y1)/(x2 – x1)
m = -300/5 = -60
Read the y-intercept from the graph: b = 300
Use your slope and y-intercept values to write an equation in slope-intercept
form.
y = -60x + 300

Question 3.
At 59 °F, crickets chirp at a rate of 76 times per minute, and at 65 °F, they chirp 100 times per minute. Write an equation in slope-intercept form that represents the situation.
Type below:
____________

Answer:
y = 4x – 160

Explanation:
Input variable: Temperature
Output variable: Number of chirps per minute
Slope:
m = (y2 -y1)/(x2 – x1) = (100 – 76)/(65 – 59) = 24/6 = 4
100 = 4(65) + b
y-intercept:
b = -160
Substituting the value of the slope m and y-intercept in the slope-intercept form. y = mx + b where, m = 4 and b = -160.
y = 4x – 160

Essential Question Check-In

Question 4.
Explain what m and b in the equation y = mx + b tell you about the graph of the line with that equation.
Type below:
____________

Answer:
The equation of any straight line, called a linear equation, can be written as: y = mx + b, where m is the slope of the line and b is the y-intercept.
m = change in y-value/change in x-value
The y-intercept of this line is the value of y at the point where the line crosses the y-axis.

Independent Practice – Writing Linear Equations from Situations and Graphs – Page No. 131

Question 5.
A dragonfly can beat its wings 30 times per second. Write an equation in slope-intercept form that shows the relationship between flying time in seconds and the number of times the dragonfly beats its wings
Type below:
____________

Answer:
y = 30x

Explanation:
The linear equation is y = mx + b where m is the slope and b is the y-intercept.
y variable will be the number of times the dragonfly has beat its wings and the x variable will be the time.
A dragonfly can beat its wings 30 times per second.
To find b, let’s consider how many times the dragonfly has beat its wings at time 0s.
So, the equation of the line is y = 30x

Question 6.
A balloon is released from the top of a platform that is 50 meters tall. The balloon rises at the rate of 4 meters per second. Write an equation in slope-intercept form that tells the height of the balloon above the ground after a given number of seconds.
Type below:
____________

Answer:
y = 4x + 50

Explanation:
Input variable: Number of seconds
Output variable: Height of the balloon
The balloon rises at a rate of 4 meters per second. m = 4;
A balloon is released from the top of a platform that is 50 meters tall. b = 50.
y = 4x + 50

The graph shows a scuba diver’s ascent over time.
Go Math Grade 8 Answer Key Chapter 5 Writing Linear Equations Lesson 1: Writing Linear Equations from Situations and Graphs img 2

Question 7.
Use the graph to find the slope of the line. Tell what the slope means in this context.
Slope = _______ meters per second

Answer:
Slope = 1/8 or 0.125 meters per second

Explanation:
10m/80sec = 1/8
The scuba diver’s ascent gives up 1 meter per 16 seconds.

Question 8.
Identify the y-intercept. Tell what the y-intercept means in this context.
y-intercept = _______ meters

Answer:
y-intercept = -10 meters

Explanation:
The scuba divers start 10 meters below the water surface

Question 9.
Write an equation in slope-intercept form that represents the diver’s depth over time.
Type below:
____________

Answer:
y = 1/8 x – 10

Question 10.
The formula for converting Celsius temperatures to Fahrenheit temperatures is a linear equation. Water freezes at 0°C, or 32°F, and it boils at 100°C, or 212°F. Find the slope and y-intercept for a graph that gives degrees Celsius on the horizontal axis and degrees Fahrenheit on the vertical axis. Then write an equation in slope-intercept form that converts degrees Celsius into degrees Fahrenheit.
Type below:
____________

Answer:
Input variable: Celsius
Output variable: Fahrenheit
(0, 32) (100, 212)
m = (y2 -y1)/(x2 – x1) = (212 – 32)/(100 – 0) = 180/100 = 1.8
y intercepts = 32 when x = 0
F = 1.8C + 32

Question 11.
The cost of renting a sailboat at a lake is $20 per hour plus $12 for lifejackets. Write an equation in slope-intercept form that can be used to calculate the total amount you would pay for using this sailboat.
Type below:
____________

Answer:
y = 20x + 12

Explanation:
Input variable: Number of hours sailboat is rented
Output variable: Total cost
The cost of renting a sailboat at a lake is $20 per hour plus $12 for lifejackets.
Slope m = 20; y-intercept b = 12
Substituting the value of the slope m and y-intercept in the slope-intercept form. y = mx + b where, m = 20 and b = 12.
y = 20x + 12

Writing Linear Equations from Situations and Graphs – Page No. 132

The graph shows the activity in a savings account.
Go Math Grade 8 Answer Key Chapter 5 Writing Linear Equations Lesson 1: Writing Linear Equations from Situations and Graphs img 3

Question 12.
What was the amount of the initial deposit that started this savings account?
$ _______

Answer:
$1000

Explanation:
The amount of the initial deposit that started this savings account is $1000

Question 13.
Find the slope and y-intercept of the graphed line.
Type below:
____________

Answer:
slope = 500
y-intercept = 1000

Explanation:
slope m = (y2 -y1)/(x2 – x1) = (2000 – 1500)/(2 – 1) = 500/1 = 500
y-intercept = 1000

Question 14.
Write an equation in slope-intercept form for the activity in this savings account.
Type below:
____________

Answer:
y = 500x + 1000

Explanation:
Substituting the value of the slope m and y-intercept in the slope-intercept form. y = mx + b where, m = 500 and b = 1000.
y = 500x + 1000

Question 15.
Explain the meaning of the slope in this graph.
Type below:
____________

Answer:
The slope represents the amount of money saved in dollars per month in the plan.

H.O.T.

Focus on Higher Order Thinking

Question 16.
Communicate Mathematical Ideas
Explain how you decide which part of a problem will be represented by the variable x, and which part will be represented by the variable y in a graph of the situation.
Type below:
____________

Answer:
y=mx+b
M-slope
B-Y intercept
and
(x,y)- would be any point on the graph and if you were to plug those points in for x and y the equation would solve if done properly

Question 17.
Represent Real-World Problems
Describe what would be true about the rate of change in a situation that could not be represented by a graphed line and an equation in the form y = mx + b.
Type below:
____________

Answer:
The rate of change would not be constant. Using different pairs of points in the slope formula would give different results.

Question 18.
Draw Conclusions
Must m, in the equation y = mx + b, always be a positive number? Explain.
Type below:
____________

Answer:
No; The slope m can be negative or positive. If the slope of the number is positive (the graph goes upward from left to right), then m will be positive, but if the slope is negative (the graph goes down from left to right), then m is negative.

Guided Practice – Writing Linear Equations from a Table – Page No. 136

Question 1.
Jaime purchased a $20 bus pass. Each time he rides the bus, a certain amount is deducted from the pass. The table shows the amount, y, left on his pass after x rides. Graph the data, and find the slope and y-intercept from the graph or from the table. Then write the equation for the graph in slope-intercept form.
Go Math Grade 8 Answer Key Chapter 5 Writing Linear Equations Lesson 2: Writing Linear Equations from a Table img 4
Type below:
____________

Answer:
Grade 8 Chapter 5 image 1
y = -5/4 x + 20

Explanation:
Slope = -20/16 = -5/4 = -1.25
y- intercepts = 20
Substituting the value of the slope m and y-intercept in the slope-intercept form. y = mx + b where, m = -1.25 and b = 20.
y = -5/4 x + 20

The table shows the temperature (y) at different altitudes (x). This is a linear relationship.
Go Math Grade 8 Answer Key Chapter 5 Writing Linear Equations Lesson 2: Writing Linear Equations from a Table img 5

Question 2.
Find the slope for this relationship.
_______

Answer:
Slope m = (y2 -y1)/(x2 – x1) = (51 – 59)/(2000 – 0) = -8/2000 = -0.004

Explanation:
(x1, y1) = (0, 59), (x2, y2) = (2000, 51)
Slope m = (y2 -y1)/(x2 – x1) = (51 – 59)/(2000 – 0) = -8/2000 = -0.004

Question 3.
Find the y-intercept for this relationship.
y-intercept = _______

Answer:
b = 50

Explanation:
y-intercept = 59 when x = 0

Question 4.
Write an equation in slope-intercept form that represents this relationship.
Type below:
____________

Answer:
y = -0.004x + 59

Explanation:
Substituting the value of the slope m and y-intercept in the slope-intercept form. y = mx + b where, m = -0.004 and b = 59.

Question 5.
Use your equation to determine the temperature at an altitude of 5000 feet.
_______ °F

Answer:
39°F

Explanation:
y = -0.004x + 59
y = -0.004(5000) + 59 = -20 + 59 = 39
The temperature is 39°F at the altitude of 5000 feet.

Essential Question Check-In

Question 6.
Describe how you can use the information in a table showing a linear relationship to find the slope and y-intercept for the equation.
Type below:
____________

Answer:
Use any two points from the table to fins teh slope.
Slope m = (y2 -y1)/(x2 – x1)
If the value of y-intercept, when x=0 is not given in the table, use the slope and any ordered pair from the table and substitute in slope-intercept from y=mx + b to find b.

Independent Practice – Writing Linear Equations from a Table – Page No. 137

Question 7.
The table shows the costs of a large cheese pizza with toppings at a local pizzeria. Graph the data, and find the slope and y-intercept from the graph. Then write the equation for the graph in slope-intercept form.
Go Math Grade 8 Answer Key Chapter 5 Writing Linear Equations Lesson 2: Writing Linear Equations from a Table img 6
Type below:
____________

Answer:
Grade 8 Chapter 5 image 2

Slope m = (y2 -y1)/(x2 – x1) = (10 – 8)/(1 – 0) = 2/1 = 2
y-intercept b = 8
Substituting the value of the slope m and y-intercept in the slope-intercept form. y = mx + b where, m = 2 and b = 8.
y = 2x + 8

Question 8.
The table shows how much an air-conditioning repair company charges for different numbers of hours of work. Graph the data, and find the slope and y-intercept from the graph. Then write the equation for the graph in slope-intercept form.
Go Math Grade 8 Answer Key Chapter 5 Writing Linear Equations Lesson 2: Writing Linear Equations from a Table img 7
Type below:
____________

Answer:
Grade 8 Chapter 5 image 3

Slope m = (y2 -y1)/(x2 – x1) = (100 – 50)/(1 – 0) = 50/1 = 50
y-intercept b = 50
Substituting the value of the slope m and y-intercept in the slope-intercept form. y = mx + b where, m = 50 and b = 50.
y = 50x + 50

Question 9.
A friend gave Ms. Morris a gift card for a local car wash. The table shows the linear relationship of how the value left on the card relates to the number of car washes.
a. Write an equation that shows the number of dollars left on the card.
Go Math Grade 8 Answer Key Chapter 5 Writing Linear Equations Lesson 2: Writing Linear Equations from a Table img 8
Type below:
____________

Answer:
y = -1.50x + 30

Explanation:
Slope m = (y2 -y1)/(x2 – x1) = (18 – 30)/(8 – 0) = -12/8 = -1.5
y-intercept b = 30
Substituting the value of the slope m and y-intercept in the slope-intercept form. y = mx + b where, m = -1.50 and b = 30.
y = -1.50x + 30

Question 9.
b. Explain the meaning of the negative slope in this situation.
Type below:
____________

Answer:
The negative slope means that the amount on the card decreased by $1.5 per car wash

Question 9.
c. What is the maximum value of x that makes sense in this context? Explain.
x = _______

Answer:
x = 20

Explanation:
0 = -1.50x + 30
1.5x = 30
x = 30/1.5 = 20
the maximum value of x = 20

The tables show linear relationships between x and y. Write an equation in slope-intercept form for each relationship.

Question 10.
Go Math Grade 8 Answer Key Chapter 5 Writing Linear Equations Lesson 2: Writing Linear Equations from a Table img 9
Type below:
____________

Answer:
Slope m = (y2 -y1)/(x2 – x1) = (3 – 1)/(2 – 0) =  2/2 = 1
y-intercept b = 1
Substituting the value of the slope m and y-intercept in the slope-intercept form. y = mx + b where, m = 1 and b = 1.
y =  x + 1

Question 11.
Go Math Grade 8 Answer Key Chapter 5 Writing Linear Equations Lesson 2: Writing Linear Equations from a Table img 10
Type below:
____________

Answer:
Slope m = (y2 -y1)/(x2 – x1) = (6 – 4)/(0 – 1) =  -2/1 = -2
y-intercept b = 6
Substituting the value of the slope m and y-intercept in the slope-intercept form. y = mx + b where, m = -2 and b = 6.
y = -2x + 6

Writing Linear Equations from a Table – Page No. 138

Question 12.
Finance
Desiree starts a savings account with $125.00. Every month, she deposits $53.50.
a. Complete the table to model the situation.
Go Math Grade 8 Answer Key Chapter 5 Writing Linear Equations Lesson 2: Writing Linear Equations from a Table img 11
Type below:
____________

Answer:
Grade 8 Chapter 5 image 4

Question 12.
b. Write an equation in slope-intercept form that shows how much money Desiree has in her savings account after x months.
Type below:
____________

Answer:
y = 53.5x + 125

Explanation:
Desiree starts a savings account with $125.00. Every month, she deposits $53.50.
Substituting the value of the slope m and y-intercept in the slope-intercept form. y = mx + b where, m = 53.5 and b = 125.
y = 53.5x + 125

Question 12.
c. Use the equation to find how much money Desiree will have in savings after 11 months.
$ ________

Answer:
$713.5

Explanation:
y = 53.5x + 125
The value of x is 11
y = 53.5 (11) + 125 = 588.5 + 125 = 713.5
Desiree will have $713.5 in savings after 11 months.

Question 13.
Monty documented the amount of rain his farm received on a monthly basis, as shown in the table.
Go Math Grade 8 Answer Key Chapter 5 Writing Linear Equations Lesson 2: Writing Linear Equations from a Table img 12
a. Is the relationship linear? Why or why not?
Type below:
____________

Answer:
No

Explanation:
The change in the months is constant but the change in rainfall is not constant.

Question 13.
b. Can an equation be written to describe the amount of rain? Explain.
Type below:
____________

Answer:
No

Explanation:
There is no apparent pattern in the given data.

H.O.T.

Focus on Higher Order Thinking

Question 14.
Analyze Relationships
If you have a table that shows a linear relationship, when can you read the value for b, in y = mx + b, directly from the table without drawing a graph or doing any calculations? Explain.
Type below:
____________

Answer:
You can read the value of b directly from the table whrn the table contains the input value of 0 and its corresponding output values (value of y when x = 0)

Question 15.
What If?
Jaíme graphed linear data given in the form (cost, number). The y-intercept was 0. Jayla graphed the same data given in the form (number, cost). What was the y-intercept of her graph? Explain.
Type below:
____________

Answer:
Jaíme graphed linear data given in the form (cost, number). The y-intercept was 0. Jayla graphed the same data given in the form (number, cost).
Jaíme’s graph contained (0, 0). Since Jayal’s data were the same y-intercept is 0 but x and y are switched.

Guided Practice – Linear Relationships and Bivariate Data – Page No. 144

Use the following graphs to find the equation of the linear relationship.

Question 1.
Go Math Grade 8 Answer Key Chapter 5 Writing Linear Equations Lesson 3: Linear Relationships and Bivariate Data img 13
Type below:
____________

Answer:
y = 30x

Explanation:
Grade 8 Chapter 5 image 5
Find the slope using two points from the grapgh by
Slope m = (y2 -y1)/(x2 – x1) where (x1, y1) = (1, 30) and (x2, y2) = (2, 60)
Slope m = (y2 -y1)/(x2 – x1) = (60 – 30)/(2 – 1) = 30/1 = 30
y-intercept b = 0
Substituting the value of the slope m and y-intercept in the slope-intercept form. y = mx + b where, m = 30 and b = 0.
y = 30x

Question 2.
Go Math Grade 8 Answer Key Chapter 5 Writing Linear Equations Lesson 3: Linear Relationships and Bivariate Data img 14
Type below:
____________

Answer:
y = 2.5x + 2

Explanation:
Grade 8 Chapter 5 image 6
Find the slope using two points from the grapgh by
Slope m = (y2 -y1)/(x2 – x1) where (x1, y1) = (2, 7) and (x2, y2) = (4, 12)
Slope m = (y2 -y1)/(x2 – x1) = (12 – 7)/(4 – 2) = 5/2 = 2.5
y-intercept b = 2
Substituting the value of the slope m and y-intercept in the slope-intercept form. y = mx + b where, m = 2.5 and b = 2.
y = 2.5x + 2

Question 3.
The graph shows the relationship between the number of hours a kayak is rented and the total cost of the rental. Write an equation of the relationship. Then use the equation to predict the cost of a rental that lasts 5.5 hours.
Go Math Grade 8 Answer Key Chapter 5 Writing Linear Equations Lesson 3: Linear Relationships and Bivariate Data img 15
Type below:
____________

Answer:
y = 20x + 30
The cost of a rental that lasts 5.5 hours is $140

Explanation:
Find the slope using two points from the grapgh by
Slope m = (y2 -y1)/(x2 – x1) where (x1, y1) = (2, 70) and (x2, y2) = (4, 110)
Slope m = (y2 -y1)/(x2 – x1) = (110 – 70)/(4 – 2) = 40/2 = 20
y-intercept b = 30
Substituting the value of the slope m and y-intercept in the slope-intercept form. y = mx + b where, m = 20 and b = 30.
y = 20x + 30
Substitue the value of x is 5.5 hrs
y = 20 (5.5) + 30
y = 110 + 30 = 140
The cost of a rental that lasts 5.5 hours is $140

Does each of the following graphs represent a linear relationship? Why or why not?

Question 4.
Go Math Grade 8 Answer Key Chapter 5 Writing Linear Equations Lesson 3: Linear Relationships and Bivariate Data img 16
____________

Answer:
Yes; The graph has a constant rate of change

Explanation:
Find the slope using two points from the grapgh by
Slope m = (y2 -y1)/(x2 – x1) where (x1, y1) = (2, 6) and (x2, y2) = (5, 15)
Slope m = (y2 -y1)/(x2 – x1) = (15 – 6)/(5 – 2) = 9/3 = 3
y-intercept b = 6
Substituting the value of the slope m and y-intercept in the slope-intercept form. y = mx + b where, m = 3 and b = 6.
y = 3x + 6
The values changing according to the constant change in the x values.

Question 5.
Go Math Grade 8 Answer Key Chapter 5 Writing Linear Equations Lesson 3: Linear Relationships and Bivariate Data img 17
____________

Answer:
No; The graph does not have a constant rate of change

Essential Question Check-In

Question 6.
How can you tell if a set of bivariate data shows a linear relationship?
Type below:
____________

Answer:
It is a linear relationship if the rate of change is constant or the graph is a straight line.

Independent Practice – Linear Relationships and Bivariate Data – Page No. 145

Does each of the following tables represent a linear relationship? Why or why not?

Question 7.
Go Math Grade 8 Answer Key Chapter 5 Writing Linear Equations Lesson 3: Linear Relationships and Bivariate Data img 18
____________

Answer:
Linear Relationship

Explanation:
Find Rate of Change using Slope m = (y2 -y1)/(x2 – x1)
Rate of Change = (45 – 15)/(9 – 3) = 30/6 = 5
Rate of Change = (105 – 45)/(21 – 9) = 60/12 = 5
It is a Linear Relationship as the rate of the change is constant.

Question 8.
Go Math Grade 8 Answer Key Chapter 5 Writing Linear Equations Lesson 3: Linear Relationships and Bivariate Data img 19
____________

Answer:
not a Linear Relationship

Explanation:
Find Rate of Change using Slope m = (y2 -y1)/(x2 – x1)
Rate of Change = (76.8 – 30)/(8 – 5) = 46./3 = 15.6
Rate of Change = (235.2 – 76.8)/(14 – 8) = 158.4/6 = 26.4
It is not a Linear Relationship as the rate of the change is varies.

Explain whether or not you think each relationship is linear.

Question 9.
the cost of equal-priced DVDs and the number purchased
____________

Answer:
y = cx

Explanation:
The relation between the cost of equal-priced DVDs and the number purchased is linear because the change rate is constant. If we denote with c the cost of a DVD, with x the number of purchased DVDs and with y the total cost of the purchased DVDs, we get an equation of the form:
y = cx

Question 10.
the height of a person and the person’s age
____________

Answer:
Non – Linear relationship

Explanation:
The height of a person does not increase at a constant rate with a person’s age

Question 11.
the area of a square quilt and its side length
____________

Answer:
Non – Linear relationship

Explanation:
The rate of change in the area of a square quilt increase as the side length increases.

Question 12.
the number of miles to the next service station and the number of kilometers
____________

Answer:
Linear relationship

Explanation:
The number of miles increases at a constant rate with the number of kilometers.

Question 13.
Multistep
The Mars Rover travels 0.75 feet in 6 seconds. Add the point to the graph. Then determine whether the relationship between distance and time is linear, and if so, predict the distance that the Mars Rover would travel in 1 minute.
Go Math Grade 8 Answer Key Chapter 5 Writing Linear Equations Lesson 3: Linear Relationships and Bivariate Data img 20
Distance traveled in 1 min.: _______ ft.

Answer:
Relationship is linear
Distance traveled in 1 min.: 7.5 ft.

Explanation:
Grade 8 Chapter 5 image 7
The graph is a straight line. So, the Relationship is linear
The equation representing the relationship between distance and time.
y = 0.125x
The value of x is 1 min = 60 sec
y = 0.125 (60) = 7.5 ft
Distance traveled in 1 min.: 7.5 ft.

Linear Relationships and Bivariate Data – Page No. 146

Question 14.
Make a Conjecture
Zefram analyzed a linear relationship, found that the slope-intercept equation was y=3.5x+16, and made a prediction for the value of y for a given value of x. He realized that he made an error calculating the y-intercept and that it was actually 12. Can he just subtract 4 from his prediction if he knows that the slope is correct? Explain.
____________

Answer:
Yes

Explanation:
The value of y is calculated using y = 3.5x+ 16. Since the slope of the point remains the same, 4 can be subtracted from the predicted answer as the value of y would be: y = 3.5x+ 16 – 4

H.O.T.

Focus on Higher Order Thinking

Question 15.
Communicate Mathematical Ideas
The table shows a linear relationship. How can you predict the value of y when x = 6 without finding the equation of the relationship?
Go Math Grade 8 Answer Key Chapter 5 Writing Linear Equations Lesson 3: Linear Relationships and Bivariate Data img 21
Type below:
____________

Answer:
57

Explanation:
Find Rate of Change using Slope m = (y2 -y1)/(x2 – x1)
Rate of Change = (76 – 38)/(8 – 4) = 38/4 = 9.5
Since the difference between 8 and S is 2, subtract twice the value of the rate of change from 76
y = 76 – (9.5 × 2) =57

Question 16.
Critique Reasoning
Louis says that if the differences between the values of x are constant between all the points on a graph, then the relationship is linear. Do you agree? Explain.
____________

Answer:
No

Explanation:
The difference between y should be constant for a constant interval of x.

Question 17.
Make a Conjecture
Suppose you know the slope of a linear relationship and one of the points that its graph passes through. How could you predict another point that falls on the graph of the line?
Type below:
____________

Answer:
Find the equation of the linear relationship using the slope and given point. The insert any x-value to find a y value on the graph of the line.

Question 18.
Explain the Error
Thomas used (7, 17.5) and (18, 45) from a graph to find the equation of a linear relationship as shown. What was his mistake?
m = \(\frac{45-7}{18-17.5}=\frac{38}{0.5}\) = 79
y = 79x + b =
49 = 79 × 18 + b
45 = 1422 + b, so b = −1337
The equation is y = 79x − 1377
Type below:
____________

Answer:
He subtracted the x value of the first point from the y of the second point and the y value of the second point from the x value of the first point.
Hence, the slope is incorrect and the equation is incorrect as well. The correct slope is
(45 – 17.5)/(18 – 7) = 27.5/11 = 2.5

5.1 Writing Linear Equations from Situations and Graphs – Model Quiz – Page No. 147

Write the equation of each line in slope-intercept form.

Question 1.
Go Math Grade 8 Answer Key Chapter 5 Writing Linear Equations Model Quiz img 22
Type below:
____________

Answer:
y = 30x + 20

Explanation:
Find the slope using two points from the graph by m = (y2 -y1)/(x2 – x1) where (x1, y1) = (0, 20), (x2, y2) = (2, 80)
Substitute the value of m and (x1, y1) = (0, 20), (x2, y2) = (2, 80)
Slope m = (y2 -y1)/(x2 – x1) = (80 – 20)/(2 – 0) = 60/2 = 30
Substituting the value of slope (m) and (x, y) in the slope intercept form to find y intercept (b):
80 = 30(2) + b
y-intercept b = 20
Substituting the value of the slope m and y-intercept in the slope-intercept form. y = mx + b where, m = 30 and b = 20.
y = 30x + 20

Question 2.
Go Math Grade 8 Answer Key Chapter 5 Writing Linear Equations Model Quiz img 23
Type below:
____________

Answer:
y = -10x + 60

Explanation:
Find the slope using two points from the graph by m = (y2 -y1)/(x2 – x1) where (x1, y1) = (6, 0), (x2, y2) = (0, 60)
Substitute the value of m and (x1, y1) = (6, 0), (x2, y2) = (0, 60)
Slope m = (y2 -y1)/(x2 – x1) = (60 – 0)/(0 – 6) = -60/6 = -10
Substituting the value of slope (m) and (x, y) in the slope intercept form to find y intercept (b):
60 = -10(0) + b
y-intercept b = 60
Substituting the value of the slope m and y-intercept in the slope-intercept form. y = mx + b where, m = -10 and b = 60.
y = -10x + 60

5.2 Writing Linear Equations from a Table

Write the equation of each linear relationship in slope-intercept form.

Question 3.
Go Math Grade 8 Answer Key Chapter 5 Writing Linear Equations Model Quiz img 24
Type below:
____________

Answer:
y = 0.35x + 1.5

Explanation:
Find the slope using two points from the graph by m = (y2 -y1)/(x2 – x1) where (x1, y1) = (0, 1.5), (x2, y2) = (100, 36.5)
Substitute the value of m and (x1, y1) = (0, 1.5), (x2, y2) = (100, 36.5)
Slope m = (y2 -y1)/(x2 – x1) = (36.5 – 1.5)/(100 – 0) = 35/100 = 0.35
Substituting the value of slope (m) and (x, y) in the slope intercept form to find y intercept (b):
1.5 =0.35(0) + b
y-intercept b = 1.5
Substituting the value of the slope m and y-intercept in the slope-intercept form. y = mx + b where, m = 0.35 and b = 1.5.
y = 0.35x + 1.5

Question 4.
Go Math Grade 8 Answer Key Chapter 5 Writing Linear Equations Model Quiz img 25
Type below:
____________

Answer:
y = -0.6x + 109

Explanation:
Find the slope using two points from the graph by m = (y2 -y1)/(x2 – x1) where (x1, y1) = (25, 94), (x2, y2) = (35, 88)
Substitute the value of m and (x1, y1) = (25, 94), (x2, y2) = (35, 88)
Slope m = (y2 -y1)/(x2 – x1) = (88 – 94)/(35 – 25) = -6/10 = -0.6
Substituting the value of slope (m) and (x, y) in the slope intercept form to find y intercept (b):
88 = -0.6(35) + b
y-intercept b = 109
Substituting the value of the slope m and y-intercept in the slope-intercept form. y = mx + b where, m = -0.6 and b = 109.
y = -0.6x + 109

5.3 Linear Relationships and Bivariate Data

Write the equation of the line that connects each set of data points.

Question 5.
Go Math Grade 8 Answer Key Chapter 5 Writing Linear Equations Model Quiz img 26
Type below:
____________

Answer:
y = 2/3x + 26 2/3

Explanation:
Grade 8 Chapter 5 image 8
Find the slope using two points from the graph by m = (y2 -y1)/(x2 – x1) where (x1, y1) = (20, 40), (x2, y2) = (50, 60)
Substitute the value of m and (x1, y1) = (20, 40), (x2, y2) = (50, 60)
Slope m = (y2 -y1)/(x2 – x1) = (60 – 40)/(50 – 20) = 20/30 = 2/3
Substituting the value of slope (m) and (x, y) in the slope intercept form to find y intercept (b):
40 = 2/3(20) + b
y-intercept b = 26 2/3
Substituting the value of the slope m and y-intercept in the slope-intercept form. y = mx + b where, m = 2/3 and b = 26 2/3.
y = 2/3x + 26 2/3

Question 6.
Go Math Grade 8 Answer Key Chapter 5 Writing Linear Equations Model Quiz img 27
Type below:
____________

Answer:
y = -3x + 140

Explanation:
Grade 8 Chapter 5 image 9
Find the slope using two points from the graph by m = (y2 -y1)/(x2 – x1) where (x1, y1) = (25, 65), (x2, y2) = (30, 50)
Substitute the value of m and (x1, y1) = (25, 65), (x2, y2) = (30, 50)
Slope m = (y2 -y1)/(x2 – x1) = (50 – 65)/(30 – 25) = -15/5 = -3
Substituting the value of slope (m) and (x, y) in the slope intercept form to find y intercept (b):
50 = -3(30) + b
y-intercept b = 140
Substituting the value of the slope m and y-intercept in the slope-intercept form. y = mx + b where, m = -3 and b = 140.
y = -3x + 140

Essential Question

Question 7.
Write a real-world situation that can be represented by a linear relationship.
Type below:
____________

Answer:
There is an hourly fee of $15 for renting the ski gears per hour and an entry fee of $10 for the ski lodge.

Selected Response – Mixed Review – Page No. 148

Question 1.
An hourglass is turned over with the top part filled with sand. After 3 minutes, there are 855 mL of sand in the top half. After 10 minutes, there are 750 mL of sand in the top half. Which equation represents this situation?
Options:
a. y = 285x
b. y = −10.5x + 900
c. y = −15x + 900
d. y = 75x

Answer:
c. y = −15x + 900

Explanation:
Identify the input and output variable
Input: Number of minutes
Output: Quantity of sand in the hourglass
Write the given information as ordered pair (3, 855), (10, 750)
Find the slope using two points from the graph by m = (y2 -y1)/(x2 – x1) where (x1, y1) = (10, 750), (x2, y2) =(3, 855)
Substitute the value of m and (x1, y1) = (10, 750), (x2, y2) =(3, 855)
Slope m = (y2 -y1)/(x2 – x1) = (855 – 750)/(3 – 10) = -105/7 = -15
Substituting the value of slope (m) and (x, y) in the slope intercept form to find y intercept (b):
750 = -15(10) + b
y-intercept b = 900
Substituting the value of the slope m and y-intercept in the slope-intercept form. y = mx + b where, m = -15 and b = 900.
y = -15x + 900

Question 2.
Which graph shows a linear relationship?
Go Math Grade 8 Answer Key Chapter 5 Writing Linear Equations Mixed Review img 28
Options:
a. A
b. B
c. C
d. D

Answer:
b. B

Explanation:
Graph B represents the linear relationship
The data appears to lie on a straight line

Question 3.
What are the slope and y-intercept of the relationship shown in the table?
Go Math Grade 8 Answer Key Chapter 5 Writing Linear Equations Mixed Review img 29
Options:
a. slope = 0.05, y-intercept = 1,500
b. slope = 0.5, y-intercept = 1,500
c. slope = 0.05, y-intercept = 2,000
d. slope = 0.5, y-intercept = 2,000

Answer:
c. slope = 0.05, y-intercept = 2,000

Explanation:
Find the slope using two points from the graph by m = (y2 -y1)/(x2 – x1) where (x1, y1) = (10,000, 2,500), (x2, y2) =(20,000, 3,000)
Substitute the value of m and (x1, y1) = (10,000, 2,500), (x2, y2) =(20,000, 3,000)
Slope m = (y2 -y1)/(x2 – x1) = (3,000 – 2,500)/(20,000 – 10,000) = 500/10,000 = 0.05
Substituting the value of slope (m) and (x, y) in the slope intercept form to find y intercept (b):
3,000 = 0.05(20,000) + b
y-intercept b = 2,000

Question 4.
Which is the sum of 3.15 × 107 + 9.3 × 106? Write your answer in scientific notation.
Options:
a. 4.08 × 107
b. 4.08 × 106
c. 0.408 × 108
d. 40.8 × 106

Answer:
a. 4.08 × 107

Explanation:
Given 3.15 × 107 + 9.3 × 106?
(3.15 + 0.93) × 107
4.08 × 107

Mini-Task

Question 5.
Franklin’s faucet was leaking, so he put a bucket underneath to catch the water. After a while, Franklin started keeping track of how much water was in the bucket. His data is in the table below.
Go Math Grade 8 Answer Key Chapter 5 Writing Linear Equations Mixed Review img 30
a. Is the relationship linear or nonlinear?
_____________

Answer:
The relationship linear

Explanation:
Find the rate of change by Difference in Quarts/Hours
(6.5 – 5)/(3 – 2) = 1.5
(8 – 6.5)/(4 – 3) = 1.5
(9.5 – 8)/(5 – 4) = 1.5
Since the rate of change is constant, the relationship is linear.

Question 5.
b. Write the equation for the relationship.
Type below:
_____________

Answer:
Rate of change is the slope of the linear equation
m = 1.5

Question 5.
c. Predict how much water will be in the bucket after 14 hours if Franklin doesn’t stop the leak.
_______ quarts

Answer:
23 quarts

Explanation:
Substituting the value of slope (m) and (x, y) in the slope intercept form to find y-intercept (b)
y = mx + b
5 = 1.5(2) + b
y-intercept b = 2
Substituting the value of the slope m and y-intercept in the slope-intercept form. y = mx + b where, m = 1.5 and b = 2.
y = 1.5x + 2
The x value is 2
y = 1.5(2) + 2 = 23
There will be 23 quarts after 14 hrs.

Conclusion:

Every expert’s choice is utilizing Go math Grade 8 Chapter 5 Writing Linear Equations Answer Key PDF. Practice the question from our Go Math 8th Grade Chapter 5 Writing Linear Equations Answers to score the highest marks in the exams. Compare the questions with the real-time problems and solve the problems with the help of Go Math Grade 8 Chapter 5 Answer Key Chapter 5 Writing Linear Equations. Stay tuned to our ccssmathanswers.com to get the all Go Math Grade 8 Answer Key Chapterwise pdfs.

Go Math Grade 8 Answer Key Chapter 3 Proportional Relationships

go-math-grade-8-chapter-3-proportional-relationships-answer-key

Who doesn’t love the best learning? HMH Grade 8 Go Math Answer Key Chapter 3 Proportional Relationships is the perfect guide for students who love maths. Pupils can habituate practice maths if they start their practice with Go Math 8th Grade Chapter 3 Proportional Relationships Solution Key. The Go Math Grade 8 Answer Key includes an explanation for all the questions where students can easily understand the methods in an easy manner.

Go Math Grade 8 Chapter 3 Proportional Relationships Answer Key

Go Math Grade 8 Answer Key Chapter 3 Proportional Relationships offers the best way to practice math with simple techniques. The students can download and practice the questions from Go Math Grade 8 Text Book for free. The HMH Go Math Grade 8 Answer Key is provided will help the students to score the highest marks in the exams.

Lesson 1: Representing Proportional Relationships

Lesson 2: Rate of Change and Slope

Lesson 3: Interpreting the Unit As Slope

Lesson 4: Representing Proportional Relationships – Model Quiz

Mixed Review

Guided Practice – Representing Proportional Relationships – Page No. 74

Question 1.
Vocabulary
A proportional relationship is a relationship between two quantities in which the ratio of one quantity to the other quantity is/is not constant.
______ constant

Answer:
is constant

Explanation:
The rent would be proportional so therefore it is.

Question 2.
Vocabulary
When writing an equation of a proportional relationship in the form y = kx, k represents the __________________________.
______________

Answer:
constant of proportionality

Explanation:
When writing an equation of a proportional relationship in the form y = kx, k represents the constant of proportionality.

Question 3.
Write an equation that describes the proportional relationship between the number of days and the number of weeks in a given length of time.
a. Complete the table.
Go Math Grade 8 Answer Key Chapter 3 Proportional Relationships Lesson 1: Representing Proportional Relationships img 1
Type below:
______________

Answer:
Grade 8 Chapter 3 image 1

Question 3.
b. Let x represent _____.
Let y represent _____.
The equation that describes the relationship is _____.
Type below:
______________

Answer:
Let x represent the time in weeks.
Let y represent the time in days.
The equation that describes the relationship is y = 7x.

Each table or graph represents a proportional relationship. Write an equation that describes the relationship.

Question 4.
Physical Science
The relationship between the numbers of oxygen atoms and hydrogen atoms in water.
Go Math Grade 8 Answer Key Chapter 3 Proportional Relationships Lesson 1: Representing Proportional Relationships img 2
Type below:
______________

Answer:
y = 2x
Grade 8 Chapter 3 image 2

Explanation:
x represents the Oxygen atoms
y represents the Hydrogen atoms
For every point of the x-axis, the y-axis is varying with 2x times.
y = 2x

Question 5.
Go Math Grade 8 Answer Key Chapter 3 Proportional Relationships Lesson 1: Representing Proportional Relationships img 3
Type below:
______________

Answer:
y = 30x

Explanation:
x represents the Distance in inches
y represents the Actual Distance in miles
For every point of the x-axis, the y-axis is varying with 30x times.
y = 30x

Essential Question Check-In

Question 6.
If you know the equation of a proportional relationship, how can you draw the graph of the equation?
Type below:
______________

Answer:
Use the equation to make a table with x-values and y-values. Then graph the points (x, y) and draw a line through the points.

Independent Practice – Representing Proportional Relationships – Page No. 75

The table shows the relationship between temperatures measured on the Celsius and Fahrenheit scales.
Go Math Grade 8 Answer Key Chapter 3 Proportional Relationships Lesson 1: Representing Proportional Relationships img 4

Question 7.
Is the relationship between the temperature scales proportional? Why or why not?
______________

Answer:
No. The ratios of the numbers in each column are not equal.

Explanation:
Fahrenheit Temperature/Celsius Temperature = 50/10 = 5
86/30 = 2.87
122/50 = 2.44
The relationship is not propotional as the ratio is not constant.

Question 8.
Describe the graph of the Celsius-Fahrenheit relationship.
Type below:
______________

Answer:
A line starting at (0, 32) and slanting upward to the right.

Explanation:
The graph is a straight line with a y-intercept of 32.

Question 9.
Analyze Relationships
Ralph opened a savings account with a deposit of $100. Every month after that, he deposited $20 more.
a. Why is the relationship described not proportional?
Type below:
______________

Answer:
The account had a balance of $100, to begin with.

Question 9.
b. How could the situation be changed to make the situation proportional?
Type below:
______________

Answer:
Have Ralph open the account with no money, to begin with, and then put $20 every month.

Question 10.
Represent Real-World Problems
Describe a real-world situation that can be modeled by the equation y = \(\frac{1}{20}\)x. Be sure to describe what each variable represents.
Type below:
______________

Answer:
If x is the number of nickels you have, y = \(\frac{1}{20}\)x is the amount of money you have in dollars.

Look for a Pattern

The variables x and y are related proportionally.

Question 11.
When x = 8, y = 20. Find y when x = 42.
_______

Answer:
y = 105

Explanation:
x = 8, y = 20
y/x = 20/8
y = 20x/8
when x = 42
y = (20 × 42)/8
y = 105

Question 12.
When x = 12, y = 8. Find x when y = 12.
_______

Answer:
x = 18

Explanation:
x/y = 12/8
x = 12y/8
when y = 12
x = (12 × 12)/8
x = 18

Representing Proportional Relationships – Page No. 76

Question 13.
The graph shows the relationship between the distance that a snail crawls and the time that it crawls.
a. Use the points on the graph to make a table.
Go Math Grade 8 Answer Key Chapter 3 Proportional Relationships Lesson 1: Representing Proportional Relationships img 5
Go Math Grade 8 Answer Key Chapter 3 Proportional Relationships Lesson 1: Representing Proportional Relationships img 6
Type below:
______________

Answer:
Grade 8 Chapter 3 image 10

Question 13.
b. Write the equation for the relationship and tell what each variable represents.
Type below:
______________

Answer:
y = 1/10 x, where y is the time in minutes and x is the distance in inches

Question 13.
c. How long does it take the snail to crawl 85 inches?
_______ minutes

Answer:
8.5 minutes

H.O.T.

Focus on Higher Order Thinking

Question 14.
Communicate Mathematical Ideas
Explain why all of the graphs in this lesson show the first quadrant but omit the other three quadrants.
Type below:
______________

Answer:
All of the graphs represent real-world data for which both x and y take on only nonnegative values, which graph in the first quadrant or on the axes. If either x or y or both could be negative, then other quadrants would be needed.

Question 15.
Analyze Relationships
Complete the table.
Go Math Grade 8 Answer Key Chapter 3 Proportional Relationships Lesson 1: Representing Proportional Relationships img 7

Grade 8 Chapter 3 image 10
a. Are the length of a side of a square and the perimeter of the square related proportionally? Why or why not?
______________

Answer:
Yes. The ratio of the perimeter of a square to its side length is always 4.

Question 15.
b. Are the length of a side of a square and the area of the square related proportionally? Why or why not?
______________

Answer:
No. The ratio of the area of a square to its side length is not constant

Question 16.
Make a Conjecture
A table shows a proportional relationship where k is the constant of proportionality. The rows are then switched. How does the new constant of proportionality relate to the original one?
Type below:
______________

Answer:
It is the reciprocal of the original constant of proportionality

Guided Practice – Rate of Change and Slope – Page No. 80

Tell whether the rates of change are constant or variable.

Question 1.
building measurements _____
Go Math Grade 8 Answer Key Chapter 3 Proportional Relationships Lesson 2: Rate of Change and Slope img 8
______________

Answer:
constant

Explanation:
Input variable: Feet
Output variable: Yard
For every point of the Yard, the Feet is increasing 3 times.
So, the answer is constant.

Question 2.
computers sold _____
Go Math Grade 8 Answer Key Chapter 3 Proportional Relationships Lesson 2: Rate of Change and Slope img 9
______________

Answer:
variable

Explanation:
Identify the input and output variables
Input variable: Week
Output variable: The number sold
x-axis and y-axis points are not varying constantly. So, the answer is variable.

Question 3.
distance an object falls _____
Go Math Grade 8 Answer Key Chapter 3 Proportional Relationships Lesson 2: Rate of Change and Slope img 10
______________

Answer:
variable

Explanation:
Identify the input and output variables
Input variable: Time
Output variable: Distance
x-axis and y-axis points are not varying constantly. So, the answer is variable.

Question 4.
cost of sweaters _____
Go Math Grade 8 Answer Key Chapter 3 Proportional Relationships Lesson 2: Rate of Change and Slope img 11
______________

Answer:
constant

Explanation:
Identify the input and output variables
Input variable: Number
Output variable: Cost
x-axis and y-axis points are varying constantly. So, the answer is constant.

Erica walks to her friend Philip’s house. The graph shows Erica’s distance from home over time.

Question 5.
Find the rate of change from 1 minute to 2 minutes.
Go Math Grade 8 Answer Key Chapter 3 Proportional Relationships Lesson 2: Rate of Change and Slope img 12
\(\frac{\text { change in distance }}{\text { change in time }}=\frac{400-?}{2-?}=\frac{?}{?}=?\) ft per min
________ feet per minute

Answer:
200 feet per minute

Explanation:
(400 – 200)/(2 – 1)
200/1
200 feet per minute

Question 6 (request help)
Find the rate of change from 1 minute to 4 minutes.
______ feet per minute

Answer:
200 ft per min

Explanation:
change in distance/change in time
(800 – 200)/(4 – 1)
600/3 = 200 ft per min

Find the slope of each line.

Question 7.
Go Math Grade 8 Answer Key Chapter 3 Proportional Relationships Lesson 2: Rate of Change and Slope img 13
slope = _____
________

Answer:
slope = -2

Explanation:
From the given image, the x-axis is separated with 1 unit.
the y-axis is separated with 1 unit.
For every 1 unit of the x-axis, the slope is taken -2 units on the y-axis
The slope of the line is -2

Question 8.
Go Math Grade 8 Answer Key Chapter 3 Proportional Relationships Lesson 2: Rate of Change and Slope img 14
slope = _____
\(\frac{□}{□}\)

Answer:
\(\frac{3}{2}\)

Explanation:
From the given image, the slope is 3/2

Essential Question Check-In

Question 9.
If you know two points on a line, how can you find the rate of change of the variables being graphed?
Type below:
_____________

Answer:
Find the coordinates of two points on the line. Then divide the change in y-values from one point to the next by the change in x-values.

Independent Practice – Rate of Change and Slope – Page No. 81

Question 10.
Rectangle EFGH is graphed on a coordinate plane with vertices at E(-3, 5), F(6, 2), G(4, -4), and H(-5, -1).
a. Find the slopes of each side.
Type below:
_____________

Answer:
Slope EF = 1/3
slope FG = 3
slope GH = -1/3
slope HE =3

Question 10.
b. What do you notice about the slopes of opposite sides?
Type below:
_____________

Answer:
They are the same.

Question 10.
c. What do you notice about the slopes of adjacent sides?
Type below:
_____________

Answer:
They are negative reciprocals of one another.

Question 11.
A bicyclist started riding at 8:00 A.M. The diagram below shows the distance the bicyclist had traveled at different times. What was the bicyclist’s average rate of speed in miles per hour?
Go Math Grade 8 Answer Key Chapter 3 Proportional Relationships Lesson 2: Rate of Change and Slope img 15
_______ miles per hour

Answer:
15 miles per hour

Explanation:
Total distance traveled by bicyclist = 4.5 mile + 7.5 mile = 12 mile
Total time taken by bicyclist = (8:48 A.M – 8:00 A.M) = 48 min = 0.8 hr
conversion used : ( 1 hour = 60 minute)
Average rate of speed = total distance/total time = 15 mile/hr

Question 12.
Multistep
A line passes through (6, 3), (8, 4), and (n, -2). Find the value of n.
_______

Answer:
n = -4

Explanation:
A line passes through (6, 3), (8, 4), and (n, -2).
From the given information, for every 2 points on x-axis, the y-values are changing one point.
(4, 2), (2, 1), (0, 0), (-2, -1), (-4, -2)

Question 13.
A large container holds 5 gallons of water. It begins leaking at a constant rate. After 10 minutes, the container has 3 gallons of water left.
a. At what rate is the water leaking?
_______ gallons per minute

Answer:
1 gallon every 5 minutes, or 0.2 gal/min

Explanation:
Rate = (5 – 3)/(0 – 10)
= 2/-10
= -0.2
The rate of water leaking is 1 gallon every 5 minutes, or 0.2 gal/min

Question 13.
b. After how many minutes will the container be empty?
_______ minutes

Answer:
25 minutes

Explanation:
Number of minutes = 5/0.2 = 25
It will take 25 minutes for the container to be empty.

Question 14.
Critique Reasoning
Billy found the slope of the line through the points (2, 5) and (-2, -5) using the equation \(\frac{2-(-2)}{5-(-5)}=\frac{2}{5}\). What mistake did he make?
Type below:
_____________

Answer:
He used the change in x over the change in y instead of the change in y over the change in x.

Rate of Change and Slope – Page No. 82

Question 15.
Multiple Representations
Graph parallelogram ABCD on a coordinate plane with vertices at A(3, 4), B(6, 1), C(0, -2), and D(-3, 1).
a. Find the slope of each side.
Go Math Grade 8 Answer Key Chapter 3 Proportional Relationships Lesson 2: Rate of Change and Slope img 16
Type below:
_____________

Answer:
slope AB = -1;
slope BC = 1/2
slope CD = -1;
slope DA = 1/2

Explanation:
Grade 8 Chapter 3 image 5

Question 15.
b. What do you notice about the slopes?
Type below:
_____________

Answer:
The slopes of the opposite sides are the same.

Question 15.
c. Draw another parallelogram on the coordinate plane. Do the slopes have the same characteristics?
Type below:
_____________

Answer:
Yes; opposite sides still have the same slope.

H.O.T.

Focus on Higher Order Thinking

Question 16.
Communicate Mathematical Ideas
Ben and Phoebe are finding the slope of a line. Ben chose two points on the line and used them to find the slope. Phoebe used two different points to find the slope. Did they get the same answer? Explain.
_____________

Answer:
Yes. The slope of a line is constant. Therefore, the slope that you calculate will be the same no matter which two points you choose.

Question 17.
Analyze Relationships
Two lines pass through the origin. The lines have slopes that are opposites. Compare and contrast the lines.
Type below:
_____________

Answer:
One line has a positive slope and one has a negative slope. The lines are equally steep, but one slants upward left to right while the other slants downward left to right. The lines cross at the origin.

Question 18.
Reason Abstractly
What is the slope of the x-axis? Explain.
_____________

Answer:
Zero. The rise along the x-axis is zero, while the run along the x-axis is not zero. The slope zero/run or zero.

Guided Practice – Interpreting the Unit As Slope – Page No. 86

Give the slope of the graph and the unit rate.

Question 1.
Jorge: 5 miles every 6 hours
Go Math Grade 8 Answer Key Chapter 3 Proportional Relationships Lesson 3: Interpreting the Unit As Slope img 17
\(\frac{□}{□}\) miles per hour

Answer:
\(\frac{5}{6}\) miles per hour

Explanation:
Grade 8 Chapter 3 image 11
Slope = rise/run = 5/6
The unit rate a the distance traveled and the slope a the graph of the relationship is equal, 5/6 miles per hour
Calculate miles PER hour
5/6

Question 2.
Akiko
Go Math Grade 8 Answer Key Chapter 3 Proportional Relationships Lesson 3: Interpreting the Unit As Slope img 18
\(\frac{□}{□}\) miles per hour

Answer:
\(\frac{5}{4}\) miles per hour

Explanation:
Calculate miles PER hour
5 miles/4hours = 5/4 miles per hour

Question 3.
The equation y = 0.5x represents the distance Henry hikes, in miles, over time, in hours. The graph represents the rate that Clark hikes. Determine which hiker is faster. Explain.
Go Math Grade 8 Answer Key Chapter 3 Proportional Relationships Lesson 3: Interpreting the Unit As Slope img 19
___________

Answer:
Clark is faster.

Explanation:
From the equation, Henry’s rate is equal to 0.5, or 1/2 mile per hour. Clark’s rate is the slope of the line, which is 3/2 or 1.5 miles per hour.

Write an equation relating the variables in each table.

Question 4.
Go Math Grade 8 Answer Key Chapter 3 Proportional Relationships Lesson 3: Interpreting the Unit As Slope img 20
Type below:
___________

Answer:
y = 15x

Explanation:
y/x = 15/1
y = 15x
Multiply 15 with the x values to get the y values.
y = 15x

Question 5.
Go Math Grade 8 Answer Key Chapter 3 Proportional Relationships Lesson 3: Interpreting the Unit As Slope img 21
Type below:
___________

Answer:
y = 3/8 x

Explanation:
y/x = 6/16
y/x = 3/8
y = 3/8 x

Essential Question Check-In

Question 6.
Describe methods you can use to show a proportional relationship between two variables, x and y. For each method, explain how you can find the unit rate and the slope.
Type below:
___________

Answer:
The ratio of y to x gives the unit rate and slope.

Explanation:
If the equation can be written as y = mx, then m is the unit rate and the slope. Graph: When the line passes through the origin, then the value of r at the point (1, r) is the unit rate and the slope.

Independent Practice – Interpreting the Unit As Slope – Page No. 87

Question 7.
A Canadian goose migrated at a steady rate of 3 miles every 4 minutes.
a. Fill in the table to describe the relationship.
Go Math Grade 8 Answer Key Chapter 3 Proportional Relationships Lesson 3: Interpreting the Unit As Slope img 22
Type below:
___________

Answer:
Grade 8 Chapter 3 image 6

Explanation:
Canadian goose migrated at a steady rate of 3 miles every 4 minutes
y/x = 3/4; x/y = 4/3
y = 3/4 x; x = 4/3 y
If x = 8, y = 3/4 × 8 = 6
If y = 9, x = 4/3 × 9 = 12
If y = 12, x = 4/3 × 12 = 16
If x = 20, y = 3/4 × 20 = 15

Question 7.
b. Graph the relationship.
Go Math Grade 8 Answer Key Chapter 3 Proportional Relationships Lesson 3: Interpreting the Unit As Slope img 23
Type below:
___________

Answer:
Grade 8 Chapter 3 image 7

Explanation:
The points are (4, 3), (8, 6), (12, 9), (16, 12), (20, 15)

Question 7.
c. Find the slope of the graph and describe what it means in the context of this problem.
Slope: \(\frac{□}{□}\) miles per minute

Answer:
Slope: \(\frac{3}{4}\) miles per minute

Explanation:
The unit rate of migration of the goose and the slope of the graph both equal 3/4 mi/min

Question 8.
Vocabulary
A unit rate is a rate in which the first quantity / second quantity in the comparison is one unit.
___________

Answer:
second quantity

Explanation:
A unit rate is a rate in which the “second quantity” in the comparison is one unit

Question 9.
The table and the graph represent the rate at which two machines are bottling milk in gallons per second.
Go Math Grade 8 Answer Key Chapter 3 Proportional Relationships Lesson 3: Interpreting the Unit As Slope img 24
a. Determine the slope and unit rate of each machine.
Type below:
___________

Answer:
Machine 1: slope = unit rate = 0.6/1 = 0.6 gal/s
Machine 2: slope = unit rate = 3/4 = 0.75 gal/s

Question 9.
b. Determine which machine is working at a faster rate.
___________

Answer:
Machine 2 is working at a faster rate since 0.75 > 0.6

Interpreting the Unit As Slope – Page No. 88

Question 10.
Cycling
The equation y = \(\frac{1}{9}\) x represents the distance y, in kilometers, that Patrick traveled in x minutes while training for the cycling portion of a triathlon. The table shows the distance y Jennifer traveled in x minutes in her training. Who has the faster training rate?
Go Math Grade 8 Answer Key Chapter 3 Proportional Relationships Lesson 3: Interpreting the Unit As Slope img 25
___________

Answer:
Jennifer has a faster training rate

Explanation:
Patrick’s rate is 1/9 kilometer per minute. Jennifer’s rate is 1/8 kilometer per minute. 1/9 < 1/8. So, Jennifer has a faster training rate.

H.O.T.

Focus on Higher Order Thinking

Question 11.
Analyze Relationships
There is a proportional relationship between minutes and dollars per minute, shown on a graph of printing expenses. The graph passes through the point (1, 4.75). What is the slope of the graph? What is the unit rate? Explain.
______ dollars per minute

Answer:
$4.75/min

Explanation:
slope = unit rate = 4.75.
If the graph of a proportional relationship passes through the point (1, r), then r equals the slope and the unit rate, which is $4.75/min.

Question 12.
Draw Conclusions
Two cars start at the same time and travel at different constant rates. A graph for Car A passes through the point (0.5, 27.5), and a graph for Car B passes through (4, 240). Both graphs show distance in miles and time in hours. Which car is traveling faster? Explain.
___________

Answer:
Car B

Explanation:
The slope and unit rate of speed of Car A is (27.5 – 0)/(0.5 – 0) = 27.5/0.5 = 55 mi/h.
The slope and unit rate of speed of Car B is (240 – 0)/(4 – 0) = 240/4 = 60 mi/h.
60 > 55, so Car B is traveling faster.

Question 13.
Critical Thinking
The table shows the rate at which water is being pumped into a swimming pool. Use the unit rate and the amount of water pumped after 12 minutes to find how much water will have been pumped into the pool after 13 \(\frac{1}{2}\) minutes. Explain your reasoning.
Go Math Grade 8 Answer Key Chapter 3 Proportional Relationships Lesson 3: Interpreting the Unit As Slope img 26
______ gallons

Answer:
243 gallons

Explanation:
The unit rate is 36/2 = 18gal/min.
So, 1 1/2 minutes after 12 minutes, an additional 18 × 1 1/2 = = 27 gallons will be pumped in.
So, the total is 216 + 27 = 243 gal.

3.1 Representing Proportional Relationships – Model Quiz – Page No. 89

Question 1.
Find the constant of proportionality for the table of values.
Go Math Grade 8 Answer Key Chapter 3 Proportional Relationships Model Quiz img 27
______

Answer:
1.5

Explanation:
constant of proportionality
k = y/x = 3/2 = 1.5
k = y/x = 4.5/3 = 1.5
k = y/x = 6/4 = 1.5
k = y/x = 7.5/5 = 1.5

Question 2.
Phil is riding his bike. He rides 25 miles in 2 hours, 37.5 miles in 3 hours, and 50 miles in 4 hours. Find the constant of proportionality and write an equation to describe the situation.
Type below:
___________

Answer:
The constant of proportionality is 12.5 miles per hour.

Explanation:
The equation is d = 12.5 × t
25 miles ÷ 2 hours = 12.5 miles/hour
A direct proportionality d = 12.5 × t

3.2 Rate of Change and Slope

Find the slope of each line.

Question 3.
Go Math Grade 8 Answer Key Chapter 3 Proportional Relationships Model Quiz img 28
______

Answer:
Slope = 3

Question 4.
Go Math Grade 8 Answer Key Chapter 3 Proportional Relationships Model Quiz img 29
______

Answer:
Slope = -5

3.3 Interpreting the Unit Rate as Slope

Question 5.
The distance Train A travels is represented by d = 70t, where d is the distance in kilometers and t is the time in hours. The distance Train B travels at various times is shown in the table. What is the unit rate of each train? Which train is going faster?
Go Math Grade 8 Answer Key Chapter 3 Proportional Relationships Model Quiz img 30
____________

Answer:
Train A: 70 km per hour; Train B: 75 km per hour; Train B is faster.

Explanation:
The distance Train A travels is represented by d = 70t
where d is the distance in kilometers and t is the time in hours
The speed of train A is 70 kilometer per hour
To find the speed of train B use the table
Slope = (300 – 150)/(4 – 2) = 75
The speed of train B is 75 km per hour
The speed of train B is more

Essential Question

Question 6.
What is the relationship among proportional relationships, lines, rates of change, and slope?
Type below:
____________

Answer:
The relationship between the x-axis and y-axis of any graph is a proportional Relationship which is defined by slope i.e. calculating the rate of change of the plotted line.
1) Here proportional change and rate of change are algebraic quantities which specify how one quantity changes with respect to another.
2) Line and Slope are geometric quantities which describe the graph of any equation.

Selected Response – Mixed Review – Page No. 90

Question 1.
Which of the following is equivalent to 5-1?
Options:
a. 4
b. \(\frac{1}{5}\)
c. −\(\frac{1}{5}\)
d. -5

Answer:
b. \(\frac{1}{5}\)

Explanation:
5-1
1/5

Question 2.
Prasert earns $9 an hour. Which table represents this proportional relationship?
Go Math Grade 8 Answer Key Chapter 3 Proportional Relationships Mixed Review img 31
Options:
a. A
b. B
c. C
d. D

Answer:
a. A

Explanation:
a. 36/4 = 9; 54/6 = 9; 72/8 = 9
b. 36/4 = 9; 45/6 = 7.5; 54/8 = 6.75
c. 9/2 = 4.5
d. 18/2 = 9; 27/3 = 9; 54/4 = 13.5

Question 3.
A factory produces widgets at a constant rate. After 4 hours, 3,120 widgets have been produced. At what rate are the widgets being produced?
Options:
a. 630 widgets per hour
b. 708 widgets per hour
c. 780 widgets per hour
d. 1,365 widgets per hour

Answer:
c. 780 widgets per hour

Explanation:
A factory produces widgets at a constant rate. After 4 hours, 3,120 widgets have been produced.
3,120/4 = 780 widgets per hour

Question 4.
A full lake begins dropping at a constant rate. After 4 weeks it has dropped 3 feet. What is the unit rate of change in the lake’s level compared to its full level?
Options:
a. 0.75 feet per week
b. 1.33 feet per week
c. -0.75 feet per week
d. -1.33 feet per week

Answer:
c. -0.75 feet per week

Explanation:
A full lake begins dropping at a constant rate. After 4 weeks it has dropped 3 feet.
(-3 ft)/(4 weeks) = -3/4 ft/wk = -0.75 ft/wk

Question 5.
What is the slope of the line below?
Go Math Grade 8 Answer Key Chapter 3 Proportional Relationships Mixed Review img 32
Options:
a. -2
b. −\(\frac{1}{2}\)
c. \(\frac{1}{2}\)
d. 2

Answer:
c. \(\frac{1}{2}\)

Explanation:
(-1.5, 1.5) & (1.5, 0)
(0 – 1..5)/(1.5 – (-1.5))
1.5/3
= 1/2

Question 6.
Jim earns $41.25 in 5 hours. Susan earns $30.00 in 4 hours. Pierre’s hourly rate is less than Jim’s, but more than Susan’s. What is his hourly rate?
Options:
a. $6.50
b. $7.75
c. $7.35
d. $8.25

Answer:
b. $7.75

Explanation:
Jim earns $41.25 in 5 hours.
$41.25/5 = 8.25
Jim’s unit rate is $8.25 per hour
30/4 = 7.5
Pierre’s hourly rate is is less than $8.25 but more than $7.50
$7.75

Mini-Task

Question 7.
Joelle can read 3 pages in 4 minutes, 4.5 pages in 6 minutes, and 6 pages in 8 minutes.
a. Make a table of the data.
Go Math Grade 8 Answer Key Chapter 3 Proportional Relationships Mixed Review img 33
Type below:
______________

Answer:
Grade 8 Chapter 3 image 8

Explanation:
Joelle can read 3 pages in 4 minutes,
y/x = 3/4
y = 3/4 x
If x = 6, y = 3/4 × 6 = 4.5
If x = 8, y = 3/4 × 8 = 6

Question 7.
b. Use the values in the table to find the unit rate.
_______ pages per minute

Answer:
0.75 pages per minute

Explanation:
3/4 = 0.75
4.5/6 = 0.75
6/8 = 0.75
The unit rate is 0.75 pages per minute

Question 7.
c. Graph the relationship between minutes and pages read.
Type below:
______________

Answer:
Grade 8 Chapter 3 image 9

Conclusion:

I wish the info prevailed regarding the Go Math Grade 8 Answer Key Chapter 3 Proportional Relationships PDF is beneficial for you. Share this pdf with your besties in order to help them to overcome the difficulties in maths. Students can definitely score good marks in the exam with the help of the Go Math 8th Grade Ch 3 Proportional Relationships Solution Key.All the Best!!!

Go Math Grade 3 Answer Key Chapter 12 Two-Dimensional Shapes

go-math-grade-3-chapter-12-two-dimensional-shapes-answer-key

Access the Go Math Grade 3 Answer Key Chapter 12 Two-Dimensional Shapes with Solutions. Seek Homework Help needed as a part of your preparation and avail 3rd Grade Go Math Answer Key to clear your queries. Enhance your problem-solving skills with the help of Go Math Grade 3 Answer Key and understand the concepts better. Begin your preparation and make the most out of the Grade 3 Chapter 12 Two-Dimensional Shapes Answer Key to score better grades.

Go Math Grade 3 Chapter 12 Two-Dimensional Shapes Answer Key

Practice is the perfect key to success and we have provided simple tricks to solve the Problems in Chapter 12 Two Dimensional Shapes. Go Math Grade 3 Answer Key Chapter 12 acts as a one stop destination to enhance your conceptual knowledge. You will know how to solve different Problems in Chapter 12 using the simple approaches by accessing our Go Math Grade 3 Answer Key Chapter 12 Two-Dimensional Shapes.

Lesson 1: Describe Plane Shapes

Lesson 2: Describe Angles in Plane Shapes

Lesson 3: Identify Polygons

Chapter: Unlock the Problem – Page No. 715

Lesson 4: Describe Sides of Polygons

Mid -Chapter Checkpoint

Lesson 5: Classify Quadrilaterals

Lesson 6: Draw Quadrilaterals

Lesson 7: Describe Triangles

Lesson 8: Problem Solving Classify Plane Shapes

Lesson 9: Relate Shapes, Fractions, and Area

Review/Test

Describe Plane Shapes – Page No. 701

Write how many line segments the shape has.

Question 1.
Go Math Grade 3 Answer Key Chapter 12 Two-Dimensional Shapes Describe Plane Shapes img 1
4 line segments

Answer:
4 line segments

Explanation:
Chapter 12 Describe Plane Shapes image 1 701

Question 2.
Go Math Grade 3 Answer Key Chapter 12 Two-Dimensional Shapes Describe Plane Shapes img 2
________ line segments

Answer:
6 line segments

Explanation:
Chapter 12 Describe Plane Shapes image 2 701

Question 3.
Go Math Grade 3 Answer Key Chapter 12 Two-Dimensional Shapes Describe Plane Shapes img 3
________ line segments

Answer:
8 line segments

Explanation:
Chapter 12 Describe Plane Shapes image 3 701

Question 4.
Go Math Grade 3 Answer Key Chapter 12 Two-Dimensional Shapes Describe Plane Shapes img 4
________ line segments

Answer:
5 line segments

Explanation:
Chapter 12 Describe Plane Shapes image 4 701

Write whether the shape is open or closed.

Question 5.
Go Math Grade 3 Answer Key Chapter 12 Two-Dimensional Shapes Describe Plane Shapes img 5
________

Answer:
Open

Explanation:
The given shape does not start and ends at the same point. So, the shape is open.

Question 6.
Go Math Grade 3 Answer Key Chapter 12 Two-Dimensional Shapes Describe Plane Shapes img 6
________

Answer:
Closed

Explanation:
The given shape started and ends at the same point. So, the shape is closed.

Problem Solving

Question 7.
Carl wants to show a closed shape in his drawing. Show and explain how to make the drawing a closed shape.
Go Math Grade 3 Answer Key Chapter 12 Two-Dimensional Shapes Describe Plane Shapes img 7
Type below:
___________

Answer:
chapter 12 image 1 702
Add a fourth line segment, so the shape starts and ends at the same point.

Explanation:
Add a fourth line segment, so the shape starts and ends at the same point.

Question 8.
The shape of a fish pond at a park is shown below. Is the shape open or closed?
Go Math Grade 3 Answer Key Chapter 12 Two-Dimensional Shapes Describe Plane Shapes img 8
________

Answer:
Closed

Explanation:
The given shape started and ends at the same point. So, the shape is closed.

Describe Plane Shapes – Page No. 702

Lesson Check

Question 1.
How many line segments does this shape have?
Go Math Grade 3 Answer Key Chapter 12 Two-Dimensional Shapes Describe Plane Shapes img 9
Options:
a. 2
b. 3
c. 4
d. 5

Answer:
d. 5

Explanation:
Chapter 12 Describe Plane Shapes image 5 701

Question 2.
Which of these is part of a line, has one endpoint, and continues in one direction?
Options:
a. ray
b. line
c. line segment
d. point

Answer:
a. ray

Explanation:
The line has one endpoint and continues in one direction is called a ray.

Spiral Review

Question 3.
What multiplication sentence does the array show?
Go Math Grade 3 Answer Key Chapter 12 Two-Dimensional Shapes Describe Plane Shapes img 10
Options:
a. 3 × 8 = 24
b. 4 × 8 = 32
c. 8 × 5 = 40
d. 4 × 9 = 36

Answer:
b. 4 × 8 = 32

Explanation:
There are 4 rows and 8 columns available. 4 x 8 = 32 is the answer.

Question 4.
What is the unknown factor and quotient?
9 × ■ = 27
27 ÷ 9 = ■
Options:
a. 3
b. 4
c. 5
d. 6

Answer:
a. 3

Explanation:
9 x 3 = 27.
27 ÷ 9 = 3.
The answer is 3.

Question 5.
Which fraction is equivalent to \(\frac{4}{8}\)?
Go Math Grade 3 Answer Key Chapter 12 Two-Dimensional Shapes Describe Plane Shapes img 11
Options:
a. \(\frac{3}{4}\)
b. \(\frac{1}{2}\)
c. \(\frac{1}{4}\)
d. \(\frac{1}{8}\)

Answer:
b. \(\frac{1}{2}\)

Explanation:
\(\frac{4}{8}\) = b. \(\frac{1}{2}\).

Question 6.
Mr. MacTavish has 30 students from his class going on a field trip to the zoo. He is placing 6 students in each group. How many groups of students from Mr. MacTavish’s class will be going to the zoo?
Options:
a. 5
b. 6
c. 7
d. 36

Answer:
a. 5

Explanation:
1 group = 6 students.
6 x number of groups = 30 students.
6 x s = 30.
s = 30 ÷ 6
s = 5.
5 groups of students from Mr. MacTavish’s class will be going to the zoo.

Describe Angles in Plane Shapes – Page No. 707

Use the corner of a sheet of paper to tell whether the angle is a right angle, less than a right angle, or greater than a right angle.

Question 1.
Go Math Grade 3 Answer Key Chapter 12 Two-Dimensional Shapes Describe Angles in Plane Shapes img 12
less than a right angle

Answer:
less than a right angle

Question 2.
Go Math Grade 3 Answer Key Chapter 12 Two-Dimensional Shapes Describe Angles in Plane Shapes img 13
________

Answer:
Right Angle:

Question 3.
Go Math Grade 3 Answer Key Chapter 12 Two-Dimensional Shapes Describe Angles in Plane Shapes img 14
________

Answer:
Greater than a right angle

Write how many of each type of angle the shape has.

Question 4.
Go Math Grade 3 Answer Key Chapter 12 Two-Dimensional Shapes Describe Angles in Plane Shapes img 15
________ right
________ less than a right
________ greater than a right

Answer:
4 right
0 less than a right
0 greater than a right

Explanation:
Chapter 12 Describe Angles in Plane Shapes image 1 707
4 right Angles.
0 less than right angles.
0 greater than right angles.

Question 5.
Go Math Grade 3 Answer Key Chapter 12 Two-Dimensional Shapes Describe Angles in Plane Shapes img 16
________ right
________ less than a right
________ greater than a right

Answer:
0 right
0 less than a right
5 greater than a right

Explanation:
Chapter 12 Describe Angles in Plane Shapes image 2 707
0 right Angles
0 less than right angles
5 greater than right angles

Question 6.
Go Math Grade 3 Answer Key Chapter 12 Two-Dimensional Shapes Describe Angles in Plane Shapes img 17
________ right
________ less than a right
________ greater than a right

Answer:
2 right
0 less than a right
4 greater than a right

Explanation:
Chapter 12 Describe Angles in Plane Shapes image 4 707
2 right angles.
0 less than right angles
4 greater than right angles.

Problem Solving

Question 7.
Jeff has a square piece of art paper. He cuts across it from one corner to the opposite corner to make two pieces. What is the total number of sides and angles in both of the new shapes?
________ sides
________ angle

Answer:
The new shape has 6 sides and 6 angles

Explanation:
Chapter 12 Describe Angles in Plane Shapes image 8 707
The new shape has 6 sides and 6 angles

Question 8.
Kaylee tells Aimee that the shape of a stop sign has at least one right angle. Aimee says that there are no right angles. Who is correct? Explain.
Go Math Grade 3 Answer Key Chapter 12 Two-Dimensional Shapes Describe Angles in Plane Shapes img 18
________

Answer:
Aimee’s answer is correct

Explanation:
Chapter 12 Describe Angles in Plane Shapes image 5 707
All the angles are greater than a right angle. Aimee’s answer is correct.

Describe Angles in Plane Shapes – Page No. 708

Lesson Check

Question 1.
What describes this angle?
Go Math Grade 3 Answer Key Chapter 12 Two-Dimensional Shapes Describe Angles in Plane Shapes img 19
Options:
a. right angle
b. less than a right angle
c. greater than a right angle
d. small angle

Answer:
c. greater than a right angle

Explanation:
The given shape has the greater angle than the right angle. So, the answer is greater than a right angle.

Question 2.
How many right angles does this shape have?
Go Math Grade 3 Answer Key Chapter 12 Two-Dimensional Shapes Describe Angles in Plane Shapes img 20
Options:
a. 1
b. 2
c. 3
d. 4

Answer:
c. 3

Explanation:
Chapter 12 Describe Angles in Plane Shapes image 5 708

Spiral Review

Question 3.
What fraction of the group is shaded?
Go Math Grade 3 Answer Key Chapter 12 Two-Dimensional Shapes Describe Angles in Plane Shapes img 21
Options:
a. \(\frac{5}{6}\)
b. \(\frac{1}{3}\)
c. \(\frac{1}{6}\)
d. \(\frac{1}{8}\)

Answer:
c. \(\frac{1}{6}\)

Explanation:
There are 6 circles. And one circle is shaded out of 6 circles. So, answer is \(\frac{1}{6}\).

Question 4.
Compare.
\(\frac{4}{8}\) _____ \(\frac{3}{8}\)
Options:
a. >
b. <
c. =
d. ÷

Answer:
a. >

Explanation:
Given fractions have the same denominators. So, compare numerators to get the answer.
4 > 3. The answer is \(\frac{4}{8}\) > \(\frac{3}{8}\).

Question 5.
Which of the following does NOT describe a line segment?
Options:
a. does not end
b. is straight
c. is part of a line
d. has 2 endpoints

Answer:
a. does not end

Explanation:
The line segment is a straight line that has two end points. a. does not end not describes a line segment.

Question 6.
How many line segments does this shape have?
Go Math Grade 3 Answer Key Chapter 12 Two-Dimensional Shapes Describe Angles in Plane Shapes img 22
Options:
a. 5
b. 6
c. 7
d. 8

Answer:
c. 7

Explanation:
Chapter 12 Describe Angles in Plane Shapes image 6 708

Identify Polygons – Page No. 713

Is the shape a polygon? Write yes or no.

Question 1.
Go Math Grade 3 Answer Key Chapter 12 Two-Dimensional Shapes Identify Polygons img 23
no

Answer:
No

Explanation:
No, The given image doesn’t have line segments.

Question 2.
Go Math Grade 3 Answer Key Chapter 12 Two-Dimensional Shapes Identify Polygons img 24
_____

Answer:
Yes

Explanation:
Yes, A polygon is a closed plane shape that is made up of line segments that meet only at their endpoints. Each line segment in a polygon is a side.

Question 3.
Go Math Grade 3 Answer Key Chapter 12 Two-Dimensional Shapes Identify Polygons img 25
_____

Answer:
No

Explanation:
No, the given shape is an open shape. It is not a polygon.

Question 4.
Go Math Grade 3 Answer Key Chapter 12 Two-Dimensional Shapes Identify Polygons img 26
_____

Answer:
No

Explanation:
No, it has a curved path. So, the given shape is not a polygon.

Write the number of sides and the number of angles. Then name the polygon.

Question 5.
Go Math Grade 3 Answer Key Chapter 12 Two-Dimensional Shapes Identify Polygons img 27
_______ sides
_______ angles
_______

Answer:
6 sides; 6 angles; hexagon

Explanation:
Chapter 12 Identify Polygons image 1 713

Question 6.
Go Math Grade 3 Answer Key Chapter 12 Two-Dimensional Shapes Identify Polygons img 28
_______ sides
_______ angles
_______

Answer:
4 sides; 4 angles; quadrilateral

Explanation:
Chapter 12 Identify Polygons image 2 713

Problem Solving

Question 7.
Mr. Murphy has an old coin that has ten sides. If its shape is a polygon, how many angles does the old coin have?
_______ angles

Answer:
10 angles

Explanation:
Given that the shape has ten sides. So, the old coin will have 10 angles.

Question 8.
Lin says that an octagon has six sides. Chris says that it has eight sides. Whose statement is correct?
_______ statement

Answer:
Chris’s statement is correct.

Explanation:
Chris’s statement is correct. An octagon has eight sides.

Identify Polygons – Page No. 714

Lesson Check

Question 1.
Which is a name for this polygon?
Go Math Grade 3 Answer Key Chapter 12 Two-Dimensional Shapes Identify Polygons img 29
Options:
a. hexagon
b. octagon
c. quadrilateral
d. pentagon

Answer:
c. quadrilateral

Explanation:
Chapter 12 Identify Polygons image 1 714
It has 4 sides and 4 angles. The name of the polygon is quadrilateral.

Question 2.
How many sides does this polygon have?
Go Math Grade 3 Answer Key Chapter 12 Two-Dimensional Shapes Identify Polygons img 30
Options:
a. 4
b. 5
c. 6
d. 7

Answer:
c. 6

Explanation:
Chapter 12 Identify Polygons image 2 714

Spiral Review

Question 3.
How many right angles does this shape have?
Go Math Grade 3 Answer Key Chapter 12 Two-Dimensional Shapes Identify Polygons img 31
Options:
a. 4
b. 3
c. 2
d. 0

Answer:
a. 4

Explanation:
Chapter 12 Identify Polygons image 3 714

Question 4.
Erica has 8 necklaces. One fourth of the necklaces are blue. How many necklaces are blue?
Options:
a. 2
b. 3
c. 4
d. 8

Answer:
a. 2

Explanation:
Erica has 8 necklaces. One-fourth of the necklaces are blue. So, 2 necklaces are blue.

Question 5.
Which of these is straight, is part of a line, and has 2 endpoints?
Options:
a. line
b. line segment
c. point
d. ray

Answer:
b. line segment

Question 6.
What describes this angle?
Go Math Grade 3 Answer Key Chapter 12 Two-Dimensional Shapes Identify Polygons img 32
Options:
a. greater than a right angle
b. large angle
c. less than a right angle
d. right angle

Answer:
c. less than a right angle

Unlock the Problem – Page No. 715

Question 1.
Look at the polygon. How many pairs of sides are parallel?
Go Math Grade 3 Answer Key Chapter 12 Two-Dimensional Shapes Unlock the Problem img 33
________ pair(s)

Answer:
1 pair of sides are parallel

Question 2.
How do you know the shape is a polygon?
Type below:
___________

Answer:
A polygon is a closed plane shape that is made up of line segments that meet only at their endpoints. Each line segment in a polygon is a side.

Question 3.
Lines that cross or meet are intersecting lines. Intersecting lines form angles.
Go Math Grade 3 Answer Key Chapter 12 Two-Dimensional Shapes Unlock the Problem img 34

Answer:
Either less than the right angle or more than the right angle.

Question 4.
Go Math Grade 3 Answer Key Chapter 12 Two-Dimensional Shapes Unlock the Problem img 35
The orange and blue line segments meet and form an angle. So, they are __________________
___________

Answer:
Intersecting lines

Question 5.
Intersecting lines that cross or meet to form right angles are perpendicular lines.
Go Math Grade 3 Answer Key Chapter 12 Two-Dimensional Shapes Unlock the Problem img 36

Answer:
90 degrees

Question 6.
Go Math Grade 3 Answer Key Chapter 12 Two-Dimensional Shapes Unlock the Problem img 37
The red and blue line segments meet to form a right angle. So, they are ________________
___________

Answer:
Perpendicular lines

Question 7.
Lines that never cross or meet and are always the same distance apart are parallel lines. They do not form any angles.
Go Math Grade 3 Answer Key Chapter 12 Two-Dimensional Shapes Unlock the Problem img 38
So, the polygon above has _ pair of parallel sides
_________ pair(s)

Answer:
The polygon has 1 pair of parallel sides

Question 8.
Go Math Grade 3 Answer Key Chapter 12 Two-Dimensional Shapes Unlock the Problem img 39
The green and blue line segments would never cross or meet. They are always the same distance apart. So, they appear to be _________________
_________

Answer:
Parallel lines

Describe Sides of Polygons – Page No. 719

Look at the dashed sides of the polygon. Tell if they appear to be intersecting, perpendicular, or parallel. Write all the words that describe the sides.

Question 1.
Go Math Grade 3 Answer Key Chapter 12 Two-Dimensional Shapes Describe Sides of Polygons img 40
parallel

Answer:
parallel lines

Explanation:
The dashed sides are not intersecting with each other. So, the given lines are parallel lines.

Question 2.
Go Math Grade 3 Answer Key Chapter 12 Two-Dimensional Shapes Describe Sides of Polygons img 41
_______
_______

Answer:
perpendicular lines

Explanation:
The dashed sides are meeting to form a right angle. So, they are perpendicular lines.

Question 3.
Go Math Grade 3 Answer Key Chapter 12 Two-Dimensional Shapes Describe Sides of Polygons img 42
_______

Answer:
intersecting lines

Explanation:
The dashed line segments meet and form an angle. So, they are intersecting lines.

Question 4.
Go Math Grade 3 Answer Key Chapter 12 Two-Dimensional Shapes Describe Sides of Polygons img 43
_______

Answer:
intersecting lines

Explanation:
The dashed line segments meet and form an angle. So, they are intersecting lines.

Question 5.
Go Math Grade 3 Answer Key Chapter 12 Two-Dimensional Shapes Describe Sides of Polygons img 44
_______
_______

Answer:
perpendicular lines

Explanation:
The dashed line segments meet to form a right angle. So, they are perpendicular lines.

Question 6.
Go Math Grade 3 Answer Key Chapter 12 Two-Dimensional Shapes Describe Sides of Polygons img 45
_______

Answer:
parallel lines

Explanation:
The dashed line segments would never cross or meet. They are always the same distance apart. So, they are parallel lines.

Question 7.
Go Math Grade 3 Answer Key Chapter 12 Two-Dimensional Shapes Describe Sides of Polygons img 46
_______

Answer:
parallel lines

Explanation:
The dashed line segments would never cross or meet. They are always the same distance apart. So, they are parallel lines.

Question 8.
Go Math Grade 3 Answer Key Chapter 12 Two-Dimensional Shapes Describe Sides of Polygons img 47
_______

Answer:
intersecting lines

Explanation:
The dashed line segments meet and form an angle. So, they are intersecting lines.

Question 9.
Go Math Grade 3 Answer Key Chapter 12 Two-Dimensional Shapes Describe Sides of Polygons img 48
_______
_______

Answer:
perpendicular lines

Explanation:
The dashed line segments meet to form a right angle. So, they are perpendicular lines.

Problem Solving

Use shapes A–D for 10–11.
Go Math Grade 3 Answer Key Chapter 12 Two-Dimensional Shapes Describe Sides of Polygons img 49

Question 10.
Which shapes appear to have parallel sides?
_______ ; _______ ; _______

Answer:
A, C, D

Explanation:
A, C, D have lines that appear to never meet and are always the same distance apart. So, they have parallel lines.

Question 11.
Which shapes appear to have perpendicular sides?
_______ ; _______

Answer:
B, C

Explanation:
B, C have lines that meet to form right angles. So, they have perpendicular sides.

Describe Sides of Polygons – Page No. 720

Lesson Check

Question 1.
How many pairs of parallel sides does the quadrilateral appear to have?
Go Math Grade 3 Answer Key Chapter 12 Two-Dimensional Shapes Describe Sides of Polygons img 50
Options:
a. 1
b. 2
c. 3
d. 4

Answer:
b. 2

Explanation:
Chapter 12 Identify Polygons image 5 714
Side a and a are one pair of parallel lines.
Side b and b are one pair of parallel lines.
The quadrilateral has 2 pairs of parallel sides.

Question 2.
Which sides appear to be parallel?
Go Math Grade 3 Answer Key Chapter 12 Two-Dimensional Shapes Describe Sides of Polygons img 51
Options:
a. a and c only
b. b and d only
c. a and b, c and d
d. a and c, b and d

Answer:
d. a and c, b and d

Explanation:
a and c are parallel lines. They appear to never meet and are always the same distance apart. They do not form any angles.
Similarly, b and d are parallel lines.

Spiral Review

Question 3.
Mr. Lance designed a class banner shaped like the polygon shown. What is the name of the polygon?
Go Math Grade 3 Answer Key Chapter 12 Two-Dimensional Shapes Describe Sides of Polygons img 52
Options:
a. pentagon
b. octagon
c. hexagon
d. decagon

Answer:
a. pentagon

Explanation:
Chapter 12 Describe Sides of Polygons image 1 720
The polygon has 5 sides and 5 angles. So, the given polygon is a pentagon.

Question 4.
How many angles greater than a right angle does this shape have?
Go Math Grade 3 Answer Key Chapter 12 Two-Dimensional Shapes Describe Sides of Polygons img 53
Options:
a. 0
b. 1
c. 2
d. 3

Answer:
b. 1

Explanation:
Chapter 12 Describe Sides of Polygons image 2 720
One angle is greater than the right angles.

Question 5.
How many line segments does this shape have?
Go Math Grade 3 Answer Key Chapter 12 Two-Dimensional Shapes Describe Sides of Polygons img 54
Options:
a. 6
b. 7
c. 8
d. 9

Answer:
c. 8

Explanation:
Chapter 12 Describe Sides of Polygons image 3 720
The given shape has 8 line segments.

Question 6.
Which fraction names the shaded part?
Go Math Grade 3 Answer Key Chapter 12 Two-Dimensional Shapes Describe Sides of Polygons img 55
Options:
a. \(\frac{1}{3}\)
b. \(\frac{1}{4}\)
c. \(\frac{1}{6}\)
d. \(\frac{5}{6}\)

Answer:
c. \(\frac{1}{6}\)

Explanation:
The given image has 6 parts. 1 part is shaded out of 1 part. The answer is \(\frac{1}{6}\)

Mid -Chapter Checkpoint – Page No. 721

Vocabulary

Choose the best term from the box to complete the sentence.
Go Math Grade 3 Answer Key Chapter 12 Two-Dimensional Shapes Mid -Chapter Checkpoint img 56

Question 1.
An __________ is formed by two rays that share an endpoint.
__________

Answer:
Angle

Explanation:

Question 2.
A _____________ is a closed plane shape made up of line segments.
________

Answer:
Polygon

Question 3.
A ____________ forms a square corner.
________

Answer:
Right Angle

Concepts and Skills

Use the corner of a sheet of paper to tell whether the angle is a
right angle, less than a right angle, or greater than a right angle.

Question 4.
Go Math Grade 3 Answer Key Chapter 12 Two-Dimensional Shapes Mid -Chapter Checkpoint img 57
________

Answer:
less than a right angle

Question 5.
Go Math Grade 3 Answer Key Chapter 12 Two-Dimensional Shapes Mid -Chapter Checkpoint img 58
________

Answer:
right angle

Question 6.
Go Math Grade 3 Answer Key Chapter 12 Two-Dimensional Shapes Mid -Chapter Checkpoint img 59
________

Answer:
greater than a right angle

Write the number of sides and the number of angles.
Then name the polygon.

Question 7.
Go Math Grade 3 Answer Key Chapter 12 Two-Dimensional Shapes Mid -Chapter Checkpoint img 60
________ sides
________ angles
________

Answer:
6 sides; 6 angles

Explanation:
Chapter 12 Identify Polygons image 1 713
6 sides; 6 angles

Question 8.
Go Math Grade 3 Answer Key Chapter 12 Two-Dimensional Shapes Mid -Chapter Checkpoint img 61
________ sides
________ angles
________

Answer:
4 sides; 4 angles

Explanation:
Chapter 12 Concepts and Skills image 2 721
4 sides; 4 angles

Question 9.
Go Math Grade 3 Answer Key Chapter 12 Two-Dimensional Shapes Mid -Chapter Checkpoint img 62
________ sides
________ angles
________

Answer:
5 sides; 5 angles

Explanation:
Chapter 12 Concepts and Skills image 3 721
5 sides; 5 angles

Mid -Chapter Checkpoint – Page No. 722

Question 10.
Anne drew the shape at the right. Is her shape an open shape or a closed shape?
Go Math Grade 3 Answer Key Chapter 12 Two-Dimensional Shapes Mid -Chapter Checkpoint img 63
________

Answer:
Closed Shape

Explanation:
The given shape started and ends at the same point. So, the shape is closed.

Question 11.
This sign tells drivers there is a steep hill ahead. Write the number of sides and the number of angles in the shape of the sign. Then name the shape.
Go Math Grade 3 Answer Key Chapter 12 Two-Dimensional Shapes Mid -Chapter Checkpoint img 64
________ sides
________ angles
________

Answer:
4 sides; 4 angles

Explanation:
Chapter 12 Concepts and Skills image 4 721
4 sides; 4 angles

Question 12.
Why is this closed plane shape NOT a polygon?
Go Math Grade 3 Answer Key Chapter 12 Two-Dimensional Shapes Mid -Chapter Checkpoint img 65
Type below:
________

Answer:
2 × 5 = 10

Explanation:
The given shape started and ends at the same point. So, the shape is closed. A polygon is made up of line segments that meet only at their endpoints. So, the given shape is not a polygon.

Question 13.
Sean drew a shape with 2 fewer sides than an octagon. Which shape did he draw?
________

Answer:
pentagon

Explanation:
octagon has 8 sides.
2 + 8 = 10 sides

Question 14.
John drew a polygon with two line segments that meet to form a right angle. Circle the words that describe the line segments.
Go Math Grade 3 Answer Key Chapter 12 Two-Dimensional Shapes Mid -Chapter Checkpoint img 66
Type below:
________

Answer:
perpendicular

Classify Quadrilaterals – Page No. 727

Click all the words that describe the quadrilateral.

Question 1.
Go Math Grade 3 Answer Key Chapter 12 Two-Dimensional Shapes Classify Quadrilaterals img 67

Answer:
Square; Rectangle; Rhombus

Question 2.
Go Math Grade 3 Answer Key Chapter 12 Two-Dimensional Shapes Classify Quadrilaterals img 68
Options:
a. square
b. rectangle
c. rhombus
d. trapezoid

Answer:
b. rectangle

Explanation:
The given shape has two pairs opposite with the same length. Also, all the angles are right angles. The given shape is a rectangle.

Question 3.
Go Math Grade 3 Answer Key Chapter 12 Two-Dimensional Shapes Classify Quadrilaterals img 69
Options:
a. square
b. rectangle
c. rhombus
d. trapezoid

Answer:
d. trapezoid

Explanation:
Even though the given shape has four sides, they are not equal. Also, it has only two right angles. The given shape is a trapezoid.

Use the quadrilaterals below for 4–6.
Go Math Grade 3 Answer Key Chapter 12 Two-Dimensional Shapes Classify Quadrilaterals img 70

Question 4.
Which quadrilaterals appear to have no right angles?
______ ; ______ ; ______

Answer:
B, D, and E

Explanation:
B, D, E don’t have right angles.

Question 5.
Which quadrilaterals appear to have 4 right angles?
______ ; ______

Answer:
A and C

Explanation:
A and C both shapes have right angles.

Question 6.
Which quadrilaterals appear to have 4 sides of equal length?
______ ; ______ ; ______

Answer:
B, C, and D

Explanation:
The B, C, and D have all the equal lengths.

Problem Solving

Question 7.
A picture on the wall in Jeremy’s classroom has 4 right angles, 4 sides of equal length, and 2 pairs of opposite sides that are parallel. What quadrilateral best describes the picture?
_______

Answer:
Square

Explanation:
A square has 4 right angles, 4 sides of equal length, and 2 pairs of opposite sides that are parallel.

Question 8.
Sofia has a plate that has 4 sides of equal length, 2 pairs of opposite sides that are parallel, and no right angles. What quadrilateral best describes the plate?
_________

Answer:
rhombus

Explanation:
A rhombus has 4 sides of equal length, 2 pairs of opposite sides that are parallel, and no right angles.

Classify Quadrilaterals – Page No. 728

Lesson Check

Question 1.
Which word describes the quadrilateral?
Go Math Grade 3 Answer Key Chapter 12 Two-Dimensional Shapes Classify Quadrilaterals img 71
Options:
a. square
b. trapezoid
c. rhombus
d. rectangle

Answer:
b. trapezoid

Explanation:
The given quadrilateral doesn’t have right angles. So, it is not a square and a rectangle. Also, the quadrilateral doesn’t have equal sides. So, it is not a rhombus.

Question 2.
Which quadrilaterals appear to have 2 pairs of opposite sides that are parallel?
Go Math Grade 3 Answer Key Chapter 12 Two-Dimensional Shapes Classify Quadrilaterals img 72
Options:
a. A and B
b. A, B, and C
c. A only
d. B only

Answer:
a. A and B

Explanation:
The A and B shapes have 2 pairs of opposite sides that are parallel.

Spiral Review

Question 3.
Aiden drew the polygon shown. What is the name of the polygon he drew?
Go Math Grade 3 Answer Key Chapter 12 Two-Dimensional Shapes Classify Quadrilaterals img 73
Options:
a. decagon
b. hexagon
c. octagon
d. pentagon

Answer:
a. decagon

Explanation:
Chapter 12 Classify Quadrilaterals image 1 728
10 sides; 10 angles

Question 4.
How many pairs of parallel sides does this shape appear to have?
Go Math Grade 3 Answer Key Chapter 12 Two-Dimensional Shapes Classify Quadrilaterals img 74
Options:
a. 4
b. 2
c. 1
d. 0

Answer:
b. 2

Explanation:
Chapter 12 Identify Polygons image 5 714

Side a and a are one pair of parallel lines.
Side b and b are one pair of parallel lines.
The quadrilateral has 2 pairs of parallel sides.

Question 5.
What word describes the dashed sides of the shape shown?
Go Math Grade 3 Answer Key Chapter 12 Two-Dimensional Shapes Classify Quadrilaterals img 75
Options:
a. intersecting
b. parallel
c. perpendicular
d. right

Answer:
b. parallel

Explanation:
The dashed sides of the shape would never cross or meet. They are always the same distance apart. So, they appear to be parallel lines.

Question 6.
How many right angles does this shape have?
Go Math Grade 3 Answer Key Chapter 12 Two-Dimensional Shapes Classify Quadrilaterals img 76
Options:
a. 0
b. 1
c. 2
d. 3

Answer:
a. 0

Explanation:
There are no right angles available for the given shape.

Draw Quadrilaterals – Page No. 733

Draw a quadrilateral that is described.
Name the quadrilateral you drew.

Question 1.
4 sides of equal length
Go Math Grade 3 Answer Key Chapter 12 Two-Dimensional Shapes Draw Quadrilaterals img 77

Answer:
square

Explanation:
A square has four equal sides and four equal angles

Question 2.
1 pair of opposite sides that are parallel
Go Math Grade 3 Answer Key Chapter 12 Two-Dimensional Shapes Draw Quadrilaterals img 78
_________

Answer:
trapezoid

Explanation:
Chapter 12 Draw Quadrilaterals image 1 733
trapezoid

Draw a quadrilateral that does not belong.
Then explain why.

Question 3.
Go Math Grade 3 Answer Key Chapter 12 Two-Dimensional Shapes Draw Quadrilaterals img 79

Answer:
Chapter 12 Classify Quadrilaterals image 2 728

Explanation:
The shape is a trapezoid. It has only 1 pair of opposite sides that are parallel. The 3 quadrilaterals shown have 2 pairs of sides that are parallel.

Problem Solving

Question 4.
Layla drew a quadrilateral with 4 right angles and 2 pairs of opposite sides that are parallel. Name the quadrilateral she could have drawn.
_________

Answer:
square or rectangle

Explanation:
The square or rectangle has 4 right angles and 2 pairs of opposite sides that are parallel.

Question 5.
Victor drew a quadrilateral with no right angles and 4 sides of equal length. What quadrilateral could Victor have drawn?
_________

Answer:
rhombus.

Explanation:
A rhombus has no right angles and 4 sides of equal length.

Draw Quadrilaterals – Page No. 734

Lesson Check

Question 1.
Chloe drew a quadrilateral with 2 pairs of opposite sides that are parallel. Which shape could NOT be Chloe’s quadrilateral?
Options:
a. rectangle
b. rhombus
c. square
d. trapezoid

Answer:
d. trapezoid

Explanation:
A trapezoid will have only one pair of parallel lines.

Question 2.
Mike drew a quadrilateral with four right angles. Which shape could he have drawn?
Options:
a. rectangle
b. hexagon
c. trapezoid
d. triangle

Answer:
a. rectangle

Explanation:
A rectangle will have four right angles.

Spiral Review

Question 3.
A quadrilateral has 4 right angles and 4 sides of equal length. What is the name of the quadrilateral?
Options:
a. pentagon
b. square
c. trapezoid
d. hexagon

Answer:
b. square

Explanation:
A square has 4 right angles, 4 sides of equal length, and 2 pairs of opposite sides that are parallel.

Question 4.
Mark drew two lines that form a right angle. Which word describes the lines Mark drew?
Options:
a. perpendicular
b. parallel
c. acute
d. obtuse

Answer:
a. perpendicular

Explanation:
Intersecting lines that cross or meet to form right angles are perpendicular lines.

Question 5.
Dennis drew the rectangle on grid paper. What is the perimeter of the rectangle Dennis drew?
Go Math Grade 3 Answer Key Chapter 12 Two-Dimensional Shapes Draw Quadrilaterals img 80
Options:
a. 7 units
b. 12 units
c. 14 units
d. 15 units

Answer:
b. 12 units

Explanation:
Chapter 12 Draw Quadrilaterals image 1 734
3 + 3 + 3 + 3 = 12 units.

Question 6.
Jill drew the rectangle on grid paper. What is the area of the rectangle Jill drew?
Go Math Grade 3 Answer Key Chapter 12 Two-Dimensional Shapes Draw Quadrilaterals img 81
Options:
a. 12 square units
b. 15 square units
c. 16 square units
d. 18 square units

Answer:
b. 15 square units

Explanation:
Chapter 12 Draw Quadrilaterals image 3 734
5 x 3 = 15 square units.

Describe Triangles – Page No. 739

Use the triangles for 1–3. Write A, B, or C.
Then complete the sentences.
Go Math Grade 3 Answer Key Chapter 12 Two-Dimensional Shapes Describe Triangles img 82

Question 1.
Triangle B has 3 angles less than a right angle and
appears to have 3 sides of equal length.

Answer:
B

Question 2.
Triangle _________ has 1 right angle and appears to have
_________ sides of equal length.

Answer:
Triangle C has 1 right angle and appears to have 0 sides of equal length.

Question 3.
Triangle _________ has 1 angle greater than a right angle and appears to have
_________ sides of equal length.

Answer:
Triangle A has 1 angle greater than a right angle and appears to have 2 sides of equal length.

Question 4.
Kyle, Kathy, and Kelly each drew a triangle. Who drew the triangle that has 1 angle greater than a right angle and appears to have no sides of equal length?
Go Math Grade 3 Answer Key Chapter 12 Two-Dimensional Shapes Describe Triangles img 83
_________

Answer:
Kathy

Explanation:
Kathy triangle has 1 angle greater than a right angle and has no sides of equal length.

Problem Solving

Question 5.
Matthew drew the back of his tent. How many sides appear to be of equal length?
Go Math Grade 3 Answer Key Chapter 12 Two-Dimensional Shapes Describe Triangles img 84
_________ sides

Answer:
2 sides

Explanation:
Chapter 12 Describe Triangles image 1 739
a and b are 2 sides have equal lengths.

Question 6.
Sierra made the triangular picture frame shown. How many angles are greater than a right angle?
Go Math Grade 3 Answer Key Chapter 12 Two-Dimensional Shapes Describe Triangles img 85
_________

Answer:
0

Explanation:
Chapter 12 Describe Triangles image 3 739
0 angles are greater than a right angle.

Describe Triangles – Page No. 740

Lesson Check

Question 1.
How many angles less than a right angle does this triangle have?
Go Math Grade 3 Answer Key Chapter 12 Two-Dimensional Shapes Describe Triangles img 86
Options:
a. 0
b. 1
c. 2
d. 3

Answer:
d. 3

Explanation:
Chapter 12 Describe Triangles image 1 740
3 angles less than a right angle.

Question 2.
How many sides of equal length does this triangle appear to have?
Go Math Grade 3 Answer Key Chapter 12 Two-Dimensional Shapes Describe Triangles img 87
Options:
a. 0
b. 1
c. 2
d. 3

Answer:
c. 2

Explanation:
The two lines forming the right angle are equal in length.

Spiral Review

Question 3.
A quadrilateral has 4 right angles and 2 pairs of opposite sides that are parallel. Which quadrilateral could it be?
Options:
a. trapezoid
b. hexagon
c. triangle
d. rectangle

Answer:
d. rectangle

Explanation:
A rectangle has 4 right angles and 2 pairs of opposite sides that are parallel.

Question 4.
Mason drew a quadrilateral with only one pair of opposite sides that are parallel. Which quadrilateral did Mason draw?
Options:
a. square
b. rhombus
c. trapezoid
d. rectangle

Answer:
c. trapezoid

Explanation:
A trapezoid will have only one pair of opposite sides that are parallel.

Question 5.
Which shape has an area of 8 square units and a perimeter of 12 units?
Options:
a. Go Math Grade 3 Answer Key Chapter 12 Two-Dimensional Shapes Describe Triangles img 88
b. Go Math Grade 3 Answer Key Chapter 12 Two-Dimensional Shapes Describe Triangles img 89
c. Go Math Grade 3 Answer Key Chapter 12 Two-Dimensional Shapes Describe Triangles img 90
d. Go Math Grade 3 Answer Key Chapter 12 Two-Dimensional Shapes Describe Triangles img 91

Answer:
a

Explanation:
a. Area = 2 x 4 = 8 square units; Perimeter = 2 + 4 + 2 + 4 = 12 units.

Question 6.
What fraction of the square is shaded?
Go Math Grade 3 Answer Key Chapter 12 Two-Dimensional Shapes Describe Triangles img 92
Options:
a. \(\frac{3}{5}\)
b. \(\frac{5}{3}\)
c. \(\frac{3}{8}\)
d. \(\frac{8}{5}\)

Answer:
c. \(\frac{3}{8}\)

Explanation:
The image has 8 parts. 3 parts are shaded out of 8 parts. So, \(\frac{3}{8}\) is the answer.

Problem Solving Classify Plane Shapes – Page No. 745

Solve each problem.

Question 1.
Steve drew the shapes below. Write the letter of each shape where it belongs in the Venn diagram.
Go Math Grade 3 Answer Key Chapter 12 Two-Dimensional Shapes Problem Solving Classify Plane Shapes img 93
Go Math Grade 3 Answer Key Chapter 12 Two-Dimensional Shapes Problem Solving Classify Plane Shapes img 94

Answer:
Go Math Grade 3 Answer Key Chapter 12 Two-Dimensional Shapes Problem Solving Classify Plane Shapes img 94

Question 2.
Janice drew the shapes below. Write the letter of each shape where it belongs in the Venn diagram.
Go Math Grade 3 Answer Key Chapter 12 Two-Dimensional Shapes Problem Solving Classify Plane Shapes img 95
Go Math Grade 3 Answer Key Chapter 12 Two-Dimensional Shapes Problem Solving Classify Plane Shapes img 96

Answer:
Chapter 12 Problem Solving Classify Plane Shapes image 1 745

Question 3.
Beth drew the shapes below. Write the letter of each shape where it belongs in the Venn diagram.
Go Math Grade 3 Answer Key Chapter 12 Two-Dimensional Shapes Problem Solving Classify Plane Shapes img 97
Go Math Grade 3 Answer Key Chapter 12 Two-Dimensional Shapes Problem Solving Classify Plane Shapes img 98

Answer:

Chapter 12 Problem Solving Classify Plane Shapes image 2 745

Problem Solving Classify Plane Shapes – Page No. 746

Lesson Check

Go Math Grade 3 Answer Key Chapter 12 Two-Dimensional Shapes Problem Solving Classify Plane Shapes img 99

Question 1.
Which shape would go in the section where the two circles overlap?
Options:
a. triangle
b. trapezoid
c. square
d. hexagon

Answer:
c. square

Explanation:
A square is a rectangle with equal lengths of all sides.

Question 2.
Which shape could NOT go in the circle labeled All Sides of Equal Length?
Options:
a. rhombus
b. trapezoid
c. square
d. triangle

Answer:
b. trapezoid

Explanation:
A trapezoid will not have All Sides of Equal Length.

Spiral Review

Question 3.
How many angles greater than a right angle does this triangle have?
Go Math Grade 3 Answer Key Chapter 12 Two-Dimensional Shapes Problem Solving Classify Plane Shapes img 100
Options:
a. 0
b. 1
c. 2
d. 3

Answer:
b. 1

Explanation:
Chapter 12 Problem Solving Classify Plane Shapes image 3 745
1 angle is greater than a right angle.

Question 4.
How many sides of equal length does this triangle appear to have?
Go Math Grade 3 Answer Key Chapter 12 Two-Dimensional Shapes Problem Solving Classify Plane Shapes img 101
Options:
a. 0
b. 1
c. 2
d. 3

Answer:
d. 3

Explanation:
The triangle appears to have 3 sides of equal length.

Question 5.
Madison drew this shape. How many angles less than a right angle does it have?
Go Math Grade 3 Answer Key Chapter 12 Two-Dimensional Shapes Problem Solving Classify Plane Shapes img 102
Options:
a. 0
b. 1
c. 3
d. 5

Answer:
a. 0

Explanation:
Chapter 12 Describe Angles in Plane Shapes image 2 707
0

Question 6.
How many dots are in \(\frac{1}{2}\) of this group?
Go Math Grade 3 Answer Key Chapter 12 Two-Dimensional Shapes Problem Solving Classify Plane Shapes img 103
Options:
a. 6
b. 8
c. 9
d. 18

Answer:
c. 9

Explanation:
9 dots are in \(\frac{1}{2}\) of this group.

Relate Shapes, Fractions, and Area – Page No. 751

Draw lines to divide the shape into equal parts that show the fraction given.

Question 1.
Go Math Grade 3 Answer Key Chapter 12 Two-Dimensional Shapes Relate Shapes, Fractions, and Area img 104
\(\frac{1}{3}\)

Answer:
\(\frac{1}{3}\)

Question 2.
Go Math Grade 3 Answer Key Chapter 12 Two-Dimensional Shapes Relate Shapes, Fractions, and Area img 105
\(\frac{1}{8}\)

Answer:

Chapter 12 Relate Shapes, Fractions, and Area image 1 751

Question 3.
Go Math Grade 3 Answer Key Chapter 12 Two-Dimensional Shapes Relate Shapes, Fractions, and Area img 106
\(\frac{1}{2}\)

Answer:
Chapter 12 Relate Shapes, Fractions, and Area image 2 751

Draw lines to divide the shape into parts with equal area. Write the area of each part as a unit fraction.

Question 4.
Go Math Grade 3 Answer Key Chapter 12 Two-Dimensional Shapes Relate Shapes, Fractions, and Area img 107
4 equal parts
\(\frac{□}{□}\)

Answer:
Chapter 12 Relate Shapes, Fractions, and Area image 4 751
\(\frac{1}{4}\)

Question 5.
Go Math Grade 3 Answer Key Chapter 12 Two-Dimensional Shapes Relate Shapes, Fractions, and Area img 108
6 equal parts
\(\frac{□}{□}\)

Answer:
Chapter 12 Relate Shapes, Fractions, and Area image 5 751
\(\frac{1}{6}\)

Question 6.
Go Math Grade 3 Answer Key Chapter 12 Two-Dimensional Shapes Relate Shapes, Fractions, and Area img 109
3 equal parts
\(\frac{□}{□}\)

Answer:
Chapter 12 Relate Shapes, Fractions, and Area image 6 751
\(\frac{1}{3}\)

Problem Solving

Question 7.
Robert divided a hexagon into 3 equal parts. Show how he might have divided the hexagon. Write the fraction that names each part of the whole you divided.
Go Math Grade 3 Answer Key Chapter 12 Two-Dimensional Shapes Relate Shapes, Fractions, and Area img 110
\(\frac{□}{□}\)

Answer:
Chapter 12 Relate Shapes, Fractions, and Area image 7 751
\(\frac{1}{3}\)

Question 8.
Show how you might divide the shape into 8 equal parts. What fraction names the area of each part of the divided shape?
Go Math Grade 3 Answer Key Chapter 12 Two-Dimensional Shapes Relate Shapes, Fractions, and Area img 111
\(\frac{□}{□}\)

Answer:
Chapter 12 Relate Shapes, Fractions, and Area image 8 751
\(\frac{1}{8}\)

Relate Shapes, Fractions, and Area – Page No. 752

Lesson Check

Question 1.
What fraction names each part of the divided whole?
Go Math Grade 3 Answer Key Chapter 12 Two-Dimensional Shapes Relate Shapes, Fractions, and Area img 112
Options:
a. \(\frac{1}{2}\)
b. \(\frac{1}{3}\)
c. \(\frac{1}{4}\)
d. \(\frac{1}{6}\)

Answer:
b. \(\frac{1}{3}\)

Explanation:
Chapter 12 Relate Shapes, Fractions, and Area image 2 752
The shape has 3 parts. One part is shaded out of 3.

Question 2.
What fraction names the whole area that was divided?
Go Math Grade 3 Answer Key Chapter 12 Two-Dimensional Shapes Relate Shapes, Fractions, and Area img 113
Options:
a. \(\frac{1}{8}\)
b. \(\frac{1}{2}\)
c. \(\frac{8}{8}\)
d. \(\frac{8}{1}\)

Answer:
c. \(\frac{8}{8}\)

Explanation:
Chapter 12 Relate Shapes, Fractions, and Area image 9 752
8 parts are shaded out of 8 parts. So, \(\frac{8}{8}\) is the answer.

Spiral Review

Question 3.
Lil drew the figure below. Which word does NOT describe the shape?
Go Math Grade 3 Answer Key Chapter 12 Two-Dimensional Shapes Relate Shapes, Fractions, and Area img 114
Options:
a. plane shape
b. closed shape
c. open shape
d. curved path

Answer:
c. open shape

Explanation:
The given shape is started and ended at the same point. So, it is not an open shape.

Question 4.
How many line segments does this shape have?
Go Math Grade 3 Answer Key Chapter 12 Two-Dimensional Shapes Relate Shapes, Fractions, and Area img 115
Options:
a. 6
b. 5
c. 4
d. 3

Answer:
b. 5

Explanation:
Chapter 12 Describe Plane Shapes image 4 701
5

Use the Venn diagram for 5–6.
Go Math Grade 3 Answer Key Chapter 12 Two-Dimensional Shapes Relate Shapes, Fractions, and Area img 116

Question 5.
Which shape would go in the section where the two circles overlap?
Options:
a. triangle
b. square
c. trapezoid
d. pentagon

Answer:
b. square

Explanation:
A square have right angles and all sides with equal lengths.

Question 6.
Which shape could NOT go in the circle labeled All Sides of Equal Length?
Options:
a. square
b. rhombus
c. triangle
d. rectangle

Answer:
d. rectangle

Explanation:
A rectangle doesn’t have All Sides of Equal Length.

Review/Test – Page No. 753

Question 1.
Which words describe this shape? Mark all that apply.
Go Math Grade 3 Answer Key Chapter 12 Two-Dimensional Shapes Review/Test img 117
Options:
a. polygon
b. open shape
c. pentagon
d. quadrilateral

Answer:
a. polygon
c. pentagon

Explanation:
Chapter 12 Concepts and Skills image 3 721
A polygon is a closed plane shape that is made up of line segments that meet only at their endpoints. Each line segment in a polygon is a side. So, the shape is a polygon. Also, the shape has five sides and five angles. So, it is a pentagon.

Question 2.
Umberto drew one side of a quadrilateral with 4 equal sides and no right angles. Draw the other 3 sides to complete Umberto’s shape.
Go Math Grade 3 Answer Key Chapter 12 Two-Dimensional Shapes Review/Test img 118
Type below:
____________

Answer:
Chapter 12 Review image 1 753

Question 3.
Mikael saw a painting that included this shape.
Go Math Grade 3 Answer Key Chapter 12 Two-Dimensional Shapes Review/Test img 119
For numbers 3a–3d, select True or False for each statement about the shape.
a. The shape has no right angles.
i. True
ii. False

Answer:
ii. False

Question 3.
b. The shape has 2 angles greater than a right angle.
i. True
ii. False

Answer:
ii. False

Question 3.
c. The shape has 2 right angles.
i. True
ii. False

Answer:
i. True

Question 3.
d. The shape has 1 angle greater than a right angle.
i. True
ii. False

Answer:
i. True

Review/Test – Page No. 754

Question 4.
Fran used a Venn Diagram to sort shapes.
Go Math Grade 3 Answer Key Chapter 12 Two-Dimensional Shapes Review/Test img 120
Part A
Draw another plane shape that belongs inside the left circle of the diagram but NOT in the section where the circles overlap.
Type below:
____________

Answer:
Chapter 12 Review image 1 754

Question 4.
Part B
How can you describe the shapes in the section where the circles overlap?
Type below:
____________

Answer:
The shapes in the circle overlap are polygons with right angles. Also, those shapes have 4 sides and 4 angles named as quadrilaterals.

Question 5.
Match each object in the left column with its name in the right column.
Go Math Grade 3 Answer Key Chapter 12 Two-Dimensional Shapes Review/Test img 121
Type below:
____________

Answer:
Chapter 12 Review image 2 754

Question 6.
Describe the angles and sides of this triangle.
Go Math Grade 3 Answer Key Chapter 12 Two-Dimensional Shapes Review/Test img 122
Type below:
____________

Answer:
The triangle has 3 sides of equal length. Also, the triangle has 3 angles with less than a right angle.

Review/Test – Page No. 755

Question 7.
Which words describe this shape. Mark all that apply.
Go Math Grade 3 Answer Key Chapter 12 Two-Dimensional Shapes Review/Test img 123
Options:
a. rectangle
b. rhombus
c. quadrilateral
d. square

Answer:
a. rectangle
c. quadrilateral
d. square

Explanation:
The given shape is a square or rectangle with 4 sides and 4 angles.

Question 8.
Divide each shape into the number of equal parts shown. Then write the fraction that describes each part of the whole.
Go Math Grade 3 Answer Key Chapter 12 Two-Dimensional Shapes Review/Test img 124
Type below:
____________

Answer:
\(\frac{1}{3}\)
\(\frac{1}{6}\)
\(\frac{1}{8}\)

Explanation:
Chapter 12 Review image 1 755
\(\frac{1}{3}\)
\(\frac{1}{6}\)
\(\frac{1}{8}\)

Question 9.
Han drew a triangle with 1 angle greater than a right angle.
For numbers 9a–9d, choose Yes or No to tell whether the triangle could be the triangle Han drew.
a. Go Math Grade 3 Answer Key Chapter 12 Two-Dimensional Shapes Review/Test img 125
i. yes
ii. no

Answer:
ii. no

Explanation:
It has all the angles less than a right angle.

Question 9.
b. Go Math Grade 3 Answer Key Chapter 12 Two-Dimensional Shapes Review/Test img 126
i. yes
ii. no

Answer:
ii. no

Explanation:
It has one right angle and two angles with less than a right angle.

Question 9.
c. Go Math Grade 3 Answer Key Chapter 12 Two-Dimensional Shapes Review/Test img 127
i. yes
ii. no

Answer:
i. yes

Explanation:
It has one angle greater than a right angle

Question 9.
d. Go Math Grade 3 Answer Key Chapter 12 Two-Dimensional Shapes Review/Test img 128
i. yes
ii. no

Answer:
i. yes

Explanation:
It has one angle greater than a right angle

Review/Test – Page No. 756

Question 10.
Look at this group of pattern blocks.
Go Math Grade 3 Answer Key Chapter 12 Two-Dimensional Shapes Review/Test img 129
Chapter 12 Review image 6 756
Part A
Sort the pattern blocks by sides. How many groups did you make? Explain how you sorted the shapes.
Type below:
____________

Answer:
A has 6 sides 6 angles = hexagon.
B has 4 sides 4 angles = quadrilateral
C has 4 sides 4 angles = quadrilateral
D has 3 sides 3 angles = triangle
E has 4 sides 4 angles = quadrilateral
F has 4 sides 4 angles = quadrilateral
1 triangle, 4 quadrilateral, and 1 hexagon blocks available.

Question 10.
Part B
Sort the pattern blocks by angles. How many groups did you make? Explain how you sorted the shapes.
Type below:
____________

Answer:
A and F have greater than a right angle.
B has 4 right angles.
C, D, and E have all the angles less than a right angle.

Question 11.
Teresa drew a quadrilateral that had 4 sides of equal length and no right angles. What quadrilateral did she draw?
____________

Answer:
rhombus

Review/Test – Page No. 757

Question 12.
Rhea used a Venn diagram to sort shapes. What label could she use for circle A?
Go Math Grade 3 Answer Key Chapter 12 Two-Dimensional Shapes Review/Test img 130
Type below:
____________

Answer:
Right Angle

Question 13.
Colette drew lines to divide a rectangle into equal parts that each represent \(\frac{1}{6}\) of the whole area. Her first line is shown. Draw lines to complete Colette’s model.
Go Math Grade 3 Answer Key Chapter 12 Two-Dimensional Shapes Review/Test img 131
Type below:
____________

Answer:
Chapter 12 Review image 1 757

Question 14.
Brad drew a quadrilateral. Select the pairs of sides that appear to be parallel. Mark all that apply.
Go Math Grade 3 Answer Key Chapter 12 Two-Dimensional Shapes Review/Test img 132
Options:
a. a and b
b. b and d
c. c and a
d. d and c

Answer:
b. b and d
c. c and a

Review/Test – Page No. 758

Question 15.
Give two reasons that this shape is not a polygon.
Go Math Grade 3 Answer Key Chapter 12 Two-Dimensional Shapes Review/Test img 133
Type below:
____________

Answer:
1. The shape is not a closed shape.
2. The shape has no line segments.

Question 16.
A triangle has 1 angle greater than a right angle. What must be true about the other angles? Mark all that apply.
Options:
a. At least one must be less than a right angle.
b. One could be a right angle.
c. Both must be less than a right angle.
d. One must be greater than a right angle.

Answer:
c. Both must be less than a right angle.

Question 17.
Ava drew a quadrilateral with 2 pairs of opposite sides that are parallel. The shape has at least 2 right angles. Draw a shape that Ava could have drawn.
Type below:
____________

Answer:
Chapter 12 Describe Angles in Plane Shapes image 1 707

Question 18.
For 18a–18d, select True or False for each description of a ray.
Go Math Grade 3 Answer Key Chapter 12 Two-Dimensional Shapes Review/Test img 134
a. straight
i. True
ii. False

Answer:
i. True

Question 18.
b. has 2 endpoints
i. True
ii. False

Answer:
ii. False

Question 18.
c. part of a line
i. True
ii. False

Answer:
i. True

Question 18.
d. continues in 1 direction
i. True
ii. False

Answer:
i. True

Conclusion

Go Math Grade 3 Answer Key Chapter 12 Two-Dimensional Shapes makes it easy for you to get a grip on the subjects. Refer to the HMH Go Math Grade 3 Answer Key for quick reference and resolve your queries. Get Go Math Grade 3 Answer Key Chapter 12 Two-Dimensional Shapes Extra Practice and test your knowledge on the Chapter.